JAC Class 12 Political Science Important Questions Chapter 7 समकालीन विश्व में सरक्षा

Jharkhand Board JAC Class 12 Political Science Important Questions Chapter 7 समकालीन विश्व में सरक्षा Important Questions and Answers.

JAC Board Class 12 Political Science Important Questions Chapter 7 समकालीन विश्व में सरक्षा

बहुचयनात्मक प्रश्न

1. एंटी बैलेस्टिक मिसाइल संधि (ABM) किस वर्ष हुई ?
(अ) 1975
(ब) 1978
(स) 1956
(द) 1972
उत्तर:
(द) 1972

2. निम्नलिखित में से कौन-सी संधि अस्त्र नियंत्रण संधि थी
(अ) अस्त्र परिसीमन संधि – 2 ( SALT – II)
(ब) सामरिक अस्त्र न्यूनीकरण संधि (स्ट्रेटजिक आर्म्स रिडक्शन ट्रीटी – SIART)
(स) परमाणु अप्रसार संधि (NPT)
(द) उपरोक्त सभी
उत्तर:
(द) उपरोक्त सभी

3. जैविक हथियार संधि कब की गई ?
(अ) 1975
(ब) 1992
(स) 1972
(द) 1968
उत्तर:
(स) 1972

4. सुरक्षा की अवधारणा कितने प्रकार की है ?
(अ) तीन
(ब) चार
(स) दो
(द) एक
उत्तर:
(स) दो

5. परमाणु अप्रसार संधि जिस सन् में हुई वह है-
(अ) 1968
(ब) दो
(स) 1972
(द) एक
उत्तर:
(अ) 1968

JAC Class 12 Political Science Important Questions Chapter 7 समकालीन विश्व में सरक्षा

6. निम्न में से किस संधि ने अमरीका और सोवियत संघ को बैलेस्टिक मिसाइलों को रक्षा कवच के रूप में इस्तेमाल करने से रोक:
(अ) जैविक हथियार संधि
(ब) एंटी बैलेस्टिक मिसाइल संधि
(स) रासायनिक हथियार संधि
(द) परमाणु अप्रसार संधि
उत्तर:
(ब) एंटी बैलेस्टिक मिसाइल संधि

7. अमरीका के वर्ल्ड ट्रेड सेंटर पर आतंकवादियों ने हमला किया
(अ) 11 सितंबर, 2001
(ब) 10 अक्टूबर, 2001
(स) 11 नवम्बर, 2002
(द) 9 दिसम्बर, 2002
उत्तर:
(अ) 11 सितंबर, 2001

8. भारत ने पहला परमाणु परीक्षण किया
(अ) 1974 में
(ब) 1975 में
(स) 1978 में
(द) 1980 में
उत्तर:
(अ) 1974 में

9. पाकिस्तान ने भारत पर अब तक कुल कितनी बार हमला किया है?
(अ) तीन
(ब) दो
(स) चार
(द) पाँच
उत्तर:
(अ) तीन

10. क्योटो के प्रोटोकॉल पर हस्ताक्षर कब किया गया?.
(अ) 1998
(ब) 1997
(स) 1991
(द) 1992
उत्तर:
(ब) 1997

रिक्त स्थानों की पूर्ति कीजिए:

1. सुरक्षा की ………………………. धारणा में माना जाता है कि किसी देश की सुरक्षा को ज्यादातर खतरा उसकी सीमा के बाहर से होता है।
उत्तर:
परंपरागत

2. सुरक्षा – नीति का संबंध युद्ध की आशंका को रोकने में होता है जिसे ………………………….. कहा जाता है।
उत्तर:
अपरोध

3. …………………….. सुरक्षा नीति का एक तत्त्व शक्ति संतुलन है।
उत्तर:
परम्परागत

4. जैविक हथियार संधि पर ………………………. से ज्यादा देशों ने संधि पर हस्ताक्षर किए।
उत्तर:
155

5. …………………………संधि ने परमाणविक आयुधों को हासिल कर सकने वाले देशों की संख्या कम की।
उत्तर:
परमाणु अप्रसार

6. सुरक्षा की …………………… धारणा को ‘मानवता की सुरक्षा’ अथवा ……………………………. कहा जाता है।
उत्तर:
अपारंपरिक, विश्व- रक्षा

अतिलघूत्तरात्मक प्रश्न

प्रश्न 1.
सुरक्षा का बुनियादी अर्थ लिखिए।
उत्तर:
सुरक्षा का बुनियादी अर्थ है। खतरे से आजादी।

प्रश्न 2.
सुरक्षा की कितनी धारणाएँ हैं?
उत्तर:
सुरक्षा की दो धारणाएँ हैं। पारंपरिक और अपारंपरिक।

प्रश्न 3.
लोग पलायन क्यों करते हैं? कोई एक कारण बताएँ।
उत्तर:
लोग आजीविका हेतु पलायन करते हैं।

JAC Class 12 Political Science Important Questions Chapter 7 समकालीन विश्व में सरक्षा

प्रश्न 4.
भारत के किन दो पड़ौसी देशों के पास परमाणु हथियार हैं?
उत्तर:
भारत के दो पड़ौसी देशों – पाकिस्तान और चीन के पास परमाणु हथियार हैं।

प्रश्न 5.
आतंकवाद सुरक्षा के लिए खतरे की किस श्रेणी में आता है?
उत्तर:
अपरम्परागत श्रेणी में।

प्रश्न 6.
एन. पी. टी. का पूरा नाम क्या है? यह किस वर्ष में हुई?
उत्तर:
एन. पी. टी. का पूरा नाम है। न्यूक्लियर नॉन प्रोलिफेरेशन ट्रीटी । यह सन् 1968 में हुई।

प्रश्न 7.
सैन्य शक्ति का आधार क्या है?
उत्तर:
सैन्य – शक्ति का आधार आर्थिक और प्रौद्योगिकी की ताकत है।

प्रश्न 8.
ओसामा बिन लादेन किस आतंकवादी समूह का था?
उत्तर:
अल-कायदा।

प्रश्न 9.
पारम्परिक सुरक्षा की धारणा के अन्तर्गत ‘अपरोध’ का क्या अभिप्राय है?
उत्तर:
पारम्परिक सुरक्षा की धारणा के अन्तर्गत ‘अपरोध’ का अर्थ है – युद्ध की आशंका को रोकना।

प्रश्न 10.
पारम्परिक बाह्य सुरक्षा नीति के कोई दो तत्त्व लिखिये।
उत्तर:
शक्ति सन्तुलन और गठबंधन बनाना।

प्रश्न 11.
एशिया- अफ्रीका के नव-स्वतंत्र देशों में आंतरिक सुरक्षा के लिए खतरा पैदा करने वाली किसी एक समस्या का नाम लिखिये।
उत्तर:
अलगाववादी आंदोलन|

प्रश्न 12.
सुरक्षा की अपारंपरिक धारणा को क्या कहा जाता है?
उत्तर:
सुरक्षा की अपारंपरिक धारणा को ‘मानवता की सुरक्षा’ अथवा ‘विश्व- रक्षा’ कहा जाता है।

प्रश्न 13.
मानवता की सुरक्षा का प्राथमिक लक्ष्य क्या है?
उत्तर:
मानवता की सुरक्षा का प्राथमिक लक्ष्य व्यक्तियों की संरक्षा है।

JAC Class 12 Political Science Important Questions Chapter 7 समकालीन विश्व में सरक्षा

प्रश्न 14.
व्यापकतम अर्थ में मानवता की रक्षा से क्या आशय है?
उत्तर:
व्यापकतम अर्थ में मानवता की रक्षा से आशय ‘अभाव से मुक्ति’ और ‘भय से मुक्ति’ है।

प्रश्न 15.
युद्ध के सिवाय मानव सुरक्षा के किन्हीं अन्य चार खतरों का उल्लेख कीजिए।
उत्तर:
मानव सुरक्षा के खतरे निम्नलिखित हैं।

  1. पर्यावरण ह्रास
  2. ग्रीन हाउस गैसों का अत्यधिक उत्सर्जन
  3. नाभिकीय युद्ध का भय
  4. बढ़ती हुई जनसंख्या।

प्रश्न 16.
सुरक्षा के खतरे के किन्हीं दो नये स्रोतों को सूचीबद्ध कीजिए।
उत्तर:

  1. वैश्विक ताप वृद्धि
  2. अन्तर्राष्ट्रीय आतंकवाद।

प्रश्न 17.
किन्हीं दो शक्तियों के नाम लिखें जो सैनिक शक्ति का आधार हैं।
उत्तर:
आर्थिक शक्ति एवं, तकनीकी शक्ति।

प्रश्न 18.
सुरक्षा के मुख्य दो रूपों के नाम लिखिये।
उत्तर:
सुरक्षा के दो रूप हैं।

  1. पारम्परिक सुरक्षा और
  2. अपारंपरिक सुरक्षा।

प्रश्न 19.
पारम्परिक सुरक्षा से क्या आशय है?
उत्तर:
पारम्परिक सुरक्षा में यह स्वीकार किया गया है कि हिंसा का प्रयोग जहाँ तक हो सके कम से कम होना

प्रश्न 20.
अपारम्परिक सुरक्षा से क्या आशय है?
उत्तर:
अपारम्परिक सुरक्षा की धारणा सैन्य खतरों से सम्बन्धित न होकर मानवीय अस्तित्व को चोट पहुँचाने वाले व्यापक खतरों से है।

प्रश्न 21.
परम्परागत सुरक्षा और अपरम्परागत सुरक्षा में एक अंतर लिखें।
उत्तर:
परम्परागत सुरक्षा का दृष्टिकोण संकुचित है जबकि अपरम्परागत सुरक्षा का दृष्टिकोण व्यापक है।

प्रश्न 22.
निःशस्त्रीकरण से आप क्या समझते हैं?
उत्तर:
निशस्त्रीकरण से अभिप्राय हथियारों के निर्माण या उनको हासिल करने पर अंकुश लगाना है।

प्रश्न 23.
विश्व तापन से आप क्या समझते हैं?
उत्तर:
विश्व तापन से अभिप्राय विश्व स्तर पर पारे में लगातार होने वाली वृद्धि है, जिसके कारण विश्व का वातावरण गर्म होता जा रहा है।

JAC Class 12 Political Science Important Questions Chapter 7 समकालीन विश्व में सरक्षा

प्रश्न 24.
निरस्त्रीकरण के दो उदाहरण दीजिए।
उत्तर:

  1. जैविक हथियार संधि
  2. रासायनिक हथियार संधि।

प्रश्न 25.
आतंकवाद के कोई दो रूप लिखिये।
उत्तर:
आतंकवाद के दो रूप हैं।

  1. विमान अपहरण करके आतंकवाद फैलाना।
  2. भीड़ भरी जगहों पर विस्फोट करना।

प्रश्न 26.
सुरक्षा की अपारंपरिक धारणा के दो पक्ष बताइये।
उत्तर:
सुरक्षा की अपारंपरिक धारणा के दो पक्ष हैं।

  1. मानवता की सुरक्षा और
  2. विश्व सुरक्षा।

प्रश्न 27.
सुरक्षा नीति के दो घटक बताइये।
उत्तर:

  1. सैन्य क्षमता को मजबूत करना।
  2. अपने सुरक्षा हितों को बचाने के लिये अन्तर्राष्ट्रीय कायदों और संस्थाओं को मजबूत करना।

प्रश्न 28.
ऐसी दो संधियों के नाम बताइये जो अस्त्र नियंत्रण से सम्बन्धित हैं।
उत्तर:

  1. सामरिक अस्त्र परिसीमन संधि
  2. परमाणु अप्रसार संधि।

प्रश्न 29.
विश्व सुरक्षा का क्या अर्थ है?
उत्तर:
विश्व सुरक्षा से आशय है- पृथ्वी के बढ़ते तापमान, अन्तर्राष्ट्रीय आतंकवाद, एड्स ‘जैसे असाध्य रोगों पर रोक लगाना।

प्रश्न 30.
क्षेत्रीय सुरक्षा से क्या आशय है?
उत्तर:
क्षेत्रीय सुरक्षा से आशय है। सशस्त्र विद्रोहियों तथा विदेशी आक्रमणकारियों से किसी भू भाग तथा उसके निवासियों के जान-माल की रक्षा करना।

प्रश्न 31.
राष्ट्रीय सुरक्षा, सुरक्षा की किस अवधारणा से जुड़ी हुई है?
उत्तर:
सुरक्षा की पारम्परिक अवधारणा से।

प्रश्न 32.
आतंकवाद का क्या अभिप्राय है?
उत्तर:
आतंकवाद का अभिप्राय है। राजनीतिक हिंसा, जिसका निशाना नागरिक होते हैं ताकि समाज में दहशत पैदा की जा सके।

प्रश्न 33.
मानवाधिकार की पहली कोटि कौन-सी है?
उत्तर:
राजनैतिक अधिकारों की।

JAC Class 12 Political Science Important Questions Chapter 7 समकालीन विश्व में सरक्षा

प्रश्न 34.
केमिकल वीपन्स कन्वेंशन (CWC) संधि पर कितने देशों ने हस्ताक्षर किये थे?
उत्तर:
181 देशों ने।

प्रश्न 35.
अस्त्र नियंत्रण से आपका क्या तात्पर्य है?
उत्तर:
अस्त्र नियंत्रण का आशय है हथियारों को विकसित करने अथवा उनको हासिल करने के संबंध में कुछ कानून का पालन करना।

प्रश्न 36.
सुरक्षा की अपारंपरिक धारणा को ‘मानवता की सुरक्षा’ अथवा ‘विश्व – रक्षा’ क्यों कहा जाता है?
उत्तर:
क्योंकि सुरक्षा की जरूरत सिर्फ राज्य ही नहीं व्यक्तियों और समुदायों अपितु समूची मानवता को है।

प्रश्न 37.
परम्परागत धारणा के अनुसार सुरक्षा के कितने प्रकार होते हैं?
उत्तर:
दो – बाह्य सुरक्षा, आंतरिक सुरक्षा।

प्रश्न 38.
मानवाधिकार को कितने कोटियों में रखा गया है?
उत्तर:
तीन।

प्रश्न 39.
राजनैतिक अधिकारों के उदाहरण दीजिए।
उत्तर:
अभिव्यक्ति और सभा करने की आजादी।

प्रश्न 40.
सहयोगमूलक सुरक्षा से आपका क्या अभिप्राय है?
उत्तर:
अपरम्परागत खतरों के लिए सैन्य संघर्ष की बजाय आपसी सहयोग अपनाना।

प्रश्न 41.
‘क्योटो प्रोटोकॉल’ क्या है?
उत्तर:
‘क्योटो प्रोटोकॉल’ में वैश्विक तापवृद्धि पर काबू पाने तथा ग्रीनहाऊस गैसों के उत्सर्जन को कम करने के संबंध में दिशा-निर्देश दिए गए हैं।

JAC Class 12 Political Science Important Questions Chapter 7 समकालीन विश्व में सरक्षा

प्रश्न 42.
क्योटो प्रोटोकॉल पर कितने देशों ने हस्ताक्षर किए हैं?
उत्तर:
160

लघूत्तरात्मक प्रश्न

प्रश्न 1.
सुरक्षा का अर्थ स्पष्ट कीजिये।
उत्तर:
सुरक्षा का बुनियादी अर्थ है। खतरे से आजादी संकीर्ण दृष्टिकोण के अनुसार इसका अभिप्राय व्यक्तिगत मूल्यों की सुरक्षा से है और व्यापक दृष्टिकोण के अनुसार इसका अभिप्राय बड़े और गंभीर खतरों से सुरक्षा है।

प्रश्न 2.
निःशस्त्रीकरण के मार्ग में आने वाली दो कठिनाइयाँ लिखें।
उत्तर:
निःशस्त्रीकरण के मार्ग में आने वाली दो कठिनाइयाँ ये हैं।

  1. महाशक्तियों में अस्त्र-शस्त्रों के आधुनिकीकरण के प्रति मोह विद्यमान है।
  2. महाशक्तियों में एक-दूसरे के प्रति अविश्वास की भावना भी अभी बनी हुई है।

प्रश्न 3.
‘सुरक्षा’ की धारणा अपने आप में भुलैयादार धारणा है। कैसे?
उत्तर:
‘सुरक्षा’ की धारणा अपने आप में भुलैयादार है क्योंकि इसकी धारणा हर सदी में एकसमान नहीं होती है। विश्व के सारे नागरिकों के लिए सुरक्षा के मायने अलग-अलग होते हैं। विकासशील देशों को बेरोजगार, भुखमरी तथा आर्थिक व सामाजिक पिछड़ेपन से खुद की सुरक्षा करनी होती है तो विकसित देशों को पर्यावरण प्रदूषण, वैश्विक तापवृद्धि जैसे समस्याओं से स्वयं की सुरक्षा करनी होती है।

प्रश्न 4.
आतंकवाद क्या है?
उत्तर:
आतंकवाद का अर्थ है। राजनीतिक हिंसा, जिसका निशाना नागरिक होते हैं। ताकि समाज में दहशत पैदा की जा सके। इसकी चिर-परिचित तकनीकें हैं। विमान अपहरण, भीड़भरी जगहों, जैसे रेलवे स्टेशनों, होटल, बाजार, धर्मस्थल आदि जगहों में बम लगाकर विस्फोट करना।

प्रश्न 5.
आतंकवादी दहशत क्यों पैदा करते हैं?
उत्तर:
आतंकवादी सरकार से अपनी मांगों को मनवाने के लिए दहशत पैदा करते हैं। दूसरे, उन्हें दहशत पैदा करने के लिए ही अपने संगठन से धन व अन्य सुविधायें मिलती हैं।

JAC Class 12 Political Science Important Questions Chapter 7 समकालीन विश्व में सरक्षा

प्रश्न 6.
पारम्परिक सुरक्षा से क्या आशय है?
उत्तर;
पारम्परिक सुरक्षा:
पारम्परिक सुरक्षा में यह स्वीकार किया गया है कि हिंसा का प्रयोग जहाँ तक हो सके कम से कम होना चाहिए। युद्ध के लक्ष्य और साधन दोनों का इससे सम्बन्ध है। यह न्याय युद्ध की परम्परा का विस्तार, निःशस्त्रीकरण, अस्त्र- नियंत्रण और विश्वास बहाली के उपायों पर आधारित है।

प्रश्न 7.
सुरक्षा की परम्परागत तथा गैर-परम्परागत धारणाओं में क्या अन्तर है?
उत्तर:
सुरक्षा की परम्परागत धारणा में सिर्फ भूखण्ड तथा उसमें रहने वाले लोगों की जान-माल की रक्षा करना तथा सशस्त्र सैन्य हमलों को रोकना है जबकि अपरम्परागत धारणा में भू-भाग, प्राणियों और सम्पत्ति की सुरक्षा के साथ- साथ पर्यावरण तथा मानवाधिकारों की सुरक्षा भी शामिल है।

प्रश्न 8.
अमरीका तथा सोवियत संघ जैसी महाशक्तियों ने अस्त्र- नियंत्रण का सहारा क्यों लिया?
उत्तर:
मरीका तथा सोवियत संघ सामूहिक संहार के अस्त्र यानी परमाण्विक हथियार का विकल्प नहीं छोड़ना चाहती थीं इसलिए दोनों ने अस्त्र-नियंत्रण का सहारा लिया।

प्रश्न 9.
अस्त्र नियंत्रण का अभिप्राय क्या है?
उत्तर:
अस्त्र नियंत्रण के अंतर्गत हथियारों को विकसित करने अथवा उनको हासिल करने के संबंध में कुछ कायदे-कानूनों का पालन करना पड़ता है। उदाहरण के लिए सामरिक अस्त्र परिसीमन संधि – 2, सामरिक अस्त्र न्यूनीकरण संधि इत्यादि संधियाँ अस्त्र नियंत्रण के उदाहरण हैं.

प्रश्न 10.
एंटी बैलेस्टिक संधि कब और क्यों की गई?
उत्तर:
सन् 1972 में एंटी बैलेस्टिक संधि की गई। इस संधि ने अमरीका और सोवियत संघ को बैलेस्टिक मिसाइलों को रक्षा कवच के रूप में इस्तेमाल करने से रोका। इस संधि में दोनों देशों को सीमित संख्या में ऐसी रक्षा प्रणाली तैनात करने की अनुमति थी लेकिन इस संधि ने दोनों देशों को ऐसी रक्षा प्रणाली के व्यापक उत्पादन से रोक दिया।

प्रश्न 11.
अपरोध नीति क्या है?
उत्तर:
युद्ध की आशंका को रोकने की सुरक्षा नीति को अपरोध नीति कहा जाता है। इसके अन्तर्गत एक पक्ष द्वारा . युद्ध से होने वाले विनाश को इस हद तक बढ़ाने के संकेत दिये जाते हैं ताकि दूसरा पक्ष सहम कर हमला करने से रुक जाये।

प्रश्न 12.
सुरक्षा की पारंपरिक अवधारणा में किस खतरे को सर्वाधिक खतरनाक माना जाता है?
उत्तर:
सुरक्षा की पारंपरिक अवधारणा में सैन्य खतरे को किसी देश के लिए सबसे ज्यादा खतरनाक माना जाता है। इसका स्रोत कोई दूसरा देश होता है जो सैन्य हमले की धमकी देकर संप्रभुता, स्वतंत्रता और क्षेत्रीय अखंडता जैसे किसी देश के केन्द्रीय मूल्यों के लिए खतरा पैदा करता है।

JAC Class 12 Political Science Important Questions Chapter 7 समकालीन विश्व में सरक्षा

प्रश्न 13.
बायोलॉजिकल वैपन्स कन्वेंशन, 1972 द्वारा क्या निर्णय लिया गया?
उत्तर:
सन् 1972 की जैविक हथियार संधि (बायोलॉजिकल वैपन्स कन्वेंशन) ने जैविक हथियारों को बनाना और रखना प्रतिबंधित कर दिया गया। 155 से अधिक देशों ने इस पर हस्ताक्षर किए हैं जिनमें विश्व की सभी महाशक्तियाँ शामिल हैं।.

प्रश्न 14.
आपकी दृष्टि में बुनियादी तौर पर किसी सरकार के पास युद्ध की स्थिति में कौनसे विकल्प हो सकते हैं? कोई दो स्पष्ट कीजिए।
अथवा
किसी सरकार के पास युद्ध की स्थिति में सुरक्षा के कितने विकल्प होते हैं?
उत्तर:
किसी सरकार के पास युद्ध की स्थिति में तीन विकल्प होते हैं।

  1. आत्म-समर्पण करना तथा दूसरे पक्ष की बात को बिना युद्ध किये मान लेना।
  2. युद्ध से होने वाले नाश को इस हद तक बढ़ाने के संकेत देना कि दूसरा पक्ष सहम कर हमला करने से रुक जाये।
  3. यदि युद्ध ठन जाये तो अपनी रक्षा करना।

प्रश्न 15.
बाहरी सुरक्षा हेतु गठबंधन बनाने से क्या आशय है?
उत्तर:
गठबंधन बनाना:
गठबंधन में कई देश शामिल होते हैं और सैन्य हमले को रोकने अथवा उससे रक्षा करने के लिए समवेत कदम उठाते हैं। अधिकांश गठबंधनों को लिखित संधि से एक औपचारिक रूप मिलता है जिसमें यह स्पष्ट होता है कि खतरा किससे है? गठबंधन राष्ट्रीय हितों पर आधारित होते हैं।

प्रश्न 16.
एक उदाहरण देकर यह स्पष्ट कीजिये कि राष्ट्रीय हितों के बदलने पर गठबंधन भी बदल जाते हैं।
उत्तर:
राष्ट्रीय हितों के बदलने पर गठबंधन भी बदल जाते हैं। उदाहरण के लिए, अमरीका ने 1980 के दशक में सोवियत संघ के खिलाफ इस्लामी उग्रवादियों को समर्थन दिया, लेकिन 9/11 के आतंकवादी हमले के बाद उसने उन्हीं इस्लामी उग्रवादियों के खिलाफ मोर्चा खोल दिया।

JAC Class 12 Political Science Important Questions Chapter 7 समकालीन विश्व में सरक्षा

प्रश्न 17.
निरस्त्रीकरण से क्या आशय है?
उत्तर:
निरस्त्रीकरण-निरस्त्रीकरण सुरक्षा की इस धारणा पर आधारित है कि देशों के बीच एक न एक रूप में सहयोग हो। निरस्त्रीकरण की मांग होती है कि सभी राज्य चाहे उनका आकार, ताकत और प्रभाव कुछ भी हो कुछ ख़ास किस्म के हथियारों से बाज आयें।

प्रश्न 18.
आप वर्तमान विश्व में सुरक्षा को किससे खतरा मानते हैं? किन्हीं दो कारणों का उल्लेख कीजिये।
उत्तर:
हम वर्तमान विश्व में सुरक्षा को खतरा निम्न दो कारणों को मानते हैं।

  1. वैश्विक ताप वृद्धि: वर्तमान में विश्व में वैश्विक ताप वृद्धि सम्पूर्ण मानव जाति के लिए खतरा है।
  2. प्रदूषण: पर्यावरण में तीव्रता से बढ़ रहे प्रदूषण से विश्व की सुरक्षा के समक्ष एक गंभीर खतरा उत्पन्न हो गया है।

प्रश्न 19.
सुरक्षा के पारंपरिक तरीके के रूप में अस्त्र नियंत्रण को स्पष्ट कीजिये।
उत्तर:
अस्त्र नियंत्रण के अन्तर्गत हथियारों के संबंध में कुछ कायदे-कानूनों का पालन करना पड़ता है। जैसे, सन् 1972 की एंटी बैलिस्टिक मिसाइल संधि (ABM) ने अमरीका और सोवियत संघ को बैलेस्टिक मिसाइलों को रक्षा- कवच के रूप में इस्तेमाल करने से रोका।

प्रश्न 20.
मानवाधिकारों को कितनी कोटियों में रखा गया है?
उत्तर:
मानवाधिकारों को तीन कोटियों (श्रेणियों) में रखा गया है। ये हैं।

  1. राजनैतिक अधिकार, जैसे अभिव्यक्ति और सभा करने की स्वतंत्रता।
  2. आर्थिक और सामाजिक अधिकार।
  3. उपनिवेशीकृत जनता अथवा जातीय और मूलवासी अल्पसंख्यकों के अधिकार।

प्रश्न 21.
आपकी दृष्टि में मानवता की सुरक्षा के व्यापकतम अर्थ में कौन-कौनसी सुरक्षा को शामिल करेंगे?
उत्तर:
मानवतावादी सुरक्षा के व्यापकतम अर्थ में हम युद्ध, जनसंहार, आतंकवाद, अकाल, महामारी, प्राकृतिक आपदा से सुरक्षा के साथ-साथ ‘अभाव से मुक्ति’ और ‘भय से मुक्ति’ को भी शामिल करेंगे।

प्रश्न 22.
आप भारत की सुरक्षा नीति के दो घटक बताइये।
उत्तर:
भारत की सुरक्षा नीति के दो घटक ये हैं-

  1. सैन्य क्षमता को मजबूत करना – अपने चारों तरफ परमाणु हथियारों से लैस देशों को देखते हुए भारत ने 1974 तथा 1998 में परमाणु परीक्षण कर अपनी सैन्य क्षमता का विकास किया है।
  2. अन्तर्राष्ट्रीय संस्थाओं को मजबूत करना – भारत ने अपने सुरक्षा हितों को बचाने के लिए अन्तर्राष्ट्रीय कायदों एवं संस्थाओं को मजबूत करने की नीति अपनायी है।

प्रश्न 23.
‘आंतरिक रूप से विस्थापित जन’ से क्या आशय है?
उत्तर:
जो लोग राजनीतिक उत्पीड़न, जातीय हिंसा आदि किसी कारण से अपना घर-बार छोड़कर अपने ही देश या राष्ट्र की सीमा के भीतर ही रह रहे हैं, उन्हें ‘आंतरिक रूप से विस्थापित जन’ कहा जाता है। जैसे कश्मीर घाटी छोड़ने वाले कश्मीरी पंडित।

JAC Class 12 Political Science Important Questions Chapter 7 समकालीन विश्व में सरक्षा

प्रश्न 24.
युद्ध और शरणार्थी समस्या के आपसी सम्बन्ध पर प्रकाश डालिये।
उत्तर:
युद्ध और शरणार्थी समस्या के बीच आपस में सकारात्मक सम्बन्ध है क्योंकि युद्ध या सशस्त्र संघर्षों के कारण ही शरणार्थी की समस्या बढ़ती है। उदाहरण के लिए सन् 1990 के दशक में कुल 60 जगहों से शरणार्थी प्रवास करने को मजबूर हुए और इनमें से तीन को छोड़कर शेष सभी के मूल में सशस्त्र संघर्ष था।

प्रश्न 25.
परमाणु अप्रसार संधि, 1968 की एक अस्त्र नियंत्रण संधि के रूप में व्याख्या कीजिये।
उत्तर:
परमाणु अप्रसार संधि, 1968 इस अर्थ में एक अस्त्र नियंत्रक संधि थी क्योंकि इसने परमाणविक हथियारों के उपार्जन को कायदे-कानूनों के दायरे में ला दिया। जिन देशों ने सन् 1967 से पहले परमाणु हथियार बना लिये थे उन्हें इस संधि के अन्तर्गत इस हथियारों को रखने की अनुमति दी गई। लेकिन अन्य देशों को ऐसे हथियारों को हासिल करने के अधिकार से वंचित किया गया।

प्रश्न 26.
शक्ति संतुलन को कैसे बनाए रखा जा सकता है?
उत्तर:
शक्ति सन्तुलन को बनाए रखने के अनेक साधन हैं।

  1. शक्ति संतुलन बनाए रखने के लिए सैन्य शक्ति को बढ़ाना एवं आर्थिक और प्रौद्योगिकी विकास महत्त्वपूर्ण हैं।
  2. राष्ट्रों द्वारा सैनिक या सुरक्षा संधियाँ कर गठबंधन कर शक्ति सन्तुलन बनाए रखा जा सकता है।
  3. ‘फूट डालो और राज करो’ की नीति अपना कर भी राष्ट्रों द्वारा शक्ति सन्तुलन बनाए रखा जा सकता है।
  4. कई बार एक राष्ट्र दूसरे राष्ट्र में हस्तक्षेप कर वहां अपनी मित्र सरकार बनाकर भी शक्ति सन्तुलन स्थापित करते हैं।
  5. शस्त्रीकरण और निःशस्त्रीकरण द्वारा भी शक्ति सन्तुलन बनाए रखा जा सकता है।

प्रश्न 27.
सुरक्षा का अर्थ स्पष्ट कीजिये।
उत्तर:
सुरक्षा का अर्थ- सुरक्षा का बुनियादी अर्थ है खतरे से आजादी मानव का अस्तित्व और किसी देश का जीवन खतरों से भरा होता है लेकिन इसका अभिप्राय यह नहीं कि हर तरह के खतरे को सुरक्षा पर खतरा माना जाये अतः सुरक्षा के अर्थ को दो दृष्टिकोणों से स्पष्ट किया जा सकता है।

  1. संकीर्ण दृष्टिकोण: इस दृष्टिकोण के अनुसार व्यक्तिगत मूल्यों की सुरक्षा अर्थात् समाज में प्रत्येक मनुष्य की अपनी सोच व मूल्य होते हैं। जब इन मूल्यों को बचाने का प्रयास किया जाता है तो यह सुरक्षा का संकीर्ण दृष्टिकोण कहलाता है।
  2. व्यापक दृष्टिकोण: इस दृष्टिकोण के अनुसार सुरक्षा का सम्बन्ध बड़े तथा गंभीर खतरों से है। इसमें वे खतरे सम्मिलित होते हैं जिन्हें रोकने के उपाय नहीं किये गये तो हमारे केन्द्रीय मूल्यों को अपूरणीय हानि पहुँचेगी।

प्रश्न 28.
अमेरिका और सोवियत संघ ने नियंत्रण से जुड़ी जिन संधियों पर हस्ताक्षर किये उन्हें संक्षेप में लिखिये।
उत्तर:
अमेरिका और सोवियत संघ ने अस्त्र – नियंत्रण की कई संधियों पर हस्ताक्षर किये जिसमें सामरिक अस्त्र परिसीमन संधि – 2 ( स्ट्रेटजिक आर्म्स लिमिटेशन ट्रीटी – SALT-II) और सामरिक अस्त्र न्यूनीकरण संधि ( स्ट्रेटजिक आर्म्स रिडक्शन ट्रीटी-(START) शामिल हैं। परमाणु अप्रसार संधि (न्यूक्लियर नॉन प्रोलिफेरेशन ट्रीटी – NPT (1968) भी एक अर्थ में अस्त्र नियंत्रण संधि ही थी क्योंकि इसने परमाण्विक हथियारों के उपार्जन को कायदे-कानून के दायरे में ला खड़ा किया। सन् 1972 की एंटी बैलेस्टिक मिसाइल संधि (ABM) ने अमेरिका और सोवियत संघ को बैलेस्टिक मिसाइलों को रक्षा कवच के रूप में प्रयोग करने से रोका।

प्रश्न 29.
सुरक्षा की दृष्टि से निरस्त्रीकरण के महत्त्व को बताइए।
उत्तर:
वर्तमान में विश्व शांति तथा सुरक्षा की दृष्टि से निरस्त्रीकरण का बहुत महत्त्व है। आज यह अनुभव किया गया है कि राष्ट्रों की मारक क्षमता को कम करने वाला निःशस्त्रीकरण तथा शस्त्र नियंत्रण न कि मारक क्षमता बढ़ाने वाले तथा आतंक संतुलन बनाने वाली शस्त्र दौड़, आज के युग में अधिक प्रभावशाली व लाभकारी शक्ति संतुलन का साधन है। एक व्यापक निःशस्त्रीकरण संधि, परमाणु निःशस्त्रीकरण तथा शस्त्र नियंत्रण, 1972 की जैविक हथियार संधि, 1992 की रासायनिक हथियार संधि तथा 181 देशों के CWC संधि पर हस्ताक्षर, इस संतुलन को सुदृढ़ करने के लिये अधिक सहायक हो सकते हैं।

प्रश्न 30.
सुरक्षा की पारंपरिक धारणा में विश्वास बहाली के उपायों को स्पष्ट कीजिये।
उत्तर:
सुरक्षा की पारंपरिक धारणा में यह बात भी मानी गई है कि विश्वास बहाली के उपायों से देशों के बीच हिंसाचार कम किया जा सकता है। विश्वास बहाली के उपाय अग्र हैं।
उत्तर:
युद्ध और शरणार्थी समस्या के बीच आपस में सकारात्मक सम्बन्ध है क्योंकि युद्ध या सशस्त्र संघर्षों के कारण ही शरणार्थी की समस्या बढ़ती है। उदाहरण के लिए सन् 1990 के दशक में कुल 60 जगहों से शरणार्थी प्रवास करने को मजबूर हुए और इनमें से तीन को छोड़कर शेष सभी के मूल में सशस्त्र संघर्ष था।

प्रश्न 25.
परमाणु अप्रसार संधि, 1968 की एक अस्त्र नियंत्रण संधि के रूप में व्याख्या कीजिये।
उत्तर:
परमाणु अप्रसार संधि, 1968 इस अर्थ में एक अस्त्र नियंत्रक संधि थी क्योंकि इसने परमाणविक हथियारों के उपार्जन को कायदे-कानूनों के दायरे में ला दिया। जिन देशों ने सन् 1967 से पहले परमाणु हथियार बना लिये थे उन्हें इस संधि के अन्तर्गत इस हथियारों को रखने की अनुमति दी गई। लेकिन अन्य देशों को ऐसे हथियारों को हासिल करने के अधिकार से वंचित किया गया।

JAC Class 12 Political Science Important Questions Chapter 7 समकालीन विश्व में सरक्षा

प्रश्न 26.
शक्ति संतुलन को कैसे बनाए रखा जा सकता है?
उत्तर:
शक्ति सन्तुलन को बनाए रखने के अनेक साधन हैं।

  1. शक्ति संतुलन बनाए रखने के लिए सैन्य शक्ति को बढ़ाना एवं आर्थिक और प्रौद्योगिकी विकास महत्त्वपूर्ण हैं।
  2. राष्ट्रों द्वारा सैनिक या सुरक्षा संधियाँ कर गठबंधन कर शक्ति सन्तुलन बनाए रखा जा सकता है।
  3. ‘फूट डालो और राज करो’ की नीति अपना कर भी राष्ट्रों द्वारा शक्ति सन्तुलन बनाए रखा जा सकता है।
  4. कई बार एक राष्ट्र दूसरे राष्ट्र में हस्तक्षेप कर वहां अपनी मित्र सरकार बनाकर भी शक्ति सन्तुलन स्थापित करते हैं।
  5. शस्त्रीकरण और निःशस्त्रीकरण द्वारा भी शक्ति सन्तुलन बनाए रखा जा सकता है।

प्रश्न 27.
सुरक्षा का अर्थ स्पष्ट कीजिये।
उत्तर:
सुरक्षा का अर्थ- सुरक्षा का बुनियादी अर्थ है खतरे से आजादी मानव का अस्तित्व और किसी देश का जीवन खतरों से भरा होता है लेकिन इसका अभिप्राय यह नहीं कि हर तरह के खतरे को सुरक्षा पर खतरा माना जाये अतः सुरक्षा के अर्थ को दो दृष्टिकोणों से स्पष्ट किया जा सकता है।

  1. संकीर्ण दृष्टिकोण: इस दृष्टिकोण के अनुसार व्यक्तिगत मूल्यों की सुरक्षा अर्थात् समाज में प्रत्येक मनुष्य की अपनी सोच व मूल्य होते हैं। जब इन मूल्यों को बचाने का प्रयास किया जाता है तो यह सुरक्षा का संकीर्ण दृष्टिकोण कहलाता है।
  2. व्यापक दृष्टिकोण: इस दृष्टिकोण के अनुसार सुरक्षा का सम्बन्ध बड़े तथा गंभीर खतरों से है। इसमें वे खतरे सम्मिलित होते हैं जिन्हें रोकने के उपाय नहीं किये गये तो हमारे केन्द्रीय मूल्यों को अपूरणीय हानि पहुँचेगी।

प्रश्न 28.
अमेरिका और सोवियत संघ ने नियंत्रण से जुड़ी जिन संधियों पर हस्ताक्षर किये उन्हें संक्षेप में लिखिये।
उत्तर;
अमेरिका और सोवियत संघ ने अस्त्र – नियंत्रण की कई संधियों पर हस्ताक्षर किये जिसमें सामरिक अस्त्र परिसीमन संधि – 2 ( स्ट्रेटजिक आर्म्स लिमिटेशन ट्रीटी – SALT-II) और सामरिक अस्त्र न्यूनीकरण संधि ( स्ट्रेटजिक आर्म्स रिडक्शन ट्रीटी-(START) शामिल हैं। परमाणु अप्रसार संधि (न्यूक्लियर नॉन प्रोलिफेरेशन ट्रीटी – NPT (1968) भी एक अर्थ में अस्त्र नियंत्रण संधि ही थी क्योंकि इसने परमाण्विक हथियारों के उपार्जन को कायदे-कानून के दायरे में ला खड़ा किया। सन् 1972 की एंटी बैलेस्टिक मिसाइल संधि (ABM) ने अमेरिका और सोवियत संघ को बैलेस्टिक मिसाइलों को रक्षा कवच के रूप में प्रयोग करने से रोका।

प्रश्न 29.
सुरक्षा की दृष्टि से निरस्त्रीकरण के महत्त्व को बताइए।
उत्तर:
वर्तमान में विश्व शांति तथा सुरक्षा की दृष्टि से निरस्त्रीकरण का बहुत महत्त्व है। आज यह अनुभव किया गया है कि राष्ट्रों की मारक क्षमता को कम करने वाला निःशस्त्रीकरण तथा शस्त्र नियंत्रण न कि मारक क्षमता बढ़ाने वाले तथा आतंक संतुलन बनाने वाली शस्त्र दौड़, आज के युग में अधिक प्रभावशाली व लाभकारी शक्ति संतुलन का साधन है। एक व्यापक निःशस्त्रीकरण संधि, परमाणु निःशस्त्रीकरण तथा शस्त्र नियंत्रण, 1972 की जैविक हथियार संधि, 1992 की रासायनिक हथियार संधि तथा 181 देशों के CWC संधि पर हस्ताक्षर, इस संतुलन को सुदृढ़ करने के लिये अधिक सहायक हो सकते हैं।

JAC Class 12 Political Science Important Questions Chapter 7 समकालीन विश्व में सरक्षा

प्रश्न 30.
सुरक्षा की पारंपरिक धारणा में विश्वास बहाली के उपायों को स्पष्ट कीजिये।
उत्तर:
सुरक्षा की पारंपरिक धारणा में यह बात भी मानी गई है कि विश्वास बहाली के उपायों से देशों के बीच हिंसाचार कम किया जा सकता है। विश्वास बहाली के उपाय अग्र हैं।

  1. विश्वास बहाली से दोनों देशों के बीच हिंसा को कम किया जा सकता है।
  2. विश्वास बहाली से दोनों देशों के बीच सूचनाओं तथा विचारों का आदान-प्रदान किया जाता है।
  3. ऐसे में दोनों देश एक-दूसरे को सैनिक साजो-सामान की जानकारी व अपने सैनिक मकसद के बारे में जानकारी देते हैं
  4. इस प्रक्रिया से दोनों देशों के बीच गलतफहमी से बचा जा सकता है।

प्रश्न 31.
आपकी दृष्टि में सुरक्षा की अपारंपरिक धारणा क्या है?
अथवा
सुरक्षा की अपारंपरिक धारणा क्या है? संक्षेप में लिखिये।
उत्तर:
अपारंपरिक धारणा का अर्थ- सुरक्षा की अपारंपरिक धारणा में न केवल सैन्य खतरों को बल्कि इसमें मानवीय अस्तित्व पर चोट करने वाले अन्य व्यापक खतरों और आशंकाओं को भी शामिल किया गया है। इसमें राज्य ही नहीं बल्कि व्यक्तियों और संप्रदायों या कहें कि संपूर्ण मानवता की सुरक्षा होती है।

प्रश्न 32.
परम्परागत सुरक्षा के किन्हीं चार तत्त्वों का उल्लेख कीजिये।
उत्तर:
परम्परागत सुरक्षा के चार तत्त्व निम्नलिखित हैं।

  1. परम्परागत खतरे: सुरक्षा की परम्परागत धारणा में सैन्य खतरों को किसी भी देश के लिए सर्वाधिक घातक माना जाता है। इसका स्रोत कोई अन्य देश होता है जो सैनिक हमले की धमकी देकर देश की स्वतंत्रता, संप्रभुता तथा अखण्डता को प्रभावित करता है।
  2.  युद्ध: युद्ध से साधारण लोगों के जीवन पर भी खतरा मंडराता है क्योंकि युद्ध में जन सामान्य को भी काफी नुकसान पहुँचता है।
  3.  शक्ति सन्तुलन: प्रत्येक सरकार दूसरे देशों से अपने शक्ति सन्तुलन को लेकर अत्यधिक संवेदनशील रहती है।
  4. गठबंधन: इसमें विभिन्न देश सैनिक हमले को रोकने अथवा उससे रक्षा करने के लिए मिलजुलकर कदम उठाते हैं।

प्रश्न 33.
आतंकवाद से आप क्या समझते हैं? उदाहरण सहित स्पष्ट कीजिये।
उत्तर:
आतंकवाद-आतंकवाद का आशय राजनीतिक खून-खराबे से है जो जानबूझकर बिना किसी मुरौव्वत के नागरिकों को अपना निशाना बनाता है। अन्तर्राष्ट्रीय आतंकवाद एक से ज्यादा देशों में व्याप्त आतंकवाद है और उसके निशाने पर कई देशों के नागरिक हैं। कोई राजनीतिक स्थिति पसंद न होने पर आतंकवादी समूह उसे बल-प्रयोग या बल-प्रयोग की धमकी देकर बदलना चाहते हैं। जनमानस को आतंकित करने के लिए नागरिकों को निशाना बनाया जाता है। आतंकवाद की चिर-परिचित तकनीकें हैं। विमान अपहरण, भीड़ भरी जगहों, जैसे रेलगाड़ी, होटल, बाजार, धर्म स्थल आदि जगहों पर बम लगाना सितम्बर सन् 2001 में आतंकवादियों ने अमरीका के वर्ल्ड ट्रेड सेंटर पर हमला बोला। इस घटना के बाद लगभग सभी देश आतंकवाद पर ज्यादा ध्यान देने लगे हैं।

प्रश्न 34.
मानव अधिकारों के हनन की स्थिति में क्या संयुक्त राष्ट्र संघ को हस्तक्षेप करना चाहिए?
उत्तर:
मानव अधिकारों की हनन की स्थिति में संयुक्त राष्ट्र संघ को हस्तक्षेप करना चाहिए या नहीं, इस सम्बन्ध में विवाद है।

  1. कुछ देशों का तर्क है कि राष्ट्र संघ का घोषणा पत्र अन्तर्राष्ट्रीय जगत् को अधिकार देता है कि वह मानवाधिकारों की रक्षा के लिए हथियार उठाये अर्थात् राष्ट्र संघ को इस क्षेत्र में दखल देना चाहिए।
  2. कुछ देशों का तर्क है यह संभव है कि मानवाधिकार हनन का मामला ताकतवर देशों के हितों से निर्धारित होता है और इसी आधार पर यह निर्धारित होता है कि संयुक्त राष्ट्र संघ मानवाधिकार उल्लंघन के लिए मामले में कार्रवाई करेगा और किसमें नहीं? इससे ताकतवर देशों को मानवाधिकार के बहाने उसके अंदरूनी मामलों में दखल देने का आसान रास्ता मिल जायेगा।

प्रश्न 35.
सुरक्षा की पारंपरिक अवधारणा में सैन्य खतरे को किसी भी देश के लिए खतरनाक क्यों माना जाता है?
उत्तर:
सुरक्षा की पारंपरिक अवधारणा में सैन्य खतरे को किसी भी देश के लिए खतरनाक माना जाता है क्योंकि इस खतरे का स्रोत कोई दूसरा मुल्क होता है जो सैन्य हमले की धमकी देकर संप्रभुता, स्वतंत्रता और क्षेत्रीय अखंडता जैसे किसी देश के केन्द्रीय मूल्यों के लिए खतरा पैदा करता है। सैन्य कार्रवाई से आम नागरिकों के जीवन को भी खतरा होता है। युद्ध में सिर्फ सैनिक ही घायल नहीं होते हैं अपितु आम नागरिकों को भी हानि उठानी पड़ती है। अक्सर निहत्थे और आम नागरिकों को जंग का निशाना बनाया जाता है; उनका और उनकी सरकार का हौंसला तोड़ने की कोशिश होती है।

प्रश्न 36.
हर सरकार दूसरे देश से अपने शक्ति संतुलन को लेकर बहुत संवेदनशील रहती है। इस कथन को स्पष्ट कीजिए।
उत्तर:
शक्ति – संतुलन परंपरागत सुरक्षा नीति का एक तत्त्व है। हर देश के पड़ोस में छोटे या बड़े मुल्क होते हैं इससे भविष्य के खतरे का अंदाजा लगाया जा सकता है। उदाहरण के लिए कोई पड़ोसी देश संभवतः यह जाहिर ना करे कि वह हमले की तैयारी कर रहा है अथवा हमले का कोई प्रकट कारण भी ना हो। तथापि यह देखकर कि कोई देश बहुत ताकतवर है यह अंदाजा लगाया जा सकता है कि भविष्य में वह हमलवार हो सकता है। इस वजह से हर सरकार दूसरे देश से अपने शक्ति संतुलन को लेकर बहुत संवेदनशील रहती है।

प्रश्न 37.
सरकारें दूसरे देशों से शक्ति-संतुलन का पलड़ा अपने पक्ष में बैठाने हेतु किस प्रकार की कोशिशें करती हैं? यथा-
उत्तर:
सरकारें दूसरे देशों से शक्ति-संतुलन का पलड़ा अपने पक्ष में बैठाने हेतु जी-तोड़ कोशिशें करती हैं।

  1. वो नजदीक देश जिनके साथ किसी मुद्दे पर मतभेद हो या अतीत में युद्ध हो चुका हो उनके साथ शक्ति संतुलन को अपने पक्ष में करने के लिए अपनी सैन्य शक्ति बढ़ाने का प्रयत्न किया जाता है।
  2. सैन्य शक्ति के साथ आर्थिक और प्रौद्योगिकी की ताकत को बढ़ाने पर भी जोर दिया जाता है क्योंकि सैन्य- शक्ति का यही आधार है।

प्रश्न 38.
गठबंधन बनाना पारंपरिक सुरक्षा नीति का चौथा तत्त्व है। संक्षेप में व्याख्या कीजिए।
उत्तर:
पारंपरिक सुरक्षा नीति का चौथा तत्त्व है गठबंधन बनाना गठबंधन में कई देश शामिल होते हैं जो सैन्य हमले को रोकने अथवा उससे रक्षा करने के लिए समवेत कदम उठाते हैं। गठबंधन लिखित रूप में होते हैं उनको औपचारिक रूप मिलता है और ऐसे गठबंधनों को यह बात स्पष्ट रहती है कि उन्हें खतरा किस देश से है। किसी देश अथवा गठबंधन की तुलना में अपनी ताकत बढ़ाने के लिए देश गठबंधन बनाते हैं। गठबंधन राष्ट्रीय हितों पर आधारित होते हैं। राष्ट्रीय हितों के बदलने के साथ ही गठबंधन भी बदल जाते हैं।

JAC Class 12 Political Science Important Questions Chapter 7 समकालीन विश्व में सरक्षा

प्रश्न 39.
संयुक्त राष्ट्रसंघ विश्व: राजनीति में ऐसी केन्द्रीय सत्ता है जो सर्वोपरि है। यह सोचना बस एक लालचमात्र है। इस कथन को स्पष्ट कीजिए।
उत्तर:
संयुक्त राष्ट्रसंघ विश्व:
राजनीति में ऐसी केन्द्रीय सत्ता है जो सर्वोपरि है यह सोचना बस एक लालचमात्र है क्योंकि अपनी बनावट के अनुरूप संयुक्त राष्ट्रसंघ अपने सदस्य देशों का दास है ओर इसके सदस्य दशों का दास है ओर इसके सदस्य देश जितनी सत्ता इसको सौंपते और स्वीकारते हैं उतनी ही सत्ता इसे हासिल होती है। अतः विश्व- राजनीति में हर देश को अपनी सुरक्षा ही सत्ता इसे हासलि होती है। अतः विश्व – राजनीति में हर देश को अपनी सुरक्षा की जिम्मेदारी खुद उठानी होती है।

प्रश्न 40.
एशिया और अफ्रीका के नव स्वतंत्र देशों के सामने खड़ी सुरक्षा की चुनौतियाँ यूरोपीय देशों के मुकाबले किन दो मायनों में विशिष्ट थीं?
उत्तर:
एशिया और अफ्रीका के नव स्वतंत्र देशों के सामने खड़ी सुरक्षा की चुनौतियाँ यूरोपीय देशों के मुकाबले निम्न दो मायनों में विशिष्ट थीं।

  1. इन देशों को अपने पड़ोसी देश से सैन्य हमले की आशंका थी।
  2. इन्हें अंदरूनी सैन्य संघर्ष की भी चिंता करनी थी।

प्रश्न 41.
नव-स्वतंत्र देशों के सामने पड़ोसी देशों से युद्ध और आंतरिक संघर्ष की सबसे बड़ी चुनौती थे। स्पष्ट कीजिए।
उत्तर:
नव-स्वतंत्र देशों के सामने सीमापार से खतरे के साथ ही पड़ोसी देशों से भी खतरा था। साथ ही भीतर से भी खतरे की आशंका थी अनेक नव-स्वतंत्र देश संयुक्त राज्य अमरीका या सोवियत संघ अथवा औपनिवेशिक ताकतों से कहीं ज्यादा अपने पड़ोसी देशों से आशंकित थे। इनके बीच सीमा रेखा और भूक्षेत्र अथवा आबादी पर नियंत्रण को लेकर या एक-एक करके सभी सवालों पर झगड़े हुए।

अलग राष्ट्र बनाने पर तुले अंदर के अलगावादी आंदोलनों से भी इन देशों को खतरा था। कोई पड़ोसी देश यदि ऐसे अलगाववादी आंदोलन को हवा दे अथवा उसकी सहायता करे तो दो पड़ोसी देशों के बीच तनाव की स्थिति बन जाती थी । इस प्रकार पड़ोसी देशों से युद्ध और आंतरिक संघर्ष नवस्वतंत्र देशों के सामने सुरक्षा की सबसे बड़ी चुनौती थे।

प्रश्न 42.
‘न्याय-युद्ध’ की यूरोपीय परंपरा को स्पष्ट कीजिए।
उत्तर:
सुरक्षा की परंपरागत धारणा में यह माना गया है कि जितना हो सके हिंसा का इस्तेमाल सीमित होना चाहिए। युद्ध के लक्ष्य और दोनों से इसका संबंध है। न्याय-युद्ध की यूरोपीय परम्परा को आज पूरा विश्व मानता है। इस परंपरा के अनुसार किसी भी देश को युद्ध उचित कारणों अर्थात् आत्मरक्षा अथवा दूसरों को जनसंहार से बचाने के लिए ही करना चाहिए। इस दृष्टिकोण का मानना है कि।

  1. किसी भी देश को युद्ध में युद्ध साधनों का सीमित इस्तेमाल करना चाहिए।
  2. युद्धरत सेना को संघर्षविमुख शत्रु, निहत्थे व्यक्ति अथवा आत्मसमर्पण करने वाले शत्रु को मारना नहीं चाहिए।
  3. सेना को उतने ही बल का प्रयोग करना चाहिए जितना आत्मरक्षा के लिए आवश्यक हो और हिंसा का सहारा एक सीमा तक लेना चाहिए। बल प्रयोग तभी किया जाये जब बाकी के उपाय असफल हो गए हों।

प्रश्न 43.
भारत ने परमाणु परीक्षण करने के फैसले को अंतर्राष्ट्रीय पटल पर सत्यापित कैसे किया?
उत्तर:
भारतीय सुरक्षा नीति का पहला घटक सैन्य शक्ति को मजबूत करना है क्योंकि भारत पर पड़ोसी देशों से हमले होते रहे हैं। पाकिस्तान ने तीन तथा चीन ने भारत पर एक बार हमला किया है। दक्षिण एशियाई इलाके में भारत के चारों तरफ परमाणु हथियारों से लैस देश है। ऐसे में भारतीय सरकार ने परमाणु परीक्षण करने के भारत के फैसले को उचित ठहराते हुए राष्ट्रीय सुरक्षा का तर्क दिया था। भारत ने सन् 1974 में पहला तथा 1998 में दूसरा परमाणु परीक्षण किया था।

प्रश्न 44.
अप्रवासी और शरणार्थी में अंतर स्पष्ट कीजिए।
उत्तर:
अप्रवासी उन्हें कहा जाता है जो अपनी इच्छा से स्वदेश छोड़ते हैं और शरणार्थी हम उन्हें कहते हैं जो युद्ध. प्राकृतिक आपदा अथवा राजनीतिक उत्पीड़न के कारण स्वदेश छोड़ने पर मजबूर होते हैं

प्रश्न 45.
1990 के दशक में विश्व सुरक्षा की धारणा उभरने की क्या वजहें हैं? उदाहरण सहित स्पष्ट कीजिए।
उत्तर:
विश्वव्यापी खतरे जैसे वैश्विक तापवृद्धि, अंतर्राष्ट्रीय आतंकवाद तथा एड्स और बर्ड फ्लू जैसी महामारियों को ध्यान में रखते हुए 1990 के दशक में विश्व सुरक्षा की धारणा उभरी। क्योंकि कोई भी देश इन समस्याओं का समाधान अकेले नहीं कर सकता। ऐसा भी हो सकता है कि किन्हीं स्थितियों में किसी एक देश को इन समस्याओं की मार बाकियों की अपेक्षा ज्यादा झेलनी पड़े।

JAC Class 12 Political Science Important Questions Chapter 7 समकालीन विश्व में सरक्षा

प्रश्न 46.
भारत को अपनी परिस्थिति के अनुसार परम्परागत या अपरम्परागत सुरक्षा, किसे वरीयता देनी चाहिए?
उत्तर:
भारत को दोनों प्रकार की सुरक्षा को वरीयता देनी चाहिए।

  1. परम्परागत सुरक्षा के कारण : स्वतंत्रता के बाद भारत ने अनेक युद्ध लड़े तथा भारत के अनेक आंतरिक भाग में अलगाववादी गतिविधियाँ व्याप्त हैं।
  2. अपरम्परागत सुरक्षा के कारण: भारत एक विकासशील देश है और इसके साथ इसमें गरीबी, बेकारी, साम्प्रदायिकता, सामाजिक और आर्थिक पिछड़ापन भी व्याप्त है।

प्रश्न 47.
वैश्विक गरीबी असुरक्षा का स्रोत है। स्पष्ट कीजिए।
उत्तर:
वैश्विक गरीबी निर्धनता असुरक्षा का स्रोत है। वैश्विक गरीबी का आशय है आर्थिक विकास में कमी, राष्ट्रीय आय में कमी और यह विकासशील या विकसित देशों के जीवनस्तर को प्रभावित करती हैं। दुनिया की आधी आबादी का विकास सिर्फ 6 देशों में होता है भारत, चीन, पाकिस्तान, नाइजीरिया, बांग्लादेश और इंडोनेशिया, जिन्हें विकासशील देश माना जाता है और अनुमान है कि अगले 50 सालों में दुनिया के गरीब देशों में जनसंख्या तीन गुना बढ़ेगी।

विश्व स्तर पर यह विषमता दुनिया के उत्तरी और दक्षिणी गोलार्द्ध के देशों के बीच की खाई में योगदान करती है। दक्षिणी गोलार्द्ध के देशों में मौजूद गरीबी के कारण अधिकाधिक लोग बेहतर जीवन की तलाश में उत्तरी गोलार्द्ध के देशों में प्रवास कर रहे हैं। उपर्युक्त कारणों ने अंतर्राष्ट्रीय राजनीतिक घर्षण पैदा किया क्योंकि अंतर्राष्ट्रीय कानून अप्रवासी और शरणार्थी में भेद करते हैं।

निबन्धात्मक प्रश्न

प्रश्न 1.
सुरक्षा की पारम्परिक धारणा की विवेचना कीजिये।
उत्तर:
सुरक्षा की पारम्परिक धारणा: सुरक्षा की पारम्परिक धारणा को दो भागों में विभाजित किया गया है।
(अ) बाहरी सुरक्षा की धारणा और
(ब) आन्तरिक सुरक्षा की धारणा। यथा।
(अ) बाहरी सुरक्षा की धारणा: सैन्य खतरा-
सुरक्षा की पारंपरिक अवधारणा में सैन्य खतरे को किसी देश के लिए सबसे ज्यादा खतरनाक माना जाता है। इस खतरे का स्रोत कोई दूसरा देश होता है जो सैन्य हमले की धमकी देकर संप्रभुता,  तंत्रता और क्षेत्रीय अखंडता तथा जन-धन की हानि का खतरा पैदा करता है। सैन्य खतरे अर्थात् युद्ध से बचने के उपाय- सरकार के पास सैन्य खतरे से बचने के प्रमुख उपाय होते हैं।

  1. आत्मसमर्पण करना
  2. अपरोध की नीति अपनाना
  3. रक्षा नीति अपनाना
  4. शक्ति सन्तुलन की स्थापना करना तथा
  5. गठबन्धन बनाने की नीति अपनाना।

(ब) आंतरिक सुरक्षा की धारणा:
दूसरे विश्व युद्ध के बाद से ‘सुरक्षा के आंतरिक पक्ष पर दुनिया के अधिकांश ताकतवर देश अपनी अंदरूनी सुरक्षा के प्रति कमोबेश आश्वस्त थे। लेकिन एशिया और अफ्रीका के नव-स्वतंत्र देशों को बाह्य सुरक्षा के साथ-साथ अन्दरूनी सैन्य संघर्ष की भी समस्याओं का सामना करना पड़ रहा था क्योंकि ये देश सीमा पार से अपने पड़ौसी देशों से सैन्य हमले की आशंका से ग्रस्त थे तो दूसरी तरफ अलग राष्ट्र बनाने पर तुले अन्दर के अलगाववादी आंदोलनों से भी इन देशों को खतरा था।

प्रश्न 2.
सुरक्षा के पारंपरिक तरीके कौन-कौन से हैं? उनमें से प्रत्येक की संक्षिप्त व्याख्या कीजिए।
अथवा
सुरक्षा की पारंपरिक अवधारणा को स्पष्ट कीजिये तथा सुरक्षा के पारम्परिक तरीकों का वर्णन कीजिये।
उत्तर:
सुरक्षा की पारंपरिक अवधारणा- सुरक्षा की पारम्परिक अवधारणा में निम्न तरीकों पर बल दिया गया है।

  • न्याय युद्ध की परम्परा का विस्तार- सुरक्षा की पारंपरिक अवधारणा ‘न्याय युद्ध’ की यूरोपीय परम्परा का विस्तार है। इसकी प्रमुख बातें ये हैं-
    1. किसी देश को युद्ध आत्म-रक्षा अथवा दूसरों से जन-संहार से बचाने के लिए ही करना चाहिए।
    2. साधनों का सीमित प्रयोग करना चाहिए।
    3. निहत्थे व्यक्ति या आत्मसमर्पण वाले शत्रु को नहीं मारना चाहिए तथा
    4. बल प्रयोग तभी किया जाये जब अन्य उपाय असफल हो गये हों।
  • निरस्त्रीकरण: देशों के बीच सहयोग में सुरक्षा का सबसे महत्त्वपूर्ण तरीका है। निरस्त्रीकरण। इसमें कुछ खास किस्म के हथियारों का त्याग करने पर बल दिया जाता है।
  • अस्त्र – नियंत्रण – अस्त्र- नियंत्रण के अन्तर्गत हथियारों को विकसित करने अथवा उनको प्राप्त करने के सम्बन्ध में कुछ कायदे-कानूनों का पालन करना पड़ता है। सन् 1972 की ‘एंटी बैलेस्टिक मिसाइल संधि’, ‘साल्ट-2’ तथा ‘परमाणु अप्रसार संधि 1968 ‘ इसके प्रमुख उदाहरण हैं।
  •  विश्वास बहाली का उपाय: विश्वास बहाली की प्रक्रिया यह सुनिश्चित करती है कि प्रतिद्वन्द्वी देश किसी गलतफहमी या गफलत में पड़कर जंग के लिए आमादा न हो जाएँ। विश्वास बहाली की प्रक्रिया के अन्तर्गत सैन्य टकराव और प्रतिद्वन्द्विता वाले देश एक-दूसरे को अपने फौजी मकसद, अपनी सैन्य योजनाओं, सैन्य बलों के स्वरूप तथा उनके तैनाती के स्थानों आदि प्रकार की सूचनाओं और विचारों का नियमित आदान-प्रदान करने का फैसला करते हैं।

JAC Class 12 Political Science Important Questions Chapter 7 समकालीन विश्व में सरक्षा

प्रश्न 3.
सुरक्षा की अपारंपरिक धारणा की विवेचना कीजिए।
उत्तर:
सुरक्षा की अपारंपरिक धारणा सुरक्षा की अपारंपरिक धारणा सिर्फ सैन्य खतरों से ही संबद्ध नहीं है, बल्कि इसमें मानवीय अस्तित्व पर चोट करने वाले व्यापक खतरों और आशंकाओं को शामिल किया जाता है। इसके दो प्रमुख पक्ष हैं।
1. मानवता की सुरक्षा तथा
2. विश्व सुरक्षा। यथा।

1. मानवता की सुरक्षा:
मानवता की सुरक्षा की धारणा व्यक्तियों की रक्षा पर बल देती है। मानवता की रक्षा का विचार जन – सुरक्षा को राज्यों की सुरक्षा से बढ़कर मानता है। मानवता की सुरक्षा और राज्य की सुरक्षा परस्पर पूरक होने चाहिए लेकिन व्यक्तियों की रक्षा किनसे की जाय, इस सम्बन्ध में तीन प्रकार के दृष्टिकोण सामने आये हैं। यथा।

(अ) संकीर्ण अर्थ: इस दृष्टिकोण के पैरोकारों का जोर व्यक्तियों और समुदायों को अंदरूनी खून-खराबे से बचाना है।

(ब) व्यापक अर्थ: व्यापक अर्थ लेने वाले समर्थक विद्वानों का तर्क है कि खतरों की सूची में हिंसक खतरों के साथ-साथ अकाल, महामारी और आपदाओं को भी शामिल किया जाये ।

(स) व्यापकतम अर्थ: व्यापकतम अर्थ में युद्ध, जनसंहार, आतंकवाद, अकाल, महामारी, प्राकृतिक आपदा से सुरक्षा के साथ-साथ ‘अभाव से मुक्ति’ और ‘भय से मुक्ति’ पर बल दिया गया है।

2. विश्व – सुरक्षा:
सुरक्षा की अपारंपरिक धारणा का दूसरा पक्ष है। विश्व सुरक्षा विश्वव्यापी खतरे, वैश्विक ताप वृद्धि (ग्लोबल वार्मिंग), अन्तर्राष्ट्रीय आतंकवाद, एड्स तथा बर्ड फ्लू जैसी समस्याओं की प्रकृति वैश्विक है, इसलिए अन्तर्राष्ट्रीय सहयोग अत्यन्त महत्त्वपूर्ण हो जाता है।

प्रश्न 4.
सुरक्षा की अपारंपरिक धारणा में सुरक्षा के प्रमुख खतरों पर एक निबंध लिखिये।
उत्तर:
सुरक्षा की अपारंपरिक धारणा में खतरे के नये स्त्रोत: सुरक्षा की अपारंपरिक धारणा के संदर्भ में खतरों की बदलती प्रकृति पर जोर दिया जाता है। ऐसे खतरों के प्रमुख नये स्त्रोत निम्नलिखित हैं।

  1. अन्तर्राष्ट्रीय आतंकवाद: जब आतंकवाद का कोई संगठन एक से अधिक देशों में व्याप्त हो जाता है, तो उसे अन्तर्राष्ट्रीय आतंकवाद कहते हैं। अन्तर्राष्ट्रीय आतंकवाद के निशाने पर कई देशों के नागरिक हैं। आतंकवाद की चिर-परिचित तकनीकें हैं। विमान अपहरण करना, भीड़-भरी जगहों में बम लगाना।
  2. मानवाधिकारों का हनन: मानवता की सुरक्षा का एक नया स्रोत राष्ट्रीय सरकारों द्वारा मानवाधिकारों का हनन है।
  3. वैश्विक निर्धनता: मानवता की सुरक्षा के लिए वैश्विक गरीबी एक बड़ा खतरा है।
  4. र्थिक असमानता: विश्व स्तर पर आर्थिक असमानता पूरे विश्व को उत्तरी गोलार्द्ध व दक्षिणी गोलार्द्ध में विभाजित करती है । दक्षिणी गोलार्द्ध में यह आर्थिक असमानता और अधिक व्याप्त है।
  5. आप्रवासी, शरणार्थी और आंतरिक रूप से विस्थापित लोगों की समस्या – आप्रवासी, शरणार्थी तथा आन्तरिक रूप से विस्थापित लोगों की समस्याएँ भी सुरक्षा की अपारम्परिक धारणा के अन्तर्गत आती हैं।
  6. महामारियाँ: एड्स, बर्ड- ड-फ्लू, सार्स जैसी महामारियों के फैलाव को रोकने में किसी एक देश की सफलता अथवा असफलता का प्रभाव दूसरे देशों में होने वाले संक्रमण पर पड़ता है।

प्रश्न 5.
सुरक्षा पर मंडराते अनेक अपारंपरिक खतरों से निपटने के लिए क्या किया जाना आवश्यक है?
उत्तर:
सुरक्षा के अपारंपरिक खतरों से निपटने के उपाय : सहयोगात्मक सुरक्षा सुरक्षा पर मंडराते अनेक अपारंपरिक खतरों, जैसे- अन्तर्राष्ट्रीय आतंकवाद, वैश्विक ताप वृद्धि, वैश्विक गरीबी, वैश्विक असमानता, महामारियाँ, मानवाधिकारों के हनन तथा शरणार्थी समस्या आदि-से निपटने के लिए अन्तर्राष्ट्रीय सहयोग की रणनीतियाँ बनाने की आवश्यकता है। इन्हें निम्नलिखित बिन्दुओं के अन्तर्गत स्पष्ट किया गया है।

  1. राज्यस्तरीय द्विपक्षीय, क्षेत्रीय, महादेशीय और वैश्विक सहयोग: इन अपारंपरिक खतरों से निपटने के लिए विभिन्न देश द्विपक्षीय, क्षेत्रीय, महादेशीय या वैश्विक स्तर पर सहयोग की रणनीति बना सकते हैं।
  2. अन्तर्राष्ट्रीय संस्थाओं द्वारा सुरक्षात्मक रणनीतियाँ: सहयोगमूलक सुरक्षा में विभिन्न देशों के अतिरिक्त अन्तर्राष्ट्रीय स्तर की संस्थाएँ, जैसे संयुक्त राष्ट्र संघ व उसकी विभिन्न एजेन्सियाँ, अन्तर्राष्ट्रीय स्वयंसेवी संगठन, बहुराष्ट्रीय व्यावसायिक संगठन और निगम तथा जानी-मानी हस्तियाँ शामिल हो सकती हैं।
  3. बल-प्रयोग-सहयोगमूलक सुरक्षा में भी अंतिम उपाय के रूप में बल-प्रयोग किया जा सकता है। अन्तर्राष्ट्रीय बिरादरी उन सरकारों से निपटने के लिए बल-प्रयोग की अनुमति दे सकती है जो अपनी ही जनता को मार रही हो अथवा उसके दुःख-दर्द की उपेक्षा कर रही हो। लेकिन बल-प्रयोग सामूहिक स्वीकृति से और सामूहिक रूप में किया जाए।

JAC Class 12 Political Science Important Questions Chapter 7 समकालीन विश्व में सरक्षा

प्रश्न 6.
भारत की ‘सुरक्षा रणनीति’ के विभिन्न घटकों का उल्लेख कीजिये।
अथवा
भारत की सुरक्षा नीति के प्रमुख घटकों की विवेचना कीजिये।
उत्तर:
भारत की सुरक्षा नीति के प्रमुख घटक: भारत की सुरक्षा नीति के चार बड़े घटक हैं और अलग-अलग समयों में इन्हीं घटकों के हेर-फेर से सुरक्षा की रणनीति बनायी गई है। यथा

  1. सैन्य क्षमता को मजबूत करना: भारत की सुरक्षा नीति का पहला घटक है। सैन्य क्षमता को मजबूत करना क्योंकि भारत पर पड़ौसी देशों के सैन्य – आक्रमण होते रहे हैं। भारत ने परमाणु परीक्षण के औचित्य में भी राष्ट्रीय सुरक्षा का तर्क दिया है।
  2. अन्तर्राष्ट्रीय कायदों और संस्थाओं को मजबूत करना: भारत की सुरक्षा नीति का दूसरा घटक है- अपने हितों को बचाने के लिए अन्तर्राष्ट्रीय कायदों और संस्थाओं को मजबूत करना। इस हेतु भारत ने एशियाई एकता, उपनिवेशीकरण का विरोध, निरस्त्रीकरण, नव अन्तर्राष्ट्रीय अर्थव्यवस्था तथा संयुक्त राष्ट्र संघ के समर्थ की नीतियाँ अपनाई हैं।
  3. देश की आंतरिक सुरक्षा: समस्याओं से निपटना: भारत सरकार ने देश की आंतरिक सुरक्षा की समस्याओं से निबटने के लिए लोकतांत्रिक राजनीतिक व्यवस्था का पालन किया है।
  4. गरीबी और असमानता को दूर करने के प्रयास: भारत सरकार ने बहुसंख्यक नागरिकों को गरीबी और अभाव से निजात दिलाने के निरन्तर प्रयास किये हैं ताकि नागरिकों के बीच आर्थिक असमानता ज्यादा न हो।

प्रश्न 7.
शक्ति सन्तुलन को बनाए रखने वाले साधनों का विवेचन कीजिये।
उत्तर;
शक्ति सन्तुलन को बनाए रखने वाले साधन: शक्ति सन्तुलन को बनाए रखने वाले प्रमुख साधन निम्नलिखित हैं।

  1. मुआवजा या क्षतिपूर्ति: साधारणतया इसका अर्थ उस देश की भूमि को बाँटने या समामेलन से लिया जाता है जो शक्ति सन्तुलन के लिए खतरा होती है।
  2. शस्त्रीकरण तथा निःशस्त्रीकरण: प्रत्येक राष्ट्र अपने पक्ष में शक्ति सन्तुलन बनाए रखने के लिए शस्त्रीकरण पर अधिक जोर देता है। वर्तमान में शस्त्रीकरण के साथ-साथ निःशस्त्रीकरण एवं शस्त्र – नियंत्रण को भी महत्त्व दिया जाने लगा है।
  3. गठबंधन: एक गठबंधन समझौते के बाद विरोधी राष्ट्रों के समूह के बीच भी एक प्रति गठबंधन समझौता होता है। इसीलिए इसे गठबंधनों और प्रतिगठबंधनों का नाम दिया जाता है।
  4. हस्तक्षेप: कई बार कोई बड़ा देश किसी छोटे देश के आन्तरिक मामलों में हस्तक्षेप करके वहाँ पर अपनी मित्र – सरकार स्थापित कर देता है।
  5. फूट डालो और राज करो: ‘फूट डालो और राज करो’ की नीति को भी शत्रु को कमजोर करने का एक बड़ा महत्त्वपूर्ण शक्ति संतुलन का साधन माना जाता है।
  6. बफर राज्य-शक्ति प्राप्त करके और इसे बनाए रखने का एक अन्य तरीका है। ऐसे तटस्थ (बफर) राज्य की स्थापना करना जो कमजोर हो और दो बड़े प्रतिद्वन्द्वी देशों के बीच में स्थित हो।
  7. सन्तुलनधारी राज्य: संतुलनधारी राज्य वह देश होता है जो दूसरे देशों की प्रतिद्वन्द्विता से दूर रहता है और एक-दूसरे के प्रतिद्वन्द्वी देश उसकी सहायता पाने की इच्छा रखते हैं। ऐसा देश प्राय: शक्ति संतुलन हेतु कमजोर राष्ट्र का साथ देता है।

प्रश्न 8.
निरस्त्रीकरण से आप क्या समझते हैं? आधुनिक युग में इसकी आवश्यकता को स्पष्ट करते हुए इसके मार्ग की बाधाओं का उल्लेख कीजिये।
उत्तर:
निःशस्त्रीकरण का अर्थ: निःशस्त्रीकरण से हथियारों की सीमा निश्चित करने या उन पर नियंत्रण करने या उन्हें कम करने का विचार प्रकट होता है। इसका लक्ष्य उपस्थित हथियारों के प्रभाव व संख्या को घटा देना है। मॉर्गेन्थो के शब्दों में, “नि:शस्त्रीकरण कुछ या सब शस्त्रों में कटौती या उनको समाप्त करना है ताकि शस्त्रीकरण की दौड़ का अन्त हो।” निःशस्त्रीकरण की आवश्यकता निःशस्त्रीकरण की आवश्यकता या महत्त्व को निम्न प्रकार स्पष्ट किया गया है।

  1. विश्व शांति और सुरक्षा की स्थापना की दृष्टि से निःशस्त्रीकरण आवश्यक है।
  2. इससे विश्व के राष्ट्र अपने धन को आर्थिक विकास के कार्यों में लगा सकते हैं।
  3. निःशस्त्रीकरण को अपनाने पर उपनिवेशवाद व साम्राज्यवाद का अन्त होगा।
  4. विदेशी हस्तक्षेप को रोकने के लिए आवश्यक है कि सभी देश मिलकर निःशस्त्रीकरण पर बल दें।
  5. बढ़ते हुए सैनिकीकरण को रोकने के लिए निःशस्त्रीकरण बहुत आवश्यक है।
  6. नि:शस्त्रीकरण सैनिक गठबन्धनों को रोकता एवं समाप्त करता है।
  7. परमाणु युद्ध के बचाव के लिए भी निःशस्त्रीकरण आवश्यक है।

निःशस्त्रीकरण के मार्ग की बाधाएँ: निःशस्त्रीकरण के मार्ग में आने वाली प्रमुख बाधायें निम्नलिखित हैं।

  1. विश्व व्यवस्था राष्ट्रों में परस्पर अविश्वास का होना।
  2. प्रत्येक राष्ट्र द्वारा राष्ट्रीय हित को सर्वोपरि महत्त्व देना।
  3. विश्व में प्रत्येक राष्ट्र की राष्ट्रीय सुरक्षा के प्रति बढ़ती आशंका।
  4. वर्चस्व स्थापित करने की भावना।
  5. नि:शस्त्रीकरण से बड़े देशों की कंपनियों के हथियारों के व्यापार को संकट का सामना करना पड़ता है।

JAC Class 12 Political Science Important Questions Chapter 7 समकालीन विश्व में सरक्षा

प्रश्न 9.
शरणार्थी से आप क्या समझते हैं? शरणार्थी समस्या का प्रमुख कारण क्या है? संयुक्त राष्ट्र संघ के तत्वावधान में शरणार्थियों की सुरक्षा व उनके अधिकारों की रक्षा के क्या प्रयास किये गये हैं?
उत्तर:
शरणार्थी से आशय:
शरणार्थी वे व्यक्ति हैं जो युद्ध, प्राकृतिक आपदा अथवा राजनीतिक उत्पीड़न के कारण अपने देश को छोड़ने पर मजबूर होते हैं और दूसरे देश में पलायन कर जाते हैं। शरणार्थी समस्या का प्रमुख कारण – शरणार्थी समस्या का प्रमुख कारण सशस्त्र संघर्ष और युद्ध है। दक्षिणी गोलार्द्ध के देशों में सशस्त्र संघर्ष और युद्ध के कारण लाखों लोग शरणार्थी बने। शरणार्थी सुरक्षा एजेन्सियाँ – शरणार्थियों की सुरक्षा व उनके अधिकारों की रक्षा के लिए संयुक्त राष्ट्र संघ के तत्वावधान में निम्नलिखित अभिकरण कार्य कर रहे हैं।

  1. संयुक्त राष्ट्र शरणार्थी उच्चायुक्त: संयुक्त राष्ट्र शरणार्थी उच्चायुक्त ने आंतरिक रूप से विस्थापित व्यक्तियों की. सुरक्षा और सहायता का समग्र उत्तरदायित्व ग्रहण किया है। सशस्त्र संघर्ष की स्थिति में यह नागरिकों को सुरक्षित मार्ग से सुरक्षित ठिकानों पर पहुँचाता है।
  2. अन्तर्राष्ट्रीय रेडक्रास समिति: अन्तर्राष्ट्रीय रेडक्रास समिति जो खतरे की स्थिति में नागरिकों को निकालना, बंदियों की रिहाई, संरक्षित क्षेत्र बनाना, युद्ध विराम के लिए व्यवस्था करना आदि कार्य करती है।
  3. संयुक्त राष्ट्र विकास कार्यक्रम, विश्व खाद्य कार्यक्रम, विश्व स्वास्थ्य संगठन भी महिलाओं और बच्चों की सहायता करते हैं।
  4. डॉक्टर्स विदआऊट वार्ड्स और वर्ल्ड काउंसिल ऑफ चर्च्स भी देशीय विस्थापितों की सहायता करते हैं।

प्रश्न 10.
विश्वास बहाली की प्रक्रिया द्वारा यह सुनिश्चित किया जा सकता है कि प्रतिद्वन्द्वी देश किसी गलतफहमी या गफलत में पड़कर जंग के लिए आमादा न हो जाए इस कथन पर टिप्पणी लिखिए।
उत्तर:
सुरक्षा की पारंपरिक धारणा में यह बात स्वीकार की गई है विश्वास बहाली के माध्यम से देशों के बीच हिंसाचार को कम किया जा सकता है। विश्वास बहाली की प्रक्रिया: विश्वास बहाली की प्रक्रिया में सैन्य टकराव और प्रतिद्वन्द्विता वाले देश सूचनाओं तथा विचारों के नियमित आदान-प्रदान का फैसला करते हैं। दो देश एक-दूसरे को अपने फौजी मकसद तथा एक हद तक अपनी सैन्य योजनाओं के बारे में बताते हैं। इस प्रकार ये देश अपने प्रतिद्वन्द्वी देश को इस बात का भरोसा दिलाते हैं कि उनकी तरफ से किसी भी प्रकार से हमले की योजना नहीं बनायी जा रही है।

एक-दूसरे को यह भी बताते हैं कि उनके पास किस प्रकार के सैन्य बल हैं तथा इन बलों को कहाँ तैनात किया जा रहा है। इस प्रकार संक्षेप में कहें तो विश्वास बहाली की प्रक्रिया यह सुनिश्चित करती है कि प्रतिद्वन्द्वी देश किसी गलतफहमी या गफलत में पड़कर जंग के लिए आमादा न हो जाएँ। सुरक्षा की परंपरागत धारणा मुख्य रूप से सैन्य बल के प्रयोग अथवा सैन्य बलके प्रयोग की आशंका से संबंध है।

प्रश्न 11.
सुरक्षा की दृष्टि से खतरे के नए स्रोत पर टिप्पणी लिखिए।
उत्तर:
खतरे के नए स्रोत: सुरक्षा की अपारंपरिक धारणा के दो पक्ष हैं मानवता की सुरक्षा और विश्व सुरक्षा ये दोनों सुरक्षा के संदर्भ में खतरों की बदलती प्रकृति पर जोर देते हैं।
1.आतंकवाद:
आतंकवाद का आशय राजनीतिक खून-खराबे से है जो जानबूझकर और बिना किसी मुरौव्वत के नागरिकों को निशाना बनाता है। अंतर्राष्ट्रीय आतंकवाद कई देशों में व्याप्त है तथा कई देशों के निर्दोष नागरिक इसके निशाने पर है। कोई राजनीतिक संदर्भ या स्थिति नापसंद हो तो आतंकवादी समूह उसे बल प्रयोग अथवा बल-प्रयोग की धमकी देकर बदलने का प्रयत्न करते हैं। जनमानस को आतंकित करने के लिए नागरिकों को निशाना बनाया जाता है और आतंकवाद नागरिकों के असंतोष का इस्तेमाल राष्ट्रीय सरकारों अथवा संघर्षों में शामिल अन्य पक्ष के खिलाफ करता है। विमान-अपहरण अथवा भीड़ भरी जगहों जैसे रेलगाड़ी, होटल, बाजार या ऐसी ही जगहों पर बम लगाना आदि आतंकवाद के उदाहरण हैं।

2. मानवाधिकार:
मानवाधिकार को तीन कोटियों में रखा गया है। पहली कोटि राजनीतिक अधिकारों की है जैसे अभिव्यक्ति और सभा करने की आजादी । दूसरी कोटि आर्थिक और सामाजिक अधिकारों की है। अधिकारों की तीसरी कोटि में उपनिवेशीकृत जनता अथवा जातीय और मूलवासी अल्पसंख्यकों के अधिकार आते हैं। इन वर्गीकरण को लेकर सहमति तो हैं लेकिन इस बात पर सहमति नहीं बन पायी है कि इनमें से किस कोटि के अधिकारों को सार्वभौम मानवाधिकारों की संज्ञा दी जाए या इन अधिकारों के उल्लंघन की स्थिति में अंतर्राष्ट्रीय बिरादरी को क्या करना चाहिए?

3. वैश्विक निर्धनता:
खतरे का एक ओर स्रोत वैश्विक निर्धनता है। विश्व की जनसंख्या फिलहाल 760 करोड़ है और यह आँकड़ा 21वीं सदी के मध्य तक 1000 करोड़ हो जाएगी। अनुमान है कि अगले 50 सालों में दुनिया के सबसे गरीब देशों में जनसंख्या तीन गुना बढ़ेगी जबकि इसी अवधि में अनेक धनी देशों की जनसंख्या घटेगी। प्रति व्यक्ति उच्च आय और जनसंख्या की कम वृद्धि के कारण धनी देश अथवा सामाजिक समूहों को और धनी बनाने में मदद मिलती है जबकि प्रति व्यक्ति निम्न आय और जनसंख्या की तीव्र वृद्धि एक साथ मिलकर गरीब देशों और सामाजिक समूहों को और गरीब बनाते हैं।

JAC Class 12 Political Science Important Questions Chapter 7 समकालीन विश्व में सरक्षा

प्रश्न 12.
विश्व स्तर पर असमानता से किस प्रकार के खतरे उत्पन्न होते हैं? स्पष्ट कीजिए।
उत्तर:
विश्वस्तर पर अमीरी-गरीबी की यह असमानता उत्तरी गोलार्द्ध के देशों को दक्षिणी गोलार्द्ध के देशों से अलग करती है। दक्षिण गोलार्द्ध के देशों में असमानता अच्छी-खासी बढ़ी है। यहाँ कुछ देशों ने आबादी की रफ्तार को काबू कर आय को बढ़ाने में सफलता प्राप्त की है परंतु बाकी के देश ऐसा नहीं कर पाए हैं। उदाहरण के लिए दुनिया में सबसे ज्यादा सशस्त्र संघर्ष अफ्रीका के सहारा मरुस्थल के दक्षिणवर्ती देशों में होते हैं। यह इलाका दुनिया का सबसे गरीब इलाका है। 21वीं सदी के शुरुआती समय में इस इलाके के युद्धों में शेष दुनिया की तुलना में कहीं ज्यादा लोग मारे गए। दक्षिणी गोलार्द्ध के देशों में मौजूद गरीबी के कारण अधिकाधिक लोग बेहतर जीवन खासकर आर्थिक अवसरों की तलाश में उत्तरी गोलार्द्ध के देशों में प्रवास कर रहे हैं।

इससे अंतर्राष्ट्रीय स्तर पर राजनीतिक मतभेद उठ खड़ा हुआ है। अंतर्राष्ट्रीय कायदे कानून अप्रवासी और शरणार्थी में भेद करते हैं। विश्व का शरणार्थी – मानचित्र विश्व के संघर्ष – मानचित्र से लगभग हू-ब-हू मिलता है क्योंकि दक्षिणी गोलार्द्ध के देशों में सशस्त्र संघर्ष और युद्ध के कारण लाखों लोग शरणार्थी बने और सुरक्षित जगह की तलाश में निकले हैं। 1990 से 1995 के बीच 70 देशों के मध्य कुल 93 युद्ध हुए और इसमें लगभग साढ़े 55 लाख लोग मारे गए। इसके परिणामस्वरूप व्यक्ति, परिवार और कभी-कभी पूरे समुदाय को सर्वव्याप्त भय अथवा आजीविका, पहचान और जीवन- यापन के परिवेश के नाश के कारण जन्मभूमि छोड़ने पर मजबूर होना पड़ा।

प्रश्न 13.
महामारियों के फैलाव से विश्व में किस प्रकार के खतरे उत्पन्न होते हैं ? निबंध लिखिए।
उत्तर:
एचआईवी-एड्स, बर्ड फ्लू और सार्स (सिवियर एक्यूट रेसपिटॅरी सिंड्रोम – SARS) जैसी महामारियाँ अप्रवास, व्यवसाय, पर्यटन और सैन्य अभियानों के जरिए बड़ी तेजी से विभिन्न देशों में फैली हैं। इन बीमारियों के फैलाव को रोकने में किसी एक देश की सफलता अथवा असफलता का प्रभाव दूसरे देशों में होने वाले संक्रमण पर पड़ता है। एक अनुमान है कि 2003 तक पुरी दुनिया में 4 करोड़ लोग एचआईवी-एड्स से संक्रमित हो चुके थे।

इसमें दो- तिहाई लोग अफ्रीका में रहते हैं जबकि शेष के 50 फीसदी दक्षिण एशिया में। उत्तरी अमरीका तथा दूसरे औद्योगिक देशों में उपचार की नई विधियों के कारण 1990 के दशक के उत्तरार्ध के वर्षों में एचआईवी एड्स से होने वाली मृत्यु की दर में तेजी से कमी आयी है परंतु अफ्रीका जैसे गरीब इलाके के लिए ये उपचार कीमत को देखते हुए आकाश- कुसुम कहे जाएँगे जबकि अफ्रीका को ज्यादा गरीब बनाने में एचआईवी-एड्स महत्त्वपूर्ण घटक साबित हुआ है।

एबोला वायरस, हैन्टावायरस और हेपेटाइटिस – सी जैसी कुछ नयी महामारियाँ उभरी हैं जिनके बारे में खास जानकारी उपलब्ध नहीं हैं। टीबी, मलेरिया, डेंगू बुखार और हैजा जैसी पुरानी महामारियों ने औषधि प्रतिरोधक रूप धारण कर लिया है और इससे इनका उपचार कठिन हो गया है। जानवरों में महामारी फैलने से भारी आर्थिक दुष्प्रभाव होते हैं 1990 के दशक के उत्तरार्द्ध के सालों से ब्रिटेन ने ‘मेड-काऊ’ महामारी के फैलने के कारण अरबों डॉलर का नुकसान उठाया है और बर्ड फ्लू के कारण कई दक्षिण एशियाई देशों को मुर्ग-निर्यात बंद करना पड़ा। ऐसी महामारियाँ बताती हैं कि देशों के बीच पारस्परिक निर्भरता बढ़ रही हैं और राष्ट्रीय सीमाएँ पहले की तुलना में कम सार्थक रह गई हैं। इन महामारियों का संकेत है कि अंतर्राष्ट्रीय सहयोग को बढ़ाने की जरूरत है।

प्रश्न 14.
सुरक्षा की अपारंपरिक धारणा भी सुरक्षा की पारंपरिक धारणा समान स्थानीय संदर्भों के अनुकूल परिवर्तनशील है। व्याख्या कीजिए।
उत्तर:
सुरक्षा की धारणा में विस्तार करने का यह मतलब नहीं होता कि हम हर तरह के कष्ट या बीमारी को सुरक्षा विषयक चर्चा के दायरे में शामिल कर सकते हैं। ऐसा करने पर सुरक्षा की धारणा में संगति नहीं रह जाती। ऐसे में हर चीज सुरक्षा का मसला हो सकती है। इसी वजह से किसी मसले को सुरक्षा का मसला कहलाने के लिए एक सर्व स्वीकृत न्यूनतम मानक पर खरा उतरना जरूरी है। उदाहरण के लिए यदि किसी मसले से संदर्भों के अस्तित्व को खतरा हो जाए तो उसे सुरक्षा का मसला कहा जा सकता है चाहे इस खतरे की प्रकृति कुछ भी हो।

उदाहरण के लिए मालदीव को वैश्विक तापवृद्धि से खतरा हो सकता है क्योंकि समुद्रतल के ऊँचा उठने से इसका ज्यादातर हिस्सा डूब जाएगा जबकि दक्षिणी अफ्रीकी देशों में एचआईवी-एड्स से गंभीर खतरा है क्योंकि यहाँ हर 6 वयस्क व्यक्ति में 1 इस रोग से पीड़ित है। 1994 की खांडा की तुन्सी जनजाति के अस्तित्व पर खतरा मंडराया क्योंकि प्रतिद्वन्द्वी हुतु जनजाति ने कुछ हफ्तों में लगभग 5 लाख तुन्सी लोगों को मार डाला। इससे पता चलता है कि सुरक्षा की अपारंपरिक धारणा भी सुरक्षा की पारंपरिक धारणा के समान संदर्भों के अनुकूल परिवर्तनशील है।

JAC Class 12 Political Science Important Questions Chapter 7 समकालीन विश्व में सरक्षा

प्रश्न 15.
लोकतंत्र सिर्फ राजनीतिक आदर्श नहीं अपितु लोकतांत्रिक शासन जनता को ज्यादा सुरक्षा मुहैया कराने का साधन भी है। इस कथन पर प्रकाश डालिए।
उत्तर:
भारतीय सरकार ने भारत की अर्थव्यवस्था को इस तरह विकसित करने का प्रयास किया है जिससे बहुसंख्यक नागरिकों को गरीबी और अभाव से निजात मिले और नागरिकों के बीच आर्थिक असमानता ज्यादा न हो। हालांकि ये प्रयास ज्यादा सफल नहीं हो पाए हैं। हमारा देश अभी भी गरीब है और आर्थिक असमानता व्यापक रूप से व्याप्त है। फिर भी, लोकतांत्रिक राजनीति से ऐसे अवसर नागरिकों को उपलब्ध हो जाते हैं जिसके द्वारा गरीब और वंचित नागरिक अपनी आवाज उठा सके और अपने हक के लिए सरकार से माँग कर सकें। लोकतांत्रिक सरकार लोकतंत्र की रीति से निर्वाचित होती है जिसके कारण सरकार के ऊपर दबाव होता है कि वह आर्थिक संवृद्धि को मानवीय विकास का सहगामी बनाए। इस प्रकार हम कह सकते हैं कि लोकतंत्र सिर्फ राजनीतिक आदर्श नहीं है; लोकतांत्रिक शासन जनता को ज्यादा सुरक्षा मुहैया कराने का साधन भी है।

प्रश्न 16.
पारंपरिक और अपारंपरिक धारणा में अंतर स्पष्ट कीजिए।
उत्तर:

पारंपरिक धारणा अपारंपरिक धारणा
(1) पारंपरिक सुरक्षा सैन्य उपयोग के खतरे से संबंधित है। (1) अपारंपरिक सुरक्षा सैन्य खतरों से इतर है। इसमें वो शामिल हैं जो मानव अस्तित्व को खतरे में डालते हैं।
(2) इस सुरक्षा के लिए पारंपरिक खतरे की संप्रभुता, स्वतंत्रता और क्षेत्रीय अखंडता के मुख्य मूल्यों को खतरे में डालते हैं। (2) गैर पारंपरिक सुरक्षा राज्य की तुलना में मानव को खतरे में डालती है।
(3) पारंपरिक अवधारणा के तहत सैन्य बल पर जोर दिया जाता है। (3) अपारंपरिक सुरक्षा में सैन्य बल का उपयोग अंतिम उपाय के रूप में किया जाता है।
(4) सुरक्षा की पारंपरिक धारणा में माना जाता है कि किसी देश की सुरक्षा को ज्यादातर खतरा उसकी सीमा के बाहर से होता है। (4) अपारंपरिक धारणा में खतरा सामान्य वातावरण है।

JAC Class 11 Geography Important Questions Chapter 6 मृदा

Jharkhand Board JAC Class 11 Geography Important Questions Chapter 6 मृदा Important Questions and Answers.

JAC Board Class 11 Geography Important Questions Chapter 6 मृदा

बहु-विकल्पी प्रश्न(Multiple Choice Questions)

प्रश्न – दिए गए चार वैकल्पिक उत्तरों में से सही उत्तर चुनिए-
1. महाराष्ट्र राज्य में किस मृदा का अधिक विस्तार है?
(A) काली मृदा
(B) लाल मृदा
(C) जलोढ़ मृदा
(D) लेटराइट मृदा।
उत्तर:
(A) काली मृदा।

2. रेगड़ मृदा किसे कहते हैं?
(A) लाल मृदा
(B) लेटराइट मृदा
(C) काली मृदा
(D) जलोढ़ मृदा
उत्तर:
(C) काली मृदा।

3. मृदा निर्माण का प्रमुख साधन है।
(A) नदी
(B) वायु
(C) जनक सामग्री
(D) लहरें
उत्तर:
(C) जनक सामग्री।

4. मृदा की किस परत में जैव पदार्थ अधिक होते हैं?
(A) ‘क’ परत
(B) ‘ख’ परत
(C) ‘ग’ परत
(D) चट्टान
उत्तर:
(A) ‘क’ परत।

5. भारत में प्रथम मृदा सर्वेक्षण कब हुआ?
(A) 1936 में
(B) 1946 में
(C) 1956 में
(D) 1966 में।
उत्तर:
(C) 1956 में।

JAC Class 11 Geography Important Questions Chapter 6 मृदा

6. जलोढ़ मृदा का देश के क्षेत्रफल के कितने प्रतिशत पर विस्तार है?
(A) 30%
(B) 35%
(C) 40%
(D) 45%.
उत्तर:
(C) 40%.

7. नए जलोढ़क मृदा को कहते हैं
(A) तराई
(B) भाबर
(C) खादर
(D) बांगर
उत्तर;
(C) खादर।

8. ‘स्वयं जुताई मृदा’ किसे कहते हैं?
(A) जलोढ़
(B) काला
(C) लेटराइट
(D) लाल।
उत्तर:
(B) काली।

9. लेटराइट शब्द का अर्थ है।
(A) चीका
(B) ईंट
(C) तलछट
(D) पोटाश
उत्तर:
(B) ईंट

10. लवण मृदाएं अधिकतर किस राज्य में हैं?
(A) पंजाब
(B) महाराष्ट्र
(C) गुजरात
(D) केरल।
उत्तर:
(A) पंजाब।

11. किस घाटी में बीहड़ अधिक है?
(A) गंगा घाटी
(B) चम्बल घाटी
(C) उष्ण घाटी
(D) नर्मदा घाटी
उत्तर:
(B) चम्बल घाटी।

JAC Class 11 Geography Important Questions Chapter 6 मृदा

12. शिवालिक में मृदा अपरदन का मुख्य कारण है।
(A) वर्षा
(B) अति-चराई
(C) वनोन्मूलन
(D) नदी बांध|
उत्तर:
(C) वनोन्मूलन।

13. कृषि के लिए पहाड़ी क्षेत्र की ढलान कितनी है?
(A) 5-10% से कम
(B) 10–15% से कम
(C) 15-20% से कम
(D) 20-25% से कम
उत्तर:
(D) 20–25% से कम।

14. शुष्क प्रदेशों में बालू के टीलों का विस्तार कैसे रोका जा सकता है?
(A) वृक्षों की रक्षक मेखला
(B) वृक्षों की कटाई
(C) जल सिंचाई
(D) नदी बांध।
उत्तर:
(A) वृक्षों की रक्षक मेखला।

अति लघु उत्तरीय प्रश्न (Very Short Answer Type Questions)

प्रश्न 1.
मृदा महत्त्वपूर्ण क्यों है?
उत्तर:
यह सभी जीवित वस्तुओं के लिये भोजन उत्पादन करती है।

प्रश्न 2.
मिट्टी में पाये जाने वाले मुख्य आवश्यक तत्त्व लिखो।
उत्तर:
सिलीका, चीका तथा ह्यूमस।

प्रश्न 3.
मिट्टी में चीका का क्या कार्य है?
उत्तर:
यह जल को सोख लेती है।

प्रश्न 4.
मृदा की तीन परतों के नाम लिखो।
उत्तर:
A स्तर, B स्तर, C स्तर

प्रश्न 5.
मृदा की परिभाषा दें।
उत्तर:
यह असंगठित पदार्थों की पतली परत होती है।

JAC Class 11 Geography Important Questions Chapter 6 मृदा

प्रश्न 6.
उन तत्त्वों के नाम बतायें जिन पर मृदा की बनावट निर्भर करती है?
उत्तर:
मूल पदार्थ, धरातल, जलवायु

प्रश्न 7.
भारत में मृदा के तीन व्यापक प्रादेशिक भागों के नाम लिखें।
उत्तर:

  1. प्रायद्वीप की मिट्टियां।
  2. त्तरी मैदान की मिट्टियां।
  3.  हिमालय की मिट्टियां।

प्रश्न 8.
बनावट के आधार पर मृदा की तीन किस्में लिखो।
उत्तर:

  1. रेतीली मिट्टियां
  2. चीका मिट्टी
  3. दोमट मिट्टी

प्रश्न 9.
भारत में पाई जाने वाली अधिकतर व्यापक मिट्टी का नाम लिखो।
उत्तर:
जलोढ़ मिट्टी

प्रश्न 10.
उत्तरी भारत में पाई जाने वाली जलोढ़ मिट्टी की दो मुख्य किस्में लिखो।
उत्तर:
खाद्दर तथा बांगर मिट्टी।

प्रश्न 11.
भारतीय मैदानों में विशाल क्षेत्रों में पाई जाने वाली मिट्टी का नाम लिखो।
उत्तर:
जलोढ़ मिट्टी।

JAC Class 11 Geography Important Questions Chapter 6 मृदा

प्रश्न 12.
खाद्दर तथा बांगर मिट्टी के दो स्थानीय नाम लिखें।
उत्तर:
खाद्दर मिट्टी – बैठ, बांगर मिट्टी – धाया।

प्रश्न 13.
तीन क्षेत्रों के नाम बतायें जहां पर खाद्दर मिट्टी पाई जाती है।
उत्तर:
सतलुज बेसिन, गंगा का मैदान, यमुना बेसिन।

प्रश्न 14.
जलोढ़ मिट्टी में उत्पादित होने वाले दो खाद्य पदार्थों के नाम लिखो।
उत्तर:
गेहूँ, चावल।

प्रश्न 15.
पश्चिम बंगाल के जलोढ़ मिट्टी सबसे अधिक किस फसल के लिये उपयोगी है?
उत्तर:
पटसन के लिये।

प्रश्न 16.
दक्कन पठार के छोर पर कौन-सी मिट्टी मिलती है?
उत्तर:
लाल मिट्टी।

प्रश्न 17.
कौन-सी मृदा प्रायद्वीपीय भारत में पाई जाती है?
उत्तर:
लाल मिट्टी

JAC Class 11 Geography Important Questions Chapter 6 मृदा

प्रश्न 18.
उन दो राज्यों के नाम बतायें जहां पर लाल मिट्टी अधिकतर पाई जाती है।
उत्तर:
तमिलनाडु तथा छत्तीसगढ़।

प्रश्न 19.
उन तीन रंगों के नाम बतायें जिनसे लाल मिट्टी का निर्माण होता है।
उत्तर:
भूरा, चाकलेट तथा पीला।

प्रश्न 20.
रेगड़ मिट्टी का रंग बताओ।
उत्तर:
काला।

प्रश्न 21.
उन दो राज्यों के नाम लिखो जहां पर काली मिट्टी पाई जाती है?
उत्तर:
महाराष्ट्र तथा मध्य प्रदेश

प्रश्न 22.
काली मिट्टी के दो अन्य नाम लिखो।
उत्तर:
कपास मिट्टी तथा रेगड़ मिट्टी

प्रश्न 23.
काली मिट्टी का निर्माण कैसे होता है?
उत्तर:
दक्कन ट्रैप के ज्वालामुखी चट्टानों के लावा द्वारा।

प्रश्न 24.
एक फसल का नाम लिखें जिसके लिये काली मिट्टी बहुत उपयोगी है।
उत्तर:
कपास।

प्रश्न 25.
क्रिस प्रकार के जलवायु में लेटराइट मिट्टी का निर्माण होता है?
उत्तर:
उष्ण कटिबन्धीय मानसून जलवायु।

प्रश्न 26.
लेटराइट मिट्टी की दो किस्में लिखो।
उत्तर:
उच्च मैदान लेटराइट मिट्टी तथा निम्न मैदान लेटराइट मिट्टी

JAC Class 11 Geography Important Questions Chapter 6 मृदा

प्रश्न 27.
उन तीन राज्यों के नाम लिखो जहां पर लेटराइट मिट्टी पाई जाती है।
उत्तर:
आंध्र प्रदेश, तमिलनाडु तथा उड़ीसा

प्रश्न 28.
लेटराइट मिट्टी किस फसल के लिए सबसे अधिक उपयोगी है?
उत्तर:
बागानी फसल लगाने के लिये।

प्रश्न 29.
मरुस्थलीय मिट्टी किस प्रदेश में पाई जाती है?
उत्तर:
शुष्क मरुस्थल।

प्रश्न 30.
भारत के किस क्षेत्र में मरुस्थलीय मिट्टी पाई जाती है?
उत्तर:
थार मरुस्थल ( राजस्थान तथा सिंध )।

प्रश्न 31.
मरुस्थलीय मिट्टी में उपज का क्या कारण है?
उत्तर:
सिंचाई

प्रश्न 32.
रेतीले मरुस्थल में पाई जाने वाली मिट्टी के दो प्रकार लिखो।
उत्तर:
तेज़ाबी तथा नमकीन।

प्रश्न 33.
उस क्षेत्र का नाम बतायें जहां पर्वतीय मिट्टी पाई जाती है।
उत्तर:
हिमालय पर्वत।

प्रश्न 34.
पर्वतीय मिट्टी किस फसल के लिये उपयोगी है?
उत्तर:
चाय।

JAC Class 11 Geography Important Questions Chapter 6 मृदा

प्रश्न 35.
पर्वतीय मिट्टी चाय के लिये उपयोगी क्यों है?
उत्तर:
पर्वतीय मिट्टियां तेज़ाबी होती हैं जो चाय को विशेष स्वाद देती हैं।

प्रश्न 36.
भारत के किस भाग में मृदा अपरदन के कारण अस्त-व्यस्त धरातल का निर्माण होता है?
उत्तर:
चम्बल घाटी।

प्रश्न 37.
परत अपरदन का क्या कार्य है?
उत्तर:
पहाड़ी ढलानों पर कृषि

प्रश्न 38.
चो से क्या अभिप्राय है?
उत्तर:
चो शिवालिक की तलछटी में मौसमी नालों को कहा जाता है

प्रश्न 39.
जलोढ़ मृदाएँ कहां पाई जाती हैं? ये देश के कुल क्षेत्रफल के कितने भाग पर विस्तृत हैं?
उत्तर:
जलोढ़ मृदाएँ उत्तरी मैदान और नदी घाटियों के विस्तृत भागों में पाई जाती हैं। ये मृदाएँ देश के कुल क्षेत्रफल के लगभग 40 प्रतिशत भाग पर विस्तृत हैं।

प्रश्न 40.
भारत में जलोढ़ मृदाओं के क्षेत्र बताइए।
उत्तर:
भारत में जलोढ़ मृदाएँ राजस्थान के एक संकीर्ण गलियारे से होती हुई गुजरात के मैदान तक फैली हुई हैं।

प्रश्न 41.
भारत में काली मृदाएँ कहां पाई जाती हैं?
उत्तर:
भारत में काली मृदाएँ दक्कन के पठार के अधिकतर भाग पर पाई जाती हैं। इसमें महाराष्ट्र के कुछ भाग, गुजरात, आंध्र प्रदेश तथा तमिलनाडु के कुछ भाग शामिल हैं।

JAC Class 11 Geography Important Questions Chapter 6 मृदा

प्रश्न 42.
काली मृदाओं में मिलने वाले रासायनिक तत्त्व बताइये।
उत्तर:
रासायनिक दृष्टि से काली मृदाओं में चूने, लौह, मैग्नीशिया और ऐलुमिना के तत्त्व अधिक मात्रा में पाए जाते हैं।

प्रश्न 43.
भारत में लाल और काली मृदाएँ कहां पाई जाती हैं?
उत्तर:
भारत में पश्चिमी घाट के गिरिपद क्षेत्र की एक लम्बी पट्टी में लाल दुमट मृदा पाई जाती है। पीली और लाल मृदाएँ उड़ीसा तथा छत्तीसगढ़ के कुछ भागों तथा मध्य गंगा के मैदान के दक्षिणी भागों में पाई जाती हैं।

प्रश्न 44.
लैटेराइट का अर्थ बताइये लैटेराइट मृदाएँ किन क्षेत्रों में विकसित होती हैं?
उत्तर:
लैटेराइट लैटिन शब्द ‘लेटर’ से बना है जिसका शाब्दिक अर्थ ईंट होता है। लैटेराइट मृदाएँ उच्च तापमान और भारी वर्षा के क्षेत्रों में विकसित होती हैं।

प्रश्न 45.
शुष्क मृदाओं में नमी और ह्यूमस की मात्रा कम क्यों होती है?
उत्तर:
शुष्क जलवायु, उच्च तापमान और भी तीव्र गति से होने वाले वाष्पीकरण के फलस्वरूप शुष्क मृदाओं में नमी और ह्यूमस कम होती है।

प्रश्न 46.
लेटेराइट मिट्टी का सबसे अधिक उपयोग किस काम में होता है?
उत्तर:
भवन निर्माण।

लघु उत्तरीय प्रश्न (Short Answer Type Questions)

प्रश्न 1.
मृदा क्या है? इसका निर्माण कैसे होता है?
उत्तर:
भू-पृष्ठ (Crust) पर मिलने वाले असंगठित पदार्थों की ऊपरी पर्त को मृदा कहते हैं। इस पर्त की मोटाई कुछ सें० मी० से लेकर कई मीटर तक हो सकती है। इसमें कई तत्त्व जैसे मिट्टी के बारीक कण, ह्यूमस, खनिज तथा जीवाणु मिले होते हैं। इन तत्त्वों के कारण इसमें कई परतें होती हैं। मृदा का निर्माण आधार चट्टानों के मूल पदार्थों तथा वनस्पति के सहयोग से होता है। किसी प्रदेश में यान्त्रिक तथा रासायनिक ऋतु अपक्षय द्वारा मूल चट्टानों के टूटने पर मिट्टी का निर्माण आरम्भ होता है । इसमें वनस्पति के गले – सड़े अंश के मिलने से कई भौतिक तथा रासायनिक कारकों के सहयोग से मृदा का पूर्ण विकास होता है। इस प्रकार मृदा की परिभाषा है, “Soil is the end product of the physical, chemical, biological and cultural factors which act and react together.”

प्रश्न 2.
मृदा जनन की प्रक्रिया क्या है?
उत्तर:
मृदा जनन एक जटिल प्रक्रिया है जिसके द्वारा विशेष प्राकृतिक परिस्थितियों में मृदा का निर्माण होता है। वातावरण के भौतिक, रासायनिक तथा जैविका तत्त्वों के योग से इस प्रक्रिया द्वारा मृदा का निर्माण होता है। विभिन्न प्रकार की जलवायु चट्टानों, जीव-जन्तुओं तथा वनस्पति के क्षेत्र से प्राप्त पदार्थों के इकट्ठा होने से मृदा का निर्माण होता है

JAC Class 11 Geography Important Questions Chapter 6 मृदा

प्रश्न 3.
मृदा का मूल पदार्थ क्या है?
उत्तर:
आधार चट्टानों के रासायनिक तथा यान्त्रिक अपक्षरण से प्राप्त होने वाले पदार्थों को मृदा का मूल पदार्थ कहा जाता है। इन सभी पदार्थों से मृदा का निर्माण होता है। मृदा का रंग, उपजाऊपन आदि मूल पदार्थों पर निर्भर करता है। लावा चट्टानों से बनने वाली मृदा का रंग काला होता है।

प्रश्न 4.
पर्यावरण के छः तत्त्वों के नाम बताइए जो मृदा जनन की प्रक्रिया में महत्त्वपूर्ण भूमिका निभाते हैं।
उत्तर:
मृदा जनन एक जटिल प्रक्रिया है जिसके द्वारा विशेष प्राकृतिक परिस्थितियों में मृदा का निर्माण होता है। वातावरण के भौतिक, रासायनिक तथा जैविका तत्त्वों के योग से इस प्रक्रिया द्वारा मृदा का निर्माण होता है। विभिन्न प्रकार की जलवायु, चट्टानों, जीव-जन्तुओं तथा वनस्पति के क्षेत्र से प्राप्त पदार्थों के इकट्ठा होने से मृदा का निर्माण होता है। धरातल तथा भूमि की ढाल भी मृदा जनन पर प्रभाव डालते हैं।

प्रश्न 5.
जलोढ़ मृदा की विशेषताएं क्या हैं?
उत्तर:
जलोढ़ मृदा की विशेषताएं-

  1. इसका निक्षेपण मुख्यतः नदी द्वारा होता है।
  2. इसका विस्तार नदी बेसिन तथा मैदानों तक सीमित होता है।
  3. यह अत्यधिक उपजाऊ मृदा होती है।
  4. इसमें बारीक कणों वाली मृदा पाई जाती है।
  5. इसमें काफ़ी मात्रा में पोटाश पाया जाता है, परन्तु फॉस्फोरस की कमी होती है।
  6. यह मृदा बहुत गहरी होती है।

प्रश्न 6.
भारत में उपलब्ध मिट्टी के प्रकारों में प्रादेशिक विभिन्नता के क्या कारण है?
उत्तर:
भारत की मिट्टियों में पाई जाने वाली प्रादेशिक विभिन्नता निम्नलिखित घटकों पर निर्भर करती है

  1. शैल – समूह
  2. उच्चावच के धरातलीय लक्षण
  3. ढाल का रूप
  4. जलवायु तथा प्राकृतिक वनस्पति
  5. पशु तथा कीड़े-मकौड़े।

प्रश्न 7.
पठारों तथा मैदानों की मिट्टी में क्या अन्तर है?
उत्तर:
पठारों तथा मैदानों की मिट्टी में मुख्य अन्तर मूल पदार्थों में पाया जाता है। पठारों में आधार चट्टानें कठोर होती हैं। इसकी मिट्टी में मूल पदार्थों की प्राप्ति इन चट्टानों से होती है। यह मिट्टी मोटे कणों वाली तथा कम उपजाऊ होती है। मैदानों में मिट्टी का निर्माण नदियों के निक्षेपण कार्य से होता है। मैदानों में प्रायः जलोढ़ मिट्टी मिलती है। नदी में प्रत्येक बाढ़ के कारण महीन सिल्ट (Silt) तथा मृतिका (Clay) का निक्षेप होता रहता है।

प्रश्न 8.
चट्टानों में जंग लगने से कौन-सी मिट्टी का निर्माण होता है ? भारत में इस मिट्टी के तीन प्रमुख लक्षण बताओ।
उत्तर:
इस क्रिया से लाल मिट्टी का निर्माण होता है।

(i) विस्तार (Extent ):
भारत की सब मिट्टियों में से लाल मिट्टी विस्तार सबसे अधिक है। यह मिट्टी लगभग 8 लाख वर्ग किलोमीटर क्षेत्र में पाई जाती है। दक्षिणी में तमिलनाडु, कर्नाटक, आन्ध्र प्रदेश, छोटा नागपुर पठार तथा प्रायद्वीप पठार के बाहरी भागों में लाल मिट्टी का विस्तार मिलता है।

(ii) विशेषताएं (Characteristics):
इस मिट्टी का निर्माण प्रायद्वीप की आधारभूत आग्नेय चट्टानों, ग्रेनाइट तथा नीस चट्टानों की टूट-फूट से हुआ है। इस मिट्टी का रंग लौहयुक्त चट्टानों में ऑक्सीकरण (oxidation) की क्रिया से लाल हो जाता है। वर्षा के कारण ह्यूमस नष्ट हो जाता है तथा आयरन ऑक्साइड ऊपरी सतह पर आ जाता है। इस मिट्टी में लोहा और एल्यूमीनियम की अधिकता होती है, परन्तु नाइट्रोजन और फॉस्फोरस की कमी होती है। यह मिट्टी कम गहरी तथा कम उपजाऊ होती है। इस मिट्टी में ज्वार, बाजरा, कपास, दालें, तम्बाकू की कृषि होती है।

JAC Class 11 Geography Important Questions Chapter 6 मृदा

प्रश्न 9.
समोच्च रेखा बन्धन किसे कहते हैं? मृदा को विनाश से बचाने के लिए इसका किस प्रकार प्रयोग कर सकते हैं?
उत्तर:
समोच्च रेखा बन्धन (Contour Bunding) – पर्वतीय ढलानों पर सम ऊंचाई की रेखा के साथ-साथ सीढ़ीदार खेत बनाए जाते हैं ताकि मिट्टी के कटाव को रोका जा सके। ऐसे प्रदेश में खड़ी ढाल वाले खेतों में समान ऊंचाई की रेखा के साथ बांध या ढाल बनाई जाती है। ऐसे बांध को समोच्च रेखा बन्धन कहते हैं। इससे वर्षा के जल को रोक कर मृदा अपरदन से बचाया जा सकता है। इससे वर्षा के जल को नियन्त्रित करके बहते हुए पानी द्वारा मृदा अपरदन को कम किया जा सकता है।

प्रश्न 10.
मृदा की उर्वरा शक्ति को विकसित करने के लिए कौन-कौन से उपाय करने चाहिएं?
उत्तर:
मृदा की उर्वरा शक्ति का विकास करने के लिए निम्नलिखित उपाय करने चाहिएं।

  1. मृदा अपरदन को रोकने के उपाय होने चाहिएं।
  2. मिट्टी की उर्वरा शक्ति बनाए रखने के लिए उर्वरकों और खादों का प्रयोग करना चाहिए।
  3. फसलों के हेर-फेर की प्रणाली का प्रयोग करना चाहिए।
  4. कृषि की वैज्ञानिक विधियों को अपनाना चाहिए।
  5. मिट्टी के उपजाऊ तत्त्वों का संरक्षण करके रासायनिक तत्त्वों को मिलाना चाहिए।

प्रश्न 11.
किसी प्रदेश के आर्थिक विकास में मृदा की विशेषता किस प्रकार महत्त्वपूर्ण भूमिका अदा करती है? इसकी व्याख्या करने के लिए दो उदाहरण बताइए।
उत्तर:
मृदा मानव के लिए बहुत मूल्यवान् प्राकृतिक सम्पदा है। मिट्टी पर बहुत-सी मानवीय क्रियाएं आधारित हैं। मिट्टी पर कृषि, पशु पालन तथा वनस्पति जीवन निर्भर करता है। किसी प्रदेश का आर्थिक विकास मिट्टी की उर्वरा शक्ति पर निर्भर करता है। कई देशों की कृषि अर्थव्यवस्था मिट्टी की उर्वरा शक्ति पर निर्भर करती है। संसार के प्रत्येक भाग में जनसंख्या का एक बड़ा भाग भोजन की पूर्ति के लिए मिट्टी पर निर्भर करता है।

अनुपजाऊ क्षेत्रों में जनसंख्या घनत्व तथा लोगों का जीवन स्तर दोनों ही निम्न होते हैं। उदाहरण के लिए पश्चिमी बंगाल के डेल्टाई प्रदेश तथा केरल तट जलोढ़ मिट्टी से बने उपजाऊ क्षेत्र हैं। यहां उन्नत कृषि का विकास हुआ है । यह प्रदेश भारत में सबसे अधिक जनसंख्या घनत्व वाला प्रदेश है। दूसरी ओर तेलंगाना में मोटे कणों वाली मिट्टी मिलती है तथा राजस्थान के शुष्क प्रदेश में रेतीली मिट्टी कृषि के अनुकूल नहीं है। इन प्रदेशों में विरल जनसंख्या पाई जाती है।

प्रश्न 12.
मृदा की उर्वरता समाप्ति के तीन कारण बताइए।
उत्तर:
मृदा एक मूल्यवान् प्राकृतिक संसाधन है। अधिक गहरी तथा उपजाऊ मिट्टी वाले प्रदेशों में कृषि का अधिक विकास होता है। मिट्टी की उर्वरा शक्ति के ह्रास के निम्नलिखित कारण हैं।

  1. कृषि भूमि पर निरन्तर कृषि करते रहने से मिट्टी की उर्वरा शक्ति धीरे-धीरे नष्ट होने लगती है। मृदा को उर्वरा शक्ति प्राप्त करने का पूरा समय नहीं मिलता।
  2. दोषपूर्ण कृषि पद्धतियों के कारण मृदा की उर्वरा शक्ति समाप्त होने लगती ह । स्थानान्तरी कृषि के कारण मृदा के उपजाऊ तत्त्वों का नाश होने लगता है। प्रतिवर्ष एक ही फसल बोने से मृदा के
  3. विशिष्ट प्रकार के खनिज कम होने लगते है।
  4. मृदा अपरदन से, वायु तथा जल द्वारा अपरदन से मृदा की उर्वरता समाप्त होती रहती है।

प्रश्न 13.
दक्षिणी पठार में पाई जाने वाली मृदा का लाल रंग क्यों है?
उत्तर:
दक्षिणी पठार के बाह्य प्रदेशों में लाल मिट्टी का अधिकतर विस्तार है। इस मिट्टी के मूल पदार्थ पठारी प्रदेश के पुराने क्रिस्टलीय तथा रूपान्तरित चट्टानों से प्राप्त होता है। इनमें ग्रेनाइट, नाईस तथा शिष्ट की चट्टानों का विस्तार है। इन चट्टानों में लोहा तथा मैग्नीशियम की अधिक मात्रा पाई जाती है। उष्ण कटिबन्धीय जलवायु जलीकरण की क्रिया के कारण आयरन-ऑक्साइड द्वारा इस मिट्टी का रंग लाल हो जाता है।

तुलनात्मक प्रश्न ( Comparison Type Questions)

प्रश्न 1.
वनस्पति जाति और वनस्पति में क्या अन्तर है?
उत्तर:

वनस्पति जाति (Flora) वनस्पति (Vegetation)
(1) विभिन्न समय में किसी प्रदेश में उगने वाले पेड़पौधों के विभिन्न वर्गों को वनस्पति जाति कहा जाता है। (1) किसी प्रदेश में उगने वाले पेड़-पौधों की जातियों के समुच्चय को वनस्पति कहा जाता है।
(2) वनस्पति जातियां वातावरण के अनुसार पनपती हैं। (2) पेड़-पौधों की विभिन्न जातियां एक-दूसरे से सम्बन्धित होती हैं।
(3) वनस्पति जातियों को एक वर्ग में रखकर उसे वनस्पति जाति का नाम दिया जाता है। जैसे-चीन तथा तिब्बत से प्राप्त होने वाले पेड़-पौधों को बोरियल कहा जाता है। (3) किसी पारिस्थितिक ढांचे में वन, पेड़-पौधे, घास एक-दूसरे से सम्बन्धित होते हैं इसलिए इन्हें वनस्पति कहा जाता है।

प्रश्न 2.
वनस्पति और वन में अन्तर स्पष्ट करो।
उत्तर:

वनस्पति (Vegetation) वन (Forest)
(1) किसी प्रदेश में उगने वाले पेड़-पौधों की जातियों के समूह को वनस्पति कहा जाता है। (1) पेड़-पौधों तथा झाड़ियों से घिरे हुए एक बड़े क्षेत्र को वन कहा जाता है।
(2) वनस्पति में एक वातावरण में उगने वाले पेड़पौधे और, झाड़ियों को शामिल किया जाता है। ऐसे भूदृश्य को wood land, grassland आदि नाम दिए जाते हैं। (2) घने तथा एक-दूसरे के निकट उगने वाले वृक्षों के क्षेत्र को वन कहा जाता है। वन शब्द का प्रयोग भूगोलवेत्ता तथा वन रक्षक करते हैं ताकि इनका आर्थिक मूल्यांकन किया जा सके।

JAC Class 11 Geography Important Questions Chapter 6 मृदा

प्रश्न 3.
संरचना, संकेन्द्रण क्षेत्र तथा रासायनिक रचना के संदर्भ में भारत की जलोढ़ तथा काली मिट्टी में अन्तर स्पष्ट करो।
उत्तर:

जलोढ़ मिट्टी काली मिट्टी
(1) क्षेत्र-इस मिट्टी का विस्तार उत्तरी मैदान की नदी घाटियों में पंजाब, हरियाणा, उत्तर प्रदेश, बिहार तथा पश्चिमी बंगाल राज्यों में है। (1) इस मिट्टी का विस्तार दक्षिणी भारत में लावा क्षेत्र में है जो कि गुजरात, महाराष्ट्र, मध्य प्रदेश, आन्ध्र प्रदेश, तथा तमिलनाडु राज्यों में है।
(2) गठन-यह मिट्टी रेतीली, दोमट और चिकनी होती है। (2) यह मिट्टी गहरी तथा दानेदार, चिकनी तथा अप्रवेशी है।
(3) रचना-इस मिट्टी में पोटाश अधिक होती है पर फॉस्फोरस कम होती है। (3) इस मिट्टी में चूना, लोहा, एल्यूमीनियम और पोटाश अधिक होते हैं।

निबन्धात्मक प्रश्न (Essay Type Questions)

प्रश्न 1.
मृदा निर्माण के मुख्य घटकों के प्रभाव का वर्णन करो।
उत्तर:
मृदा निर्माण कई भौतिक तथा रासायनिक तत्त्वों पर निर्भर करता है। इन तत्त्वों के कारण मृदा प्रकारों के वितरण भिन्नता पाई जाती है। ये तत्त्व स्वतन्त्र रूप से या अलग से नहीं बल्कि एक-दूसरे के सहयोग से काम करते हैं।

1. मूल पदार्थ:
मृदा निर्माण करने वाले पदार्थों की प्राप्ति आधार चट्टानों से होती है। इन चट्टानों के अपक्षरण से मूल पदार्थ प्राप्त होते हैं। प्रायः पठारों की मिट्टी का सम्बन्ध आधार चट्टानों से होता है। मैदानी भागों में मृदा निर्माण का मूल पदार्थ नदियों द्वारा जमा किए जाते हैं। नदियों में बाढ़ के कारण प्रत्येक वर्ष मिट्टी की नई परत बिछ जाती है।

2. उच्चावच:
किसी प्रदेश का उच्चावच तथा ढाल मृदा निर्माण पर कई प्रकार से प्रभाव डालता है। मैदानी भागों में गहरी मिट्टी मिलती है जबकि खड़ी ढाल वाले प्रदेशों में कम गहरी मिट्टी का आवरण होता है। पठारों पर भी मिट्टी की परत कम गहरी होती है। तेज़ ढाल वाले क्षेत्रों में अपरदन के कारण मिट्टी की ऊपरी परत बह जाती है। इस प्रकार किसी प्रदेश के धरातल तथा ढाल के अनुसार जल प्रवाह की मात्रा मृदा निर्माण को प्रभावित करती है।

3. जलवायु:
वर्षा, तापमान तथा पवनें किसी प्रदेश में मृदा निर्माण पर महत्त्वपूर्ण प्रभाव डालते हैं। जलवायु के अनुसार सूक्ष्म जीव तथा प्राकृतिक वनस्पति भी मृदा पर प्रभाव डालते हैं। शुष्क प्रदेशों में वायु मिट्टी के ऊपरी परत को उड़ा ले जाती है। अधिक वर्षा वाले प्रदेशों में मिट्टी कटाव अधिक होता ह।

4. प्राकृतिक वनस्पति:
किसी प्रदेश में मिट्टी का विकास प्राकृतिक वनस्पति की वृद्धि के साथ ही आरम्भ होता है। वनस्पति के गले – सड़े अंश के कारण मिट्टी में ह्यूमस की मात्रा बढ़ जाती है जिससे मिट्टी की उर्वरता बढ़ती है। इसी कारण घास के मैदानों में उपजाऊ मिट्टी का निर्माण होता है। भारत के विभिन्न प्रदेशों में मृदा तथा वनस्पति के प्रकारों में गहरा सम्बन्ध पाया जाता है।

प्रश्न 2.
मिट्टी की परिभाषा दो भारत की मिट्टियों का वर्गीकरण कर इन मिट्टियों के वितरण का वर्णन करो।
उत्तर:
मिट्टी (Soil) मिट्टी भूतल की ऊपरी सतह का आवरण है। भू-पृष्ठ पर मिलने वाले असंगठित पदार्थों की ऊपरी पर्त को मृदा या मिट्टी कहते हैं। भारत एक कृषि प्रधान देश है। कृषि की सफलता मिट्टी के उपजाऊपन पर निर्भर है। भारत की मिट्टियों को तीन परम्परागत वर्गों में बांटा जाता है।
(A) प्रायद्वीप की मिट्टिया।
(B) उत्तरी मैदान की मिट्टियां।
(C) हिमालय प्रदेश की मिट्टियां।

(A) प्रायद्वीप की मिट्टियां (Soils of the Peninsular India):
भारतीय प्रायद्वीप की रब्बेदार चट्टानों से बनने वाली मिट्टियां निम्नलिखित प्रकार की हैं।

1. काली मिट्टी (Black Soil):
(i) विस्तार (Extent ): इस मिट्टी का विस्तार प्रायद्वीप के उत्तर-पश्चिमी भाग में लगभग 5 लाख वर्ग किलोमीटर क्षेत्र में है। यह गुजरात, महाराष्ट्र के अधिकांश भाग, मध्य प्रदेश, दक्षिणी उड़ीसा, उत्तरी कर्नाटक, दक्षिणी आंध्र प्रदेश में मिलती है। यह मिट्टी नर्मदा, ताप्ती, गोदावरी तथा कृष्णा नदी की घाटियों में पाई जाती है।

(ii) विशेषताएं (Characteristics):
इस मिट्टी का निर्माण लावा प्रवाह से बनी आग्नेय चट्टानों के टूटने-फूटने से हुआ है। यह अधिकतर दक्कन लावा (Deccan Trap) के क्षेत्र में मिलता है। इसलिए इसे लावा मिट्टी भी कहते हैं । इस मिट्टी का रंग काला होता है इसलिए इसे काली मिट्टी (Black Soil) भी कहते हैं । इसे रेगड़ मिट्टी (Regur Soil) भी कहा जाता है। इस मिट्टी में चूना, लोहा, मैग्नीशियम की मात्रा अधिक होती है, परन्तु फॉस्फोरस, पोटाश, नाइट्रोजन तथा जैविक पदार्थों की कमी होती है। इस मिट्टी में नमी धारण करने की शक्ति अधिक है। इसलिए सिंचाई की आवश्यकता कम होती है। इस मिट्टी की तुलना में इसकी ‘शर्नीज़म’ तथा अमेरिका की ‘प्रेयरी’ मिट्टी से की जाती है ।

(iii) फसलों की उपयुक्तता (Suitability to Crops ):
यह मिट्टी कपास की कृषि के लिए उपयुक्त है। इसलिए इसे ‘कपास की मिट्टी’ (Cotton Soil) भी कहते हैं। पठारों की उच्च भूमि में कम उपजाऊ होने के कारण यहां ज्वार – बाजरा, दालों आदि की कृषि होती है। मैदानी भाग में गेहूं, कपास, तम्बाकू, मूंगफली तथा तिलहन की कृषि की जाती है।

2. लाल मिट्टी (Red Soil)
(i) विस्तार (Extent ): भारत की सब मिट्टियों में से लाल मिट्टी का विस्तार सबसे अधिक है। यह मिट्टी लगभग 8 लाख वर्ग किलोमीटर क्षेत्र में पाई जाती है। दक्षिणी भारत में तमिलनाडु, कर्नाटक, आन्ध्र प्रदेश, छोटानागपुर पठार तथा प्रायद्वीप पठार के बाहरी भागों में लाल मिट्टी का विस्तार मिलता है.

(ii) विशेषताएं (Characteristics): इस मिट्टी का निर्माण प्रायद्वीप की आधारभूत आग्नेय चट्टानों ग्रेनाइट तथा नीस चट्टानों की टूट-फूट से हुआ है। इस मिट्टी का रंग लौहयुक्त चट्टानों में ऑक्सीकरण (oxidation) की क्रिया से लाल हो जाता है। वर्षा के कारण ह्यूमस नष्ट हो जाता है तथा आयरन ऑक्साइड ऊपरी सतह पर आ जाता है। इस मिट्टी में लोहा और एल्यूमीनियम की अधिकता होती है परन्तु नाइट्रोजन और फॉस्फोरस की कमी होती है । यह मिट्टी कम गहरी तथा कम उपजाऊ होती है।

(iii) फसलों की उपयुक्तता (Suitability to Crops ): उच्च प्रदेशों में यह मिट्टी कम गहरी तथा अनुपजाऊ होती है। यहां केवल ज्वार – बाजरा की फसलें उगाई जाती हैं। नदी घाटियों में अधिक उपजाऊ मिट्टी में कपास, गेहूं, दालें अलसी, मोटे अनाज, तम्बाकू की कृषि की जाती है।

3. लैटेराइट मिट्टी (Laterite Soil):
(i) विस्तार (Extent ): लैटेराइट मिट्टी का विस्तार एक विशाल क्षेत्र में लगभग 2.5 लाख वर्ग किलोमीटर क्षेत्र पर है। इस मिट्टी का विस्तार असम, राजमहल की पहाड़ियों, पूर्वी घाट तथा पश्चिमी घाट, मध्य प्रदेश में विन्ध्याचल और सतपुड़ा पहाड़ियों के बसाल्ट क्षेत्र में है।

(ii) विशेषताएं (Characteristics): इस मिट्टी का रंग ईंट (Brick) के समान लाल होता है। इस मिट्टी में लोहा, एल्यूमीनियम आदि तत्त्वों की अधिकता होती है। परन्तु नाइट्रोजन तथा वनस्पति अंश की कमी होती है। उष्ण-आर्द्र जलवायु में अपक्षालन क्रिया (Leaching of Soil) के कारण इस मिट्टी में से सिलिका की मात्रा कम रह जाती है तथा लौह पदार्थों की अधिकता हो जाती है। यह मिट्टी तुरन्त जल सोख लेती है। कणों के आधार पर यह मिट्टी दो प्रकार की है।
(क) उच्च प्रदेशों की लैटेराइट (Upland Laterite ): यह मिट्टी कम गहरी तथा कंकरीली होती है। यह कृषि के लिए उपजाऊ नहीं होती।
(ख) निचले प्रदेशों की लैटेराइट (Lowland Laterite ): यह मिट्टी बारीक कणों तथा दोमट के कारण अधिक उपजाऊ है।

(iii) फसलों की उपयुक्तता (Suitability to Crops ): मैदानी भागों में चाय, रबड़, कहवा, सिनकोना तथा मोरे- अनाज की कृषि की जाती है।

4. जलोढ़ मिट्टी (Alluvial Soil)
(i) विस्तार (Extent):
यह प्रवाहित मिट्टी है जो नदियों द्वारा डेल्टाई क्षेत्रों में पाई जाती है। इस मिट्टी का विस्तार महानदी, गोदावरी, कृष्णा, कावेरी आदि नदी घाटियों में तथा डेल्टाई प्रदेशों में मिलता है।

(ii) विशेषताएं (Characteristics):
यह मिट्टी उत्तरी मैदान की मिट्टियों की भान्ति उपजाऊ है। इसमें पोटाश तथा चूने की अधिकता है परन्तु नाइट्रोजन की कमी है। इस मिट्टी में पटसन, चावल, गन्ना आदि फसलों की कृषि की जाती है।

(B) उत्तरी मैदान की मिट्टियां (Soils of the Northern Plain): उत्तरी मैदान की अधिकांश मिट्टियां जलोढ़ तथा नवीन हैं। यह मिट्टियां नदियों द्वारा पर्वतीय भागों से लाए गए तलछट के जमाव से बनी हैं।1. जलोढ़ मिट्टियां (Aluvial Soils):
(i) विस्तार (Extent ): इन मिट्टियों का विस्तार पश्चिम में पंजाब से लेकर असम तक मिलता है। यह मिट्टियां पंजाब, उत्तर प्रदेश, बिहार, पश्चिमी बंगाल असम तथा राजस्थान के लगभग 7.7 लाख वर्ग कि०मी० क्षेत्र में पाई जाती हैं। भारत के 43.7% क्षेत्र पर जलोढ़ मिट्टियों का विस्तार है।
JAC Class 11 Geography Important Questions Chapter 6 मृदा 1

(ii) विशेषताएं (Characteristics): यह गहरी तथा उपजाऊ मिट्टी है। गंगा – सतलुज का मैदान इसी मिट्टी से बना हुआ है। यह मिट्टी लगभग 400 मीटर गहरी है। इस मिट्टी में नाइट्रोजन, फॉस्फोरस और जीवांश की कमी होती है परन्तु पोटॉश तथा चूना पर्याप्त मात्रा में पाया जाता है। उपजाऊपन तथा रचना के आधार पर जलोढ़ मिट्टियों को तीन उपविभागों में बांटा जाता है।

(क) बांगर मिट्टी (Banger Soil): यह पुरातन जलोढ़ मिट्टियां हैं। पुराने जलोढ़ निक्षेप से बने ऊंचे प्रदेश को बांगर कहते हैं, जहां बाढ़ का पानी नहीं पहुंच पाता इसे धाया भी कहते हैं।
(ख) खादर मिट्टियां (Khadar Soils ): नदी के समीप नवीन मिट्टी से बने निचले प्रदेश को खादर कहते हैं। नदियों बाढ़ कारण प्रति वर्ष जलोढ़ की नई पर्त बिछ जाने से यह अधिक उपजाऊ मिट्टी है। इसे बेट भी कहा जाता है।
(ग) नूतनतम जलोढ़ मिट्टियां (Newest Alluvial Soils): नदी डेल्टाई प्रदेशों में बारीक कणों के निक्षेप से इन मिट्टियों का विस्तार मिलता है।

(iii) फसलों की उपयुक्तता (Suitability to Crops ): इन मिट्टियों को कांप की मिट्टी तथा दोमट मिट्टी भी कहा जाता है। इसकी तुलना संसार के उपजाऊ मैदानों से की जाती है। यह वास्तव में भारत का अन्न भण्डार है। इन मिट्टियों में गेहूं, चावल, पटसन की कृषि भी की जाती है। जल सिंचाई की सहायता से गन्ना, कपास, तम्बाकू तथा तिलहन की कृषि की जाती है।

2. मरुस्थलीय मिट्टी (Desert Soil):
इस मिट्टी का विस्तार शुष्क प्रदेशों में राजस्थान, गुजरात, पंजाब, हरियाणा में मिलता है। इसका क्षेत्रफल लगभग 2 लाख वर्ग कि०मी० है। यह मोटे कणों वाली बलुआही मिट्टी होती है। इसमें जल सिंचाई की सहायता से ज्वार, बाजरा, कपास की कृषि की जाती है। राजस्थान प्रदेश में इन्दिरा नहर के निर्माण से इस मिट्टी में जल सिंचाई की सहायता से अधिक उत्पादन हो सकेगा।

3. नमकीन एवं क्षारीय मिट्टियां (Saline and Alkaline Soils):
यह शुष्क प्रदेशों तथा दलदली भागों में पाई जाती है। इनका विस्तार लगभग 1 लाख वर्ग कि० मी० में है। इन्हें प्रायः थूर, ऊसर, कल्लर, रेह आदि नामों से पुकारा जाता है। इस मिट्टी में लवणों की मात्रा अधिक जमा हो जाती है जिससे यह अनुपजाऊ बन जाती है।

(C) हिमालय प्रदेश की मिट्टियां (Soils of the Himalayas ) इन मिट्टियों की पर्याप्त खोज अभी नहीं हो पाई है। अधिकांश मिट्टियां पतली, अनुपजाऊ तथा कम गहरी हैं। हिमालय प्रदेश की मिट्टियों को तीन उपविभागों में बांटा जाता है।

  1. पथरीली मिट्टी (Stony Soil): दक्षिणी स्थानों पर मोटे कणों वाली पथरीली मिट्टी मिलती है जो अनुपजाऊ है।
  2. चाय की मिट्टी (Soil of Tea): पर्वतीय ढलानों पर तथा दून घाटियों में उपजाऊ मिट्टी मिलती है जिसमें चूना तथा लोहे का अंश अधिक होता है। इसमें चाय की कृषि अधिक की जाती है।
  3. लावा मिट्टी (Volcanic Soil): पर्वतीय ढलानों पर लावा से बनी मिट्टी मिलती है।

(D) पीटमय तथा जैव मृदाएँ
वनस्पति की अच्छी बढ़वार वाले तथा भारी वर्षा और उच्च आर्द्रता से युक्त क्षेत्रों में ये मृदाएँ पाई जाती हैं। वनस्पति की तीव्र वृद्धि के कारण इन क्षेत्रों में जैव पदार्थ भारी मात्रा में इकट्ठे हो जाते हैं। इससे मृदाओं में पर्याप्त मात्रा में जैव तत्त्व और ह्यूमस होता है। इसीलिए ये पीटमय और जैव मृदाएँ हैं। इन मृदाओं में जैव पदार्थों की मात्रा 40 से 50 प्रतिशत तक हो सकती है। ये मृदाएँ सामान्यतः भारी और काले रंग की होती है। अनेक स्थानों पर ये क्षारीय भी हैं। बिहार के उत्तरी भाग, उत्तराखंड के दक्षिणी भाग, बंगाल के तटीय क्षेत्रों, उड़ीसा और तमिलनाडु में ये मृदाएँ अधिकतर पाई जाती हैं। ये मृदाएँ हल्की और कम उर्वरता का उपभोग करने वाली फसलों की खेती के लिए उपयुक्त हैं।

(E) वन मृदाएँ
नाम के अनुरूप ये मृदाएँ पर्याप्त वर्षा वाले वन क्षत्रो में ही बनती हैं। इन मृदाओं का निर्माण पर्वतीय पर्यावरण में होता है। इस पर्यावरण में परिवर्तन के अनुसार इन मृदाओं का गठन और संरचना बदलती रहती है। घाटियों में ये दुमटी और गादयुक्त होती है तथा ऊपरी ढालों पर ये मोटे कणों वाली होती है। हिमालय के हिम से ढके क्षेत्रों में इन मृदाओं में अनाच्छादन होता रहता है तथा ये अम्लीय और कम ह्यूमस वाली होती हैं। निचली घाटियों में पाई जाने वाली मृदाएँ उपजाऊ होती हैं और इनमें चावल तथा गेहूँ की खेती की जाती है।

JAC Class 11 Geography Important Questions Chapter 6 मृदा

प्रश्न 3.
मृदा अपरदन से क्या अभिप्राय है? भारत बीहड़ों की रचना तथा विवरण बताओ
उत्तर:
मृदा अपरदन जब तक मृदा निर्माण की प्रक्रियाओं और मृदा अपरदन में सन्तुलन बना रहता है, तब तक कोई समस्या नहीं पैदा होती। इस सन्तुलन के बिगड़ते ही, मृदा अपरदन एक खतरा बन जाता है। वृक्षों की अंधाधुंध कटाई, चरागाहों में बेफिक्री से अति चराई, अवैज्ञानिक अपवाह प्रक्रियाएं तथा भूमि का अनुचित उपयोग इस सन्तुलन को बिगाड़ने के महत्त्वपूर्ण कारणों में से कुछ हैं। पश्चिम बंगाल, उत्तर प्रदेश, मध्य प्रदेश, महाराष्ट्र, तमिलनाडु, कर्नाटक,
JAC Class 11 Geography Important Questions Chapter 6 मृदा 2
दिल्ली, राजस्थान और देश के अन्य अनेक भागों में मृदा अपरदन एक समस्या रही है। पहाड़ी ढालों पर गो- पशुओं द्वारा अति चराई से मृदा अपरदन की गति तेज़ हो जाती है। मेघालय और नीलगिरि की पहाड़ियों पर आलू की खेती और हिमालय और पश्चिमी घाट पर वनों के विनाश तथा देश के विभिन्न भागों में जन- जातीय लोगों द्वारा की जाने वाली झूम कृषि के कारण मृदाओं का उल्लेखनीय क्षरण हुआ है।

बीहड़ (Ravines)

  1. चम्बल नदी की द्रोणी में बीहड़ (Ravines) बहुत विस्तृत हैं। मध्य प्रदेश के ग्वालियर, मोरैना और भिंड ज़िलों में तथा उत्तर प्रदेश के आगरा, इटावा और जालौन जिलों में बीहड़ 6 लाख हेक्टेयर भूमि में फैले हैं।
  2. तमिलनाडु के दक्षिणी व उत्तरी अर्काट, कन्याकुमारी, तिरूचिरापल्ली, चिंगलीपुत, सेलम और कोयंबटूर जिलों में भी बीहड़ खूब हैं।
  3. पश्चिमी बंगाल के पुरुलिया जिले की कंगसाबती नदी के ऊपर जलग्रहण क्षेत्रों में अनेक अवनालिकाएँ (gullies) और बीहड़ हैं। देश की लगभग 8,000 हेक्टेयर भूमि प्रतिवर्ष बीहड़ बन जाती है।

मृदा अपरदन के दुष्प्रभाव:

  1. भारत की कृषि भूमि में से 80,000 हेक्टेयर भूमि अब तक बेकार हो गई है तथा इससे भी बड़ा क्षेत्र प्रतिवर्ष मृदा अपरदन के कारण कम उत्पादक हो जाता है।
  2. मृदा अपरदन भारतीय कृषि के लिए एक राष्ट्रीय संकट बन गया है। इसके दुष्प्रभाव अन्य क्षेत्रों में भी दिखाई पड़ते हैं।
  3. नदी की घाटियों में अपरदित पदार्थों के जमा होने से उनकी जल-प्रवाह क्षमता घट जाती है, इससे प्रायः बाढ़ें आती हैं तथा कृषि भूमि को क्षति पहुंचती है।
  4. उदाहरण के लिए तमिलनाडु के तिरूचिरापल्ली में कावेरी नदी का तल क्रमशः ऊपर उठ गया है। इसके परिणामस्वरूप सिंचाई के पुराने जल कपाट और अपवाह धाराएँ अवरुद्ध हो गई हैं।
  5. ब्रह्मपुत्र नदी के उथले होने से प्रतिवर्ष बाढ़ें आती हैं।
  6. तालाबों में गाद जमा होना, मृदा अपरदन का अन्य गंभीर परिणाम हैं। देश के विभिन्न भागों में अनेक तालाबों में प्रतिवर्ष गाद जमा हो जाती है।

मृदा अपरदन के प्रकार:
भारत में मृदा अपरदन के दो सबसे अधिक सक्रिय कारक हैं- पवन और प्रवाहित जल पवन द्वारा अपरदन गुजरात, राजस्थान और हरियाणा के शुष्क और अर्द्धशुष्क क्षेत्रों में सामान्य रूप से होता है। भारी मृदाओं की तुलना में हल्की मृदाओं पर पवन – अपरदन का अधिक प्रभाव पड़ता है। पवन द्वारा उड़ाकर लाया गया बालू समीप की कृषि भूमि पर फैलकर जमा हो जाता है और उसकी उर्वरता को नष्ट कर देता है। जल- अपरदन अपेक्षाकृत अधिक गम्भीर है।

इससे भारत के विभिन्न भाग विस्तृत रूप से प्रभावित हैं। जल अपरदन के मुख्यतः दो रूप हैं- परतदार अपरदन और अवनालिका अपरदन। समतल भूमि पर मूसलाधार वर्षा के बाद परतदार अपरदन होता है। इसमें मृदा के हटने का आसानी से पता ही नहीं चलता है। तीव्र ढालों पर सामान्यतः अवनालिका अपरदन होता है। वर्षा के द्वारा अवनालिकाएं गहरी होती जाती हैं। ये कृषि भूमि को छोटे-छोटे टुकड़ों में बाँट देती है। इससे भूमि खेती के लिए अनुपयुक्त हो जाती है।

मृदा अपरदन के कारक:
मृदा के गुण ह्रास में अनेक कारकों का योगदान रहता है। उदाहरणार्थ, जब जंगल काट दिए जाते हैं, तब मृदा में ह्यूमस की आपूर्ति ठप्प हो जाती है। यही नहीं, मृदा की ऊपरी परत को हटाने में प्रवाहित जल की क्षमता बढ़ जाती है। यदि अपवाह तन्त्र गड़बड़ा जाता है, तो जल भराव या मृदा की नमी का ह्रास होने लगता है। मृदा के अत्यधिक उपयोग से इसकी उर्वरता समाप्त हो जाती है। आर्द्र क्षेत्रों में प्रवाहित जल और शुष्क क्षेत्रों में पवन द्वारा मृदा के हटाए जाने को मृदा अपरदन कहते हैं। इसके विपरीत इसके जैव और खनिज तत्त्वों के हटाए जाने को मृदाक्षय कहते हैं। मृदा के दुरुपयोग से इसका अपक्षरण होता है।

मृदा अपरदन, क्षय और अपक्षरण में लिप्त कारक हैं- प्रवाहित जल, पवन, हिम, जीव-जन्तु और मानव । मानव वनोन्मूलन अति चराई और कृषि के अवैज्ञानिक तरीकों से मृदा के पारितन्त्र को अस्त-व्यस्त कर देता है । विरल वनस्पति वाले क्षेत्रों और तीव्र ढालों पर विशेष रूप से ऊबड़-खाबड़ भूमि पर तथा नदी भागों के साथ-साथ प्रायः बीहड़ दिखाई पड़ जाते हैं। कोसी नदी के द्वारा किया गया अपरदन कुख्यात हो गया है। राजस्थान के शुष्क प्रदेश पवन – अपरदन की चपेट में हैं। गहन कृषि और अतिचराई अपरदन और मरुस्थलीकरण की प्रक्रियाओं को तेज़ कर देते हैं

वनोन्मूलन तथा मृदा अपरदन।
वनोन्मूलन मृदा अपरदन के प्रमुख कारकों में से एक है। पौधों की जड़ें मृदा को बांधे रखकर अपरदन को रोकती हैं। पत्तियां और टहनियां गिरा कर वे मृदा में ह्यूमस की मात्रा में वृद्धि करते हैं। वास्तव में सम्पूर्ण भारत में वनों का विनाश हुआ है। लेकिन देश के पहाड़ी भागों, विशेष रूप से हिमाचल प्रदेश और पश्चिमी घाट पर मृदा अपरदन में इसका बड़ा हाथ है।

राज्य & क्षेत्रफल 12.30
उत्तर प्रदेश 6.83
मध्य प्रदेश 4.52
राजस्थान 4.00
गुजरात 0.20
महाराष्ट्र 1.20
पंजाब 6.00
बिहार 0.60
तमिलनाडु  1.04
पश्चिम बंगाल 12.30

JAC Class 11 Geography Important Questions Chapter 6 मृदा

प्रश्न 4.
मृदा अपरदन किसे कहते हैं? इसके क्या कारण हैं ? मानव ने मृदा अपरदन से बचाव के कौन- कौन से तरीके अपनाए हैं?
उत्तर:
मृदा अपरदन (Soil Erosion ):
भूतल की ऊपरी सतह से उपजाऊ मिट्टी का उड़ जाना या बह जाना मृदा अपरदन कहलाता है। भूतल की मिट्टी धीरे-धीरे अपना स्थान छोड़ती रहती है जिससे कृषि के अयोग्य हो जाती है। मृदा अपरदन के प्रकार (Types of Soil Erosion) – मृदा अपरदन तीन प्रकार से होता है।

  1. धरातलीय कटाव (Sheet Erosion)
  2. नालीदार कटाव ( Guly Erosion)
  3. वायु द्वारा कटाव (Wind Erosion )

मृदा अपरदन के कारण (Causes of Soil Erosion):

  1. मूसलाधार वर्षा (Torrential Rain): पर्वतीय प्रदेशों की तीव्र ढलानों पर मूसलाधार वर्षा का जल मिट्टी की परत बहा कर ले जाता है। इससे यमुना घाटी में उत्खात भूमि की रचना हुई है।
  2. वनों की कटाई (Deforestation ): वनों के अन्धाधुन्ध कटाव से मृदा अपरदन बढ़ जाता है; जैसे – पंजाब में शिवालिक पहाड़ियों पर तथा मैदानी भाग में चो (Chos) द्वारा मृदा अपरदन एक गम्भीर समस्या है।
  3. स्थानान्तरी कृषि (Shifting Agriculture ): वनों को जला कर कृषि के लिए प्राप्त करने की प्रथा से झुमिंग (Jhumming) से उत्तर पूर्वी भारत में मृदा अपरदन होता है।
  4. नर्म मिट्टी (Soft Soils): कई प्रदेशों में नर्म मिट्टी के कारण मिट्टी की परत बह जाती है।
  5. अनियन्त्रित पशु चारण (Uncontrolled Cattle Grazing ): पर्वतीय ढलानों पर चरागाहों में अनियन्त्रित पशुचारण से मिट्टी के कण ढीले होकर बह जाते हैं।
  6. धूल भरी आन्धियां (Dust Storms ): मरुस्थलों के निकटवर्ती प्रदेशों में तेज़ धूल भरी आन्धियों के कारण मिट्टी का परत अपरदन होता है।

मृदा अपरदन रोकने के उपाय (Methods to Check Soil Erosion) मिट्टी के उपजाऊपन को कायम रखने के लिए मिट्टी का संरक्षण आवश्यक है। मृदा अपरदन रोकने के लिए कई प्रकार के उपाय प्रयोग किए जाते हैं।
(i) वृक्षारोपण (Afforestation ):
पर्वतीय ढलानों पर मिट्टी को संगठित रखने के लिए वृक्ष लगाए जाते हैं नदियों के ऊपरी भागों में वन क्षेत्रों का विस्तार करके मृदा अपरदन को रोका जा सकता है। इसी प्रकार राजस्थान मरुस्थल की सीमाओं पर वन लगाकर वायु द्वारा अपरदन रोकने के लिए उपाय किए गए हैं।

(ii) नियन्त्रित पशु चारण (Controlled Grazing ):
ढलानों पर चरागाहों में बे-रोक-टोक पशु चारण को रोका जाए ताकि घास को फिर से उगने और बढ़ने का समय मिल सके।

(iii) सीढ़ीनुमा कृषि (Terraced Agricuture ):
पर्वतीय ढलानों पर सम ऊंचाई की रेखा के साथ सीढ़ीदार खेत बनाकर वर्षा के जल को रोक कर मृदा अपरदन को रोका जा सकता है।

(iv) बांध बनाना (River Dams):
नदियों के ऊपरी भागों पर बांध बनाकर बाढ़ नियन्त्रण द्वारा मृदा अपरदन को रोका जा सकता है।

(v) अन्य उपाय:
भूमि को पवन दिशा के समकोण पर जोतना चाहिए जिससे पवन द्वारा मिट्टी कटाव कम हो सके स्थानान्तरी कृषि को रोका जाए। वायु की गति को कम करने के लिए वृक्ष लगा कर वायु रोक (wind break) क्षेत्र बनाना चाहिए। फसलों के हेर-फेर तथा भूमि को परती छोड़ देने से मृदा अपरदन कम किया जा सकता है।

JAC Class 11 Geography Important Questions Chapter 7 प्राकृतिक संकट तथा आपदाएँ

Jharkhand Board JAC Class 11 Geography Important Questions Chapter 7 प्राकृतिक संकट तथा आपदाएँ Important Questions and Answers.

JAC Board Class 11 Geography Important Questions Chapter 7 प्राकृतिक संकट तथा आपदाएँ

बहु-विकल्पी प्रश्न (Multiple Choice Questions )

प्रश्न – दिए गए चार वैकल्पिक उत्तरों में से सही उत्तर चुनिए
1. कौन – सा परिवर्तन तेज़ गति से होता है?
(A) दरार
(B) वलन
(C) भूकम्प
(D) भूमण्डलीय तापन
उत्तर:
(C) भूकम्प।

2. किस आपदा का सम्बन्ध मानवीय क्रियाओं से है?
(A) भूकम्प
(B) ज्वालामुखी
(C) पर्यावरण प्रदूषण
(D) टारनेडो
उत्तर:
(C) पर्यावरण प्रदूषण।

3. पहला भू- शिखर सम्मेलन कब हुआ ?
(A) 1974
(B) 1984 में
(C) 1994 में
(D) 1998 में।
उत्तर:
(C) 1994 में।

4. पहला भू- शिखर सम्मेलन कहां हुआ था?
(A) रियो डी जनेरो
(B) टोकियो
(C) न्यूयार्क
(D) लन्दन।
उत्तर:
(A) रियो डी जनेरो।

5. सुनामी त्रासदी कब हुई?
(A) 2001 में
(B) 2002 में
(C) 2003 में
(D) 2004 में।
उत्तर:
(D) 2004 में।

JAC Class 11 Geography Important Questions Chapter 7 प्राकृतिक संकट तथा आपदाएँ

6. भू-स्खलन किस प्रकार की आपदा है?
(A) वायु मण्डलीय
(B) भौमिक
(C) जलीय
(D) जैविक
उत्तर:
(B) भौमिक।

7. भारतीय प्लेट प्रतिवर्ष किस दर से खिसक रही है?
(A) 1 सें० मी०
(B) 2 सें० मी०
(C) 3 सें० मी०
(D) 4 सें० मी०
उत्तर:
(A) 1 सें० मी।

8. दक्षिणी भारत में भ्रंश रेखा का विकास कहां हुआ है?
(A) कोयना के निकट
(B) लातूर के निकट
(C) अहमदाबाद के निकट
(D) भोपाल के निकट
उत्तर:
(B) लातूर के निकट।

9. भूकम्प से समुद्र में उठने वाली लहरों को क्या कहते हैं?
(A) लहरें
(B) दरार
(C) सुनामी
(D) ज्वार।
उत्तर:
(C) सुनामी

10. तूफ़ान महोर्मि का मुख्य कारण है
(A) सुनामी
(B) ज्वार
(C) उष्ण कटिबन्धीय चक्रवात
(D) मानसून।
उत्तर:
(C) उष्ण कटिबन्धीय चक्रवात।

JAC Class 11 Geography Important Questions Chapter 7 प्राकृतिक संकट तथा आपदाएँ

11. भारत में कितना क्षेत्र बाढ़ प्रभावित है?
(A) 1 करोड़ हेक्टेयर
(B) 2 करोड़ हेक्टेयर
(C) 3 करोड़ हेक्टेयर
(D) 4 करोड़ हेक्टेयर
उत्तर:
(D) 4 करोड़ हेक्टेयर।

12. भारत के कुल क्षेत्र का कितने % भाग सूखाग्रस्त रहता है?
(A) 9%
(B) 12 %
(C) 19%
(D) 25 %.
उत्तर:
(C) 19 %.

13. भारत में आपदा प्रबन्धन नियम कब बनाया गया?
(A) 2001 में
(B) 2002 में
(C) 2005 में
(D) 2006 में।
उत्तर:
(C) 2005 में।

14. भूकम्प किस प्रकार की आपदा है?
(A) वायुमण्डलीय
(B) भौमिकी
(C) जलीय
(D) जीव – मण्डलीय।
उत्तर:
(B) भौमिकी .

अति लघु उत्तरीय प्रश्न (Very Short Answer Type Quesrtion)

प्रश्न 1.
भारत में प्रभाव डालने वाले प्राकृतिक आपदाओं के नाम लिखो।
उत्तर:
बाढ़ें, सूखा, भूकम्प तथा भू-स्खलन।

प्रश्न 2.
प्राकृतिक आपदाओं का मुख्य कारण क्या है?
उत्तर:
आकस्मिक भूगर्भिक हलचलें।

JAC Class 11 Geography Important Questions Chapter 7 प्राकृतिक संकट तथा आपदाएँ

प्रश्न 3.
वर्तमान समय में भारत में आये विनाशकारी भूचाल का नाम लिखो।
उत्तर:
भुज, गुजरात – 26 जनवरी, 2001.

प्रश्न 4.
भू विभिन्न भूकम्पीय तरंगों के नाम लिखो।
उत्तर:

प्रश्न 5.
भूकम्प मापक यन्त्र को क्या कहते हैं?
उत्तर:
सिज़्मोग्राफ।

प्रश्न 6.
भूकम्प की तीव्रता किस पैमाने पर मापी जाती है?
उत्तर:
रिक्टर पैमाने पर।

प्रश्न 7.
लाटूर भूकम्प (महाराष्ट्र ) का क्या कारण था?
उत्तर:
भारतीय प्लेट का उत्तर की ओर खिसकना।

प्रश्न 8.
भूकम्प किस सिद्धान्त से सम्बन्धित है?
उत्तर:
प्लेट टेक्टानिक।

प्रश्न 9.
रिक्टर पैमाने पर कितने विभाग होते हैं?
उत्तर:
1-9 तक।

JAC Class 11 Geography Important Questions Chapter 7 प्राकृतिक संकट तथा आपदाएँ

प्रश्न 10.
कोएना भूकम्प का क्या कारण था?
उत्तर:
कोयना जलाशय में अत्यधिक जलदाब।

प्रश्न 11.
सूखा किसे कहते हैं?
उत्तर:
वर्षा की कमी के कारण खाद्यान्नों की कमी होना।

प्रश्न 12.
भारत में सूखे का मुख्य कारण क्या है?
उत्तर:
अनिश्चित वर्षा।

प्रश्न 13.
भारत में कितना क्षेत्रफल भाग बाढ़ों तथा सूखे से प्रभावित है?
उत्तर:
सूखे से 10% भाग तथा बाढ़ों से 12% भाग।

प्रश्न 14.
भारत में बाढ़ों का मुख्य कारण क्या है?
उत्तर:
भारी मानसून वर्षा तथा चक्रवात।

प्रश्न 15.
दक्षिणी प्रायद्वीप में बाढ़ें कम हैं। क्यों?
उत्तर:
मौसमी नदियों के कारण।

प्रश्न 16.
भू-स्खलन किसे कहते हैं?
उत्तर:
जब कोई जलभृत भाग किसी ढलान से अचानक नीचे गिरते हैं।

JAC Class 11 Geography Important Questions Chapter 7 प्राकृतिक संकट तथा आपदाएँ

प्रश्न 17.
तीन प्रदेशों के नाम लिखो जो चक्रवातों से प्रभावित हैं।
उत्तर:
उड़ीसा, आन्ध्र प्रदेश, तमिलनाडु

प्रश्न 18.
भूकम्प के आने का मुख्य कारण क्या है?
उत्तर:
विवर्तनिक हलचलें।

प्रश्न 19.
भारत में अत्यधिक सूखा प्रभावित क्षेत्र के एक जिले का नाम लिखिए।
उत्तर:
बीकानेर

लघु उत्तरीय प्रश्न (Short Answer Type Quesrtion)

प्रमुख प्राकृतिक आपदाएं कौन-सी हैं?
उत्तर:
प्राकृतिक आपदाएं वे भूगर्भिक हलचलें हैं जो अचानक ही भू-तल पर परिवर्तन लाकर जन और धन व सम्पत्ति की हानि करती हैं। सूखा, बाढ़ें, चक्रवात, भू-स्खलन, भूकम्प विभिन्न प्रकार की मुख्य प्राकृतिक आपदाएं हैं।

प्रश्न 2.
प्राकृतिक आपदाओं से होने वाली जन-धन की हानि का वर्णन करो।
उत्तर:
विश्व में प्रतिवर्ष प्राकृतिक आपदाओं से एक लाख व्यक्तियों की जानें जाती हैं तथा 20,000 करोड़ रुपये की सम्पत्ति की हानि होती है। यह मानवीय विकास के लिए एक रुकावट है। U.NO. के अनुसार 1990-99 के दशक को प्राकृतिक आपदाओं से सुरक्षा का दशक घोषित किया गया प्राकृतिक आपदाओं से ग्रस्त विश्व के प्रमुख 10 देशों में से भारत एक देश है। प्रति वर्ष 6 करोड़ लोग इनसे प्रभावित होते हैं। विश्व की 50% प्राकृतिक आपदाएं भारत में अनुभव की जाती हैं। फिर भी भारत में इन आपदाओं से सुरक्षा के लिए एक व्यापक प्रबन्ध किया गया है जिसमें भूकम्पीय स्टेशन, चक्रवात, बाढ़ें, राडार, जल प्रवाह के बारे में सूचनाएं प्राप्त की जाती हैं तथा सुरक्षा के प्रबन्ध किए जाते हैं।

प्रश्न 3.
भू-स्खलन से क्या अभिप्राय है? इनके प्रभाव बताओ।
उत्तर- भू-स्खलन (Landslides ):
भूमि के किसी भाग के अचानक फिसल कर पहाड़ी से नीचे गिर जाने की क्रिया को भू-स्खलन कहते हैं। कई बार भूमिगत जल चट्टानों में भर कर उनका भार बढ़ा देता है। यह जल भृत चट्टानें ढलान के साथ नीचे फिसल जाती हैं। इनके कई प्रकार होते हैं।

  1. सलम्प (Slumps ): जब चट्टानें थोड़ी दूरी से गिरती हैं।
  2. राक सलाइड (Rockslide ): जब चट्टानें अधिक दूरी से अधिक भार में गिरती हैं।
  3. राक फाल (Rockfall): जब किसी भृत से चट्टानें टूट कर गिरती हैं।

कारण (Causes):

  1. जब वर्षा का जल या पिघलती हिम एक सनेहक (Lubricant ) के रूप में कार्य करता है।
  2. तीव्र ढलान के कारण।
  3. भूकम्प के कारण।
  4. किसी सहारे के हट जाने पर।
  5. भ्रंशन या खदानों के कारण।
  6. ज्वालामुखी विस्फोट के कारण।

प्रभाव (Effects):

  1. भवन, सड़कें, पुल आदि का नष्ट होना।
  2. चट्टानों के नीचे दबकर लोगों की मृत्यु हो जाना।

सड़क मार्गों का अवरुद्ध हो जाना।

  1. नदियों के मार्ग अवरुद्ध होने से बाढ़ें आना।
  2. 1957 में कश्मीर में भू-स्खलन से राष्ट्रीय मार्ग बन्द हो गया था।
  3. गत वर्षों में टेहरी गढ़वाल में बादल फटने से भू-स्खलन हुआ।

प्रश्न 4.
भारत में उष्ण कटिबन्धीय चक्रवातों पर नोट लिखो।
उत्तर:
चक्रवात (Cyclones) :
भारत में उष्ण कटिबन्धीय चक्रवात खाड़ी बंगाल तथा अरब सागर में उत्पन्न होते हैं। चक्रवात पवनों का एक भँवर होता है जो मूसलाधार वर्षा प्रदान करता है। ये प्रायः अक्तूबर-नवम्बर के महीनों में चलते हैं। इनकी दिशा परिवर्तनशील होती है। ये प्रायः पश्चिम की ओर तथा उत्तर-पश्चिम, उत्तर पूर्व की ओर चलते हैं। इनका प्रभाव तमिलनाडु आन्ध्र प्रदेश, उड़ीसा के तटों पर होता है।

प्रभाव (Effects):

  1. ये चक्रवात मूसलाधार वर्षा, तेज़ पवनें तथा घने मेघ लाते हैं। औसत रूप से 50 सें०मी० वर्षा एक दिन में होती है।
  2. ये चक्रवात जन-धन हानि व्यापक रूप से करते हैं।
  3. खाड़ी बंगाल में निम्न वायु दाब केन्द्र बनने से ये चक्रवात उत्पन्न होते हैं।
  4. ये चक्रवात एक दिन में पूर्वी तट से गुज़र कर प्रायद्वीप को पार करके पश्चिमी तट पर पहुँच जाते हैं।
  5. गोदावरी, कृष्णा, कावेरी डेल्टाओं में भारी हानि होती है।
  6. सुन्दरवन डेल्टा तथा बंगला देश में भी भारी हानि होती है।

प्रश्न 5.
(i) प्राकृतिक आपदायें किसे कहते हैं?
उत्तर:
पृथ्वी के धरातल पर आन्तरिक हलचलों द्वारा अनेक परिवर्तन होते रहते हैं। इनसे मानव पर हानिकारक प्रभाव पड़ते हैं। इन्हें प्राकृतिक आपदायें कहते हैं।

(ii) कुछ सामान्य आपदाओं के नाम बताएं।
उत्तर;
सामान्य आपदाएं इस प्रकार हैं- ज्वालामुखी विस्फोट, भूकम्प, सागरकम्प, सूखा, बाढ़, चक्रवात, मृदा अपरदन, अपवाहन, पंकप्रवाह, हिमधाव।

(iii) संकट किसे कहते हैं ?
उत्तर;
अंग्रेज़ी भाषा में प्राकृतिक आपदाओं को प्राकृतिक संकट भी कहा जाता है। फ्रैंच भाषा में डेस (Des) का अर्थ बुरा (bad) तथा (Aster) का अर्थ सितारे (Stars) से है। मानवीय जीवन और अर्थव्यवस्था को भारी हानि पहुँचाने वाली प्राकृतिक आपदाओं को संकट और महाविपत्ति कहते हैं।

(iv) भूकम्प का परिमाण क्या होता है?
उत्तर:
भूकम्प की शक्ति को रिक्टर पैमाने पर मापा जाता है, जिसे परिमाण कहते हैं । यह भूकम्प द्वारा विकसित भूकम्पीय उर्जा की माप होती है।

(v) भूकम्प की तीव्रता किसे कहते हैं?
उत्तर:
भूकम्प द्वारा होने वाली हानि की माप को तीव्रता कहते हैं।

(vi) भारत के अधिक तथा अत्यधिक भूकम्पीय खतरे वाले क्षेत्रों के नाम बताएं पूर्वी भारत,
उत्तर:
भूकम्प की दृष्टि से भारत के अत्यधिक खतरे वाले क्षेत्रों के नाम हैं- हिमालय पर्वत, उत्तर- कच्छ रत्नागिरी के आस-पास का पश्चिमी तटीय तथा अण्डमान और निकोबार द्वीप समूह। अधिक खतरे वाले क्षेत्र हैं- गंगा का मैदान, पश्चिमी राजस्थान।

(vii) चक्रवात की उत्पत्ति के लिए आधारभूत आवश्यकताएं कौन-सी हैं?
उत्तर:
-जब कमज़ोर रूप से विकसित कम दबाव क्षेत्र के चारों ओर तापमान की क्षैतिज प्रवणता बहुत अधिक होती है तब उष्ण कटिबन्धीय चक्रवात बन सकता है। चक्रवात उष्मा का इंजन है तथा इसे सागरीय तल से उष्मा मिलती है।

(viii) चक्रवात की गति और सामान्य अवधि कितनी होती है?
उत्तर:
चक्रवात की गति 150 km तथा अवधि एक सप्ताह तक होती है।

(ix) भारत के बाढ़ प्रवण क्षेत्रों के नाम बताएं।
उत्तर:

  1. गंगा बेसिन, उत्तर प्रदेश, बिहार तथा पश्चिमी बंगाल
  2. असाम में ब्रह्मपुत्र घाटी
  3. उड़ीसा प्रदेश।

(x) भू-स्खलन किसे कहते हैं?
उत्तर:
आधार शैलों का भारी मात्रा में तेज़ी से खिसकना भू-स्खलन कहलाता है। तीव्र पर्वतीय ढलानों पर भूकम्प के कारण अचानक शैलें खिसक जाती हैं।

(xi) आपदा प्रबन्धन किसे कहते हैं?
उत्तर:
आपदाओं से सुरक्षा के उपाय, तैयारी तथा प्रभाव को कम करने की क्रिया को आपदा प्रबन्धन कहते हैं। इसमें राहत कार्यों की व्यवस्था भी शामिल की जाती है।

JAC Class 11 Geography Important Questions Chapter 7 प्राकृतिक संकट तथा आपदाएँ

प्रश्न 6.
भिन्न-भिन्न क्षेत्रों में आए भू-स्खलन से निपटने के लिए आप कौन-से सुझाव देंगे?
उत्तर:

  1. अधिक भू-स्खलन सम्भावी क्षेत्रों में सड़क और बड़े बाँध बनाने जैसे निर्माण कार्य तथा प्रतिबंधित करने चाहिए।
  2. इस क्षेत्रों में कृषि नदी घाटी तथा कम ढाल वाले क्षेत्रों तक सीमित होनी चाहिए तथा बड़ी विकास परियोजनाओं पर नियन्त्रण होना चाहिए।
  3. स्थानान्तरी कृषि वाली उत्तर पूर्वी राज्यों (क्षेत्रों) में सीढ़ीनुमा खेत बनाकर खेती करनी चाहिए।

प्रश्न 7.
सूखे से बचाव के उपाय के कोई तीन कारण लिखें, जिनको हम अपनाकर सूखे के प्रभाव से बच सकते हैं?
उत्तर:

  1. सूखाग्रस्त क्षेत्रों में भूमिगत जल, वर्षा जल तथा धरातलीय जल को नष्ट होने से बचना चाहिए।
  2. जल सिंचाई की लघु परियोजनाओं पर अधिक जोर दिया जाना चाहिए।
  3. नहरों को पक्का करके पानी को नष्ट होने से बचाना चाहिए।

प्रश्न 8.
उत्तराखण्ड में आई प्राकृतिक आपदा का वर्णन करो।
उत्तर:
जून, 2013 में उत्तराखण्ड में हुई भारी वर्षा तथा बाढ़ के कारण अत्यन्त तबाही हुई। इसके कारण चल रही चाल धाम यात्रा को रोकना पड़ा 15-16 जून को अलकनंदा तथा मंदाकनी नदियों में बाढ़ के कारण नदियों ने अपने मार्ग बदल लिए। केदारनाथ धाम मन्दिर में झुके हज़ारों व्यक्ति मृत्यु को प्राप्त हो गए। बहुत से लोगों का विचार है कि बादलों के फटने से यह आपदा आई। भूस्खलन के कारण सड़क मार्ग बन्द हो गए। वायु सेना ने आपदा में फंसे लोगों की बचाया। सन् 2014 में केदारनाथ यात्रा का मार्ग खोल दिया गया है।

प्रश्न 9.
सुभेद्यता किसे कहते हैं?
उत्तर:
सुभेद्यता किसी व्यक्ति, व्यक्तियों के समूह या क्षेत्र में नुकसान पहुँचाने का भय है, जिससे वह व्यक्ति, व्यक्तियों के समूह या क्षेत्र प्रभावित होता है।

निबन्धात्मक प्रश्न (Essay Type Questions)

प्रश्न 1.
भारत में सूखाग्रस्त क्षेत्रों का वर्णन करते हुए इसके कारणों तथा प्रभाव का उल्लेख करो।
उत्तर:
सूखा (Drought): सूखा एक प्राकृतिक विपदा है। जब किसी विस्तृत क्षेत्र में खाद्यान्नों का अभाव हो जाता है तो उसे सूखा कहते हैं (Drought is defined as widespread and extreme scarcity of food.) इसके प्रभाव से जनसंख्या का अधिकांश भाग भुखमरी का शिकार हो जाता है । भारत तथा संसार के कई भागों में सूखा भयानक रूप में पड़ता रहा है जिससे लाखों लोग मृत्यु का शिकार हो जाते थे आधुनिक समय में सन् 1943 में बंगाल के अकाल से 15 लाख व्यक्ति भुखमरी से मर गए। आजकल यातायात के तीव्र साधनों द्वारा शीघ्र ही सहायता पहुंच जाने के कारण सूखा तथा अकाल पहले जैसे नहीं रहे।

इतिहास (History): भारत एक विशाल देश है जिसका क्षेत्रफल लगभग 33 करोड़ हेक्टेयर है तथा औसत वार्षिक वर्षा 117 सें०मी० तक है। अधिकतर वर्षा ग्रीष्मकाल में होती है। भारत की कृषि तथा अर्थव्यवस्था मानसून पवनों पर निर्भर करती है। मानसून वर्षा बहुत अनिश्चित तथा अनियमित है। वर्षा की परिवर्तिता (Variablity) के कारण भारत के किसी-न-किसी भाग में सूखे की हालत बनी ही रहती है। औसत रूप से भारत में प्रत्येक पांच वर्षों में एक वर्ष सूखे का होता है।

(On an average, one year in every five years is a drought year.) भारत में 1966, 1968, 1973, 1979 में भयंकर सूखा पड़ा। 1984-85 से 1987-88 तक निरन्तर तीन वर्ष सूखा पड़ने से भारत में खाद्यान्न के उत्पादन में कमी रही। 1987-88 के सूखे का प्रभाव 15 राज्यों तथा 6 संघ राज्यों पर पड़ा। इसके प्रभाव से 267 ज़िलों तथा 3 लाख गांवों में 28 करोड़ लोग तथा 17 करोड़ पशु प्रभाव ग्रस्त हुए। पिछले 100 वर्षों के इतिहास में भारत में सन् 1877, 1899, 1918, 1972 तथा 1987 के वर्षों में भयानक सूखा पड़ा।

प्रभावित क्षेत्र लाख वर्ग कि०मी०:

वर्ष प्रभावित क्षेत्र लाख वर्ग कि०मी० देश के क्षेत्रफल का $\%$ भाग
1877 20 61
1899 19 63
1918 22 70
1972 14 44
1987 16 50

JAC Class 11 Geography Important Questions Chapter 7 प्राकृतिक संकट तथा आपदाएँ 1

भारत में सूखाग्रस्त क्षेत्र (Drought Areas in India) – सामान्य स्थिति में भारत के 16% क्षेत्र तथा 12% जनसंख्या पर सूखे का प्रभाव पड़ता है। सूखे का अधिक प्रभाव उन क्षेत्रों पर पड़ता है जहां वर्षा की परिवर्तिता का गुणांक (Co-efficient of Variability of Rainfall) 20% से अधिक है । निम्नलिखित क्षेत्र प्रायः सूखाग्रस्त रहते हैं-

क्षेत्र राज्य क्षेत्रफल वार्षिक वर्षा
1. मरुस्थलीय तथा अर्द्ध-मरुस्थलीय प्रदेश राजस्थान, हरियाणा (दक्षिण-पश्चिमी) 60 लाख 10 सें॰मी०
2. पश्चिमी घाट के पूर्व में स्थित प्रदेश मध्य प्रदेश, गुजरात 37 , 15 सें॰मी०
3. अन्य क्षेत्र मध्यवर्ती महाराष्ट्र कर्नाटक, आन्ध्र प्रदेश, दक्षिण तमिलनाडु, कच्छ, बिहार, कालाहांडी (उड़ीसा), रायलसीमा (आन्ध्र प्रदेश) 10, 20 सें॰मी०

भारत में सामान्यतः सूखे की स्थिति निम्नलिखित क्षेत्रों में होती है।

  1. जब वार्षिक वर्षा 100 सें०मी० से कम हो।
  2. वर्षा की परिवर्तिता 75% से अधिक हो।
  3. जहां कुल क्षेत्रफल के 30% से कम भाग में जल सिंचाई प्राप्त हो।
  4. जहां 20% से अधिक वर्षा की परिवर्तिता का गुणांक हो तो सामान्य सूखा पड़ता है।
  5. जहां 40% से अधिक वर्षा की परिवर्तिता का गुणांक हो वहां स्थायी रूप से सूखा रहता है।

सूखे के कारण (Causes of Droughts ):
1. मानसून पवनों का कमज़ोर पड़ना (Weak Monsoons ):
भारत में अधिकतर कृषि क्षेत्र वर्षा पर निर्भर (Rainfed) है। ग्रीष्मकालीन मानसून पवनों के कमज़ोर पड़ने से सूखे की स्थिति उत्पन्न हो जाती है। मानसून पवनें जब निश्चित समय से देर से आरम्भ होती हैं तो फ़सलें नष्ट हो जाती हैं। सन् 1987 में मानसून पवनें सारे देश में 1 जुलाई की अपेक्षा 27 जुलाई को आरम्भ हुईं तथा देश के 35 जलवायु खण्डों में 25 में सामान्य से कम वर्षा हुई। इस से अधिकतर क्षेत्रों में सूखे की स्थिति उत्पन्न हो जाती है।

2. वार्षिक वर्षा का कम होना (Low Rainfall):
भारत के 12% क्षेत्रफल में वार्षिक वर्षा 60 सें०मी० से कम है। इन क्षेत्रों में सूखे की स्थिति बनी रहती है। एक अनुमान है कि भारत का 1/3 कृषि क्षेत्र वर्षा की कमी के कारण सूखाग्रस्त रहता है।

3. सिंचाई साधनों का कम होना (Inadequate means of Irrigation):
सिंचाई साधनों की कमी के कारण भी कई प्रदेशों में कृषि को नियमित जल न मिलने से सूखा पड़ता है।

4. पारिस्थितिक असन्तुलन (Ecological Imbalance):
औद्योगिक विकास, बढ़ती हुई जनसंख्या के कारण वृक्षों की अन्धाधुन्ध कटाई से पारिस्थितिक सन्तुलन बिगड़ता जा रहा है। इससे वर्षा अनियमित, अनिश्चित तथा संदिग्ध होती जा रही है।

5. वर्षा की परिवर्तिता (Variability of Rainfall):
कई प्रदेशों में वर्षा बहुत संदिग्ध है। विशेषकर कम वर्षा वाले क्षेत्रों में वर्षा की अधिक परिवर्तिता सूखे का कारण बनती है। उदाहरण के लिए हिसार नगर में अगस्त मास की औसत वर्षा 12.37 सें०मी० है। परन्तु सन् 1925 में इस मास में यहां केवल 2.03 सें०मी०, सन् 1926 से 56.36 सें०मी० वर्षा हुई। प्रायः जहां वर्षा का परिवर्तिता का गुणांक 40% से अधिक है वहां सदा सूखे की स्थिति रहती है।

6. मौसमी वर्षा (Seasonal Rainfall): शीत ऋतु के शुष्क होने के कारण भी सूखे की स्थिति बन जाती है।

7. नदियों का अभाव (Absence of Rivers ): कई प्रदेशों में नदियों के अभाव से भी सूखे में वृद्धि होती है।

8. लम्बी शुष्क ऋतु तथा उच्च तापमान (Long dry Speeds):
कई बार लम्बे समय तक शुष्क मौसम चलता रहता है। साथ-ही-साथ उच्च तापमान के कारण वाष्पीकरण भी अधिक हो जाता है जिससे सूखा भयंकर रूप धारण कर लेता है।

सूखे के प्रभाव (Effects of Drought ):

  1. कृषि (Agriculture): सूखे के कारण अधिकांश भागों में फसलों की बुआई देर से आरम्भ होती है। कई विशाल क्षेत्रों में बुआई बिल्कुल नहीं होती। इससे कृषि उत्पादन कम हो जाता है। 1987 में निर्धारित लक्ष्य में खाद्यान्नों का उत्पादन 60 लाख टन कम था।
  2. कृषि मज़दूर (Agricultural Labour ): ग्रामीण क्षेत्रों में बेरोज़गारी बढ़ जाती है। छोटे किसानों तथा कृषि मज़दूरों को कोई काम नहीं मिलता। भूमि मज़दूर भुखमरी का शिकार हो जाते हैं।
  3. खरीफ की फसल (Kharif Crop ): खरीफ की फसल जून – जुलाई में बोई जाती है। सूखे की हालत में इस फसल पर बुरा प्रभाव पड़ता है। इससे खाद्यान्न, तिलहन, दालों का उत्पादन कम हो जाता है।
  4. अर्थव्यवस्था पर प्रभाव (Effect on Economy ): सूखे की स्थिति में कीमतें बढ़ जाती हैं तथा अर्थव्यवस्था पर बुरा प्रभाव पड़ता है।
  5. पशुओं का चारा (Fodder for Cattle ): सूखे के कारण चारे की कमी होती है तथा हज़ारों पशु भुखमरी का शिकार हो जाते हैं।
  6. पानी की कमी (Shortage of Water ): पीने के पानी की कमी हो जाती है । जल सिंचाई तथा जल विद्युत् उत्पादन के लिए पानी की कमी हो जाती है। कई क्षेत्रों में भूमिगत जल स्तर नीचा हो जाता है।

सूखे से बचाव के उपाय (Measures to Control Drought): सरकार तथा जनता ने सूखे से बचाव के लिए निम्न उपाय किए हैं।

  1. सूखाग्रस्त क्षेत्रों में भूमिगत जल, वर्षा जल तथा धरातलीय जल को नष्ट होने से बचाया जा रहा है।
  2. फसलों के हेर-फेर की विधि द्वारा ऐसी फसलों की कृषि की जाती है जो सूखे को सहार सकें शुष्क कृषि पर अधिक जोर दिया जा रहा है।
  3. जल सिंचाई की लघु योजनाओं पर अधिक जोर दिया जा रहा है।
  4. नहरों को पक्का करके पानी को नष्ट होने से बचाया जा रहा है।
  5. शुष्क भागों में, ट्रिकल (Trickle) जल सिंचाई विधि का प्रयोग किया जा रहा है।
  6. सूखाग्रस्त क्षेत्रों में पीने का पानी तथा पशुओं के लिए चारे का उचित प्रबन्ध किया जा रहा है।

JAC Class 11 Geography Important Questions Chapter 7 प्राकृतिक संकट तथा आपदाएँ

प्रश्न 2.
भारत में बाढ़ग्रस्त क्षेत्रों का वर्णन करो। बाढ़ों के कारणों का उल्लेख करते हुए इनसे होने वाली क्षति का वर्णन करो। बाढ़ नियन्त्रण के उपाए बताओ।
उत्तर:
बाढ़ समस्या (Flood Problem):
सूखे की भान्ति बाढ़ भी एक प्राकृतिक विपदा है क्षेत्रों से धन-जन की हानि होती है। कई बार प्रत्येक वर्ष भारत के किसी-न-किसी भाग में बाढ़ों द्वारा विस्तृत एक भाग में भयानक सूखे की स्थिति है तो दूसरे भाग में बाढ़ की समस्या उत्पन्न हो जाती है। इससे समस्या अधिक गम्भीर हो जाती है। भारत में बाढ़ें एक मौसमी समस्या है जब मानसून की अनियमित वर्षा से नदियों में बाढ़ आ जाती है। जब नदी के किनारों के ऊपर से पानी बह कर समीपवर्ती क्षेत्रों में दूर-दूर तक फैल जाता है तो इसे बाढ़ का नाम दिया जाता है।

भारत ‘नदियों का देश’ है जहां अनेक छोटी-बड़ी नदियां बहती हैं। ये नदियां वर्षा ऋतु में भरपूर बहती हैं, परन्तु शुष्क ऋतु में इनमें बहुत कम जल होता है। निरन्तर भारी वर्षा के कारण बाढ़ें उत्पन्न होती हैं। वर्षा की तीव्रता तथा वर्षाकाल की अवधि अधिक होने से बाढ़ों को सहायता मिलती है। मानसून के पूर्व आरम्भ या देर तक समाप्त होने से
JAC Class 11 Geography Important Questions Chapter 7 प्राकृतिक संकट तथा आपदाएँ 3

बाढ़ें उत्पन्न होती हैं। ब्रह्मपुत्र नदी में मई-जून के मास में बाढ़ें साधारण बात है। उत्तरी भारत की नदियों में वर्षा ऋतु में बढ़ें आती हैं। नर्मदा नदी में अचानक बाढ़ें (flash floods ) आती हैं। तटीय भागों में चक्रवातों के कारण मई तथा अक्तूबर मास में भयानक बाढ़ें आती हैं। सन् 1990 में मई मास में आन्ध्र प्रदेश में खाड़ी बंगाल के चक्रवात से भारी क्षति हुई जिसमें लगभग 1000 व्यक्ति मर गए।

बाढ़ग्रस्त क्षेत्र (Flood Affected Areas):
भारत में मैदानी भाग तथा नदी घाटियों में अधिक बाढ़ें आती हैं। देश का लगभग 1/8 भाग बाढ़ों से प्रभावित रहता है। 60 प्रतिशत बाढ़ें अधिक वर्षा के कारण उत्पन्न होती हैं। असम, बिहार, जम्मू-कश्मीर, उत्तर प्रदेश तथा पश्चिमी बंगाल राज्य स्थायी रूप से बाढ़ग्रस्त रहते हैं। इन प्रदेशों में अधिक वर्षा तथा बड़ी-बड़ी नदियों के कारण बाढ़ समस्या गम्भीर है। एक अनुमान के अनुसार देश में 78 लाख हेक्टेयर भूमि पर प्रति वर्ष बाढ़ें आती हैं। नदी घाटियों के अनुसार बाढ़ क्षेत्रों को निम्नलिखित वर्गों में बांटा जाता है।

1. हिमालय क्षेत्र की नदियां (The Rivers of the Himalayas):
इस भाग में गंगा तथा ब्रह्मपुत्र दो प्रमुख नदियां हैं जिनमें प्रत्येक वर्ष बाढ़ें आती हैं। गंगा घाटी में यमुना, घाघरा, गंडक तथा कोसी जैसी सहायक नदियां शामिल हैं। इन नदियों में जल की मात्रा अधिक होती है। इनकी ढलान तीव्र होती है तथा इन नदियों के मार्ग में परिवर्तन होता रहता है। उत्तर प्रदेश तथा बिहार के विस्तृत क्षेत्रों में बाढ़ों से भारी क्षति पहुंचती है। देश में बाढ़ों से कुल क्षति का 33% भाग उत्तर प्रदेश में तथा 27% भाग बिहार में होता है। कोसी नदी को बाढ़ों के कारण “शोक की नदी” (River of Sorrow) कहा जाता है।

ब्रह्मपुत्र नदी असम, मेघालय तथा बंगलादेश में बाढ़ों से हानि पहुंचाती है। ब्रह्मपुत्र घाटी भारत में सबसे अधिक बाढ़ प्रभावित क्षेत्र है। यहां अधिक वर्षा तथा रेत व मिट्टी के जमाव से बाढ़ें उत्पन्न होती हैं। भूकम्प के आने के कारण नदियां अपना मार्ग बदल लेती हैं तथा बाढ़ समस्या अधिक गम्भीर हो जाती हैं। दामोदर घाटी में दामोदर नदी के कारण भयंकर बाढ़ें आती रही हैं । इस नदी को ‘बंगाल का शोक’ भी कहा जाता था परन्तु दामोदर घाटी योजना के पूरा होने के बाद बाढ़ समस्या कम हो गई है।

2. उत्तर-पश्चिमी भारत (North-Western India):
इस भाग में जम्मू कश्मीर, हिमाचल प्रदेश, पंजाब, पश्चिमी उत्तर प्रदेश शामिल हैं। यहां जेहलम, चिनाब, सतलुज, ब्यास तथा रावी नदियों के कारण बाढ़ें उत्पन्न होती हैं। बरसाती नदियों में भी बाढ़ें आती हैं।

3. मध्य भारत (Central India ):
इस भाग में मध्य प्रदेश, महाराष्ट्र, आन्ध्र प्रदेश तथा उड़ीसा शामिल हैं। यहां ताप्ती, नर्मदा तथा चम्बल नदियों में कभी-कभी बाढ़ें आती हैं। यहां अधिक वर्षा के कारण बाढ़ें उत्पन्न होती हैं।

4. प्रायद्वीपीय क्षेत्र (Peninsular Region ):
इस क्षेत्र में महानदी, गोदावरी, कृष्णा तथा कावेरी नदियों में उष्ण कटिबन्धीय चक्रवातों के कारण बाढ़ें आती हैं। कई बार ज्वार-भाटा के कारण डेल्टाई क्षेत्रों में रेत और मिट्टी के जमाव से भी बाढ़ें आती हैं।

बाढ़ों के कारण (Causes of Floods): भारत एक उष्ण कटिबन्धीय मानसूनी देश है। यहां मानसूनी वर्षा के अधिक होने से बाढ़ की समस्या गम्भीर हो जाती है। बाढ़ें निम्नलिखित कारणों से आती हैं।

  1. भारी वर्षा (Heavy Rainfall): किसी भाग में एक दिन में निरन्तर वर्षा की मात्रा 15 सें० मी० से अधिक होने से बाढ़ की स्थिति उत्पन्न हो जाती है।
  2. चक्रवात (Cyclones): भारत के पूर्वी तट पर खाड़ी बंगाल के तीव्र गति के चक्रवातों से भयानक बाढ़ें आती हैं। जैसे – मई, 1990 में आन्ध्र प्रदेश में चक्रवातों द्वारा निरन्तर वर्षा से नदी क्षेत्रों में बाढ़ उत्पन्न होने से भारी हानि हुई।
  3. वनों की कटाई (Deforestation ): नदियों के ऊपरी भागों में वृक्षों की अंधाधुंध कटाई से अचानक बाढ़ें उत्पन्न हो जाती हैं। शिवालिक की पहाड़ियों, असम, मेघालय तथा छोटा नागपुर के पठार में वृक्षों की कटाई के कारण बाढ़ की समस्या गम्भीर है।
  4. नदी तल का ऊंचा उठना (Rising of the River Bed ): रेत तथा बजरी जमाव से नदी तल ऊंचा उठ जाता है जिससे समीपवर्ती क्षेत्रों में बाढ़ का जल फैल जाता है।
  5. अपर्याप्त जल प्रवाह (Inadequate Drainage ): कई निम्न क्षेत्रों में जल प्रवाह प्रबन्ध न होने से बाढ़ें उत्पन्न हो जाती हैं।

बाढ़ों से क्षति (Damage due to Floods)”:
बाढ़ों से कृषि क्षेत्र में फसलों की हानि होती है। मकानों, संचार साधनों तथा रेलों, सड़कों को क्षति पहुंचती है। बाढ़ग्रस्त क्षेत्रों में कई बीमारियां फैल जाती हैं। देश में लगभग 2 करोड़ हेक्टेयर भूमि बाढ़ग्रस्त क्षेत्र है जिसमें से 25 लाख हेक्टेयर भूमि में फसलें नष्ट हो जाती हैं। प्रति वर्ष औसत रूप से करोड़ जनसंख्या पर बाढ़ से क्षति का प्रभाव पड़ता है। लगभग 30 हज़ार पशुओं की हानि होती है। एक अनुमान है कि औसत रूप से प्रति वर्ष 505 व्यक्तियों की बाढ़ के कारण मृत्यु हो जाती है। इस प्रकार देश में लगभग ₹1500 करोड़े की आर्थिक क्षति पहुंचती है। सन् 1990 में देश में कुल क्षति ₹ 41.25 करोड़ की थी तथा 50 लाख हेक्टेयर भूमि बाढ़ग्रस्त 162 लाख लोग बाढ़ से प्रभावित हुए। ₹28 करोड़ की फसलें नष्ट हुईं। 862 जानें गईं तथा 1,22,498 पशु मारे गये।

बाढ़ों की रोकथाम (Flood Control):
भारत में प्राचीन समय से बाढ़ों की रोकथाम के लिए उपाय किए जाते हैं। प्रायः नदियों के साथ-साथ तटबंध बनाकर बाढ़ नियन्त्रण किया जाता था। सन् 1954 में राष्ट्रीय बाढ़ नियन्त्रण योजना शुरू की गई। इस योजना के अधीन बाढ़ नियन्त्रण के लिए कई उपाय किए गए।

  1. नदियों के जल सम्बन्धी आंकड़े इकट्ठे किए गए।
  2. नदियों के साथ तटबन्ध बनाये गये। देश में लगभग 15,467 कि० मि० लम्बे तटबन्धों का निर्माण किया गया।
  3. निम्न क्षेत्रों में लगभग 30,199 कि० मी० लम्बी जल प्रवाह नलिकायें बनाई गई हैं।
  4. 762 नगरों तथा 4,700 गांवों को बाढ़ों से सुरक्षित किया गया है।
  5. कई नदियों पर जलाशय बन कर बाढ़ों पर नियन्त्रण किया गया है; जैसे- दामोदर घाटी बहुमुखी योजना तथा भाखड़ा नंगल योजना ।
  6. देश में बाढ़ों का पूर्व अनुमान लगाने के लिए (Flood Forecasting) 157 केन्द्र स्थापित किए गए हैं।
  7. नदियों के ऊपरी भागों में वन रोपण किया गया है।
  8. सातवीं पंचवर्षीय योजना के अन्त तक 2710 करोड़ बाढ़ नियन्त्रण पर व्यय किए गए जबकि आठवीं पंचवर्षीय योजना पर ₹9470 करोड़ के व्यय का अनुमान है।
  9. केरल तट पर सागरीय प्रभाव से बचाव के लिए 42 कि० मी० लम्बी समुद्री दीवारों का निर्माण किया गया तथा कर्नाटक तट पर 73 कि० मी० लम्बी समुद्री दीवारें बनाई गईं।
  10. देश में बाढ़ के पूर्व निर्माण संगठन (Flood Fore-casting Organisation) की स्थापना की गई है। इसके अधीन 157 केन्द्र स्थापित किए गए हैं जिनकी संख्या इस शताब्दी के अन्त तक 300 हो जाएगी।
  11. महानदी घाटी में हीराकुड बांध, दामोदर घाटी में कई बांध, सतलुज नदी पर भाखड़ा डैम, ब्यास नदी पर पौंग डैम तथा ताप्ती नदी पर डकई बांध बनाकर बाढ़ों की रोकथाम की गई है।

JAC Class 11 Geography Important Questions Chapter 7 प्राकृतिक संकट तथा आपदाएँ

प्रश्न 3.
भूकम्प की परिभाषा दो भारत में भूकम्प क्षेत्रों के वितरण का वर्णन करो।
उत्तर:
भूकम्प (Earthquake ):
पृथ्वी के किसी भाग के अचानक हिलने को भूकम्प कहते हैं। इस हलचल से भूपृष्ठ पर झटके (Tremors) अनुभव किए जाते हैं। भूकम्पीय तरंगें सभी दिशाओं में लहरों की भान्ति आगे बढ़ती हैं। ये तरंगें उद्गम (Focus ) से आरम्भ होती हैं। ये तरंगें तीन प्रकार की होती हैं – P- Waves, S- Waves, L-Waves.

भूकम्प के कारण (Causes of Earthquake ):
भूकम्प के सामान्य कारण ज्वालामुखी विस्फोट, भू-हलचलें, चट्टानों का लचीलापन तथा स्थानीय कारण है। आधुनिक युग में भूकम्पों को टेकटानिक प्लेटों से सम्बन्धित किया गया है। भारत में सामान्य रूप से भारतीय प्लेट तथा यूरेशियन प्लेट आपस में टकराती हैं। ये एक दूसरे के नीचे धँसने का यत्न करती हैं। हिमालय पर्वतीय क्षेत्र में इनका सम्बन्ध वलन व भ्रंशन क्रिया से है। दक्षिणी भारत एक स्थिर भूखण्ड है तथा भूकम्प बहुत कम होते हैं। भूकम्पों की तीव्रता रिक्टर पैमाने से मापी जाती है जिसका मापक 1 से 9 तक होता है। अधिक तीव्र भूकम्प भारत के निम्नलिखित क्षेत्रों में अनुभव किए जाते हैं।
JAC Class 11 Geography Important Questions Chapter 7 प्राकृतिक संकट तथा आपदाएँ 4

1. हिमालयाई क्षेत्र (Himalayan Zone ):
इस क्षेत्र में क्रियाशील भूकम्प जम्मू-कश्मीर, हिमाचल प्रदेश, उत्तराखण्ड तथा उत्तर-पूर्वी राज्यों में आते हैं जिनसे बहुत हानि होती है। यह भूकम्प भारतीय प्लेट तथा यूरेशियम प्लेट के आपसी टकराव के कारण उत्पन्न होते हैं। भारतीय प्लेट प्रति वर्ष 5 सें० मी० की गति से उत्तर तथा उत्तर-पूर्व की ओर बढ़ रही है। यहां 1905 में कांगड़ा में, 1828 में कश्मीर में, 1936 में क्वेटा में तथा 1950 में असम में भयानक भूकम्प अनुभव किए गए।

2. सिन्धु-गंगा प्रदेश (Indo-Gangetion Zone ):
इस क्षेत्र में सामान्य तीव्रता के भूकम्प अनुभव किए जाते हैं। इनकी तीव्रता 6 से 6.5 तक होती है। परन्तु इन सघन बसे क्षेत्रों में बहुत हानि होती है।

3. प्रायद्वीपीय क्षेत्र (Peninsular Zone):
यह एक स्थिर क्षेत्र है परन्तु फिर भी यहां भूकम्प अनुभव किए जाते हैं। 1967 में कोयना, 1993 में लातूर, 2001 में भुज के भूकम्प बहुत विनाशकारी थे। कोयना भूकम्प कोयला डैम के जलाशय में जल के अत्यधिक दबाव के कारण आया। परन्तु वर्तमान भूकम्प भारतीय प्लेट की उतर की ओर गति के कारण आए हैं।

4. अन्य भूकम्पीय क्षेत्र (Other Sesonic Zones)

  1. बिहार – नेपाल क्षेत्र
  2. उत्तर-पश्चिमी हिमालय
  3. गुजरात क्षेत्र
  4. कोयना क्षेत्र।

भारत के प्रमुख विनाशकारी भूकम्प

केन्द्र तीव्रता वर्ष
कच्छ 8.0 1819
कच्छ 7.5 1869
मेघालय 8.7 1865
बंगाल 8.5 1885
असम 8.0 1897
कांगड़ा 8.0 1905
असम 8.7 1950
कोयना 6.3 1967
हिमाचल प्रदेश 7.5 1973
लातूर 6.0 1993
भुज 8.0 2001

भूकम्प के परिणाम:
केवल बसे हुए क्षेत्रों के आने वला भूकम्प ही आपदा या संकट बनता है। भूकम्प का प्रभाव सदैव विध्वंसक होता है। भूकम्प के कारण प्राकृतिक पर्यावरण में कई तरह से परिवर्तन हो जाते हैं। भूकम्पीय तरंगों से धरातल में दरारें पड़ जाती हैं जिनसे कभी-कभी पानी के फव्वारे छूटने लगते हैं। इसके साथ बड़ी भारी मात्रा में रेत बाहर आ जाता है तथा इससे रेत के बांध बन जाते हैं। क्षेत्र के अपवाह तन्त्र में उल्लेखनीय परिवर्तन भी देखे जा सकते हैं। नदियों के मार्ग बदल जाने से बाढ़ आ जाती है।

पहाड़ी क्षेत्रों में भू-स्खलन हो जाते हैं तथा इनके साथ भारी मात्रा में चट्टानी मलबा नीचे आ जाता है। इससे बृहतक्षरण होता है। हिमानियाँ फट जाती हैं तथा इनके हिमधाव सुदूर स्थित स्थानों पर बिखर जाते हैं। नए जल प्रपातों और सरिताओं की उत्पत्ति भी हो जाती है। भूकम्पीय आपदाओं से मनुष्य निर्मित भवन बच नहीं पाते हैं। सड़कें, रेलमार्ग, पुल और टेलीफोन की लाइनें टूट जाती हैं। गगनचुम्बी भवनों और सघन जनसंख्या वाले कस्बों और नगरों पर भूकम्पों का सबसे बुरा असर होता है।

सुनामी लहरें (Tsunami Tidal Waves):
समुद्री तली पर भूकम्प उत्पन्न होने से 30 मीटर तक ऊंची ज्वारीय लहरें (सुनामी) उत्पन्न होती है। 26 दिसम्बर, 2004 को हिन्द महासागर में इण्डोनेशिया के निकट उत्पन्न भूकम्प के कारण भयंकर सुनामी लहरें उत्पन्न हुईं। इनका प्रभाव इण्डोनेशिया, थाइलैण्ड, म्यानमार, भारत तथा श्रीलंका के तटों पर अनुभव किया गया। इन भयंकर लहरों के कारण इन क्षेत्रों में लगभग 2 लाख लोगों की जानें गईं तथा करोड़ों रुपयों की सम्पत्ति की हानि हुई है।

यह पृथ्वी के इतिहास में सबसे भयंकर प्राकृतिक आपदा थी भूकम्प के प्रभाव को कम करना भूकम्प के प्रभाव को कम करने का सबसे अच्छा तरीका हैं । इसकी निरन्तर खोज-खबर रखना तथा लोगों को इसके आने की सम्भावना की सूचना देना इससे आशंकित क्षेत्रों से लोगों को हटाया जा सकता है। भूकम्प से अत्यधिक खतरे वाले क्षेत्र में भूकम्प रोधी भवन बनाने की आवश्यकता है। भूकम्प की आशंका वाले क्षेत्रों में लोगों को भूकम्प रोधी भवन और मकान बनाने की सलाह दी जा सकती है।

JAC Class 11 Geography Important Questions Chapter 7 प्राकृतिक संकट तथा आपदाएँ

प्रश्न 4.
चक्रवात किसे कहते हैं? चक्रवातों द्वारा क्षति का वर्णन करो।
उत्तर:
चक्रवात (Cyclones):
600 कि०मी० या इससे अधिक व्यास वाले चक्रवात, पृथ्वी के वायुमण्डलीय तूफानों में सबसे अधिक विनाशक और भयंकर होते हैं। भारतीय उपमहाद्वीप संसार में चक्रवातों द्वारा सबसे अधिक दुष्प्रभावित क्षेत्र हैं। संसार में आने वाले चक्रवातों में से 6 प्रतिशत यही आते हैं।
उत्पत्ति: जब कमज़ोर रूप से विकसित कम दबाव के क्षेत्र के चारों ओर तापमान की क्षैतिज प्रवणता बहुत अधिक होती है, तब उष्ण कटिबंधीय चक्रवात बन सकता है। चक्रवात ऊष्मा का इंजिन है तथा इसे सागरीय तल से ऊष्मा मिलती है। संघनन के बाद मुक्त ऊष्मा, चक्रवात के लिए गतिज ऊर्जा (Kinetic energy) में बदल जाती है।

चक्रवात की उत्पत्ति की निम्नलिखित अवस्थाएं हैं।

  1. महासागरीय तल का तापमान 26° से अधिक।
  2. बन्द समदाब रेखाओं का आविर्भाव।
  3. निम्न वायुदाब, 1,000 मि。बा० से कम होना।
  4. चक्रीय गति के क्षेत्रफल, प्रारम्भ में इनके अर्धव्यास 30 से 50 कि०मी० फिर क्रमश: 100-200 कि०मी० और 1,000 कि०मी० तक भी बढ़ जाते हैं।
  5. ऊर्ध्वाधर रूप में पवन की गति का प्रारम्भ में 6 कि०मी० की ऊंचाई तक बढ़ना तथा इसके बाद और भी ऊंचा उठाना।

ऊष्ण कटिबंधीय चक्रवात की संरचना:

  1. ऊष्ण कटिबंधीय चक्रवातों में बहुत अधिक दाब प्रवणता (14-17 मि०बा० / 100 कि०मी०) होती है। कुछ चक्रवातों में यह इससे भी अधिक ऊंची अर्थात् 60 मि० बा० / 100 कि०मी० होती है।
  2. पवन पट्टी केन्द्र से 10 से 150 कि०मी० या कभी – कभी इससे भी अधिक दूरी में फैली होती है। धरातल पर पवन का चक्रवातीय परिसंचरण होता है। तथा ऊंचाई पर यह प्रति चक्रवातीय बन जाता है।
  3. ऊष्ण कटिबंधीय चक्रवातों की क्रोड कोष्ण होती है। चक्रवात का केन्द्र सामान्यतः मेघ विहीन होता है। इसे चक्रवात की आंख कहते हैं। चक्रवात की आंख बहुत ऊंचाई तक फैले ऊर्ध्वाधर बादलों से घिरी होती है।
  4. ऊष्ण कटिबंधीय चक्रवात से सामान्यतः 50 सें०मी० से अधिक वर्षा होती है। कभी-कभी वर्षा 100 सें०मी० से भी अधिक हो जाती है।
  5. चक्रवात अपने पूरे तन्त्र के साथ लगभग 20 कि०मी० प्रति घंटा औसत गति से आगे बढ़ता है। जैसे-जैसे चक्रवात स्थल पर बढ़ता जाता है, समुद्री जल के अभाव में इसकी ऊर्जा घटती जाती है। इससे चक्रवात समाप्त हो जाता है। चक्रवात की जीवन अवधि 5 से 7 दिनों की होती है।

चक्रवातों द्वारा क्षति:
प्रभंजन की गति वाली पवनों, प्रभंजन की लहरों तथा मूसलाधार वर्षा से उत्पन्न बाढ़ों के कारण ऊष्ण कटिबंधीय चक्रवातों का विनाशकारी प्रभाव पड़ता है। अधिकतर तूफ़ान अत्यन्त तेज़ पवनों और तूफ़ानी लहरों के द्वारा भारी क्षति पहुंचाते हैं। पर्वतीय क्षेत्रों में ढाल पर अत्यन्त तीव्रता से बहने वाला वर्षा जल अपने सामने आने वाली हर वस्तु को अपनी चपेट में लेकर भारी नुकसान करता है। तूफ़ानी लहरों की तीव्रता, पवन की गति, दाब प्रवणता, समुद्र की तली की स्थलाकृतियों तथा तटरेखा की बनावट पर निर्भर करती है। अनेक क्षेत्रों में चक्रवातों की चेतावनी व्यवस्था के बावजूद, ऊष्ण कटिबंधीय चक्रवात धन-जन को अपार क्षति पहुंचाते हैं।

क्षेत्र: अरब सागर की तुलना में बंगाल की खाड़ी में तूफ़ानों की संख्या कहीं अधिक है। बंगाल की खाड़ी और अरब सागर में अधिकतर तूफान अक्तूबर और नवम्बर के महीनों में आते हैं। मानसून ऋतु का प्रारंभिक भाग भी बंगाल की खाड़ी और अरब सागर में ऊष्ण कटिबंधीय तूफ़ानों की उत्पत्ति के अनुकूल है। मानसून ऋतु में अधिकतर चक्रवात 10° उ० तथा 15° उ० अक्षांशों के मध्य ही उत्पन्न होते हैं। जून में बंगाल की खाड़ी के लगभग सभी तूफ़ान 92° पू० देशांतर के पश्चिम में 16° उ० और 21° उ० अक्षांश के मध्य जन्म लेते हैं। जुलाई में खाड़ी के तूफ़ानों का जन्म 18° 3० अक्षांश के उत्तर में तथा 90° पू० देशान्तर के पश्चिम में होता है। यह भी ध्यान देने योग्य बात है कि जुलाई के सभी तूफ़ान पश्चिमी पथ का अनुसरण करते हैं। ये सामान्यत: 20° उ० तथा 25° उ० अक्षांशों के मध्य तक ही सीमित रहते हैं तथा हिमालय की गिरिपद पहाड़ियों की ओर अपेक्षाकृत बहुत कम मुड़ते हैं।

क्षति का प्रभाव कम करना:
अधिकतर चक्रवातीय क्षति, तेज़ पवनों, मूसलाधार वर्षा और समुद्र में उठने वाली ऊँची तूफ़ानी, ज्वारीय लहरों के द्वारा होती है। पवनों की तुलना में चक्रवातीय वर्षा के कारण आई बाढ़ अधिक विनाशकारी होती है। आज चक्रवातों की चेतावनी व्यवस्था में उल्लेखनीय सुधार होने से तथा पर्याप्त और सामयिक कार्यवाही से चक्रवात से मरने वालों की संख्या में कमी आई है। अन्य उपाय जैसे : चक्रवातों के आने के समय सुरक्षा के लिए आय स्थलों के तटबंधों, बांधों, जलाशयों के निर्माण से और तट पर वन रोपण से भी बहुत सहायता मिलती है। फ़सलों और गो- पशुओं बी से भी लोगों को क्षति पूर्ति में काफ़ी मदद मिलती है। उपग्रहों से प्राप्त चित्रों के द्वारा चक्रवात के पथ के बारे में चेतावनी देना अब सम्भव हो गया है। कम्प्यूटर द्वारा बनाए गए मॉडलों की सहायता से चक्रवात की पवनों की दिशा और तीव्रता तथा इसके पथ की दिशा की काफ़ी हद तक सही भविष्यवाणी की जा सकती है।

प्रश्न 5.
आपदा प्रबन्धन पर एक लेख लिखें।
उत्तर:
आपदा प्रबन्धन ( Disaster Management ):
आपदा प्रबन्धन में निवारक और संरक्षी उपाय, तैयारी तथा मानवों पर आपदा के प्रभाव को कम करने के लिए राहत कार्यों की व्यवस्था तथा आपदा प्रवण क्षेत्रों के सामाजिक, आर्थिक पक्ष शामिल किए जाते हैं। आपदा प्रबन्ध की सम्पूर्ण प्रक्रिया को तीन चरणों में विभाजित किया जा सकता है। प्रभाव चरण, पुनर्वास और पुनर्निर्माण चरण तथा समन्वित दीर्घकालीन विकास और तैयारी चरण। प्रभाव चरण के तीन अंग हैं।

  1. आपदा की भविष्यवाणी करना,
  2. आपदा के प्रेरक कारकों की बारीकी से खोजबीन, तथा
  3. आपदा आने के बाद प्रबन्धन के कार्य जलग्रहण क्षेत्र में हुई वर्षा का अध्ययन करके बाढ़ की भविष्यवाणी की जा सकती है।

उपग्रहों के द्वारा चक्रवातों के मार्ग, गति आदि की खोज-खबर ली जा सकती है। इस प्रकार प्राप्त सूचनाओं के आधार पर पूर्व चेतावनी तथा लोगों को सुरक्षित स्थान पर पहुंचाने के प्रयत्न शुरू किए जा सकते हैं। आपदा के लिए ज़िम्मेदार कारकों की बारीकी से की गई खोजबीन लोगों को सुरक्षित स्थानों पर पहुंचाने, भोजन, वस्त्र और पेय जल की आपूर्ति के लिए कार्यदल नियुक्त किए जा सकते हैं। आपदाएँ मृत्यु और विनाश के चिह्न छोड़ जाती हैं। प्रभावित लोगों को चिकित्सा सुविधा और अन्य विभिन्न प्रकार की सहायता की ज़रूरत होती है। दीर्घकालीन विकास के चरण के अन्तर्गत विविध प्रकार के निवारक और सुरक्षापायों की योजना बना लेनी चाहिए। संसार के लोगों का ध्यान आकर्षित करने के लिए यूनेस्को ने 1990-2000 के दौरान प्राकृतिक आपदा राहत दशक मनाया था। संसार के अन्य देशों के साथ भारत ने भी दशक के दौरान अक्तूबर में विश्व आपदा राहत दिवस मनाया था। इस अवसर पर भूकंप, बाढ़ और चक्रवात प्रवण क्षेत्रों के लोगों के लिए भारत सरकार ने जो करणीय और अकरणीय कर्म प्रचारित किए थे, वे बहुत उपयोगी हैं।

JAC Class 11 Geography Important Questions Chapter 7 प्राकृतिक संकट तथा आपदाएँ

प्रश्न 6.
सुनामी से क्या अभिप्राय हैं? इसकी उत्पत्ति कैसे होती है? 26 दिसम्बर, 2004 को हुए सुनामी संकट के प्रभाव बताओ।
उत्तर:
सुनामी (Tsunami ):
सुनामी एक जापानी भाषा का शब्द है जिसका अर्थ है ‘तटीय तरंगें’, ‘Tsu’ शुद्ध का अर्थ है-तट तथा ‘Nami’ शब्द का अर्थ है ‘तरंगें ‘। इसे प्रायः ज्वारीय लहरें (Tidal waves) या भूकम्पीय तरंगें (Seismic waves) भी कहा जाता है।
सुनामी अचानक ही ऊंचा उठने वाली विनाशकारी तरंगें हैं। इससे गहरे जल में हलचल होती है। इसकी ऊंचाई प्रायः 10 मीटर तक होती है। सुनामी उस दशा में उत्पन्न होती है जब सागरीय तली में भूकम्पीय क्रिया के कारण हल चल होती है तथा महासागर में सतह के जल का लम्बरूप में विस्थापन होता है। हिन्द महासागर में सुनामी तरंगें बहुत कम अनुभव की गई हैं। अधिकतर सुनामी प्रशान्त महासागर में घटित होती हैं।

सुनामी की उत्पत्ति (Origin of Tsunami ):
पृथ्वी आंतरिक दृष्टि से एक क्रियाशील ग्रह है। अधिकतर भूकम्प विवर्तनिक प्लेटों (Tectomic plates) की सीमाओं पर उत्पन्न होते हैं। सुनामी अधिकतर प्रविष्ठन क्षेत्र (Subduction zone) के भूकम्प के कारण उत्पन्न होती है। यह एक ऐसा क्षेत्र हैं दो प्लेटें एक दूसरे में विलीन (converge) होती हैं। भारी पदार्थों से बनी प्लेट हल्की प्लेट के नीचे खिसक जाती है। समुद्र अधस्तल का विस्तारण होता है। यह क्रिया एक कम गहरे भूकम्प को जन्म देती है।
JAC Class 11 Geography Important Questions Chapter 7 प्राकृतिक संकट तथा आपदाएँ 6
26 दिसम्बर, 2004 की सुनामी आपदा (Tsunami Disaster of 26th December, 2004 ):
प्रात: 7.58 बजे के समय पर, काले रविवार (Black Sunday) को 26 दिसम्बर, 2004 को क्रिस्मस से एक दिन बाद सुनामी त्रासदी घटी यह विशाल, विनाशकारी सुनामी लहर हिन्द महासागर के तटीय प्रदेशों से टकराई। इस लहर के कारण इण्डोनेशिया से लेकर भारत तक के देशों में 3 लाख व्यक्ति इस त्रासदी का शिकार हो गए। महासागरी तली पर उत्पन्न एक भूकम्प उत्पन्न हुआ जिसका अधिकेंदर सुमात्रा (इण्डोनेशिया) ने 257 कि०मी० दक्षिण पूर्व में था। यह भूकम्प रिक्टर पैमाने पर 8.9 शक्ति का था।

इन लहरों के ऊंचे उठने से जल की एक ऊंची दीवार उत्पन्न हो गई। आधुनिक युग के इतिहास में यह एक महान् त्रासदी के रूप में अंकित की जाएगी। सन् 1900 के पश्चात् यह चौथा बड़ा भूकम्प था। इस भूकम्प के कारण उत्पन्न सुनामी लहरों से हीरोशिमा बम्ब की तुलना में लाखों गुणा अधिक ऊर्जा का विस्फोट हुआ। इसलिए इसे भूकम्प प्रेरित प्रलयकारी लहर भी कहा जाता है। यह भारतीय तथा बर्मा की प्लेटों के मिलन स्थान पर घटी जहां लगभग 1000 कि०मी० प्लेट सीमा खिसक गई।
JAC Class 11 Geography Important Questions Chapter 7 प्राकृतिक संकट तथा आपदाएँ 7

इसके प्रभाव से सागर तल 10 मीटर ऊंचा उठ गया तथा ऊपरी जल हज़ारों घन मीटर की मात्रा में विस्थापित हो गया । इसकी गति लगभग 700 कि०मी० प्रति घण्टा थी । इसे अपने उद्गम स्थान से भारतीय तट तक पहुंचने में दो घण्टे का समय लगा। इस त्रासदी ने तटीय प्रदेशों के इतिहास तथा भूगोल को बदल कर रख दिया है।

सुनामी त्रासदी के प्रभाव (Effects of Tsunami Disaster ):
हिन्द महासागर के तटीय देशों इण्डोनेशिया, मलेशिया, थाइलैंड, म्यांमार, भारत, श्रीलंका तथा मालदीव में विनाशकारी प्रभाव पड़े। भारत में सब से अधिक प्रभावित तमिलनाडु, पांडिचेरी, आन्ध्र प्रदेश, केरल राज्य थे अण्डमान तथा निकोबार द्वीप में इस लहर का सब से अधिक प्रभाव पड़ा। इण्डोनेशिया में लगभग 1 लाख व्यक्ति, थाइलैंड में 10,000 व्यक्ति, श्रीलंका में 30,000 व्यक्ति तथा भारत में 15,000 व्यक्ति इस प्रलय के शिकार हुए।

भारत में सब से अधिक क्षति तमिलनाडु के नागापट्टनम जिले में हुई जहां जल नगर में 1.5 कि०मी० अन्दर तक घुस गया। संचार, परिवहन साधन तथा विद्युत् सप्लाई में विघ्न पड़ा बहुत से श्रद्धालु वेलान कन्नी (Velan Kanni) के पुलिन (Beach) पर सागर जल में वह गए। विनाशकारी तट लौटती लहरें हज़ारों लोगों को वहा कर ले गईं। मैरीन पुलिन (एशिया के सब से बड़े पुलिन) पर 3 कि०मी० लम्बे क्षेत्र में सैंकड़ों लोग सागर की लपेट में आ गए। यहां लाखों रुपयों के चल-अंचल संसाधनों की बर्बादी हुई।
JAC Class 11 Geography Important Questions Chapter 7 प्राकृतिक संकट तथा आपदाएँ 5
कल्पाक्कम अणु शक्ति घर में जल प्रवेश करने से अणु शक्ति के रीएक्टरों को बन्द करना पड़ा। मामलापुरम के विश्व प्रसिद्ध मन्दिर को तूफ़ानी लहरों से बहुत क्षति हुई।

सब से अधिक मौतें अण्डमान-निकोबार द्वीप पर हुईं। ग्रेट निकोबार के दक्षिणी द्वीप पर जो कि भूकम्प के अधिकेन्द्र से केवल 150 कि०मी० दूर था, सब से अधिक प्रभाव पड़ा। निकोबार द्वीप पर भारतीय नौ सेना का एक अड्डा नष्ट हो गया। ऐसा लगता है कि इन द्वीपों का बहुत-सा क्षेत्र समुद्र ने निगल लिया है। इस प्रकार सुनामी लहरों ने इन द्वीप समूहों के भूगोल को बदल दिया तथा यहां पुनः मानचित्रण करना पड़ेगा। इस देश के आपदाओं के शब्द कोश में एक नया शद्ध सुनामी आपदा जुड़ गया है। अमेरिकन वैज्ञानिकों के अनुसार इस कारण पृथ्वी अपनी धुरी से डगमगा गई और उसका परिभ्रमण तेज़ हो गया जिससे दिन हमेशा के लिए एक सैकिंड कम हो गया। सुनामी लहरें सचमुच प्रकृति का कहर हैं।

JAC Class 11 Geography Important Questions Chapter 3 अपवाह तंत्र

Jharkhand Board JAC Class 11 Geography Important Questions Chapter 3 अपवाह तंत्र Important Questions and Answers.

JAC Board Class 11 Geography Important Questions Chapter 3 अपवाह तंत्र

बहु-विकल्पी प्रश्न (Multiple Choice Questions)

प्रश्न – दिए गए चार वैकल्पिक उत्तरों में से सही उत्तर चुनिए
1. भारत की सबसे बड़ी नदी कौन-सी है?
(A) यमुना
(B) गंगा
(C) ब्रह्मपुत्र
(D) गोदावरी।
उत्तर:
(B) गंगा।

2. वृक्ष की शाखाओं के समान कौन-सा जल प्रवाह है?
(A) केन्द्रभिमुख
(B) आरीय
(C) द्रुमाकृतिक
(D) जालीनुमा।
उत्तर:
(C) द्रुमाकृतिक।

3. गंगा तथा यमुना का संगम स्थान कहां है?
(A) कानपुर
(B) वाराणसी
(C) पटना
(D) इलाहाबाद।
उत्तर:
(D) इलाहाबाद।

4. सुन्दर वन डेल्टा किन नदियों द्वारा बनता है?
(A) गंगा
(B) कावेरी
(C) गोदावरी
(D) नर्मदा।
उत्तर:
(A) गंगा।

JAC Class 11 Geography Important Questions Chapter 3 अपवाह तंत्र

5. ट्रांस – हिमालयाई नदी कौन-सी है?
(A) गंगा
(B) चम्बल
(C) सतलुज
(D) ब्यास।
उत्तर:
(C) सतलुज।

6. प्रायद्वीपीय भारत की नदियां कहां से निकलती हैं?
(A) विन्ध्याचल
(B) पश्चिमी घाट
(C) पूर्वी घाट
(D) सतपुड़ा।
उत्तर:
(B) पश्चिमी घाट

7. किस नदी को दक्षिण की गंगा कहते हैं?
(A) महानदी
(B) गोदावरी
(C) कृष्णा
(D) कावेरी ।
उत्तर:
(B) गोदावरी।

8. किस नदी पर शिव समुद्रम जलप्रपात स्थित है?
(A) महानदी
(B) गोदावरी
(C) कावेरी
(D) नर्मदा।
उत्तर:
(C) कावेरी।

9. उड़ीसा राज्य में कौन-सी झील स्थित है?
(A) चिल्का
(B) सांभर
(C) वैवनाद
(D) कोलेरु।
उत्तर:
(A) चिल्का।

10. नर्मदा नदी का उद्गम कहां है ?
(A) सतपुड़ा
(B) अमरकण्टक
(C) ब्रह्मगिरि
(D) गोबिन्दसागर।
उत्तर:
(B) अमरकण्टक।

JAC Class 11 Geography Important Questions Chapter 3 अपवाह तंत्र

11. प्रायद्वीपीय भारत की सबसे लम्बी नदी है
(A) नर्मदा
(B) गोदावरी
(C) कृष्णा
(D) महानदी।
उत्तर:
(B) गोदावरी।

12. कौन-सी नदी दरार घाटी में बहती है?
(A) दामोदर
(B) कृष्णा
(C) तुंगभद्रा
(D) तापी।
उत्तर:
(D) तापी।

अति लघु उत्तरीय प्रश्न (Very Short Answer Type Questions)

प्रश्न 1.
भारत के दो जल-प्रवाह तन्त्र बताएं।
उत्तर:
हिमालय नदियां तथा प्रायद्वीपीय नदियां ।

प्रश्न 2.
सिन्धु नदी का कुल बेसिन क्षेत्रफल कितना है?
उत्तर:
1,165,000 वर्ग किलोमीटर

प्रश्न 3.
दरार घाटियों में बहने वाली दो नदियों के नाम लिखो।
उत्तर:
नर्मदा, ताप्ती।

प्रश्न 4.
प्रायद्वीपीय नदियों के मुख्य विभाजक का नाम लिखो।
उत्तर:
पश्चिमी घाट।

प्रश्न 5.
उत्तरी भारत तथा प्रायद्वीपीय नदियों के मध्य जल विभाजन का नाम बताएं।
उत्तर:
विंध्या – सतपुड़ा श्रेणी।

प्रश्न 6.
सिन्धु नदी का उद्गम बताएं।
उत्तर:
मानसरोवर झील (तिब्बत)।

प्रश्न 7.
सिन्धु नदी की कुल लम्बाई कितनी है?
उत्तर:
2880 किलोमीटर।

JAC Class 11 Geography Important Questions Chapter 3 अपवाह तंत्र

प्रश्न 8.
गंगा की सहायक नदी का नाम बताओ जो दक्षिण से मिलती है।
उत्तर:
सोन नदी।

प्रश्न 9.
एक ट्रांस हिमालयी नदी का नाम बताएं जो सिन्धु नदी की सहायक नदी है।
उत्तर:
सतलुज।

प्रश्न 10.
भारतीय पठार की नदी का नाम लिखो जो अरब सागर की ओर बहती है।
उत्तर:
नर्मदा तथा ताप्ती।

प्रश्न 11.
प्रायद्वीपीय भारत की एक नदी बताओ जो ज्वारनदमुख बनाती है।
उत्तर:
नर्मदा।

प्रश्न 12.
प्रायद्वीपीय भारत की सबसे लम्बी नदी का नाम लिखो।
उत्तर:
गोदावरी

प्रश्न 13.
कृष्णा नदी का स्रोत कौन-सा है?
उत्तर:
महाबलेश्वर।

प्रश्न 14.
भारत में गंगा नदी का कुल कितना बेसिन क्षेत्रफल है?
उत्तर:
8,61,404 वर्ग किलोमीटर

प्रश्न 15.
बांग्लादेश में गंगा नदी को क्या नाम दिया गया है?
उत्तर:
पदमा।

JAC Class 11 Geography Important Questions Chapter 3 अपवाह तंत्र

प्रश्न 16.
उन नदियों के नाम लिखो जो हिमालय नदी तंत्र बनाती हैं?
उत्तर:
सिंधु, गंगा, ब्रह्मपुत्र।

प्रश्न 17.
प्रायद्वीपीय भारत की बंगाल की खाड़ी में गिरने वाली नदियों के नाम लिखो।
उत्तर:
महानदी, गोदावरी, कृष्णा तथा कावेरी।

प्रश्न 18.
एक ट्रांस हिमालय नदी का नाम लिखो।
उत्तर:
सतलुज।

प्रश्न 19.
पूर्ववर्ती जल प्रवाह की एक नदी का नाम लिखें।
उत्तर:
सिन्धु।

प्रश्न 20.
प्राचीन समय में कौन-सी नदी पंजाब से असम की ओर बहती थी?
उत्तर:
सिन्ध – ब्रह्म नदी।

प्रश्न 21.
जेहलम नदी का स्रोत बताएं।
उत्तर:
बुल्लर झील।

प्रश्न 22.
गंगा नदी द्वारा निर्मित डेल्टे का नाम लिखो।
उत्तर:
सुन्दरवन।

JAC Class 11 Geography Important Questions Chapter 3 अपवाह तंत्र

प्रश्न 23.
किस नदी को तिब्बत में सांग- पो कहा जाता है?
उत्तर:
ब्रह्मपुत्र नदी।

प्रश्न 24.
किस नदी को दक्षिण की गंगा कहते हैं?
उत्तर:
कावेरी

प्रश्न 25.
जबलपुर के निकट नर्मदा नदी कौन-सा जल प्रवाह बनाती है?
उत्तर:
मार्बल रॉक।

प्रश्न 26.
प्राचीन समय में हरियाणा के शुष्क क्षेत्र में बहने वाली नदी का नाम लिखो।
उत्तर:
सरस्वती।

प्रश्न 27.
जोग जल प्रपात कहां पर स्थित है?
उत्तर:
शरबती नदी पर (कर्नाटक)।

प्रश्न 28.
अरावली से निकलने वाली नदी का नाम लिखो।
उत्तर:
साबरमती।

JAC Class 11 Geography Important Questions Chapter 3 अपवाह तंत्र

प्रश्न 29.
खम्बात की खाड़ी में गिरने वाली नदी का नाम लिखो।
उत्तर:
माही।

प्रश्न 30.
नदियों के चार प्रमुख अपवाह प्रारूप बताओ।
उत्तर:

  1. वृक्षाकार
  2. अपकेन्द्रीय
  3. जालीनुमा
  4. अभिकेन्द्रीय।

प्रश्न 31.
जल ग्रहण क्षेत्र किसे कहते हैं?
उत्तर:
जहाँ से विशाल नदी जल बहा कर लाती है।

प्रश्न 32.
जलविभाजक किसे कहते हैं?
उत्तर:
दो अपवाह द्रोणियों को अलग करने वाली सीमा।

प्रश्न 33.
अरब सागर तथा खाड़ी बंगाल में भारत की नदियों का कितने-कितने % जल गिरता है?
उत्तर:
अरब सागर – 23%,
खाड़ी बंगाल – 73%.

प्रश्न 34.
20000 वर्ग कि० मी० से अधिक अपवाह क्षेत्र वाली नदियों के नाम लिखो।
उत्तर:
गंगा, ब्रह्मपुत्र, कृष्णा, तापी, नर्मदा, माही, पेन्नार, साबरमती, बारांक

प्रश्न 35.
ब्रह्मपुत्र नदी को बंगला देश में किस नाम से जाना जाता है?
उत्तर:
मेघना।

JAC Class 11 Geography Important Questions Chapter 3 अपवाह तंत्र

प्रश्न 36.
बंगला देश में गंगा को किस नाम से जाना जाता है?
उत्तर:
पदमा।

प्रश्न 37.
किस नदी को बिहार का शोक कहा जाता है?
उत्तर:
कोसी।

प्रश्न 38.
प्रायद्वीपीय भारत का सबसे बड़ा नदी तन्त्र कौन-सा है?
उत्तर:
गोदावरी।

स्मरणीय तथ्य (Points to Remember)

  • उत्तरी भारत की नदियां:
    1. सिन्धु
    2. सतलुज
    3. ब्यास
    4. रावी
    5. चेनाव
    6. जेहलम
    7. गंगा
    8. यमुना
    9. घाघरा
    10. गण्डक
    11. कोसी
    12.  ब्रह्मपुत्र
  • दक्षिणी भारत की नदियां:
    1. नर्मदा
    2. ताप्ती
    3. महानदी
    4. गोदावरी
    5. कृष्णा
    6. कावेरी।

लघु उत्तरीय प्रश्न (Short Answer Type Questions)

प्रश्न 1.
अपवाह किसे कहते हैं?
उत्तर:
निश्चित वाहिकाओं के माध्यम से हो रहे जल प्रवाह को अपवाह कहते हैं।

प्रश्न 2.
अपवाह तन्त्र की परिभाषा लिखिए।
उत्तर:
अपवाह तन्त्र- निश्चित वाहिकाओं के माध्यम से हो रहे जल-प्रवाह के जाल को अपवाह तन्त्र कहते हैं।

प्रश्न 3.
अपवाह तन्त्र को प्रभावित करने वाले कारक बताइये।
उत्तर:
किसी क्षेत्र का अपवाह तन्त्र उस क्षेत्र की भूवैज्ञानिक समयाविधि, चट्टानों की प्रकृति एवं संरचना, स्थलाकृति, ढाल, प्रवाहित जल की मात्रा तथा बहाव की अवधि का परिणाम है।

JAC Class 11 Geography Important Questions Chapter 3 अपवाह तंत्र

प्रश्न 4.
वृक्षाकार अपवाह प्रतिरूप किसे कहते हैं?
उत्तर:
वह अपवाह प्रतिरूप जो कि पेड़ की शाखाओं के अनुरूप होता है उसे वृक्षाकार अपवाह प्रतिरूप के नाम से जाना जाता है।

प्रश्न 5.
अरीय अपवाह प्रतिरूप किसे कहते हैं? उदाहरण दीजिए।
उत्तर:
जब नदियां किसी पर्वत से निकल कर सभी दिशाओं में प्रवाहित होती हैं तो इसे अरीय अपवाह प्रतिरूप के नाम से जाना जाता है। अमरकंटक पर्वत श्रृंखला से निकलने वाली नदियां इस अपवाह प्रतिरूप का अनुसरण करती हैं।

प्रश्न 6.
जालीनुमा अपवाह प्रतिरूप किसे कहते हैं?
उत्तर:
जब मुख्य नदियां एक-दूसरे के समानान्तर प्रवाहित होती हों तथा सहायक नदियां उनसे समकोण पर मिलती हों तो ऐसे अपवाह प्रतिरूप हों तो ऐसे अपवाह को जालीनुमा अपवाह प्रतिरूप कहते हैं।

प्रश्न 7.
उत्पत्ति के आधार पर भारत की नदियों को कितने वर्गों में बांटा जाता है?
उत्तर:
भारत का जल प्रवाह देश की भू-संरचना पर निर्भर करता है। इस आधार पर देश की नदियों को दो वर्गों में बांटा जाता है।

  1. हिमालय की नदियां
  2. प्रायद्वीपीय नदियां।

प्रश्न 8.
हिमालय के तीन प्रमुख नदी तन्त्रों के नाम बताइए।
उत्तर:
हिमालय की नदियों का विकास एक लम्बे समय में हुआ है। हिमालय की नदियों को तीन मुख्य तन्त्रों (System) में बांटा जाता है।

  1. सिन्धु तन्त्र (Indus System)
  2. गंगा तन्त्र (Ganges System)
  3.  ब्रह्मपुत्र तन्त्र (Brahmaputra System)।

प्रश्न 9.
गार्ज ( महाखंड ) क्या है? दो उदाहरण दीजिए।
उत्तर:
पर्वतीय भागों में बहुत गहरे तथा तंग नदी मार्गों को गार्ज कहते हैं। इसे महाखंड भी कहा जाता है। इसके किनारे खड़ी ढाल वाले होते हैं तथा लगातार ऊपर उठते रहते हैं। इसका तल लगातार गहरा होता जाता है। हिमालय पर्वत में ऐसे कई गार्ज मिलते हैं।
जैसे – सिन्धु, सतलुज, गार्ज, ब्रह्मपुत्र ( दिहांग ) गार्ज।

JAC Class 11 Geography Important Questions Chapter 3 अपवाह तंत्र

प्रश्न 10.
गंगा की दो शीर्ष नदियों (Head Streams) के नाम बताइए जो देव प्रयाग में मिलती हैं।
उत्तर:
गंगा नदी उत्तर प्रदेश के हिमालयी क्षेत्र से निकलती है तथा दक्षिण और दक्षिण-पश्चिम दिशा में बहती है। देव प्रयाग में इसमें दो शीर्ष नदियां – अलकनन्दा और भागीरथी आ कर मिलती हैं। इसके बाद इनका नाम गंगा पड़ता है

प्रश्न 11.
प्रायद्वीप भारत की प्रमुख नदियों के नाम बताइए।
उत्तर:
प्रायद्वीप की कुछ नदियां पूर्व की ओर बहती हुई खाड़ी बंगाल में गिरती हैं। इनमें महानदी, गोदावरी, कृष्णा, कावेरी, पेनार महत्त्वपूर्ण नदियां हैं। कुछ नदियां पश्चिम की ओर बह कर अरब सागर में गिरती हैं। इसमें नर्मदा, ताप्ती प्रमुख नदियां हैं।

प्रश्न 12.
डेल्टा किसे कहते हैं? भारत से चार उदाहरण दीजिए।
उत्तर:
नदियों के मुहाने पर तलछट के निक्षेप से एक त्रिभुजाकार स्थल रूप बनता है जिसे डेल्टा कहते हैं। डेल्टा नदी के अन्तिम भाग में अपने भार के निक्षेप से बनने वाला भू-आकार है। यह एक उपजाऊ समतल प्रदेश होता है। भारत में चार प्रसिद्ध डेल्टा इस प्रकार हैं:

  1. गंगा नदी का डेल्टा
  2. कृष्णा नदी का डेल्टा
  3. महानदी का डेल्टा
  4. कावेरी नदी का डेल्टा।

प्रश्न 13.
गोदावरी को वृद्ध गंगा या दक्षिण गंगा क्यों कहा जाता है?
उत्तर:
गोदावरी नदी प्रायद्वीप की सबसे बड़ी नदी है। इसका एक विशाल अपवहन क्षेत्र है जो महाराष्ट्र, कर्नाटक, उड़ीसा तथा आन्ध्र प्रदेश में फैला हुआ है। विशाल आकार और विस्तार के कारण इसकी तुलना गंगा नदी से की जाती है। जिस प्रकार उत्तरी भारत में गंगा नदी महत्त्वपूर्ण है उसी प्रकार दक्षिणी भारत में गोदावरी नदी का महत्त्व है। गंगा नदी की तरह इसकी भी अनेक सहायक नदियां हैं।

प्रश्न 14.
पश्चिमी तट पर नदियां डेल्टा क्यों नहीं बनाती हैं जबकि वे बड़ी मात्रा में तलछट बहा कर लाती हैं?
उत्तर:
पश्चिमी तट पर नर्मदा और ताप्ती प्रमुख नदियां हैं। ये नदियां काफ़ी मात्रा में तलछट बहा कर ले जाती हैं। परन्तु ये डेल्टा नहीं बनातीं। इस तट पर मैदान की चौड़ाई बहुत कम है। प्रदेश की तीव्र ढाल है। नदियां तेज़ गति से समुद्र में गिरती हैं। इसलिए तलछट का निक्षेप नहीं होता । संकरे मैदान के कारण नदियों के अन्तिम भाग की लम्बाई कम है जिससे डेल्टे का निर्माण नहीं होता।

तुलनात्मक प्रश्न (Comparison Type Questions)

प्रश्न 1.
प्रायद्वीपीय भारत की पूर्व तथा पश्चिम की अोर बहने वाली नदियों में अन्तर स्पष्ट करो।
उत्तर:

पूर्व की ओर बहने वाली नदियां पश्चिम की ओर बहने वाली नदियां
(1) महानदी, गोदावरी, कृष्णा, कावेरी नदियां पूर्व की ओर बहती हैं। (1) नर्मदा तथा ताप्ती नदियां पश्चिम की ओर बहती हैं।
(2) ये नदियां डेल्टा बनाती हैं। (2) ये नदियां डेल्टा नहीं बनाती हैं।
(3) ये नदियां खाड़ी बंगाल में गिरती हैं। (3) ये नदियां अरब सागर में गिरती हैं

प्रश्न 2.
पूर्ववर्ती अपवाह तथा अनुवर्ती अपवाह में अन्तर बताओ।
उत्तर:

पूर्ववर्ती अपवाह (Antecedent Drainage) (1) किसी क्षेत्र में उत्थान के पश्चात् नवीन ढाल के अनुसार बहने वाली अपवाह को अनुवर्ती अपवाह कहते हैं।
(1) किसी क्षेत्र में जब नदी उत्थान से पूर्व के ढाल के अनुसार मूल दिशा में बहती रहती है तो उसे पूर्ववर्ती अपवाह कहते हैं। (2) ये नदियां उत्थान के पश्चात् जन्म लेती हैं।
(2) ये नदियां उन मोड़दार पर्वतों की अपेक्षा पुरानी होती हैं जिन पर ये बहती हैं। (3) ये नदियां गार्ज नहीं बनातीं।
(3) ये नदियां गहरे गार्ज बनाती हैं। (4) दक्षिणी पठार की पूर्व की ओर बहने वाली नदियां अनुवर्ती नदियां हैं।
(4) ट्रांस हिमालयी नदियां सिन्धु- सतलुज पूर्ववर्ती अपवाह के उदाहरण हैं। (1) किसी क्षेत्र में उत्थान के पश्चात् नवीन ढाल के अनुसार बहने वाली अपवाह को अनुवर्ती अपवाह कहते हैं।

JAC Class 11 Geography Important Questions Chapter 3 अपवाह तंत्र

प्रश्न 3.
हिमालय एवं प्रायद्वीपीय पठार की नदियों के मध्य अपवाह लक्षणों एवं जलीय विशेषताओं में कौन- सी महत्त्वपूर्ण भिन्नताएं हैं? अपने उत्तर की पुष्टि उपयुक्त उदाहरण देते हुए कीजिए।
उत्तर:

हिमालय की नदियां प्रायद्वीप की नदियां
(1) हिमालय की नदियां अधिक लम्बी हैं। (1) प्रायद्वीप की नदियां इतनी अधिक लम्बी नहीं हैं।
(2) हिमालय की नदियों की संख्या अधिक है। (2) प्रायद्वीप की नदियों की संख्या कम है।
(3) हिमालय की नदियों के बेसिन काफ़ी बड़े हैं तथा अपवहन-क्षेत्र बहुत बड़े हैं। (3) प्रायद्वीप की नदियों के बेसिन तथा अपवहन-क्षेत्र छोटे हैं।
(4) हिमालय की नदियों के जल के दो स्रोत हैं-वर्षा तथा हिमनदियों से पिघलता हुआ जल। इसलिए ये बारहमासी नदियां हैं। (4) प्रायद्वीप की नदियां मुख्यतया वर्षा पर निर्भर करती हैं इसलिए ये मानसूनी नदियां हैं।
(5) हिमालय की नदियां गहरे गार्ज बनाती हैं। (5) प्रायद्वीप नदियां उथली घाटियों में बहती हैं।
(6) हिमालय की नदियां गहरे विसर्प बनाती हैं तथा मार्ग भी बदल लेती हैं। (6) प्रायद्वीप की नदियों का मार्ग सीधा होता है।
(7) हिमालय की नदियां पूर्ववर्ती नदियां हैं। (7) प्रायद्वीप की नदियां अनुवर्ती नदियां हैं।
(8) हिमालय की नदियां जहाज़रानी तथा सिंचाई के अनुकूल हैं। (8) प्रायद्वीप की नदियां जहाज़रानी और जल सिंचाई के अनुकूल नहीं हैं।
(9) ये नदियां जलोढ़ आधार के कारण घाटी के अपरदन में लगी हुई हैं। (9) ये नदियां दृढ़ आधार के कारण अधिक अपरदन नहीं कर सकतीं।सकतीं।

निबन्धात्मक प्रश्न (Essay Type Questions)

प्रश्न 1.
भारत के विभिन्न जल प्रवाहों का वर्णन करो तथा भारत की मुख्य नदियों पर प्रकाश डालो। भारत का जल प्रवाह (Drainage System of India ):
जल प्रवाह किसी देश की भू-संरचना तथा ढलान पर निर्भर करता है। भारत की धार्मिक, सामाजिक रूप-रेखा पर नदियों का विशेष प्रभाव रहा है। भारत एक कृषि प्रधान देश है तथा इसकी आर्थिक व्यवस्था में नदियों का महत्त्वपूर्ण स्थान है। इसलिए भारत को नदियों का देश (Land of Rivers) भी कहा जाता है। विन्ध्याचल पर्वत, उत्तरी भारत तथा दक्षिणी भारत के जल प्रवाह की विभाजन सीमा माना जाता है। इस प्रकार धरातल के अनुसार भारत में जल प्रवाह को दो भागों में बांटा जाता है।

  1. उत्तरी भारत के विशाल मैदान का जल प्रवाह।
  2. दक्षिण भारत के जल प्रवाह।

1. उत्तरी भारत का जल प्रवाह (Drainage System of Northern India)
उत्तरी भारत के जल प्रवाह पर हिमालय पर्वत का विशेष प्रभाव है। अधिकांश नदियां हिमालय पर्वत से ही निकलती हैं। ये नदियां बर्फीले पर्वतों से निकलने के कारण वर्ष भर बहती हैं। कई नदियां हिमालय पर्वत से भी पुरानी हैं। पर्वतीय भागों में अनेक तंग गहरी घाटियां बनाती हैं, परन्तु मैदानी भाग में निक्षेप का कार्य अधिक करती हैं। वर्षा ऋतु में भयानक बाढ़ें आती हैं। इन नदियों द्वारा निक्षेप से ही विशाल मैदान का निर्माण हुआ है। उत्तरी मैदान को गंगा का वरदान कहा जाता है। (The northern plain is a gift of the Ganges.) इस जल प्रवाह का विस्तार पश्चिम में पंजाब से लेकर पूर्व में असम प्रदेश तक है। इस जल प्रवाह को तीन भागों में बांटा जाता है।
JAC Class 11 Geography Important Questions Chapter 3 अपवाह तंत्र 1

  1. सिन्धु जल प्रवाह क्रम।
  2. गंगा जल प्रवाह क्रम।
  3. ब्रह्मपुत्र जल प्रवाह क्रम

1. सिन्धु जल प्रवाह क्रम (The Indus Drainage System):
सिन्धु नदी के जल प्रवाह में सतलुज, ब्यास तथा रावी मुख्य नदियां हैं जो भारत में हैं परन्तु झेलम, चिनाब तथा सिन्धु नदियां पाकिस्तान में हैं।
(i) सतलुज (The Sutlej): यह नदी कैलाश पर्वत के निकट मानसरोवर झील के समीप राक्षस ताल से निकलती हैं जो 4,630 मीटर ऊंचा है। पर्वतीय भाग में एक तंग गहरी घाटी बनाने के बाद रोपड़ नामक स्थान पर मैदानी भाग में प्रवेश करती हैं। हरिके पत्तन के स्थान पर ब्यास नदी इसमें मिल जाती है। 160 किलोमीटर की दूरी तक भारत- पाकिस्तान सीमा बनाती है। इसकी कुल लम्बाई 1,448 किलोमीटर है। शिवालिक की पहाड़ियों की तंग घाटी में इस नदी पर एक प्रसिद्ध बांध भाखड़ा डैम बनाया गया है।

(ii) ब्यास (The Beas): यह नदी रोहतांग दर्रे के ऊपर से ब्यास कुण्ड से निकलती है जो 4,062 मीटर ऊंचा है। शिवालिक की पहाड़ियों को पार कर मीरथल नामक स्थान पर मैदानी भाग में प्रवेश करती हैं। इसकी कुल लम्बाई 460 किलोमीटर है। इस नदी का पूरा भाग पंजाब की सीमा के अन्दर है।

(iii) रावी (The Ravi): यह नदी चम्बा के निकट धौलाधार पर्वत श्रेणी से निकलती है। माधोपुर के निकट मैदानी भाग में प्रवेश करती है। यह नदी 720 किलोमीटर लम्बी है। भारत तथा पाकिस्तान के बीच एक प्राकृतिक सीमा रेखा है। इस नदी पर थीन बांध ( Thein Dam) योजना का कार्य चल रहा है।

2. गंगा जल प्रवाह क्रम (The Ganga Drainage System):
इसमें हिमालय पर्वत से उतरने वाली नदियां गंगा, यमुना, शारदा, गण्डक, घाघरा तथा कोसी शामिल हैं। विन्ध्याचल, सतपुड़ा पर्वत श्रेणियों से निकलने वाली नदियां चम्बल, बेतवा, केन तथा सोन भी गंगा के जल प्रवाह से मिल जाती हैं। गंगा नदी इस जल प्रवाह की मुख्य नदी है। (The Ganges is the master stream of this system.)

(i) गंगा (The Ganges):
गंगा नदी भारत की सबसे पवित्र नदी है। भारत की धार्मिक तथा सांस्कृतिक रूप-रेखा पर इसका विशेष प्रभाव है। (The story of the Ganges from her source of the sea, from old times to new, is the story of India’s civilization and culture.) यह नदी हिमालय पर्वत में गोमुख हिमनदी से निकलती है। इस स्थान पर इसे गंगोत्री कहते हैं। इसका विकास भागीरथी तथा अलकनन्दा नदियों द्वारा होता है। 290 किलोमीटर पर्वतीय प्रदेश से निकल कर हरिद्वार के निकट मैदानी भाग में प्रवेश करती है। इलाहाबाद के निकट इसमें यमुना नदी आकर मिल जाती है।

यह स्थान संगम के नाम से प्रसिद्ध है। इससे आगे उत्तर की ओर से गोमती, घाघरा, गण्डक और कोसी की सहायक नदियां इसमें मिलती हैं। दक्षिण की ओर से सोन नदी आकर मिलती है। खाड़ी बंगाल में गिरने से पहले एक विशाल डेल्टा का निर्माण करती हैं। गंगा का डेल्टा ‘सुन्दर वन’ विश्व का सबसे बड़ा डेल्टा है। उद्गम से लेकर डेल्टा तक इसकी लम्बाई 2525 किलोमीटर है। किनारे पर हरिद्वार, कानपुर, इलाहाबाद, वाराणसी, पटना, कोलकाता आदि महत्त्वपूर्ण नगर बसे हैं।

(ii) यमुना (The Yamuna ):
यह यमनोत्री हिम नदी से निकल कर गंगा के समानान्तर बहती है। इसकी कुल लम्बाई 1375 किलोमीटर है। पर्वतीय भाग को पार कर उत्तरी मैदान में एक विशाल चाप बनाती हुई इलाहाबाद में गंगा नदी में मिल जाती है। भगवान् कृष्ण की लीला भूमि मथुरा, वृन्दावन, गोकुल आदि इसी के तट पर स्थित है। दक्षिण की ओर से चम्बल नदी विन्ध्याचल पर्वत से निकल कर इटावा के निकट यमुना नदी में मिल जाती है। इसके अतिरिक्त केन तथा बेतवा नदियां यमुना की सहायक नदियां हैं

(iii) घाघरा (The Ghaghra ):
इसे ‘सरयू’ नदी भी कहते हैं । नेपाल (हिमालय) से निकल कर मैदानी भाग में बहती हुई पटना के निकट गंगा नदी में मिलती है। अयोध्या नगरी इस नदी के तट पर स्थित है। शारदा नदी इसकी मुख्य सहायक नदी है।

(iv) गण्डक (The Gandak ):
यह नदी नेपाल (हिमालय) से निकलती है। मैदानी भाग में उत्तर प्रदेश तथा बिहार की सीमा बनाती है। इस नदी ने कई बार अपना मार्ग परिवर्तन किया है। यह नदी पटना के निकट गंगा में मिल जाती है। इस नदी में भीषण बाढ़ें आती रहती हैं।

(v) कोसी (The Kosi ):
यह नदी हिमालय पर्वत में कंचनजुंगा पर्वत से निकलती है। पर्वतीय प्रदेश को पार कर चतरा नामक स्थान पर मैदानी भाग में प्रवेश करती है। यह नदी मार्ग परिवर्तन तथा भयानक बाढ़ों के कारण धन-जन को बहुत हानि पहुंचाती है। इसलिए इसे “बिहार की शोक नदी” (River of sorrow of Bihar) कहते हैं ।

3. ब्रह्मपुत्र जल प्रवाह क्रम (The Brahmputra Drainage System):
यह नदी 2880 किलोमीटर लम्बी है तथा भारत की सबसे लम्बी नदी है। मानसरोवर झील के पूर्व में कैलाश पर्वत के समीप से निकल कर हिमालय पर्वत के समानान्तर बहती हुई तिब्बत प्रदेश में बहती है। इसे 1440 किलोमीटर लम्बे मार्ग में सांपो ( Tsangpo) नदी कहते हैं । हिमालय के पूर्वी मोड़ को काट कर दिहांग गार्ज (Gorge) में से असम घाटी में प्रवेश करती है। यह एक विशाल नदी है जिसमें भयानक बाढ़ें आती हैं। बांग्ला देश में यह पद्मा नदी से मिलकर खाड़ी बंगाल में विशाल डेल्टा का निर्माण करती हैं। डिब्रूगढ़ (Dibrugarh ) से लेकर खाड़ी बंगाल तक इसमें किश्तियां चलाई जा सकती हैं।

2. दक्षिणी भारत का जल प्रवाह (The Drainage System of Southern India):
दक्षिणी भारत बहुत प्राचीन भू-खण्ड है। इसलिए इस प्रदेश की नदियां बहुत प्राचीन हैं। दक्षिणी भाग एक पठार है जो चारों ओर ढालुआ है। इसलिए यहां से पूर्व, पश्चिम तथा उत्तर की ओर नदियां बहती हैं। अधिकांश नदियां पूर्व की ओर बहती हुई खाड़ी बंगाल में गिरती हैं। केवल नर्मदा, ताप्ती पश्चिम की ओर बहती हैं। पश्चिमी घाट के पर्वत दक्षिणी भारत की नदियों को दो भागों में बांटते हैं।

  1. अरब सागर में गिरने वाली नदियां तथा
  2. खाड़ी बंगाल में गिरने वाली नदियां।

दक्षिणी भारत की नदियां नीचे पर्वतों से निकलती हैं जहां हिमपात नहीं होता। इसलिए इनको केवल वर्षा काल में ही जल प्राप्त होता है। दक्षिणी पठार की नदियां छोटी तथा कम संख्या में हैं। इन नदियों में वर्षा ऋतु में अचानक बाढ़ें आ जाती हैं। इन नदियों की घाटियां चौड़ी और उथली हैं तथा कम मात्रा में कटाव करती हैं। इनका मार्ग ऊंचा – नीचा, पथरीला होने के कारण, इनमें जहाज़ नहीं चलाए जा सकते हैं। इन नदियों से नहरें निकालना कठिन है। ये नदियां अनेक जल प्रपात बनाती हैं जो जल-विद्युत् विकास को सुविधा प्रदान करते हैं।

1. अरब सागर में गिरने वाली नदियां (The Rivers falling into Arabian Sea)
(i) नर्मदा (The Narmada):
यह नदी मध्य प्रदेश में अमर कण्टक नामक स्थान से निकलती है। इसके उत्तर में विंध्याचल तथा दक्षिण में सतपुड़ा पर्वत श्रेणियां हैं। 1300 किलोमीटर लम्बी नदी, एक तंग रिफ्ट घाटी (Rift Valley) में बहती है। तीव्र गति से बहने के कारण यह नदी डेल्टा नहीं बनाती है। मध्य प्रदेश में संगमरमर की चट्टानों में रमणीक गार्ज बहुत प्रसिद्ध हैं। नर्मदा की सहायक नदियों की कमी है।

(ii) ताप्ती (The Tapti):
यह नदी महादेव पहाड़ियों में बेतुल से निकलती है। यह नदी 724 किलोमीटर लम्बी है। नर्मदा नदी के समानान्तर बहने के पश्चात् खाड़ी खम्बात (Gulf of Cambay) में गिरती हैं। इसके मुहाने पर सूरत नगर स्थित है। नर्मदा नदी तथा ताप्ती दोनों नदियां एक दरार घाटी (Rift Valley) में बहती हैं। इसके अतिरिक्त अरब सागर में गिरने वाली महत्त्वपूर्ण नदियां, लूनी, साबरमती तथा माही हैं।

2. खाड़ी बंगाल में गिरने वाली नदियां (The Rivers falling into Bay of Bengal)
(i) दामोदर नदी (The Damodar ):
530 किलोमीटर लम्बी नदी छोटा नागपुर पठार से निकलती है। बाढ़ों तथा मार्ग परिवर्तन के कारण इसे “शोक नदी ? (River of sorrow) कहते हैं। दामोदर घाटी परियोजना के कारण इससे अब आर्थिक विकास में सहायता मिलेगी।

(ii) महानदी (The Mahanadi ):
यह नदी 857 किलोमीटर लम्बी है। अमरकण्टक पर्वत श्रेणी से निकल कर उड़ीसा राज्य में बहती हुई एक उपजाऊ मैदान तथा डेल्टा बनाती है। डेल्टा क्षेत्र में इस नदी से नहरों द्वारा जल सिंचाई की जाती है।

(iii) गोदावरी (The Godavari ):
1,440 किलोमीटर लम्बी नदी, पश्चिमी घाट में नासिक क्षेत्र से निकल कर आन्ध्र प्रदेश में पूर्व की ओर बहती है। पूर्वी घाट को पार करते समय एक तंग गहरी घाटी बनाती हैं। इस नदी का डेल्टा बड़ा उपजाऊ है।

(iv) कृष्णा (The Krishna ):
यह नदी 1, 400 किलोमीटर लम्बी है। यह नदी पश्चिमी घाट में महाबलेश्वर से निकलती है। इसको दो मुख्य सहायक नदियां उत्तर में भीमा नदी तथा दक्षिण में तुंगभद्रा नदी है।

(v) कावेरी (The Cauvery ):
यह 800 किलोमीटर लम्बी नदी ब्रह्म गिर पहाड़ियों से निकल कर खाड़ी बंगाल में गिरती है। यह नदी जल सिंचाई, परिवहन तथा जल विद्युत् के लिए उपयोगी है। यह नदी कई जल प्रपात बनाती है। शिवसुन्द्रम् जल प्रपात जल  विद्युत् विकास के लिए उपयोगी है। इसका डेल्टा बहुत विशाल उपजाऊ मैदानी है शीतकाल की वर्षा के कारण इस नदी में सारा वर्ष जल रहता है। कावेरी का जल ग्रहण क्षेत्र केरल, कर्नाटक तथा तमिलनाडु राज्यों में विस्तृत है।

 

JAC Class 11 Geography Important Questions Chapter 2 संरचना तथा भूआकृति विज्ञान

Jharkhand Board JAC Class 11 Geography Important Questions Chapter 2 संरचना तथा भूआकृति विज्ञान Important Questions and Answers.

JAC Board Class 11 Geography Important Questions Chapter 2 संरचना तथा भूआकृति विज्ञान

बहु-विकल्पी प्रश्न (Multiple Choice Questions)

प्रश्न-दिए गए चार वैकल्पिक उत्तरों में से सही उत्तर चुनिए
1. भूपृष्ठ में कितनी भूगर्भिक प्लेटें हैं?
(A) 5
(B) 6
(C) 7
(D) 8
उत्तर:
(C) 7

2. भारत का प्राचीन स्थल खण्ड कौन-सा है?
(A) उत्तरी मैदान
(B) प्रायद्वीपीय पठार
(C) हिमालय
(D) अरावली।
उत्तर:
(B) प्रायद्वीपीय पठार।

3. भारत में स्थित हिमालय पर्वत का सर्वोच्च शिखर है
(A) माऊंट एवरेस्ट
(B) कंचनजंगा
(C) K2
(D) धौलागिरि।
उत्तर:
(B) कंचनजंगा।

4. प्राचीन जलोढ़ निक्षेप को कहते हैं
(A) खादर
(B) बांगर
(C) भाबर
(D) तराई।
उत्तर:
(B) बांगर।

5. दक्षिणी भारत का सर्वोच्च शिखर है-
(A) दोदा वेटा
(B) अनाई मुदी
(C) महेन्द्रगिरि
(D) कालसूबाई।
उत्तर:
(B) अनाई मुदी।

JJAC Class 11 Geography Important Questions Chapter 2 संरचना तथा भूआकृति विज्ञान

6. पश्चिमी तटीय मैदान के दक्षिण भाग को कहते हैं
(A) कोंकण तट
(B) कोरोमण्डल तट
(C) कनारा तट
(D) मालाबार तट।
उत्तर:
(D) मालाबार तट।

7. पितली पक्षी – आश्रय स्थल कहां है?
(A) अण्डमान द्वीप
(B) निकोबार द्वीप
(C) लक्षद्वीप
(D) मालदीव
उत्तर:
(C) लक्षद्वीप।

8. किस सीमा के साथ प्लेटें मिलती हैं?
(A) अपकारी
(B) अभिसारी
(C) परिवर्तित
(D) भूगर्भिक।
उत्तर:
(B) अभिसारी।

(D) मालाबार तट।
(B) अभिसारी
9. हिमालय पर्वत के स्थान पर कौन सा प्राचीन सागर था?
(A) टैथीज़
(B) दक्षिणी
(C) अरब
(D) हिन्द महासागर।
उत्तरl
(A) टैथीज़।

10. दरार घाटी कौन-सी है?
(A) गंगा
(B) नर्मदा
(C) चम्बल
(D) दामोदर
उत्तर:
(B) नर्मदा

JJAC Class 11 Geography Important Questions Chapter 2 संरचना तथा भूआकृति विज्ञान

11. करेवां भू-आकृति कहाँ पाई जाती है?
(A) उत्तर-पूर्व हिमालय में
(B) हिमाचल उत्तराखण्ड, हिमालय में
(C) पूर्वी हिमालय में
(D) कश्मीर हिमालय में।
उत्तर:
(D) कश्मीर हिमालय में।

12. निम्न पर्वतमालाओं में से सबसे पहले किसका निर्माण हुआ है?

(A) विंध्याचल
(B) नर्मदा
(C) सतपुड़ा
(D) नीलगिरि
उत्तर:
(D) नीलगिरि।

अति लघु उत्तरीय प्रश्न (Very Short Answer Type Questions)

प्रश्न 1.
वृहद् स्तर पर भारत के धरातल को कितने भागों में बांटा जा सकता है?
उत्तर:
तीन।

प्रश्न 2.
भारत का सबसे प्राचीन पठार कौन-सा है?
उत्तर:
दक्कन पठार।

प्रश्न 3.
दक्कन पठार की पूर्वी सीमाओं के नाम बताएं।
उत्तर:
राजमहल पहाड़ियां।

प्रश्न 4.
भारत का प्रायद्वीपीय पठार का निर्माण कब हुआ?
उत्तर:
पूर्व कैम्ब्रेरियन युग में।

प्रश्न 5.
अरावली पर्वत किस युग में ऊपर उठे?
उत्तर:
विन्धयन युग।

प्रश्न 6.
दक्कन पठार में निर्मित लावा की सतहें कैसे बनीं?
उत्तर:
लावा बहने के कारण।

JJAC Class 11 Geography Important Questions Chapter 2 संरचना तथा भूआकृति विज्ञान

प्रश्न 7.
भारत में पाये जाने वाली दो रिफ्ट घाटियों के नाम लिखो।
उत्तर:
नर्मदा तथा ताप्ती घाटियां।

प्रश्न 8.
अरब सागर कब अस्तित्व में आया?
उत्तर:
प्लायोसीन युग में।

प्रश्न 9.
उस सागर का नाम बतायें जो हिमालय के किनारे पर स्थित था।
उत्तर:
टैथीज़ सागर।

प्रश्न 10.
हिमालय किस युग में ऊपर उठे?
उत्तर:
टरशरी युग में।

प्रश्न 11.
हिमालय के उत्तर में कौन-सा भू-खण्ड स्थित है?
उत्तर:
अंगारालैण्ड।

प्रश्न 12.
टरशरी युग में हिमालय के दक्षिण में स्थित भू-खण्ड का नाम बताएं।
उत्तर:
गोंडवानालैण्ड।

JJAC Class 11 Geography Important Questions Chapter 2 संरचना तथा भूआकृति विज्ञान

प्रश्न 13.
कश्मीर घाटी में पाई जाने वाली झील निक्षेप का नाम लिखो।
उत्तर:
करेवा।

प्रश्न 14.
दक्कन पठार की पश्चिमी सीमा का नाम बताएं।
उत्तर:
अरावली।

प्रश्न 15.
सिन्धु गार्ज तथा ब्रह्मपुत्र गार्ज के मध्य हिमालय का क्या विस्तार है?
उत्तर:
2400 किलोमीटर।

प्रश्न 16.
हिमालय में सबसे ऊंची चोटी माउंट एवरेस्ट की कुल उंचाई बताएं।
उत्तर:
8848 मीटर।

प्रश्न 17.
भारत के उत्तरी मैदान का पूर्व-पश्चिम विस्तार बताएं।
उत्तर:
3200 किलोमीटर।

प्रश्न 18.
गंगा मैदान में तलछट की अधिकतम ऊंचाई कितनी है?
उत्तर:
2000 मीटर।

प्रश्न 19.
हिमालय के निचले भागों में पाई जाने वाली निक्षेप बताओ।
उत्तर:
जलोढ़ पंक

JJAC Class 11 Geography Important Questions Chapter 2 संरचना तथा भूआकृति विज्ञान

प्रश्न 20.
चो के लिए प्रसिद्ध क्षेत्र का नाम बताएं।
उत्तर:
होशियारपुर (पंजाब)।

प्रश्न 21.
भारत के प्रायद्वीपीय पठार की औसत ऊंचाई बताएं।
उत्तर:
600-900 मीटर।

प्रश्न 22.
उस क्षेत्र का नाम बताएं जहां बरखान पाये जाते हैं।
उत्तर:
जैसलमेर

प्रश्न 23.
प्रायद्वीपीय भारत के उत्तर-पूर्व में पाये जाने वाले पठार का नाम बताएं।
उत्तर:
शिलांग पठार तथा कार्वी एंगलोंग पठार।

प्रश्न 24.
दक्कन पठार के ढलान की दिशा बताओ।
उत्तर:
दक्षिण पूर्व।

प्रश्न 25.
भारत के उस क्षेत्र का नाम बताएं जहां ग्रेनाइट तथा नीस चट्टानें पाई जाती हैं।
उत्तर:
कर्नाटक

प्रश्न 26.
पश्चिमी घाट पर सबसे अधिक ऊंचाई कितनी है?
उत्तर:
1600 मीटर।

JJAC Class 11 Geography Important Questions Chapter 2 संरचना तथा भूआकृति विज्ञान

प्रश्न 27.
पूर्वी घाट पर सबसे अधिक ऊंचाई कितनी है?
उत्तर:
900 मीटर।

प्रश्न 28.
अरब सागर में पाये जाने वाले मूंगे के द्वीपों के समूह का नाम बताएं।
उत्तर:
लक्षद्वीप समूह।

प्रश्न 29.
प्रायद्वीपीय भारत में सबसे ऊँची चोटी का नाम बताएं।
उत्तर:
अनाईमुदी (2695 मीटर)।

प्रश्न 30.
पश्चिमी तटीय मैदान के दो विभागों के नाम लिखो।
उत्तर:
कोंकण तट, मालाबार तट।

JJAC Class 11 Geography Important Questions Chapter 2 संरचना तथा भूआकृति विज्ञान

प्रश्न 31.
यदि आपने लक्षद्वीप तक यात्रा करनी है तो किस तटीय मैदान से गुजरेंगे?
उत्तर:
पश्चिमी तटीय मैदान।

प्रश्न 32.
भारत में शीत मरुस्थल कहां है?
उत्तर:
लद्दाख में।

प्रश्न 33.
पश्चिमी तट पर डैल्टे क्यों नहीं हैं?
उत्तर:
तीव्र गति वाली छोटी नदियां तलछट का जमाव नहीं करतीं।

प्रश्न 34.
इण्डियन प्लेट की स्थिति बताओ।
उत्तर:
भूमध्य रेखा के दक्षिण में।

प्रश्न 35.
कौन-सी भ्रंश रेखा मेघालय पठार को छोटा नागपुर पठार से अलग करती है?
उत्तर:
मालदा भ्रंश रेखा।

JJAC Class 11 Geography Important Questions Chapter 2 संरचना तथा भूआकृति विज्ञान

प्रश्न 36.
प्रायद्वीप की अवशिष्ट पहाड़ियां बताओ।
उत्तर:
अरावली, नल्लामाला, जांबादी, वेलीकोण्डा, पालकोण्डा, महेन्द्रगिरी ।

प्रश्न 37.
हिमालय की केन्द्रीय अक्षीय श्रेणी बताओ।
उत्तर:
बृहत् हिमालय।

प्रश्न 38.
उत्तर:
पश्चिमी हिमालय के दर्रे बताओ।
उत्तर:
बृहत् हिमालय में जोजीला, जास्कर में कोटला, पीर पंजाल में बनिहावा, लद्दाख श्रेणी में खुर्द भंगा।

प्रश्न 39.
उत्तर-पश्चिमी हिमालय में 3 तीर्थ स्थान बताओ।
उत्तर:
वैष्णो देवी, अमरनाथ गुफ़ा, चरार-ए-शरीफ़।

प्रश्न 40.
लघु हिमालय को हिमाचल में क्या कहते हैं?
उत्तर:
नागतीमा।

JJAC Class 11 Geography Important Questions Chapter 2 संरचना तथा भूआकृति विज्ञान

प्रश्न 41.
फूलों की घाटी कहां स्थित है?
उत्तर:
बृहत् हिमालय में।

प्रश्न 42.
मालाबार तट पर कयाल का क्या प्रयोग है?
उत्तर:
मछली पकड़ना तथा नौकायन।

प्रश्न 43.
दिसम्बर, 2004 में पूर्वी तट पर कौन-सी आपदा का प्रभाव पड़ा है?
उत्तर:
सुनामी

प्रश्न 44.
उत्तरी मैदान की रचना कैसे हुई है?
उत्तर:
गंगा, सिन्धु, ब्रह्मपुत्र नदियों द्वारा जलोढ़ निक्षेप से।

प्रश्न 45.
प्रायद्वीपीय पठार पर धरातलीय विविधता क्यों पाई जाती है?
उत्तर:
भू-उत्थान, निभज्जन व भ्रंश क्रिया के कारण।

प्रश्न 46.
गारो व खासी पहाड़ियाँ किस पर्वत श्रेणी का भाग हैं?
उत्तर:
हिमालय का। स्मरणीय तथ्य (Points to Remember):

भारत के धरातलीय भाग:

  1. उत्तरी पर्वतीय प्रदेश
  2. सतलुज-गंगा का मैदान
  3. प्रायद्वीपीय पठार
  4. तटीय मैदान
  5. द्वीप समूह
  6. मरुस्थल

हिमालय पर्वत की श्रेणियां तथा प्रादेशिक विभाग:

  1. ट्रांस हिमालय
  2. महान् हिमालय
  3. लघु हिमालय
  4. शिवालिक

(क) असम हिमालय
(ग) कुमायुँ हिमालय
(ख) नेपाल हिमालय
(घ) पंजाब हिमालय

हिमालय पर्वत के प्रमुख शिखर:

  1. माऊंट एवरेस्ट – 8848 मीटर
  2. गाडविन ऑस्टिन – 8611 मीटर
  3. कंचनजंगा – 8588 मीटर
  4. मकालू – 8481 मीटर
  5. धौलागिरी – 8172 मीटर
  6. नांगा पर्वत – 8126 मीटर
  7. अन्नपूर्णा – 8078 मीटर

लघु उत्तरीय प्रश्न (Short Answer Type Questions)

प्रश्न 1.
भांगर से आप क्या समझते हैं?
उत्तर:
प्राचीन जलोढ़ मिट्टी के बने ऊंचे मैदानी प्रदेशों को भांगर कहते हैं। इन उच्च प्रदेशों में नदियों की बाढ़ का जल पहुंच नहीं पाता। इस प्रदेश की मिट्टी में चीका मिट्टी, रेत तथा कंकड़ पाए जाते हैं। भारत के उत्तरी मैदान में नदियों द्वारा जलोढ़ मिट्टी के निक्षेप से भांगर प्रदेश की रचना हुई है।

प्रश्न 2.
भारत में भ्रंशन क्रिया (Faulting) के प्रमाण किन क्षेत्रों में मिलते हैं?
उत्तर:
भू-पृष्ठीय भ्रंशन के प्रमाण सामान्य रूप से दक्षिणी पठार पर पाए जाते हैं। गोदावरी, महानदी तथा दामोदर घाटियों में भ्रंशन के प्रमाण पाए जाते हैं। नर्मदा तथा ताप्ती नदी घाटी दरार घाटियां हैं। भारत के पश्चिमी तट पर मालाबार तट तथा मेकरान तट पर धरातल पर भ्रंशन क्रिया के प्रभाव देखे जा सकते हैं।

JJAC Class 11 Geography Important Questions Chapter 2 संरचना तथा भूआकृति विज्ञान

प्रश्न 3.
दून किसे कहते हैं? हिमालय पर्वत से तीन उदाहरण दीजिए।
उत्तर:
हिमालय पर्वत की समानान्तर श्रेणियों के मध्य सपाट तलछटी वाली संरचनात्मक घाटियां मिलती हैं। इन घाटियों द्वारा पर्वत श्रेणियां एक-दूसरे से अलग होती हैं। इन घाटियों को ‘दून’ (Doon) कहा जाता है। हिमालय पर्वत में इन उदाहरण अग्रलिखित हैं

  1. देहरादून (Dehra Dun)
  2. कोथरीदून (Kothri Dun)
  3. पटलीदून (Patli Dun)

कश्मीर घाटी को भी हिमालय पर्वत में एक दून की संज्ञा दी जाती है।

प्रश्न 4.
डेल्टा किसे कहते हैं? भारत से चार उदाहरण दीजिए।
उत्तर:
नदियों के मुहाने पर तलछट के निक्षेप से एक त्रिभुजाकार स्थल रूप बनता है जिसे डेल्टा कहते हैं। डेल्टा नदी के अन्तिम भाग में अपने भार के निक्षेप से बनने वाला भू-आकार है। यह एक उपजाऊ समतल प्रदेश होता है। भारत में चार प्रसिद्ध डेल्टा इस प्रकार हैं;

  1. गंगा नदी का डेल्टा
  2. महानदी का डेल्टा
  3. कृष्णा नदी का डेल्टा
  4. कावेरी नदी का डेल्टा।

प्रश्न 5.
तराई से आप क्या समझते हैं?
उत्तर:
हिमालय पर्वत के दामन में भाबर के मैदान के साथ-साथ तराई का संकरा मैदान स्थित है। यह मैदान लगभग 30 कि०मी० चौड़ा है। इस मैदान का अधिकतर भाग दलदली है क्योंकि भाबर प्रदेशों में लुप्त हुई नदियों का जल रिस-रिस कर इस प्रदेश को अत्यधिक आर्द्र कर देता है। इस प्रदेश में ऊंची घास तथा वन पाए जाते हैं। भाबर के दक्षिण में स्थित ये मैदान बारीक कंकड़, रेत, चिकनी मिट्टी से बना है। उत्तर प्रदेश में इस क्षेत्र में बड़े-बड़े फार्म बना कर कृषि की जा रही है।

प्रश्न 6.
भाबर क्या है? भाबर पट्टी के दो प्रमुख लक्षण बताओ।
उत्तर:
बाह्य हिमालय की शिवालिक श्रेणियों के दक्षिण में इनके गिरिपद प्रदेश को भाबर का मैदान कहते हैं। पर्वतीय क्षेत्र से बहने वाली नदियों के मन्द बहाव के कारण यहां बजरी, कंकड़ का जमाव हो जाता है। इस क्षेत्र में पहुंच कर अनेक नदियां लुप्त हो जाती हैं। क्योंकि यह प्रदेश पारगम्य चट्टानों (Pervious Rocks) का बना हुआ है। भाबर का मैदान एक संकरी पट्टी के रूप में 8 से 16 कि०मी० की चौड़ाई तक पाया जाता है। भाबर पट्टी के प्रमुख लक्षण:

  1. यह प्रदेश पारगम्य चट्टानों का बना हुआ है जिस में छोटी-छोटी नदियों का जल भूमिगत हो जाता है।
  2. इसमें बजरी और पत्थर के छोटे-छोटे टुकड़ों के निक्षेप जमा होते हैं।
  3. यह प्रदेश हिमालय पर्वत तथा उत्तरी मैदान के संगम पर स्थित है।

प्रश्न 7.
दोआब से आप क्या समझते हैं? भारतीय उपमहाद्वीप से पांच उदाहरण दीजिए।
उत्तर:
दो नदियों के मध्य के मैदानी भाग को दोआब कहते हैं। नदियों द्वारा निक्षेप से पुरानी कांप मिट्टी के प्रदेश इन नदियों को एक-दूसरे से अलग करते हैं। भारतीय उप महाद्वीप में निम्नलिखित दोआब मिलते हैं

  1. गंगा-यमुना नदियों के मध्य दोआब।
  2. ब्यास – सतलुज नदियों के मध्य बिस्त जालन्धर दोआब।
  3. ब्यास- रावी के मध्य बारी दोआब।
  4. रावी – चनाब के मध्य रचना दोआब।
  5.  चनाब – झेलम के मध्य छाज दोआब।

JJAC Class 11 Geography Important Questions Chapter 2 संरचना तथा भूआकृति विज्ञान

प्रश्न 8.
बृहत् स्तर पर भारत को कितनी भू-आकृतिक इकाइयों में विभाजित किया जा सकता है? उनके नाम लिखिए।
उत्तर:
भारत के तीन भू-आकृतिक विभागों के उच्चावच के लक्षणों का विकास एक लम्बे काल में हुआ है।

  1. उत्तर में हिमालय पर्वतीय श्रृंखला
  2. उत्तरी भारत का मैदान
  3. प्रायद्वीपीय पठार।

प्रश्न 9.
भारतीय पठार के प्रमुख भौतिक विभागों के नाम बताइए।
उत्तर:
प्रायद्वीपीय पठार की भौतिक स्थलाकृतियों में बहुत विविधता है। फिर भी इसे मोटे तौर पर अग्रलिखित भौतिक इकाइयों में बांटा जा सकता है।

  1. दक्षिणी पठारी खंड
  2. दक्कन का लावा पठार
  3. मालवा का पठार
  4. अरावली पहाड़ियां
  5. नर्मदा तथा तापी की द्रोणियां
  6. महानदी, गोदावरी तथा कावेरी की नदी घाटियां
  7. संकरे तटीय मैदान।

प्रश्न 10.
हिमालय पर्वत को किन-किन श्रेणियों में बांटा जाता है?
उत्तर:
हिमालय पर्वत में कई श्रेणियां एक-दूसरे के समानान्तर पाई जाती हैं। ये श्रेणियां एक-दूसरे से ‘दून’ नामक घाटियों द्वारा अलग-अलग हैं। भौगोलिक दृष्टि से हिमालय पर्वत के केन्द्रीय अक्ष के समानान्तर तीन पर्वत श्रेणियां हैं

  1. बृहत् हिमालय (Greater Himalayas)
  2. लघु हिमालय (Lesser Himalayas)
  3. उप-हिमालय (Sub – Himalayas) या शिवालिक श्रेणी ( Shiwaliks)

उक्त पर्वत श्रेणियों को तीन अन्य नामों से भी पुकारा जाता है

  1. आन्तरिक हिमालय ( Inner Himalayas)
  2. मध्य हिमालय (Middle Himalayas)
  3. बाह्य हिमालय ( Outer Himalayas)।

प्रश्न 11.
हिमालय पर्वत में मिलने वाले ऊंचे पर्वत शिखर तथा उनकी ऊंचाई बताओ।
उत्तर:
बृहत् हिमालय में संसार के 40 ऐसे पर्वत शिखर मिलते हैं जिनकी ऊंचाई 7000 मीटर से भी अधिक है। जैसे-

  1. एवरेस्ट (Everest ) – 8848 मीटर
  2. कंचनजंगा (Kanchenjunga ) –  8598 मीटर
  3. नांगा पर्वत (Nanga Parbat ) – 8126 मीटर
  4. नंदा देवी (Nanda Devi ) – 7817 मीटर
  5. नामचा बरवा (Namcha Barwa ) – 7756 मीटर
  6. धौलागिरी ( Dhaulagiri ) – 8172 मीटर।

JJAC Class 11 Geography Important Questions Chapter 2 संरचना तथा भूआकृति विज्ञान

प्रश्न 12.
” पश्चिमी हिमालय में पर्वत श्रेणियों की एक क्रमिक श्रृंखला पाई जाती है।” व्याख्या करो।
उत्तर: हिमालय पर्वत में कई पर्वत श्रेणियां एक-दूसरे के समानान्तर पाई जाती हैं। ये श्रेणियां एक-दूसरे से “दून” या घाटियों द्वारा अलग-अलग हैं। पश्चिमी हिमालय में ये श्रेणियां स्पष्ट क्रम से दिखाई देती हैं। पंजाब के मैदानों के पश्चात् पहली श्रेणी शिवालिक की पहाड़ियों के रूप में या बाह्य हिमालय के रूप में मिलती है। इसके पश्चात् सिन्धु नदी की सहायक नदियों की घाटियां हैं। दूसरी वेदी (Stage) के रूप में पीर पंजाल तथा धौलाधार की लघु हिमालयी श्रेणियां मिलती हैं। पीर पंजाल तथा महान् हिमालय के मध्य कश्मीर घाटी है। तीसरी वेदी के रूप में महान् हिमालय की जास्कर श्रेणी पाई जाती है। इस से आगे लद्दाख तथा कराकोरम की पर्वत श्रेणियां हैं जिसके मध्य सिन्धु घाटी मिलती है ।

प्रश्न 13.
हिमालय पर्वत श्रेणियों में पाये जाने वाले प्रमुख दर्रों के नाम लिखें।
उत्तर:

  1. खैबर दर्रा – यह पाकिस्तान और अफगानिस्तान के बीच में है।
  2. वोलन दर्रा – यह पाकिस्तान में है। प्राचीन समय में यह व्यापारिक मार्ग रहा है
  3. जोजिला – यह कशमीर से लेह को जोड़ता है।
  4.  शिपक़िला – हिमाचल प्रदेश के किन्नौर से तिब्बत को जोड़ता है।

तुलनात्मक प्रश्न (Comparison Type Questions)

प्रश्न 1.
भारतीय पठार तथा हिमालय पर्वत के उच्चावच के लक्षणों में वैषम्य बताइए।
उत्तर:
हिमालय पर्वत तथा भारती पठार की भू-आकृतियों की इकाइयों के भौतिक लक्षणों में काफ़ी अन्तर पाए जाते

हिमालय पर्वत भारतीय पठार
(1) हिमालय पर्वत एक युवा, नवीन मोड़दार पर्वत है। (1) भारतीय पठार कठोर चट्टानों का बना प्राचीन भूखण्ड है।
(2) इन पर्वतों का निर्माण विभिन्न हलचलों द्वारा वलन क्रिया से हुआ है। (2) इस पठार का निर्माण एक उत्खण्ड (Horst) के रूप में हुआ है।
(3) हिमालय पर्वत का धरातल युवा लक्षण प्रकट करता है। (3) इस पठार का धरातल जीर्ण तथा घर्षित है।
(4) हिमालय पर्वत पर ऊंची तथा समानान्तर पर्वत श्रेणियां पाई जाती हैं। (4) इस पठार पर दरारों के कारण दरार घाटियां मिलती हैं।
(5) इन पर्वतों पर गहरे गार्ज पाए जाते हैं तथा यू-आकार श्रेणियां पाई जाती हैं। (5) इस पठार पर गहरी नदी घाटियां पाई जाती हैं।
(6) ये पर्वत एक चाप के रूप में फैले हुए हैं। (6) इस पठार का आकार तिकोना है।
(7) इन पर्वतों में संसार के ऊंचे शिखर पाए जाते हैं। (7) इस पठार पर अपरदित पहाड़ियां पाई जाती हैं।
(8) ये तलछटी चट्टानों से बने हुए हैं। (8) ये आग्नेय चट्टानों से बने हैं।
(9) इनकी रचना आज से 2760 लाख वर्ष पूर्व Mesozoic Period में हुई है। (9) इसकी रचना आज से 16000 लाख वर्ष पूर्व PreCambrian Period में हुई है।

प्रश्न 2.
तराई तथा भाबर प्रदेश में अन्तर स्पष्ट करो। Cambrian Period में हुई है।
उत्तर:

तराई (Terai) भाबर (Bhabar)
(1) भाबर प्रदेश के साथ-साथ दक्षिण में तराई क्षेत्र स्थित है। (1) शिवालिक पहाड़ियों के तटीय क्षेत्र में भाबर प्रदेश स्थित है।
(2) यह एक नम, दल-दली तथा जंगलों से ढका प्रदेश है जहां कंकड़ जमा होते हैं। (2) यह प्रवेशीय चट्टानों से बना क्षेत्र है जहां भारी पत्थर तथा कंकड़ जमा होते हैं।
(3) इसकी चौड़ाई 20 से 30 कि० मी० है। (3) इसकी चौड़ाई 8 से 16 कि० मी० है।
(4) भाबर प्रदेश से रिस-रिस कर आने वाला जल यहां नदियों का रूप धारण कर लेता है। (4) प्रवेशीय चट्टानों के कारण यहां नदियां विलीन हो जाती हैं।
(5) यह प्रदेश कृषि के उपयुक्त है। (5) यह प्रदेश कृषि के उपयुक्त नहीं है।

प्रश्न 3.
बांगर तथा खादर प्रदेश में क्या अन्तर है?
उत्तर:

बांगर (Bangar) खादर (Khaddar)
(1) पुराने जलोढ़ निक्षेप से बने ऊंचे प्रदेश को बांगर कहते हैं। (1) बाढ़ की नवीन मिट्टी से बने निचले प्रदेश को खादर कहते हैं।
(2) ये प्रदेश बाढ़ के मैदान के तल से ऊंचे होते हैं। (2) यहां नदियां बाढ़ के कारण प्रति वर्ष जलोढ़ की नई परत बिछा देती हैं।
(3) ये प्रदेश चूनायुक्त कंकड़ों से बने होते हैं। (3) ये उपजाऊ चीका मिट्टी से बने प्रदेश होते हैं।
(4) ये बाढ़ का पानी नहीं पहुंच पाता। (4) ये वास्तव में नदियों के बाढ़ के मैदान हैं।
(5) कई प्रदेशों में इन्हें ‘धाया’ कहा जाता है। (5) कई प्रदेशों में इन्हें ‘बेट’ कहा जाता है।

प्रश्न 4.
पूर्वी और पश्चिमी तटीय मैदानों के बीच पाए जाने वाले तीन प्रमुख स्थलाकृतिक अन्तर बताइए।
उत्तर:

पश्चिमी तटीय मैदान पूर्वी तटीय मैदान
(1) यह मैदान 50 से 80 किलोमीटर चौड़ा है। यह एक संकरा मैदान है। (1) यह मैदान 80 से 100 किलोमीटर तक चौड़ा है। यह एक अधिक चौड़ा मैदान है।
(2) पश्चिमी तट पर कई लैगून झीलें पाई जाती हैं विशेषकर केरल तट पर। (2) पूर्वी तट पर लैगून कम संख्या में पाए जाते हैं।
(3) इस मैदान पर छोटी और तीव्र नदियों के कारण डेल्टे नहीं बनते। (3) इस मैदान में लम्बी और धीमी नदियों के कारण विशाल डेल्टे बनते हैं।

निबन्धात्मक प्रश्न (Essay Type Questions)

प्रश्न 1.
” उप-महाद्वीप के वर्तमान भू-आकृतिक विभाग एक लम्बे भूगर्भिक इतिहास के दौरे में विकसित हुए हैं।” इस कथन की व्याख्या कीजिए। भारत के भू-आकृतिक खण्ड की व्याख्या कीजिए।
अथवा
उत्तर: भारतीय उपमहाद्वीप की तीनों भू-आकृतिक इकाइयां इतिहास के लम्बे उतार-चढ़ाव के दौरे में विकसित हुई हैं। इनके निर्माण के सम्बन्ध में अनेक प्रकार के भू-वैज्ञानिक प्रमाण दिए जाते हैं। फिर भी अतीत अपना रहस्य छिपाए हुए है। इनकी रचना प्राचीन काल से लेकर कई युगों में क्रमिक रूप में हुई है।

1. प्रायद्वीपीय पठार:
इस पठार की रचना कैम्ब्रियन पूर्व युग में हुई है। कुछ विद्वानों की धारणा है कि यह एक उत्खण्ड (HORST) है जिसका उत्थान समुद्र से हुआ है। इस पठार के पश्चिमी भाग में अरावली पर्वत का उत्थान दक्षिण में नाला मलाई पर्वतमाला का उत्थान विंध्य – महायुग में हुआ । इस स्थिर भाग में एक लम्बे समय तक भू-गर्भिक हलचलों का कोई प्रभाव नहीं पड़ा । कुछ भागों में धरातल पर भ्रंश पड़ने के कारण धंसाव के प्रमाण मिलते हैं । हिमालय के निर्माण के पश्चात् पठार के उत्तर-पश्चिमी भाग के धंसने के कारण अरब सागर का निर्माण प्लिओसीन युग में हुआ। इस पठार को विशाल गोंडवाना महाद्वीप का भाग माना जाता है। इसका कुछ भाग अब भी उत्तरी मैदान के नीचे छिपा हुआ है। हिमालय के उत्थान के समय पठार के उत्तर-पश्चिमी भाग में विस्तृत रूप से ज्वालामुखी उदगार हुए जिस से दक्कन लावा क्षेत्र (Deccan Trap) का निर्माण हुआ। पठार के पश्चिम भाग में निमज्जन से पश्चिमी घाट ऊपर उभरे। दूसरी ओर पूर्वी तट शान्त क्षेत्र रहे।

2. हिमालय पर्वत:
यह एक युवा तथा नवीन मोड़दार पर्वत है। मध्यजीवी काल तक यह पर्वत एक भू-अभिनति द्वारा घिरा हुआ था इसे ‘टैथीस सागर’ कहते हैं। टरशरी युग में टैथीस सागर में जमा तलछटों में वलन पड़ने से हिमालय पर्वत तथा इसकी श्रृंखलाओं का निर्माण हुआ। उत्तरी भू-खण्ड अंगारालैण्ड की ओर से दक्षिणी भू-खण्ड गोंडवाना लैण्ड की ओर दबाव पड़ा। दक्षिणी भू-खण्ड के उत्तर अभिमुखी दबाव ने टैथीस सागर में जमा तलछट को ऊँचा उठा दिया जिससे हिमालय पर्वत में वलनों का निर्माण हुआ।

हिमालय पर्वत में पर्वत निर्माण कार्य हलचल की पहली अंवस्था अल्प नूतन युग में, दूसरी अवस्था मध्य नूतन युग में तथा तीसरी अवस्था उत्तर अभिनूतन युग में हुई। आधुनिक प्रमाणों के आधार पर ये पर्वत निर्माणकारी हलचलें (Mountain Building), प्लेट विवर्तनिकी (Plate tectonics) से सम्बन्धित है। भारतीय प्लेट उत्तर की ओर खिसकी तथा यूरेशिया प्लेट को नीचे से धक्का देने से हिमालय पर्वतमाला की उत्पत्ति हुई।

3. उत्तरी मैदान:
भारत का उत्तरी मैदान हिमालय पर्वत तथा दक्षिण पठार के मध्यवर्ती क्षेत्र में फैला है। यह मैदान एक समुद्री गर्त के भर जाने से बना है। इस गर्त में हिमालय पर्वत तथा दक्षिणी पठार से बहने वाली नदियां भारी मात्रा में मलबे के निक्षेप करती रही हैं। इस गर्त का निर्माण हिमालय पर्वत के उत्थान के समय एक अग्रगामी गर्त (Fore-deep) के रूप में हुआ। इसका निर्माण प्रायद्वीपीय पठार के उत्तर अभिमुखी दबाव के कारण हिमालय पर्वत के समान हुआ। यह सम्पूर्ण क्षेत्र निक्षेप की क्रिया द्वारा लगातार पूरित होता रहा है। यह क्रिया चतुर्थ महाकल्प तक जारी है। इस प्रकार लम्बी अवधि में भारत के वर्तमान भौगोलिक स्वरूप का विकास हुआ हैं।
JAC Class 11 Geography Important Questions Chapter 2 संरचना तथा भूआकृति विज्ञान 1

प्रश्न 2.
भारत को धरातल के आधार पर विभिन्न भागों में बांटो। हिमालय पर्वत का विस्तारपूर्वक विवरण दो।
उत्तर:
भारत एक विशाल देश है जिसमें धरातल पर अनेक विभिन्नताएं पाई जाती हैं। भारत की धरातलीय रचना एक विशेष प्रकार की है जिसमें ऊँचे-ऊँचे पर्वत, पठार तथा विशाल मैदान सभी प्रकार के भू-आकार विद्यमान हैं। भारत के कुल क्षेत्र का धरातल के अनुसार विभाजन इस प्रकार है।
पर्वत – 10.7%
पहाड़ियां – 18.6%
पठार – 27.7%
मैदान – 43.0%

प्रायद्वीप को भारत की प्राकृतिक संरचना का केन्द्र (Core of Geology of India) माना जाता है। यह भाग सबसे पुराना है। देश के अन्य भाग बाद में इसके चारों ओर बने हैं। उत्तर में हिमालय पर्वत है जो भारत का सिरताज है। (The Himalayas adorn like a crown of India.) इनके मध्य गंगा का विशाल मैदान है जिसे भारतीय सभ्यता का पलना (Cradle of Indian Civilization) कहा जाता है। भारत को धरातल के आधार पर चार स्पष्ट तथा स्वतन्त्र भागों में बांटा जा सकता है।

  1. उत्तरी पर्वतीय प्रदेश (Northern Mountain Region)
  2. सतलुज- गंगा का मैदान (Sutlej – Ganges Plain)
  3. प्रायद्वीपीय पठार (Peninsular Plateau)
  4. तटीय मैदान (Coastal Plains)

1. उत्तरी पर्वतीय क्षेत्र (Northern Mountain Region):
विस्तार (Extent ):
भारत के उत्तर में हिमालय पर्वत पश्चिम-पूर्व दिशा में एक चाप के आकार में फैला हुआ है। यह पर्वत श्रेणी 2400 किलोमीटर लम्बी है तथा 240 से 320 किलोमीटर तक चौड़ी है। ये संसार के सबसे ऊँचे पर्वत हैं जो बर्फ से ढके रहते हैं।

मुख्य विशेषताएं (Main Characteristics):
(i) ये पर्वत युवा नवीन मोड़दार (Young Fold Mountains) पर्वत हैं। अपनी अल्पायु के कारण इन्हें युवा पर्वत कहते हैं।

(ii) आज से लगभग 5 करोड़ साल पहले वहां पर टैथीज़ (Tethys) सागर था इस सागर की पेटी में जमा तलछट में मोड़ पड़ने से हिमालय पर्वत तथा इसकी श्रृंखलाएं बनीं। अब भी कुछ भागों में उठाव (Uplift) की क्रिया के कारण हिमालय पर्वत की ऊंचाई बढ़ रही है। ( “The Himalayas are still rising.”)
JAC Class 11 Geography Important Questions Chapter 2 संरचना तथा भूआकृति विज्ञान 2

(iii) हिमालय प्रदेश में एक-दूसरे के समानान्तर श्रेणियां मिलती हैं जिनके बीच घाटियां तथा पठार पाए जाते हैं। (“The Himalayas are series of parallel ranges, intersected by deep valleys and broad plateaus.”) हिमालय पर्वतमालाएं क्रमिक श्रृंखलाओं में पाई जाती हैं। उत्तरी मैदान से आगे पहली वेदी शिवालिक के रूप में, दूसरी वेदी पीर पंजाल तथा धौलाधार श्रेणी के रूप में (लघु हिमालय), तीसरी वेदी महान् हिमालय की जास्कर श्रेणी के रूप में तथा इससे आगे लद्दाख, कैलाश तथा कराकोरम (ट्रांस हिमालय श्रेणी) पर्वतमालाएं पाई जाती हैं।

(iv) इन श्रेणियों की तुलना धनुष की डोरी या तलवार से भी की जाती है।

(v) हिमालय पर्वत की औसत ऊंचाई 5000 मीटर है।

(vi) इसमें बहने वाली नदियां युवावस्था में हैं। तीव्र गति के कारण ‘V’ आकार की तंग तथा गहरी (‘V’ shaped narrow valley) घाटी बनाती है।

(vii) हिमालय पर्वत के पूर्व तथा पश्चिम में दो मोड़ हैं जिन्हें “दीर्घ पर्वतीय मोड़” या Hair pin bend कहा जाता है।

(viii) इन पर्वतों पर हिमानी के कार्य के प्रमाण पाए जाते हैं। जैसे – करेवा, यू-आकार घाटी तथा हिमनदियां मिलती हैं।

क्षेत्रीय विभाजन (Regional Division ): इन दीर्घ मोड़ों के बीच कश्मीर से लेकर असम तक, सिन्धु घाटी था ब्रह्मपुत्र नदियों के बीच फैले हुए हिमालय पर्वत को चार भागों में बांटा जाता है
(क) असम हिमालय: तिस्ता – ब्रह्मपुत्र नदी के मध्य का भाग (720 कि० मी० लम्बा क्षेत्र)
(ख) नेपाल हिमालय: काली – तिस्ता नदी के मध्य का भाग ( 800 कि० मी० लम्बा क्षेत्र)
(ग) कुमायूं हिमालय: सतलुज- काली नदी के मध्य का भाग (320 कि० मी० लम्बा क्षेत्र)
(घ) पंजाब हिमालय: सिन्धु – सतलुज नदी के मध्य का भाग ( 560 कि० मी० लम्बा क्षेत्र)

भौगोलिक दृष्टि से हिमालय पर्वत की ऊंचाई को देख कर इसे चार भागों में बांटा जा सकता है। ये भाग एक-दूसरे के समानान्तर पूर्व-पश्चिम दिशा में फैले हुए हैं:

  1. ट्रांस हिमालय (Trans Himalayas)
  2. महान् हिमालय (Great Himalayas) or (Inner Himalayas)
  3. लघु हिमालय (Lesser Himalayas) or (Middle Himalayas)
  4. उप हिमालय (Sub – Himalayas) or (Outer Himalayas)

1. ट्रांस हिमालय (Trans Himalayas):
ये भारत के उत्तर-पश्चिम में ऊंची तथा विशाल पर्वतमालाएं हैं जोकि महान् हिमालय के पीछे स्थित हैं। इनकी ऊंचाई 6000 मीटर से भी अधिक है। कराकोरम (Kara Koram), लद्दाख तथा कैलाश पर्वत मुख्य पर्वत श्रेणियां हैं। K, Mt. Godwin Austin 8611 मीटर संसार में दूसरी बड़ी ऊंची चोटी है।

2. महान् हिमालय (Great Himalayas ):
पश्चिम में सिन्धु घाटी तथा पूर्व में ब्रह्मपुत्र घाटी के बीच यह सबसे लम्बी पर्वत श्रेणी है। इसकी औसत ऊंचाई 6000 मीटर है। यह अत्यन्त दुर्गम क्षेत्र है। सदा बर्फ से ढके रहने के कारण इसे हिमाद्री (Snowy Ranges ) भी कहते हैं। इस पर्वतमाला को भीतरी हिमालय ( Inner Himalayas ) भी कहते हैं। इस भाग में झीलों के भर जाने के कारण 1500 मीटर की ऊंचाई पर दो प्रसिद्ध घाटियां हैं
JAC Class 11 Geography Important Questions Chapter 2 संरचना तथा भूआकृति विज्ञान 3
(क) काठमांडू घाटी तथा
(ख) कश्मीर घाटी।
इस श्रेणी में हिमालय के ऊंचे शिखर मिलते हैं। माऊंट एवरेस्ट (Mount Everest) (8,848 मीटर) संसार में सबसे ऊंचा शिखर है।

मुख्य शिखर (High Summits): मुख्य पर्वत शिखर इस प्रकार हैं

  1. माउंट एवरेस्ट 8,848 मीटर
  2. कंचनजंगा – 8,598 मीटर
  3. मकालू – 8,481 मीटर
  4. धौलागिरि – 8, 172 मीटर
  5. नांगा पर्वत – 8, 126 मीटर
  6. अन्नपूर्णा – 8,078 मीटर
  7. नन्दा देवी – 7,817 मीटर
  8. नामचा बरवा 7,756 मीटर

JAC Class 11 Geography Important Questions Chapter 2 संरचना तथा भूआकृति विज्ञान 4

(iii) लघु हिमालय (Lesser – Himalayas):
यह हिमालय की तीसरी श्रेणी है जिसकी औसत ऊंचाई 3700 मीटर से 4580 मीटर तक है। यह पर्वत श्रेणी 75 किलोमीटर की चौड़ाई से महान् हिमालय के दक्षिण में इसके समानान्तर फैली हुई है। इसे ” मध्य हिमालय” (Middle Himalayas) भी कहते हैं। इसमें कई पर्वत श्रेणियां शामिल हैं। जैसे—कश्मीर में पीर पंजाल, हिमाचल में धौलाधार, उत्तर प्रदेश में कुमायूं श्रेणी तथा भूटान में थिम्पू श्रेणी है। देश के कई स्वास्थ्यवर्द्धक स्थान जैसे – शिमला, मसूरी, नैनीताल, दार्जिलिंग आदि इस पर्वत श्रेणी में स्थित हैं। इन पर्वतीय ढलानों पर छोटे-छोटे घास के मैदान भी मिलते हैं। जैसे – कश्मीर में गुलमर्ग तथा सोनमर्ग।

(iv) उप हिमालय (Sub Himalayas):
यह हिमालय पर्वत की दक्षिण श्रेणी है तथा इसे शिवालिक (Shiwalik) भी कहते हैं। इन्हें हिमालय पर्वत की तलहटी (Foot hills) भी कहते हैं। यह हिमालय पर्वत से नदियों द्वारा लाई हुई मिट्टी, रेत, बजरी के निक्षेप से बनी है। इनकी औसत ऊंचाई 1000 मीटर है तथा चौड़ाई 15 से 50 किलोमीटर तक है।
JAC Class 11 Geography Important Questions Chapter 2 संरचना तथा भूआकृति विज्ञान 5
शिवालिक तथा लघु हिमालय के बीच समतल लम्बाकार घाटियां मिलती हैं जिन्हें ‘दून’ (Doons) कहते हैं। जैसे- देहरादून, कोथरीदून तथा पटलीदून।

पूर्वी शाखाएं (Eastern Off-shoots ):
दूर पूर्व की ओर हिमालय पर्वतमालाएं दक्षिण की ओर मुड़ जाती हैं। इस भाग में पटकोई (Patkoi ), लोशाई (Lushai) तथा नागा (Naga) पर्वत श्रेणियां हैं जो दक्षिण में अराकान योमा के रूप में बर्मा (म्यनमार) तथा भारत की सीमा बनाती हैं । इसकी एक शाखा में गारो (Garo ), खासी (Khasi), जैन्तिया (Jaintia) की पहाड़ियां हैं।

पश्चिमी शाखाएं (Western Off-shoots ):
दूर पश्चिम में हिमालय पर्वत मालाएं दक्षिण पर्वत की ओर मुड़ जाती हैं। इस भाग में साल्ट रेंज (Salt Range), सुलेमान, किरथार श्रेणियां हैं । इस भाग से हिन्दुकुश पर्वत मालाएं निकल कर अफगानिस्तान तक चली गई हैं।

प्रश्न 3.
सतलुज, गंगा मैदान के विस्तार, रचना तथा विभिन्न भागों का वर्णन करो।
उत्तर:
सतलुज- गंगा का मैदान (Sutlej-Ganges Plain)
विस्तार (Extent ):
हिमालय पर्वत तथा प्रायद्वीप के बीच पूर्व-पश्चिम में फैला हुआ विशाल मैदान है। यह लगभग 3,200 किलोमीटर लम्बा तथा 150 से 300 किलोमीटर तक चौड़ा है। असम में इस मैदान की चौड़ाई सब से कम 90-100 कि० मी० है। राज महल पहाड़ियों के पास 160 कि० मी० तथा इलाहाबाद के पास 280 कि० मी० है। इसकी औसत ऊंचाई, 150 मीटर है तथा इसका क्षेत्रफल 7.5 लाख वर्ग किलोमीटर है। यह मैदान सतलुज, गंगा तथा इनकी सहायक नदियों द्वारा निक्षेप के कार्य से बना है।

रचना (Formation):
गोंडवाना लैण्ड तथा हिमालय के मध्य एक लम्बी पेटी के रूप में भूमि का भाग धंस गया था। यह मैदान हिमालय पर्वत बनने के बाद बना। यह भाग धीरे-धीरे नदियों द्वारा तलछट के निक्षेप से भरता चला गया। (The great plain is an alluvium filled trough.) एक अनुमान के अनुसार गंगा नदी प्रति वर्ष 3000 लाख टन मिट्टी इस मैदान में बिछाया करती है।

विशेषताएं (Characteristics): इस मैदान की कई विशेषताएं हैं:

  1. समतल मैदान (Dead Flat Lowland):
    यह एक समतल सपाट मैदान है। ऊंचे भाग बहुत कम हैं। इसमें सबसे ऊंचा प्रदेश अम्बाला क्षेत्र है जिसकी ऊंचाई 283 मीटर है। इस मैदान के धरातल की एक रूपता प्रभाव पूर्ण है। कहीं- कहीं नदी निक्षेप से बनी वेदिकाएं (Terraces) बल्फ तथा भांगर मिलते हैं।
  2. धीमी ढाल (Gentle Slope ): गंगा के मैदान का ढाल औसत रूप से 1/4 मीटर प्रति किलोमीटर है।
  3. तलछट की गहराई (Thickness of Alluvium): हज़ारों साल से लगातार निक्षेप के कारण लगभग 2000 मीटर गहरी मिट्टी (Silt) मिलती है। यह ध्वनि से मापे जाने वाली गहराई है।
  4. अनेक नदियां (Many Rivers ): इस मैदान में नदियों का जाल बिछा हुआ है जिसके कारण सारा मैदान छोटे- छोटे टुकड़ों, दोआबों (Doabs) में बंट गया है। नदियां चौड़ी घाटियां बनाती हैं तथा धीमी बहती हैं।
  5. उपजाऊ मिट्टी ( Fertile Soils): छारी मिट्टी बहुत उपजाऊ है। धरातल में विभिन्नता केवल ‘खादर’ या ‘बांगर’ मिट्टी के कारण ही मिलती है। पुरानी कछारी मिट्टी से बने ऊंचे प्रदेशों को बांगर प्रदेश तथा नवीन कछारी मिट्टी से बने निचले प्रदेशों को खादर प्रदेश कहते हैं। यहां बाढ़ का पानी प्रति वर्ष नई मिट्टी की परत बिछा देता हैं। कहीं-कहीं कंकड़ तथा भूर की भू-रचना भी है।

उत्तरी मैदान का विभाजन (Division of Northern Plain): उत्तरी मैदान को नदी घाटियों के अनुसार निम्न- लिखित भागों में बांटा जा सकता है
(i) भाभर तथा तराई प्रदेश:
शिवालिक पहाड़ियों के दामन में तंग पेटियों के रूप में यह मैदान है। भाभर शिवालिक के साथ-साथ एक लगातार मैदान है जो सिन्धु से तिस्ता नदी तक फैला हुआ है। सरंधर चट्टानों के कारण नदियां इस क्षेत्र में विलीन हो जाती हैं। नदियों की धीमी गति के कारण बजरी तथा कंकर के निक्षेप से ‘भाभर’ का मैदान बना है, परन्तु तराई के मैदान में दलदली प्रदेश अधिक हैं। तराई क्षेत्र में नदियां पुनः धरातल पर प्रकट हो जाती हैं।

(ii) सतलुज घाटी:
सतलुज, ब्यास, रावी आदि नदियों के निक्षेप से पंजाब और हरियाणा का मैदान बना है। इस मैदान की ढाल दक्षिण-पश्चिम की ओर है। इसे पश्चिमी मैदान भी कहते हैं। दो नदियों के मध्य क्षेत्र को दोआब कहा जाता है । शिवालिक पहाड़ियों से निकलने वाली बरसाती नदियों (चो) के कारण मिट्टी कटाव की गम्भीर समस्या है। इस मैदान में ऊंचे भाग धाया तथा निचले भाग बेट कहलाते हैं।

(iii) गंगा का मैदान:
यह विशाल मैदान गंगा, यमुना, घाघरा, गोमती, गंडक आदि नदियों द्वारा निक्षेप से बना है। इसे गंगा की ऊपरी घाटी, मध्य घाटी तथा निचली घाटी नामक तीन भागों में बांटा जाता है। इस मैदान में पाए जाने वाले दोआब क्षेत्रों को गंगा-यमुना दोआब, रोहिलखण्ड तथा अवध प्रदेश के नाम से जाना जाता है। चम्बल घाटी में उत्खात भूमि (Bad land) की रचना हुई है।

(iv) ब्रह्मपुत्र घाटी:
इस मैदान के पूर्वी भाग में ब्रह्मपुत्र नदी असम घाटी बनाती है। समुद्र में गिरने से पहले गंगा तथा ब्रह्मपुत्र नदियां एक विशाल डेल्टे का निर्माण करती हैं जो 80,000 वर्ग किलोमीटर क्षेत्र में फैला हुआ है और संसार में सबसे बड़ा डेल्टाई प्रदेश है। इस डेल्टा के पुराने क्षेत्रों को चर (Char) तथा निचले क्षेत्रों को बिल (Bill) कहते हैं। उत्तरी भाग को पैरा- डेल्टा ( अपघर्षण क्षेत्र ) तथा दक्षिणी भाग को डेल्टा (निक्षेपण क्षेत्र) कहते हैं।

प्रश्न 4.
भारतीय प्रायद्वीपीय पठार की विशेषताएं तथा विभिन्न भागों का वर्णन करो।
अथवा
प्रायद्वीपीय पठार का भौगोलिक वर्णन करें।
उत्तर:
प्रायद्वीपीय पठार (Peninsular Plateau)
विस्तार (Extent ):
दक्षिण पठार भारत का सबसे प्राचीन भू-खण्ड है। यह पठार तीन ओर सागरों से घिरा हुआ है। इसलिए इसे प्रायद्वीपीय पठार कहते हैं। इसका क्षेत्रफल 16 लाख वर्ग किलोमीटर है। इस प्रायद्वीपीय पठार की औसत ऊंचाई 600 से 900 मीटर है। उत्तर पश्चिम में दिल्ली रिज से अरावली तथा कच्छ तक, गंगा यमुना के समानान्तर राजमहल की पहाड़ियां तथा शिलांग पठार इसकी मूल उत्तरी सीमा है। इसका शीर्ष कन्याकुमारी है । अरावली, शिलांग पठार, राजमहल पहाड़ियां इस त्रिभुज की भुजाएं हैं।

विशेषताएं (Characteristics):
1. यह भाग प्राचीन, कठोर रवेदार चट्टानों से बना हुआ है। यह एक स्थिर भू-भाग (Stable Block) है। यह पठार क्रमिक अपरदन के कारण घिस गया है तथा यहां वृद्धावस्था के चिन्ह मिलते हैं।

2. यह भाग प्राचीन समय में गोंडवाना लैण्ड (Gondwana Land) का ही भाग था जिसमें अफ्रीका, अरब, ऑस्ट्रेलिया तथा दक्षिणी अमेरिका के पठार शामिल थे।

3. इस प्रदेश का मुख्य भाग ज्वालामुखी उद्गार के कारण लावा के जमने से बना है।

4. यह प्रदेश भारत की खनिज सम्पत्ति का भण्डार है।

5. इस पठार का भीतरी भाग नदी घाटियों में कटा-छटा है। सपाट शिखर तथा गहरी द्रोणी घाटियां हैं। सीढ़ीदार ढाल,
Img-1
दरारें (Faults) प्राचीन पर्वतों के अवशेष, नीची पहाड़ियों की एक मुद्रिका, घिसे हुए पठार तथा भ्रांशित द्रोणियां मिलती हैं। यह अनाच्छादित पठार कगारों की एक श्रृंखला के रूप में उठा हुआ है।

6. यह पठार कई छोटे-छोटे भागों में बंट गया है।

7. प्रायद्वीपीय उच्च भूमि में विविधता के मुख्य कारण हैं- पृथ्वी की हलचलें, अपरदन क्रिया, उत्थान, निमज्जन, भ्रंशन तथा विभंगन क्रियाएं।

दक्षिणी पठार का विभाजन (Division of Deccan Plateau ) :
21° उत्तर तथा 24° उत्तरी अक्षांश के बीच पूर्व- पश्चिम दिशा में फैली हुई सतपुड़ा पहाड़ियों का क्रम (Line of Satpura Ranges) दक्षिणी पठार को दो भागों में बांटता

  1. मालवा का पठार (Malwa Plateau )
  2. दक्षिण का मुख्य पठार (Deccan Trap)

(i) मालवा का पठार:
पश्चिम में अरावली पर्वत, पूर्व में गंगा घाटी तथा दक्षिण में विन्ध्याचल पर्वत इस त्रिभुजाकार की सीमाएं बनाते हैं। अरावली पर्वत बचे-खुचे पर्वत (Relict Mountains) है। जो दिल्ली – रिज तक विस्तृत हैं। सबसे ऊंचा शिखर गुरु शिखर (Guru Shikhar ) 1,772 मीटर ऊंचा है। अरावली के पश्चिम में थार की बलुई मरुभूमि है जहां बरखान टीले मिलते हैं। इस पठार में बुन्देलखण्ड, चम्बल घाटी तथा मालवा की अस्त-व्यस्त धरातल तथा जल प्रवाह है। यह प्रदेश नीस व क्वार्टज़ाइट की प्राचीन कठोर चट्टानों का बना हुआ है। मालवा के पठार का पूर्वी भाग महादेव, मैकाल, राजमहल की पहाड़ियों के रूप में फैला हुआ है। सोन नदी के पूर्व में छोटा नागपुर का कटा छटा पठार है जो भारत का खनिज भण्डार है। यह पठार 1070 मीटर तक ऊंचा है।

(ii) दक्कन का मुख्य लावा पठार:
यह पठार नर्मदा नदी के दक्षिण में है। इसके तीन ओर पर्वत श्रेणियां हैं। पश्चिम में पश्चिमी घाट, पूर्व में पूर्वी घाट तथा उत्तर में सतपुड़ा की ओर पहाड़ियां इस पठार की सीमाएं बनाती हैं। विन्ध्य तथा सतपुड़ा पहाड़ियों के बीच नर्मदा एवं ताप्ती की द्रोणियां मिलती हैं। पूर्वी घाट पश्चिमी घाट के मध्य कर्नाटक पठार है। कर्नाटक पठार के दो भाग हैं मालन्द तथा मैदान मालन्द उच्च भूमि पर बाबा बूदन पहाड़ियां हैं। पूर्व में महानदी बेसिन में छत्तीसगढ़ के मैदान प्रसिद्ध हैं। इस पठार की ढाल उत्तर-पश्चिम (North West) से दक्षिण पूर्व (South East ) की ओर है जो इस प्रदेश की नदियों की दिशा से स्पष्ट है। (It is a titled plateau with a general eastward slope.) इन नदियों ने इस पठार को कई भागों में बांट दिया है

(क) पश्चिमी घाट (Western Ghats):
यह ताप्ती घाटी से लेकर कुमारी अन्तरीप तक 1500 किलोमीटर लम्बी तथा निरन्तर पर्वत श्रेणी है। इसमें केवल तीन दर्रे हैं- थाल घाट, भोरघाट तथा पालघाट । इस पर्वत की पश्चिमी ढाल पर छोटी तथा तीव्र बहने वाली नदियां हैं, परन्तु पूर्वी ढाल से उतरने वाली नदियां धीमी गति तथा अधिक लम्बाई के कारण डेल्टे (Deltas) बनाती हैं। अधिक वर्षा के कारण गोदावरी, कृष्णा, कावेरी नदियां पूर्व की ओर बहती हैं। इस पर्वतीय भाग की औसत ऊंचाई 1000 मीटर है। इस भाग में कई भौतिक इकाइयां मेज़नुमा उच्च भूमियां लगती हैं; जैसे- अजन्ता तथा बालाघाट।

(ख) पूर्वी घाट (Eastern Ghats ):
महानदी घाटी से लेकर नीलगिरि की पहाड़ियों तक 800 किलोमीटर में फैले हुए पूर्वी घाट पठार की पूर्वी सीमा बनाते हैं। इनकी औसत ऊंचाई 500 मीटर है। ये पर्वत मालाएं पश्चिमी घाट की अपेक्षा कम ऊंची तथा अधिक कटी-छटी हैं। इस पर्वत में कई भागों के बीच Wide Gaps मिलते हैं जिनमें से कई नदियां बहती हैं। उत्तर में ये छोटा नागपुर के पठार में मिल जाते हैं तथा दक्षिण में नीलगिरि की पहाड़ियों में । दक्षिण भाग में जावादी (Javadi), पालकोंडा ( Palkonda ), शिवराय (Shevaroy), नालामलाई (Naillamalai) की पहाड़ियां हैं।

(ग) नीलगिरि की पहाड़ियां (Nilgiri Hills):
पश्चिमी तथा पूर्वी घाट नीलगिरि की पहाड़ियों में मिल जाते हैं। ये भूखण्ड ग्रेनाइट तथा नीस चट्टानों से बना है। अनाई मुदी 2698 सबसे ऊंचा शिखर है। यह एक पर्वतीय गाठ (knot) है। इसके दक्षिण में अनामलाई ( Anaimalai ), पलनी (Palni) तथा कार्डामम (Cardamom) की पहाड़ियां हैं।

प्रश्न 5.
भारत के तटीय मैदानों तथा द्वीपों का वर्णन करें।
उत्तर:
तटीय मैदान (Coastal Plains): दक्षिणी पठार के पूर्वी तथा पश्चिमी किनारे पर तंग तटीय मैदान हैं जिन्हें पूर्वी तटीय मैदान तथा पश्चिमी तटीय मैदान कहते हैं।

  1. इन मैदानों में चावल की खेती की जाती है।
  2. तटों पर नारियल के कुंज पाए जाते हैं।
  3. मछलियां पकड़ने के उत्तम केन्द्र हैं।
  4. इन तटों पर भारत की प्रमुख बन्दरगाहें पाई जाती हैं।
  5. इन तटों पर नमक, मोनोज़ाइट खनिज, पेट्रोलियम के भण्डार प्राप्त हैं।

1. पूर्वी तटीय मैदान (Eastern Coastal Plain:
यह मैदान महानदी डेल्टा से लेकर कन्याकुमारी तक फैला हुआ है। यह 50 किलोमीटर से 500 किलोमीटर तक चौड़ा है। इस भाग में महानदी, गोदावरी, कृष्णा तथा कावेरी नदियों के चौड़े तथा उपजाऊ डेल्टाई प्रदेश हैं। इस तट पर रेत के टीले (Sand Dunes) मिलते हैं जिनके कारण चिलका झील तथा पुलीकट झील का निर्माण हुआ है। इस तट को कोरोमण्डल तट भी कहते हैं।

2. पश्चिमी तटीय मैदान (Western Coastal Plain ):
अब अरब सागर तथा पश्चिमी घाट के बीच एक तंग मैदान है जो उत्तर दक्षिण में फैला हुआ है। इस मैदान की चौड़ाई 50 किलोमीटर तक है। इस तट पर गिरने वाली नदियां तीव्र ढाल के कारण डेल्टा नहीं बनाती हैं। इस तटीय मैदान के उत्तरी भाग (गोआ से मुम्बई तक ) को कोंकण (Konkan) तट कहते हैं । गोआ से आगे दक्षिणी भाग को मालाबार (Malabar ) तट कहते हैं। इस तट के साथ-साथ लम्बी झीलें है (Lagoons) मिलती हैं जिन्हें एक-दूसरे से मिलाकर जल मार्ग के रूप में प्रयोग किया जाता

द्वीप (Islands):
हिन्द महासागर में लगभग 550 द्वीप पाए जाते हैं। इन द्वीपों के तीन समूह निम्नलिखित हैं।
(i) अंडमान-निकोबार द्वीप समूह (Andaman Nicobar Islands ):
ये द्वीप 60°N से 14°N अक्षांश के मध्य स्थित है। अंडमान द्वीप समूह में 214 द्वीप हैं जबकि निकोबार द्वीप समूह में 19 द्वीप हैं। 10° चैनल इन द्वीप समूहों को पृथक् करती हैं। ये द्वीप 500 कि० मी० उत्तर दक्षिण में फैले हुए हैं। महासागरीय तट में डूबी पहाड़ियों के शिखर द्वीप के रूप में स्थित हैं।
JAC Class 11 Geography Important Questions Chapter 2 संरचना तथा भूआकृति विज्ञान 7

(ii) बैरन द्वीप तथा नारकण्डम द्वीप (Barren Island and Norcondam Island): ये ज्वालामुखी द्वीप हैं। भारत में केवल एक सक्रिय ज्वालामुखी बैरन द्वीप हैं।

(iii) लक्षद्वीप (Laskhadweep): अरब सागर में प्रवाल निक्षेपों से बने लक्षद्वीप समूह हैं। ये केरल तट से 320 कि० मी० दूर 8°N से 12°N तक फैले हुए हैं। इन्हें अटाल (Atoll) भी कहते हैं। इनकी संख्या 27 है।

JAC Class 11 Geography Important Questions Chapter 2 संरचना तथा भूआकृति विज्ञान 8

JAC Class 11 Geography Important Questions Chapter 1 भारत – स्थिति 

Jharkhand Board JAC Class 11 Geography Important Questions Chapter 1 भारत – स्थिति Important Questions and Answers.

JAC Board Class 11 Geography Important Questions Chapter 1 भारत – स्थिति

बहु-विकल्पी प्रश्न (Multiple Choice Questions)

प्रश्न-दिए गए चार वैकल्पिक उत्तरों में से सही उत्तर चुनिए
1. भारत का कुल भौगोलिक क्षेत्रफल (लाख km) है
(A) 32.80
(B) 22.80
(C) 42.08
(D) 30.80.
उत्तर:
(A) 32.80.

2. कौन-सी अक्षांश रेखा भारत को दो भागों में बांटती है?
(A) भूमध्य रेखा
(B) कर्क रेखा
(C) मकर रेखा
(D) आर्कटिक वृत्त।
उत्तर:
(B) कर्क रेखा।

3. भारत का (क्षेत्रफल के अनुसार) सबसे बड़ा राज्य है
(A) महाराष्ट्र
(B) उत्तर प्रदेश
(C) राजस्थान
(D) मध्य प्रदेश।
उत्तर:
(C) राजस्थान।

4. स्वेज नहर किस वर्ष आरम्भ हुई?
(A) 1849
(B) 1859
(C) 1869
(D) 1879
उत्तर:
(C) 1869.

5. सिक्किम राज्य की राजधानी है
(A) दिसपुर
(B) शिलांग
(C) गंगटोक

6. भारत में कुल राज्य हैं-
(A) 18
(B) 24
(C) 28
(D) 30.
उत्तर:
(C) 28.

JAC Class 11 Geography Important Questions Chapter 1 भारत – स्थिति

7. क्षेत्रफल के अनुसार भारत का विश्व में स्थान है
(A) पांचदां
(B) छठा
(C) सातवां
(D) आठवां।
उत्तर:
(C) सातवां।

8. लक्षद्वीप कहां स्थित है?
(A) खाड़ी बंगाल
(B) अरब सागर
(C) हिन्द महासागर
(D) खम्बात खाड़ी
उत्तर:
(B) अरब सागर।

9. भारतीय संघ का दक्षिणतम बिन्दु है-
(A) कन्याकुमारी
(B) इन्दिरा पुआइंट
(C) रामेश्वरम
(D) बैरन द्वीप।
उत्तर:
(B) इन्दिरा पुआइंट।

10. भारत की कुल स्थल सीमा है-
(A) 12200 कि० मी०
(B) 13202 कि० मी०
(C) 14200 कि० मी०
(D) 15200 कि० मी०
उत्तर:
(D) 15200 कि० मी०।

11. भारत की प्रामाणिक देशान्तर रेखा कहां से गुज़रती है?
(A) श्रीनगर
(B) दिल्ली
(C) मिर्ज़ापुर
(D) कोलकाता।
उत्तर:
(C) मिर्ज़ापुर।

12. भारत की तट रेखा है
(A) 10500 कि० मी०
(B) 7500 कि० मी०
(C) 3500 कि० मी०
(D) 9500 कि० मी०।
उत्तर:
(B) 7500 कि० मी०

13. कर्क रेखा किस राज्य से नहीं गुजरती है?
(A) राजस्थान
(B) उड़ीसा
(C) छत्तीसगढ़
(D) त्रिपुरा
उत्तर:
(B) उड़ीसा।

JAC Class 11 Geography Important Questions Chapter 1 भारत – स्थिति

14. ग्रीष्मावकाश में आप यदि कवरत्ती जाना चाहते हैं तो किस केन्द्र शासित क्षेत्र में जाएंगे?
(A) पुड्डुचेरी
(B) लक्षद्वीप
(C) अण्डमान और निकोबार
(D) दीव और दमन।
उत्तर:
(B) लक्षद्वीप।

15. मेरे मित्र एक ऐसे देश के निवासी हैं जिस देश की सीमा भारत के साथ नहीं लगती है। आप बताइए, वह कौन-सा देश है?
(A) भूटान
(B) तज़ाकिस्तान।
(C) बांग्लादेश
(D) नेपाल
उत्तर:
(B) तज़ाकिस्तान

अति लघु उत्तरीय प्रश्न (Very Short Answer Type Questions)

प्रश्न 1.
कौन-सी अक्षांश रेखा भारत के केन्द्र से गुज़रती है?
उत्तर:
कर्क रेखा (\(23 \frac{1}{2} \circ\) उत्तर)

प्रश्न 2.
कर्क रेखा द्वारा भारत में निर्मित दो प्रदेशों के नाम लिखो।
उत्तर:
उष्ण कटिबन्ध तथा शीतोष्ण कटिबन्ध

प्रश्न 3.
भारत की तट रेखा की कुल लम्बाई लिखो।
उत्तर:
7516.6 किलोमीटर।

प्रश्न 4.
कौन-सी स्ट्रेट भारत को श्रीलंका से अलग करती है?
उत्तर:
पाक स्ट्रेट।

प्रश्न 5.
कौन-सा महासागरीय मार्ग भारत को यूरोप से जोड़ता है?
उत्तर:
स्वेज़ नहर मार्ग।

प्रश्न 6.
भारत का सबसे बड़ा राज्य ( क्षेत्रफल ) कौन-सा है?
उत्तर:
राजस्थान।

प्रश्न 7.
भारत का सबसे छोटा राज्य कौन-सा है?
उत्तर:
गोआ।

JAC Class 11 Geography Important Questions Chapter 1 भारत – स्थिति

प्रश्न 8.
भारत में सबसे छोटे केन्द्र प्रशासित प्रदेश का नाम लिखें।
उत्तर:
लक्षद्वीप।

प्रश्न 9.
कौन-सा राज्य पांच राज्यों से घिरा हुआ है?
उत्तर:
मध्य प्रदेश।

प्रश्न 10.
भारत में कितने तटीय राज्य हैं?
उत्तर:
नौ राज्य तटीय राज्य हैं गुजरात, महाराष्ट्र, कर्नाटक, गोआ, केरल, तमिलनाडु, आंध्र प्रदेश, उड़ीसा तथा पश्चिमी बंगाल।

प्रश्न 11.
अण्डमान-निकोबार द्वीप समूह में कुल कितने द्वीप हैं?
उत्तर:
204.

प्रश्न 12.
लक्षद्वीप में कुल कितने द्वीप हैं?
उत्तर:
36.

प्रश्न 13.
मूंगे के द्वीपों के समूह का नाम बताएं
उत्तर:
लक्षद्वीप।

प्रश्न 14.
भारत की वास्तविक शक्ति क्या है?
उत्तर:
अनेकता में एकता।

प्रश्न 15.
भारत का कुल कितना क्षेत्रफल है?
उत्तर:
32,67,263 किलोमीटर 2

JAC Class 11 Geography Important Questions Chapter 1 भारत – स्थिति

प्रश्न 16.
भारत उपमहाद्वीप किस अक्षांश तथा देशांतर के मध्य स्थित है?
उत्तर:
8° उत्तर से 37° उत्तर तथा 68° पूर्व से 97° पूर्व के मध्य।

प्रश्न 17.
भारत के किस राज्य से कर्क रेखा तथा प्रामाणिक रेखाएं अधिक दूरी तय करती हैं?
उत्तर:
मध्य प्रदेश।

प्रश्न 18.
भारत का पूर्व-पश्चिम तथा उत्तर दक्षिण विस्तार लिखो
उत्तर:
पूर्व पश्चिम विस्तार = 2933 किलोमीटर
उत्तर दक्षिण विस्तार = 3214 किलोमीटर।

प्रश्न 19.
भारत के मध्य से कौन-सी अक्षांश रेखा गुज़रती है?
उत्तर:
कर्क रेखा

प्रश्न 20.
भारत में कर्क रेखा पर स्थित दो शहरों के नाम लिखो। अहमदाबाद तथा जबलपुर।
उत्तर:
अहमदाबाद तथा जबलपुर।

प्रश्न 21.
कौन-सी देशांतरीय रेखा भारत के मध्य से गुज़रती है?
उत्तर;
\( 82 \frac{1}{2}^{\circ}\) पूर्व देशांतर।

प्रश्न 22.
\( 82 \frac{1}{2}^{\circ}\) पूर्व देशांतर पर स्थित दो शहरों के नाम लिखो।
उत्तर:
इलाहाबाद तथा राँची।

प्रश्न 23.
भारत-पाक सीमा पर स्थित राज्य बताओ।
उत्तर:

  1. गुजरात
  2. राजस्थान
  3. पंजाब
  4. जम्मू तथा कश्मीर।

प्रश्न 24.
भारत तथा चीन के मध्य सीमा का नाम लिखो।
उत्तर:
मैक्मोहन लाइन।

JAC Class 11 Geography Important Questions Chapter 1 भारत – स्थिति

प्रश्न 25.
भारत-चीन सीमा पर स्थित राज्य व केन्द्र शासित राज्य बताओ।
उत्तर:

  1. उत्तराखण्ड
  2. हिमाचल प्रदेश
  3. सिक्किम
  4. अरुणाचल प्रदेश
  5. लद्दाख ( केन्द्र शासित राज्य )।

प्रश्न 26
भारत-बांग्लादेश सीमा पर स्थित राज्य बताओ।
उत्तर:

  1. पश्चिमी बंगाल
  2. असम
  3. मेघालय
  4. त्रिपुरा|

प्रश्न 27.
भारत के पश्चिमी तट पर तटीय राज्यों के नाम बतायें।
उत्तर:

  1. गुजरात
  2. महाराष्ट्र
  3. गोआ
  4. कर्नाटक
  5. केरल।

प्रश्न 28.
भारत के पूर्वी तटों पर तटीय राज्यों के नाम बतायें।
उत्तर:

  1. तमिलनाडु
  2. आंध्र प्रदेश
  3. उड़ीसा
  4. पश्चिमी बंगाल।

प्रश्न 29.
उत्तर-पूर्वी भारत के पहाड़ी राज्य बताओ।
उत्तर:

  1. अरुणाचल प्रदेश
  2. असम
  3. नागालैंड
  4. मणिपुर
  5. मिज़ोरम
  6. त्रिपुरा
  7. मेघालय।

प्रश्न 30.
किस राज्य को Land of Dawn कहते हैं?
उत्तर;
अरुणाचल प्रदेश

प्रश्न 31.
क्षेत्रफल के आधार पर भारत का विश्व में कौन-सा स्थान है?
उत्तर:
सातवां

JAC Class 11 Geography Important Questions Chapter 1 भारत – स्थिति

प्रश्न 32.
कौन-सी अक्षांश रेखाएं भारत का उत्तरी तथा दक्षिणी विस्तार बनाती हैं?
उत्तर:
37° उत्तर तथा 8° उत्तर।

प्रश्न 33.
इन्दिरा प्वाइंट का अक्षांश क्या है?
उत्तर:
6°04′.

प्रश्न 34.
भारत के नाम के पीछे किस समुद्र का नाम पड़ा है?
उत्तर:
हिन्द महासागर

प्रश्न 35.
भारत के पूर्वी तथा पश्चिमी सिरे में कितने समय का अन्तर है?
उत्तर:
2 घण्टे।

प्रश्न 36.
उस राज्य का नाम बतायें जिसकी सबसे लम्बी तट रेखा है?
उत्तर:
गुजरात।

प्रश्न 37.
उस केन्द्र प्रशासित प्रदेश का नाम बताएं जिसका क्षेत्रफल पूर्वी तट तथा पश्चिमी तट पर मिलता है?
उत्तर:
पुड्डूचेरी।

प्रश्न 38.
भारत के दो दक्षिणी पड़ोसी देशों के नाम लिखो।
उत्तर:
श्रीलंका तथा मालदीव।

प्रश्न 39.
भारत में क्रियाशील ज्वालामुखी द्वीप का नाम लिखें।
उत्तर:
निकोबार द्वीप के नज़दीक बैरन द्वीप।

प्रश्न 40.
कौन – सा चैनल निकोबार द्वीप को अण्डमान द्वीप से अलग करता है?
उत्तर:
10° चैनल।

प्रश्न 41.
अण्डमान तथा निकोबार द्वीपों का कैसे निर्माण हुआ है?
उत्तर:
जलमग्न पहाड़ियों के शिखरों के कारण।

JAC Class 11 Geography Important Questions Chapter 1 भारत – स्थिति

प्रश्न 42.
हिन्द महासागर के पूर्वी तथा पश्चिमी भाग में सागरों के नाम लिखो।
उत्तर:
अरब सागर तथा खाड़ी बंगाल।

प्रश्न 43.
हिन्द महासागर के साथ कौन-से महाद्वीप हैं?
उत्तर:
अफ्रीका, एशिया, ऑस्ट्रेलिया, अण्टार्कटिका।

प्रश्न 44.
क्षेत्रफल और जनसंख्या की दृष्टि से संसार में भारत का कौन-सा स्थान है?
उत्तर:
7वां तथा दूसरा।

प्रश्न 45.
भारत ने पश्चिम और पूर्व में स्थित दो-दो देशों के नाम बताइए।
उत्तर:
पश्चिम में पाकिस्तान, अफगानिस्तान। पूर्व में म्यनमार तथा बंगला देश।

प्रश्न 46.
तारिम बेसिन कहां स्थित है?
उत्तर:
मध्य एशिया में।

प्रश्न 47.
भारत के उत्तरी दक्षिणी अक्षांशों के नाम लिखो।
उत्तर:
37° N तथा 80° N.

प्रश्न 48.
भारत पूर्वी सिरे तथा पश्चिमी सिरे के देशांतर लिखें।
उत्तर:
97° E तथा 68°E.

प्रश्न 49.
भारत का कौन-सा राज्य सबसे घना वसा है तथा कौन-सा राज्य सबसे कम?
उत्तर:
पश्चिमी बंगाल तथा अरुणाचल प्रदेश।

प्रश्न 50.
भारत का क्षेत्रफल बताइए यह विश्व के स्थलीय भाग का कितने प्रतिशत है?
उत्तर:
क्षेत्रफल 32.8 लाख वर्ग कि०मी० विश्व के स्थलीय धरातल का 2.4 प्रतिशत भाग है।

प्रश्न 51.
जून 2014 को बने नए राज्य का नाम बताओ।
तेलंगाना।
उत्तर:

प्रश्न 52.
तेलंगाना की राजधानी बताओ।
उत्तर:
हैदराबाद।

JAC Class 11 Geography Important Questions Chapter 1 भारत – स्थिति

प्रश्न 53.
संसार की छत (Roof of the world) किसे कहा जाता है?
उत्तर:
पामीर की गाँठ।

प्रश्न 54.
भारत के किस राज्य की तटीय सीमा रेखा की लम्बाई सबसे अधिक है?
उत्तर:
गुजरात।

प्रश्न 55.
कर्क रेखा भारत के कुल कितने राज्यों से गुज़रती है?
उत्तर:
आठ।

प्रश्न 56.
भारत में कुल कितने राज्य व केन्द्र शासित प्रदेश हैं?
उत्तर:
28 राज्य व 9 केन्द्र शासित प्रदेश

प्रश्न 57.
तीन भारतीय शहरों के नाम बताएं जो कर्क रेखा पर बसे हुए हैं?
उत्तर:

  1. गांधी नगर
  2. जबलपुर
  3. रांची

प्रश्न 58.
जम्मू-कश्मीर व लद्दाख कब केन्द्र शासित राज्य बने?
उत्तर:
31 अक्तूबर, 2019

प्रश्न 59.
लक्षद्वीप (केन्द्र शासित) एवं मणिपुर राज्य की राजधानियों का नाम लिखें।
उत्तर:
लक्षद्वीप – कवरति
मणिपुर – इम्फाल

प्रश्न 60.
लद्दाख (केन्द्र शासित राज्य ) की राजधानी क्या है?
उत्तर:
लेहं।

भारत के पड़ोसी देश भारत का विस्तार
1. पाकिस्तान कुल क्षेत्रफल 32,87,263 वर्ग कि॰मी०
2. बांग्लादेश अक्षांशीय विस्तार 8°4 उत्तर से 37°6 उत्तर
3. नेपाल देशान्तरीय विस्तार 68°7′ पूर्व से 97°25’पूर्व
4. भूटान
5. श्रीलंका पूर्व-पश्चिम लम्बाई 2933 कि॰मी०
6. मालदीव उत्तर-दक्षिण लम्बाई 3214 कि॰मी०


लघु उत्तरीय प्रश्न (Short Answer Type Questions)

प्रश्न 1.
उप-महाद्वीप किसे कहते हैं? इसकी व्याख्या दक्षिण एशिया की हिमालय पर्वत श्रेणी के दक्षिण स्थित देशों के सन्दर्भ में कीजिए।
उत्तर:
उप-महाद्वीप एक विशाल स्वतन्त्र भौगोलिक इकाई को कहा जाता है। यह स्थल खण्ड मुख्य महाद्वीप से स्पष्ट रूप से अलग होता है। इस विशालता के कारण इस भू-भाग में आर्थिक, सामाजिक तथा सांस्कृतिक स्वरूपों में विभिन्नताएं पाई जाती हैं। भू-भाग की सीमाएं विभिन्न स्थलाकृतियों द्वारा बनाई जाती हैं जो इसे सीमावर्ती प्रदेश से पृथक् करती हैं। भारत एक महान् देश है। इसे प्राय: भारतीय उप महाद्वीप (Indian Sub-Continent) भी कहा जाता है। हिमालय पर्वत की प्राकृतिक सीमा भारतीय उप महाद्वीप को एक परिबद्ध चरित्र देकर विलगता प्रदान करती है। यह भौगोलिक इकाई इस भूखण्ड को एशिया महाद्वीप से अलग करती है। इसमें पाकिस्तान, भारत, नेपाल, भूटान, बांग्लादेश, श्रीलंका, अफगानिस्तान तथा मालदीव के देश स्थित हैं। इन्हें सारक (SAARC) देश भी कहते हैं।

JAC Class 11 Geography Important Questions Chapter 1 भारत – स्थिति

प्रश्न 2.
क्षेत्र के आधार पर संसार के देशों में भारत की स्थिति क्या है?
उत्तर:
क्षेत्रफल के आधार पर भारत संसार में सातवां बड़ा देश है। भारत से अधिक क्षेत्रफल वाले छः देश रूस,
JAC Class 11 Geography Important Questions Chapter 1 भारत – स्थिति  1

Based upon the Survey of India map with the permission of the Surveyor General of India. The responsibility for the correctness of internal details rests with the publisher. The territorial waters of India extend into the sea to a distance of twelve nautical miles measured from the appropriate base line.
ब्राज़ील, कनाडा, संयुक्त राज्य अमेरिका, चीन, ऑस्ट्रेलिया हैं। भारत का क्षेत्रफल रूस को छोड़ कर पूरे यूरोप के बराबर है। यह इंग्लैण्ड से 13 गुना तथा जापान से 9 गुना बड़ा है, परन्तु रूस भारत से 7 गुना तथा संयुक्त राज्य अमेरिका तीन गुना बड़ा है। भारत का पूर्व – पश्चिम तथा उत्तर-दक्षिण विस्तार पृथ्वी की परिधि का लगभग 1 / 12 भाग है।

प्रश्न 3.
उत्तर-पश्चिमी भारत में स्थित दर्रे तथा इनका महत्त्व बताओ।
उत्तर:
विदेशी उत्तर-पश्चिम में स्थित खैबर और बोलन दर्रों से होकर ही भारत में प्रवेश कर सकते थे खैबर, हिन्दुकुश पर्वत में सफेद कोह के निकट तथा बोलन, सुलेमान और किरथर पर्वत श्रेणियों के मध्य स्थित है। पहले तो मध्य और पश्चिम एशिया की जन-जातियां इन्हीं मार्गों द्वारा भारत में आईं और बाद में सिकंदर, अफ़गानी तथा फारसी फ़ौजों ने भी इन्हीं भागों का अनुसरण किया। व्यापार के लिए भारत पश्चिम एशिया, पूर्व अफ्रीका और दक्षिण – पूर्व एशिया से समुद्री मार्गों द्वारा जुड़ा था।

प्रश्न 4.
भारतीय उपमहाद्वीप के हिमालय पर्वत को पार करने वाले चार दरों के नाम बताओ।
उत्तर:
भारत के उत्तर में हिमालय एक पर्वतीय दीवार के रूप में आवागमन साधनों में एक रुकावट है। फिर भी इस पर्वत को पार करने के लिए कई दर्रे लाभदायक हैं, जैसे-

  1. सतलुज गार्ज से शिपकी लॉ दर्रा (भारत-तिब्बत सड़क मार्ग)।
  2. कराकोरम दर्रे से कश्मीर लेह मार्ग।
  3. सिक्किम में नाथूला दर्रा।
  4. सिक्किम में जैल्पला दर्रा (लहासा – कालिम्पोंग मार्ग )।

प्रश्न 5.
उन राज्यों और संघीय प्रदेशों के नाम बताइए जिनकी सीमा बांग्लादेश से मिलती है।
अथवा
भारत की स्थल सीमाओं का वर्णन करो। भारत के कौन-से राज्य सीमावर्ती देशों के साथ लगते हैं?

उत्तर:
1. बांग्लादेश के साथ स्थल सीमा:
भारत तथा बांग्लादेश के मध्य पूर्व में एक स्थलीय सीमा है। बांग्ला देश के पूर्व में असम, मेघालय, त्रिपुरा राज्य तथा मिज़ोरम प्रदेश की सीमाएं हैं। बांग्लादेश के पश्चिम में पश्चिमी बंगाल राज्य की सीमा है।

2. पाकिस्तान के साथ स्थल सीमा:
भारत तथा पाकिस्तान के बीच कश्मीर से लेकर खाड़ी कच्छ तक एक स्थलीय सीमा है। इस सीमा के साथ-साथ जम्मू-कश्मीर, पंजाब, राजस्थान तथा गुजरात राज्यों की सीमाएं मिलती हैं।

3. नेपाल के साथ स्थल सीमा:
भारत के उत्तर में हिमालय पर्वतों में स्थित नेपाल देश है। इन देशों के बीच यह एक प्राकृतिक सीमा है। इस सीमा के साथ-साथ उत्तर प्रदेश, उत्तराखण्ड, बिहार, पश्चिमी बंगाल तथा सिक्किम राज्यों की सीमाएं मिलती हैं।

4. म्यनमार के साथ स्थल सीमा:
हिमालय पर्वत की पूर्वी शाखाएं भारत- बर्मा सीमा बनाती हैं। यह एक प्राकृतिक स्थलीय सीमा है। इस सीमा के साथ-साथ नागालैण्ड, मणिपुर राज्य, अरुणाचल और मिज़ोरम प्रदेश सीमाएं बनाते हैं।

5. पामीर गांठ के शीर्ष के साथ देश:
भारत की उत्तरी सीमा के शीर्ष ( पामीर गांठ) पर पांच देशों की सीमाएं आपस में मिलती हैं। इस मिलन बिन्दु पर भारत, चीन, तजाकिस्तान, अफगानिस्तान तथा पाकिस्तान की सीमाएं मिलती हैं। पामीर गांठ को ‘संसार की छत’ (Roof of the World) कहते हैं।

प्रश्न 6.
मैक्मोहन रेखा किसे कहते हैं? इसका क्या महत्त्व है? इसका निर्धारण किस सिद्धान्त पर किया गया है?
‘उत्तर:
मैक्मोहन रेखा भारत तथा चीन के मध्य सीमा रेखा है। यह सीमा रेखा हिमालय रेखा के साथ-साथ कश्मीर से लेकर अरुणाचल प्रदेश तक फैली हुई है। इस सीमा के पार चीन के सिक्यांग तथा तिब्बत पठार स्थित हैं। इसके उत्तर- पूर्वी भाग में म्यनमार (बर्मा), चीन एवं भारत आपस में मिलते हैं। यह सीमा रेखा अधिकांशत: प्राकृतिक है तथा ऐतिहासिक रूप से निर्धारित है। हिमालय पर्वत हमारी उत्तरी सीमा का प्रहरी है। उच्च हिमालय के शिखर भारत तथा चीन को अलग-अलग करते हैं। ये शिखर एक जल विभाजक के रूप में फैले हुए हैं तथा भारत-चीन सीमा रेखा को प्राकृतिक रूप देते हैं।

प्रश्न 7.
भारत के दक्षिण में स्थित महासागर को ‘हिन्द महासागर’ क्यों कहा जाता है? हिन्द महासागर भारत को किन देशों से जोड़ता है?
उत्तर:
हिन्द महासागर सचमुच ‘हिन्द’ (भारत) का महासागर है। यह संसार में एकमात्र महासागर है जिसका नाम किसी देश के नाम के कारण है। भारत की तट रेखा हिन्द महासागर के अधिकतर भाग को घेरती है। इस क्षेत्र में भारत जैसे महत्त्वपूर्ण देश का प्रभाव है। प्राचीन काल में इस क्षेत्र में भारत ही सबसे उन्नत देश था इस महत्त्व के कारण ही इसे हिन्द महासागर कहा जाता है। हिन्द महासागर भारत को पूर्वी अफ्रीका, दक्षिण पश्चिमी एशिया, यूरोप तथा उत्तरी अमेरिका से स्वेज़ मार्ग द्वारा जोड़ता है। पूर्व में यह चीन, जापान तथा इण्डोनेशिया से जुड़ा हुआ है।

JAC Class 11 Geography Important Questions Chapter 1 भारत – स्थिति

प्रश्न 8.
जब अरुणाचल में सूर्योदय हो चुका होता है तब सौराष्ट्र में रात होती है। कारण बताओ।
अथवा
भारत के सबसे पूर्वी भाग अरुणाचल प्रदेश और सबसे पश्चिमी भाग गुजरात के स्थानीय समय में दो घण्टे का अन्तर क्यों है?
अथवा
भारत का देशान्तरीय विस्तार हमें किस प्रकार प्रभावित करता है?
उत्तर:
भारत पूर्व से पश्चिम की ओर लगभग तीन हज़ार किलोमीटर की दूरी में फैला हुआ है। इसकी सबसे पश्चिमी सीमा बिन्दु सौराष्ट्र में है, जबकि पूर्वी सीमा बिन्दु अरुणाचल प्रदेश में है। इस प्रकार भारत का पूर्व – पश्चिम विस्तार 30° देशान्तर है। सूर्य को 1° देशान्तर पार करने के लिए 4 मिनट का समय लगता है। इसलिए 30° देशान्तर के लिए ( 30 x 4 120) मिनट या दो घण्टे का समय लगेगा। अरुणाचल प्रदेश पूर्व में है। वह भाग सूर्य के सामने पहले आता है, इसलिए वहां सूर्योदय पहले होता है। पश्चिम में स्थित होने के कारण सौराष्ट्र में सूर्योदय बाद में अर्थात् दो घण्टे देर से होता है।

इसलिए जब अरुणाचल में सूर्य उदय हो चुका होता है तो सौराष्ट्र में रात होती है। इसलिए अरुणाचल को ‘सूर्योदय का प्रदेश’ (Land of Dawn) भी कहते हैं । इस तथ्य से भारत की विशालता का ज्ञान होता है परन्तु आधुनिक जेट युग में दूरियां अपना महत्त्व खो चुकी हैं। आप श्रीनगर में नाश्ता करके दोपहर के खाने पर तिरुवनन्तपुरम पहुंच सकते हैं । जामनगर और गुवाहाटी के मध्य की यात्रा उतना ही समय लेगी जितनी देर में आप एक भारतीय फिल्म देखते हैं

प्रश्न 9.
‘भारत न तो दानव है और न बौना’ इस कथन की व्याख्या कीजिए।
उत्तर:
अथवा
‘भारत न तो संसार का सबसे बड़ा देश है और न ही सबसे छोटा ।” उदाहरण सहित व्याख्या करो – भारत एक विशाल देश है। क्षेत्रफल की दृष्टि से भारत का विश्व में सातवां स्थान है। भारत पृथ्वी के धरातल के लगभग 2.2% क्षेत्रफल में फैला हुआ है। फिर भी कई देशों का आकार भारत से बड़ा है। रूस भारत से लगभग सात गुना तथा संयुक्त राज्य अमेरिका लगभग तीन गुना बड़ा है। भारत इंग्लैण्ड से 13 गुना तथा जापान से नौ गुना बड़ा है।
JAC Class 11 Geography Important Questions Chapter 1 भारत – स्थिति  2
इस प्रकार क्षेत्रफल के आधार पर भारत न बहुत बड़ा और न ही बहुत छोटा देश है । इसलिए यह कथन सही है कि ” भारत न तो दानव है और न ही बौना ।” (“India is neither a giant nor a pigmy.”)

प्रश्न 10.
” भारत को हिन्द महासागर में सर्वाधिक केन्द्रीय स्थिति प्राप्त है।” यह कथन कहां तक सही है?
उत्तर:
हिन्द महासागर का विस्तार 40° पूर्व से 120° पूर्व देशान्तर तक है। भारत का दक्षिणी सिरा कन्याकुमारी लगभग 80° पूर्वी देशान्तर पर स्थित है। इस प्रकार भारत को हिन्द महासागर में केन्द्रीय स्थिति प्राप्त है। भारतीय प्रायद्वीप अरब सागर तथा खाड़ी बंगाल के मध्य में स्थित है। हिन्द महासागर में किसी भी देश की तट रेखा भारतीय तट रेखा जितनी लम्बी नहीं है। सभी समुद्री मार्ग भारत के तट को छू कर गुज़रते हैं। भारत पूर्व तथा पश्चिम दोनों दिशाओं में स्थित देशों के मध्य में स्थित है। इसलिए भारत को हिन्द महासागर में सर्वाधिक केन्द्रीय स्थिति प्राप्त है। भारत हिन्द महासागर में है। अतः हिन्द महासागर वास्तव में “हिन्द महासागर ” है।

प्रश्न 11.
भारत का प्रायद्वीपीय आकार किस प्रकार लाभदायक है? तीन उदाहरण देकर स्पष्ट करो।
अथवा
भारत की प्रायद्वीपीय स्थिति के तीन प्रभाव बताओ।
उत्तर:
भारतीय प्रायद्वीप त्रिभुजाकार है। इससे भारत के तीन पड़ोसी सागरों तक (बंगाल की खाड़ी, अरब सागर, हिन्द महासागर) पहुंच बहुत सुगम है। इस आकार के कारण मालाबार तट तथा कोरोमण्डल तट पर मत्स्य क्षेत्रों का विकास हुआ है। दोनों तटों पर कई प्राकृतिक बंदरगाहों जैसे – विशाखापट्टनम, चेन्नई, कोचीन, मुम्बई आदि का विकास हुआ है जहाँ से कई अन्तर्राष्ट्रीय समुद्री मार्ग गुज़रते हैं।

निबन्धात्मक प्रश्न (Essay Type Questions)

प्रश्न 1.
क्या भारत को एक उप-महाद्वीप कहा जा सकता है?
उत्तर:
भारत – एक उप-महाद्वीप (India-A Sub-Continent):
भारत एक विशाल देश है। क्षेत्रफल की दृष्टि से भारत का विश्व में सातवां स्थान है। भारत से अधिक क्षेत्रफल वाले छः देश रूस, ब्राज़ील, कनाडा, संयुक्त राज्य अमेरिका, चीन, ऑस्ट्रेलिया हैं। भारत पृथ्वी के धरातल के लगभग 2.2% क्षेत्रफल में फैला हुआ है। फिर भी कई देशों का आकार भारत से बड़ा है। रूस भारत से लगभग सात गुना तथा संयुक्त राज्य अमेरिका लगभग तीन गुना बड़ा है। भारत इंग्लैण्ड से 13 गुना तथा जापान से नौ गुना बड़ा है। इस प्रकार क्षेत्रफल के आधार पर भारत न बहुत बड़ा और न ही बहुत छोटा देश है। इसलिए यह कथन सही है कि भारत न तो ” दानव है और न ही बौना ।” (India is neither a giant nor a pigmy.)

उप-महाद्वीप एक विशाल स्वतन्त्र भौगोलिक इकाई को कहा जाता है। यह स्थल खण्ड मुख्य महाद्वीप से स्पष्ट रूप से अलग होता है। इस विशालता के कारण इस भू-भाग में आर्थिक, सामाजिक तथा सांस्कृतिक स्वरूपों में विभिन्नताएं पाई जाती हैं। भू-भाग की सीमाएं विभिन्न स्थलाकृतियों द्वारा बनाई जाती हैं जो इसे सीमावर्ती प्रदेश से पृथक् करती हैं। भारत एक महान् देश है। इसे प्रायः भारतीय उप महाद्वीप ( Indian Sub – Continent) भी कहा जाता है। डॉ० क्रैसी के अनुसार भारत को यूरोप की भांति एक महाद्वीप कहलाने का अधिकार है। प्रायः ये कथन विशाल क्षेत्रफल तथा जनसंख्या के आधार पर कहे जाते हैं। ग्लोब पर एशिया महाद्वीप के दक्षिणी भाग में एक विशाल स्थलखण्ड के रूप में भारतीय उप महाद्वीप दिखाई देता है। इसे उप-महाद्वीप कहे जाने के कई कारण हैं:

1. प्राकृतिक सीमाएं:
भारत की प्राकृतिक सीमाएं इसे एक विलगता का स्वरूप प्रदान करती हैं। उत्तर में हिमालय पर्वत, दक्षिण में हिन्द महासागर, पूर्व में घने वन तथा पश्चिम में थार मरुस्थल इसे मुख्य महाद्वीप से पृथक् करके उप- महाद्वीप का स्वरूप प्रदान करते हैं।

2. परिबद्ध चरित्र:
भारत चारों ओर से एशिया के मुख्य क्षेत्रों से घिरा है। इसे विशाल पर्वतों ने हज़ारों किलोमीटर तक अखंड रूप से घेर कर परिबद्ध (Enclosed) चरित्र दे दिया है। इस पर्वतीय घेरे के कारण यह एशिया के अन्य क्षेत्रों से व्यावहारिक रूप से अलग-थलग है।

3. क्षेत्रफल तथा जनसंख्या:
क्षेत्रफल की दृष्टि से भारत संसार का सातवां बड़ा देश है। यह देश भू-मण्डल के एक बड़े भाग में फैला हुआ है। चीन को छोड़कर यह संसार में सबसे अधिक जनसंख्या वाला देश है। यहां के लोगों की शारीरिक बनावट, रहन-सहन तथा संस्कृति संसार के दूसरे प्रदेशों से भिन्न है।

4. विविधता में एकता:
भारत एक विभिन्नताओं का देश है, फिर भी भारतीय सभ्यता में एक विशिष्ट एकरूपता विद्यमान है । इस आधार पर कई लेखकों ने इस भू-भाग को एक उप महाद्वीप की संज्ञा दी है।

5. जलवायु:
जलवायु के आधार पर सम्पूर्ण देश में उष्ण मानसूनी जलवायु पाई जाती है। इस भू-भाग पर मानसून पवनें एक स्वतन्त्र रूप में उत्पन्न होती हैं। मानसून पवनों का पूर्ण रूप इसी उप-महाद्वीप पर मिलता है । सम्पूर्ण देश में ऋतुओं का एक जैसा क्रम पाया जाता है। ये पवनें इसे एशिया महाद्वीप में पृथक् प्रकार की जलवायु प्रदान करके उप- महाद्वीप का स्वरूप प्रदान करने में सहायक हैं।

6. प्राकृतिक संसाधन:
भारत में प्राकृतिक साधनों की प्रचुरता है। सारे देश की आर्थिकता कृषि पर आधारित है। ये साधन किसी महाद्वीप में मिलने वाले साधनों की तुलना में कम नहीं हैं। इन विशेषताओं के आधार पर भारत को एक उप महाद्वीप कहना सही है।

JAC Class 11 Geography Important Questions Chapter 1 भारत – स्थिति

प्रश्न 2.
‘भारत की सीमाएं अधिकांशतः प्राकृतिक हैं और वे ऐतिहासिक रूप से निर्धारित हैं।” उदाहरण सहित स्पष्ट करो।
उत्तर:
भारतीय सभ्यता बहुत प्राचीन है। इसकी सीमाएं ऐतिहासिक हैं तथा अधिकांशतः प्राकृतिक हैं।

  1. हिन्द महासागर भारत की दक्षिणी सीमा बनाता है। समुद्र के पार हमारा निकटतम पड़ोसी देश श्रीलंका है जिसे पाक जलडमरू मध्य भारत से अलग करता है।
  2. इण्डोनेशिया निकोबार द्वीप के दक्षिण में अलग-थलग स्थित है।
  3. भारत की पूर्वी सीमा पर बंगाल की खाड़ी के पार बांग्लादेश, मलेशिया, म्यानमार, थाईलैण्ड, कम्बोडिया, वियतनाम तथा लाओस स्थित हैं। यह सीमा घने जंगलों तथा पूर्वांचल की पहाड़ियों द्वारा बनी हुई है।
  4. पश्चिम की ओर अरब सागर से परे ईराक, ईरान, अरब, मिस्र, सूडान, इथोपिया, केनिया आदि देश स्थित हैं।
  5. भारत की उत्तरी सीमा पर हिमालय पर्वत की एक अखण्ड दीवार के परे तिब्बत, चीन, सिक्यिांग बेसिन, तज़ाकिस्तान तथा अफगानिस्तान स्थित हैं। मैक्मोहन रेखा भारत तथा चीन के मध्य एक प्राकृतिक सीमा है।
  6. भारत की उत्तरी सीमा पर नेपाल तथा भूटान स्थित हैं।
  7.  हमारी पश्चिमी सीमा पाकिस्तान से लगती है। ये देश ऐतिहासिक रूप से प्राचीन सभ्यता के समय से भारत का सहभागी रहा है। पांच नदियों का देश (पंजाब) तथा राजस्थान मरुभूमि एवं सिंध (पाकिस्तान ) ऐतिहासिक रूप से समकाली प्रदेश हैं। इससे स्पष्ट है कि भारत की सीमाएं अधिकांशतः प्राकृतिक हैं तथा ऐतिहासिक रूप से निर्धारित हैं।

प्रश्न 3.
भारत की भौगोलिक स्थिति का महत्त्व बताओ।
उत्तर:
भारत की भौगोलिक स्थिति का महत्त्व (Importance of the Geographical Location of India) भारत की भौगोलिक स्थिति अनेक प्रकार से सुविधाजनक तथा महत्त्वपूर्ण है

  1. केन्द्रीय स्थिति (Central Location): भारत पूर्वी गोलार्द्ध के मध्य में स्थित है। यूरोप तथा अमेरिका के पश्चिमी भाग से भारत लगभग समान दूरी पर है।
  2. व्यापारिक मार्ग (Trade Routes ): अन्तर्राष्ट्रीय व्यापार की दृष्टि से भी भारत की स्थिति महत्त्वपूर्ण है। यहां से अनेक व्यापारिक मार्ग (Trade Routes) वायु तथा जल मार्ग से होकर जाते हैं।
  3. कर्क रेखा से समीपता (Nearness of Tropic of Cancer): कर्क रेखा देश के मध्य से होकर गुज़रती है, इसलिए भारत समूचे रूप से एक गर्म देश है। अधिक तापमान के कारण वर्ष भर खेती की सुविधा है, इसलिए भारत एक कृषि प्रधान देश है।
  4. लम्बी तट रेखा (Long Coast Line ): लम्बी तट रेखा के कारण अनेक बन्दरगाहों की सुविधा है।
  5. सुरक्षा (Defence ): देश की प्राकृतिक सीमाएं सुरक्षा की दृष्टि से भी महत्त्वपूर्ण हैं।
  6.  हिन्द महासागर का प्रभाव (Effect of Indian Ocean): हिन्द महासागर के किनारे पर स्थित होने के कारण ही ग्रीष्म ऋतु की मानसून पवनों से वर्षा प्राप्त होती है।
  7. हिमालय पर्वत का प्रभाव (Effect of Himalayas ): हिमालय पर्वत अपनी स्थिति के कारण ही मानसून पवनों को रोक कर वर्षा करता है तथा शीत ऋतु में ठण्डी ध्रुवीय पवनों से उत्तरी भारत की रक्षा करता है।

प्रश्न 4.
भारत में बनाए गए नवीन राज्य व केन्द्र शासित प्रदेश का नाम लिखें।
उत्तर:
2 जून, 2014 को आन्ध्र प्रदेश राज्य का पुनर्गठन करके दो राज्य बनाए गए। तेलंगाना तथा आन्ध्र प्रदेश । तेलंगाना भारतीय पठार के मध्यवर्ती भाग में बनाया गया नया राज्य है । इस राज्य का क्षेत्रफल 114,800 वर्ग कि०मी० है। इसमें कृष्णा तथा गोदावरी प्रमुख नदियां हैं। हैदराबाद नगर को दोनों राज्यों की संयुक्त राजधानी बनाया गया है जो 10 वर्षों तक ऐसे ही रहेगा। जम्मू-कश्मीर राज्य को भी 31 अक्तूबर, 2019 को केन्द्र शासित प्रदेश बनाया गया है। जम्मू कश्मीर को दो केन्द्र शासित राज्यों लदाख व जम्मू-कश्मीर में बांट दिया गया। जम्मू-कश्मीर की राजधानी श्रीनगर तथा लदाख की राजधानी लेह है।
JAC Class 11 Geography Important Questions Chapter 1 भारत – स्थिति  3

JAC Class 11 Geography Important Questions Chapter 1 भारत – स्थिति  4

JAC Class 11 Geography Important Questions Chapter 11 वायुमंडल में जल 

Jharkhand Board JAC Class 11 Geography Important Questions Chapter 11 वायुमंडल में जल Important Questions and Answers.

JAC Board Class 11 Geography Important Questions Chapter 11 वायुमंडल में जल

बहु-विकल्पी प्रश्न (Multiple Choice Questions)

दिए गए चार वैकल्पिक उत्तरों में से सही उत्तर चुनिए
1. संसार में सबसे अधिक वार्षिक वर्षा का क्षेत्र कौन-सा है?
(क) भूमध्य रेखीयं खण्ड
(ख) ध्रुवीय प्रदेश
(ग), 20°-30° अक्षांश
(घ) उष्ण कटिबन्ध।
उत्तर:
(क) भूमध्य रेखीय खण्ड।

2. किस खण्ड में निरपेक्ष आर्द्रता अधिक होती है?
(क) भूमध्य रेखीय खण्ड
(ख) मरुस्थल
(ग) ध्रुवीय प्रदेश
(घ) पर्वतीय प्रदेश।
उत्तर:
(क) भूमध्य रेखीय खण्ड।

3. वायु के जलवाष्प के जल में बदल जाने की क्रिया को क्या कहते हैं?
(क) संघनन
(ग) विकिरण
(ख) वाष्पीकरण
(घ) संवहन।
उत्तर:
(क) संघनन।

4. भारत के दक्षिणी पठार पर कम वर्षा का मुख्य कारण है?
(क) अधिक तापमान
(ख) पठारी धरातल
(ग) वर्षा छाया में स्थित होना
(घ) कम वाष्पीकरण।
उत्तर:
(ग) वर्षा छाया में स्थित होना।

5. किसी वायु में 4 ग्रेन जलवाष्प उपस्थित है उसी तापमान पर उस वायु में 8 ग्रेन जल वाष्प समा सकते हैं तो उस वायु की सापेक्ष आर्द्रता कितनी होगी?
(क) 10 प्रतिशत
(ख) 50 प्रतिशत
(ग) 100 प्रतिशत
(घ) 200 प्रतिशत।
उत्तर:
(ख) 50 प्रतिशत।

6. सापेक्षिक आर्द्रता मापी जाती है:
(क) हाइड्रोमीटर से
(ख) हाइग्रोमीटर से
(ग) बैरोमीटर से
(घ) एनिमोमीटर से।
उत्तर:
(ख) हाइग्रोमीटर से।

JAC Class 11 Geography Important Questions Chapter 11 वायुमंडल में जल 

7. संघनन से क्या होता है?
(क) जल जलवाष्प में बदल जाता है।
(ख) वायुमण्डल में विद्यमान जलवाष्प जलकणों में परिवर्तित होते हैं।
(ग) जल हिम – कणों में बदल जाता है।
(घ) हिम-कण पिघलने लगते हैं।
उत्तर:
(ख) वायु-मण्डल में विद्यमान जलवाष्प जलकणों में परिवर्तित होते हैं।

8. वह प्रक्रिया जिससे संघनित जलवाष्प जल-कणों या हिमकणों के रूप में वायुमण्डल से भूतल पर गिरने लगे।
(क) संघनन
(ख) वाष्पीकरण
(ग) वाष्पोत्सर्जन
(घ) वृष्टि।
उत्तर:
(घ) वृष्टि

9. भूतल के निकट जलवाष्प के संघनन से बने जल-कणों या हिम-कणों के झुण्ड जिससे दृश्यता बहुत कम हो जाती है:
(क) आर्द्रता
(ख) मेघ
(ग) कोहरा
(घ) वर्षा।
उत्तर:
(ग) कोहरा।

10. पंजाब में शीतकाल में किस प्रकार की वर्षा होती है-
(क) संवहनीय वर्षा
(ख) पर्वतकृत वर्षा
(ग) चक्रवातीय वर्षा
(घ) वाताग्रीय वर्षा।
उत्तर:
(ग) चक्रवातीय वर्षा।

अति लघु उत्तरीय प्रश्न (Very Short Answer Type Questions)

प्रश्न 1.
ऊँचाई के साथ जलवाष्प शीघ्रता से क्यों कम होते जाते हैं?
उत्तर:
ऊँचाई पर जल स्रोतों की कमी होती है तथा तापमान भी कम होता है। इसलिए वाष्पन क्रिया कम होती है।

प्रश्न 2.
मेघों के तीन प्रमुख वर्गों में वर्गीकृत करने का समान्य आधार क्या है?
उत्तर:
ऊँचाई के अनुसार मेघों को तीन वर्गों में बांटा जाता है

  1. उच्च मेघ (5-14 Km )
  2. मध्य मेघ ( 2-7 Km )
  3. निम्न मेघ ( 2 Km से नीचे )।

प्रश्न 3.
वर्षण क्या है? वर्षण के रूप को कौन-सी दशाएं निर्धारित करती हैं?
उत्तर:
वर्षण का अर्थ है जल वाष्प का नीचे गिरना । इसमें केवल वर्षा तथा हिमपात ही शामिल नहीं बल्कि ओले, बजरी तथा कोहरा भी आते हैं। यह निम्नलिखित दिशाओं पर निर्भर हैं-

  1. संघनन का तापमान
  2. वायुमण्डल की दशा
  3. मेघों के प्रकार तथा ऊँचाई
  4. वर्षण को उत्पन्न करने वाली प्रक्रिया

JAC Class 11 Geography Important Questions Chapter 11 वायुमंडल में जल 

प्रश्न 4.
आर्द्रता से क्या अभिप्राय है?
उत्तर:
आर्द्रता (Humidity ):
वायु में उपस्थित जलवाष्प की मात्रा को आर्द्रता कहते हैं। ( Humidity is the amount of water vapour present in air.) वायु में सदा जल वाष्प होते हैं जिससे वायु नमी प्राप्त करती है। वायुमण्डल के कुल भार का 2% भाग जलवाष्प के रूप में मौजूद है। यह जलवाष्प महासागरों, समुद्रों, झीलों, नदियों आदि के जल से प्राप्त होता है।

प्रश्न 5.
विशिष्ट आर्द्रता से क्या अभिप्राय है?
उत्तर:
वायु के प्रति इकाई भार में जलवाष्प के भार को विशिष्ट आर्द्रता (Specific humidity) कहते हैं। यह कुल वायु के आयतन में जलवाष्प के आयतन का अनुपात है। इस प्रकार विशिष्ट आर्द्रता पर तापमान तथा वायुदाब के परिवर्तन का कोई प्रभाव नहीं पड़ता।

प्रश्न 6.
वृष्टि छाया किसे कहते हैं?
उत्तर:
वृष्टि छाया (Rain Shadow ): पर्वतों की पवन मुखी ढाल पर काफ़ी वर्षा होती है, परन्तु पवन विमुखी ढाल शुष्क रह जाते हैं। पवन मुखी ढाल पर जब पवनें नीचे उतरती हैं तो गर्म होकर शुष्क हो जाती हैं। इसलिए पवन विमुखी ढाल शुष्क रहते हैं। इन्हें वर्षा छाया क्षेत्र कहते हैं।

प्रश्न 7.
सहिम वृष्टि से क्या अभिप्राय है?
उत्तर:
सहिम वृष्टि (Sleet): सहिम वृष्टि वर्षा की बूंदों के जमने से या हिम कणों से पिघले जल के पुनः जमने से होती है। यह जल वृष्टि तथा हिम वृष्टि का मिश्रण होता है। (Sleet is a mixture of Rain and Snow. ) धरती से ऊँचाई पर हवा की परत गर्म होती है। इस परत से वर्षा की बूंदें नीचे गिरती हैं। इससे नीचे हवा की ठण्डी परत होती है। इस ठण्डी परत में जल की बूंदें गिर कर जम जाती हैं तथा हिम के रूप में गिरती हैं।

प्रश्न 8.
जल विज्ञान क्या है?
उत्तर:
जल विज्ञान विज्ञान की वह शाखा है जो जल के उत्पादन, आदान-प्रदान, स्रोत, विलीनता, वाष्पता, हिमपात, संभरण तथा मापन आदि से सम्बन्धित है

लघु उत्तरीय प्रश्न (Short Answer Type Questions)

प्रश्न 1.
वाष्पीकरण के परिमाण एवं दर को प्रभावित करने वाले घटकों का नाम लिखो ।
उत्तर:
जिस क्रिया के द्वारा तरल पदार्थ एवं जल-गैसीय पदार्थ जल वाष्प में बदल जाते हैं, उसे वाष्पीकरण कहते हैं। वाष्पीकरण की दर तथा परिमाण चार घटकों पर निर्भर करते हैं

1. शुष्कता (Aridity): शुष्क वायु में जलवाष्प ग्रहण करने की क्षमता अधिक होती है। आर्द्र वायु होने पर वाष्पीकरण की दर एवं मात्रा कम हो जाती है।
2. तापमान (Temperature ): धरातल का तापक्रम बढ़ जाने से वाष्पीकरण बढ़ जाता है तथा ठण्डे धरातल पर कम वाष्पीकरण होता है।
3. वायु परिसंचरण (Movement of air ): वायु संचरण से वाष्पीकरण की मात्रा में वृद्धि होती है।
4. जल खण्ड (Water-bodies): विशाल जल-खण्डों के ऊपर वाष्पीकरण स्थल की अपेक्षा अधिक होता है।

प्रश्न 2.
ओसांक क्या है? नमी की मात्रा से इसका क्या सम्बन्ध है?
उत्तर:
ओसांक (Dew-point ):
संतृप्त वायु के ठण्डे होने से जल वाष्प के रूप में बदल जाती है। जिस तापमान पर किसी वायु का जलवाष्प जल रूप में बदलना शुरू हो जाता है, उस तापमान को ओसांक कहते हैं। जिस तापमान पर संघनन की क्रिया आरम्भ होती है उसे ओसांक या संघनन तापमान कहा जाता है। ओसांक तथा नमी की मात्रा में प्रत्यक्ष सम्बन्ध पाया जाता है, जब सापेक्ष आर्द्रता अधिक होती है तो संघनन की क्रिया जल्दी होती है। थोड़ी मात्रा में शीतलन की क्रिया से वायु ओसांक पर पहुंच जाती है। परन्तु कम सापेक्ष आर्द्रता के कारण संघनन नहीं होता संघनन के लिए तापमान का ओसांक के निकट या उससे नीचे होना आवश्यक है।

JAC Class 11 Geography Important Questions Chapter 11 वायुमंडल में जल 

प्रश्न 3.
संघनन कब और कैसे होता है?
उत्तर:
संघनन (Condensation):
जिस क्रिया द्वारा वायु के जल वाष्प जल के रूप में बदल जाएं, उसे संघनन कहते हैं। जल कणों के वाष्प का गैस से तरल अवस्था में बदलने की क्रिया को संघनन कहते हैं । (“Change of water-vapour into water is called condensation.”) वायु का तापमान कम होने से उस वायु की वाष्प धारण करने की शक्ति कम हो जाती है। कई बार तापमान इतना कम हो जाता है कि वायु जल वाष्प को सहार नहीं सकती और जल वाष्प तरल रूप में वर्षा के रूप में गिरता है। इस क्रिया के उत्पन्न होने के कई कारण हैं

  1. जब वायु लगातार ऊपर उठ कर ठण्डी हो जाए ।
  2. जब नमी से लदी हुई वायु किसी पर्वत के सहारे ऊँची उठ कर ठण्डी हो जाए ।
  3. जब गर्म तथा ठण्डी वायु-राशियां आपस में मिलती हैं। कोहरा, धुन्ध, मेघ, हिमपात, ओस, पाला, उपल वृष्टि इसके विभिन्न रूप हैं।

प्रश्न 4.
बादल कितने प्रकार के होते हैं?
उत्तर:
बादलों के प्रकार (Types of Clouds ) ऊँचाई के अनुसार मेघ तीन प्रकार के होते हैं:

  1. उच्च स्तरीय मेघ (High Clouds ): 6,000 से 10,000 मीटर तक ऊँचे, जैसे- पक्षाभ, स्तरी तथा कपासी मेघ।
  2. मध्यम स्तरीय मेघ (Medium Clouds ): 3,000 से 6,000 मीटर तक ऊँचे, जैसे-मध्य कपासी शिखर मेघ।
  3. निम्न स्तरीय मेघ (Low Clouds ): 3,000 मीटर तक ऊँचे मेघ जिनमें स्तरी कपासी मेघ, वर्षा स्तरी मेघ, कपासी मेघ, कपासी वर्षा मेघ शामिल हैं।

प्रश्न 5.
बादल कैसे बनते हैं?
उत्तर:
मुक्त वायु में, ऊँचाई पर, धूलि – कणों पर लदे जलकणों या हिम कणों के समूह को बादल कहते हैं। वायु के तापमान के ओसांक से नीचे गिरने पर बादल बनते हैं। वायु के ऊपर उठने तथा फैलने से उसका तापमान ओसांक से नीचे हो जाता है। इससे हवा में संघनन होता है। ये जलकण वायु में आर्द्रताग्राही कणों पर जम कर बादल बन जाते हैं।

प्रश्न 6.
उपल वृष्टि तथा पाले की रचना पर नोट लिखो।
उत्तर:
उपल वृष्टि ( Hail Stones ): जब संघनन की क्रिया 0°C या 32°F से कम तापमान पर होती है तो जल – वाष्प हिम कणों में बदल जाते हैं। कई बार तूफान (Thunder Storm) के कारण होने वाली वर्षा में से ये हिम- कण ठोस ओलों के रूप में गिरते हैं।

पाला (Frost ): जब संघनन की क्रिया हिमांक 0°C (32°F) कम तापमान पर हो तो जलवाष्प ओस की बूंदों के रूप में नहीं गिरता अपितु छोटे-छोटे सफेद हिम कणों के रूप में पृथ्वी तल पर परत के रूप में फैल जाता है । इसे पाला कहते हैं ।

प्रश्न 7.
सापेक्ष आर्द्रता तथा वर्षा में क्या सम्बन्ध है?
उत्तर:
सापेक्ष आर्द्रता तथा वर्षा में सम्बन्ध (Relation between Relative Humidity and Rainfall) सापेक्ष आर्द्रता बढ़ने से वायु संतृप्त होती चली जाती है। जब सापेक्ष आर्द्रता 100% हो जाती है तो वायु पूर्ण रूप से संतृप्त हो जाती है। वह वायु नमी को सहार नहीं सकती और उसे नमी वर्षा के रूप में छोड़नी पड़ती है। इस प्रकार सापेक्ष आर्द्रता के बढ़ने से वर्षा की सम्भावना अधिक रहती है। इस प्रकार सापेक्ष आर्द्रता वाष्पीकरण की मात्रा एवं दर निर्धारित करती है अतः यह एक महत्त्वपूर्ण जलवायविक घटक है।

प्रश्न 8.
किसी स्थान की वर्षा किन तत्त्वों पर निर्भर करती है?
उत्तर:
किसी स्थान की वर्षा निम्नलिखित तत्त्वों पर निर्भर करती है-

  1. भूमध्य रेखा से दूरी: भूमध्य रेखीय क्षेत्रों से अधिक गर्मी के कारण वर्षा अधिक होती है।
  2. समुद्र से दूरी: तटवर्ती प्रदेशों में वर्षा अधिक होती है परन्तु महाद्वीपों के भीतरी भागों में कम।
  3. समुद्र तल से ऊँचाई: पर्वतीय भागों में मैदानों की अपेक्षा अधिक वर्षा होती है।
  4. प्रचलित पवनें: सागर से आने वाली पवनें वर्षा करती हैं परन्तु स्थल से आने वाली पवनें शुष्क होती हैं।
  5. महासागरीय धाराएं: उष्ण धाराओं के ऊपर से गुजरने वाली पवनें अधिक वर्षा करती हैं, इसके विपरीत शीतल पवनों के ऊपर से गुजरने वाली पवनें शुष्क होती हैं।

प्रश्न 9.
संसार में वर्षा का वार्षिक वितरण बताओ।
उत्तर:
संसार में वर्षा का वितरण समान नहीं है। भूपृष्ठ पर होने वाली कुछ वर्षा का 19 प्रतिशत महाद्वीपों पर तथा 81 प्रतिशत महासागरों पर प्राप्त होता है। संसार की औसत वार्षिक वर्षा 975 मिलीमीटर है।
1. अधिक वर्षा वाले क्षेत्र: इन क्षेत्रों में वार्षिक वर्षा का औसत 200 सेंटीमीटर है। भूमध्यरेखीय क्षेत्र तथा मानसूनी प्रदेशों के तटीय भागों में अधिक वर्षा होती है।
2. सामान्य वर्षा वाले क्षेत्र: इन क्षेत्रों में 100 से 200 सेंटीमीटर वार्षिक वर्षा होती है। यह क्षेत्र उष्ण कटिबन्ध में स्थित है तथा मध्यवर्ती पर्वतों पर मिलते हैं।
3. कम वर्षा वाले क्षेत्र: महाद्वीपों के मध्यवर्ती भाग तथा शीतोष्ण कटिबन्ध के पूर्वी तटों पर 25 से 100 सेंटीमीटर वर्षा होती है।
4. वर्षा विहीन प्रदेश: गर्म मरुस्थल, ध्रुवीय प्रदेश तथा वृष्टि छाया प्रदेशों में 25 सेंटीमीटर से कम वर्षा होती है।

तुलनात्मक प्रश्न (Comparison Type Questions )

प्रश्न 1.
ओस तथा ओसांक में अन्तर स्पष्ट करो।
उत्तर:

ओस (Dew) ओसांक (Dew Point)
(1) भू-तल तथा वनस्पति पर संघनित जल की छोटी – छोटी बूँदों को ओस कहते हैं। (1) जिस तापमान पर संघनन की क्रिया आरम्भ होती है उसे ओसांक कहते हैं।
(2) ओस बनने के लिए लम्बी रातें, स्वच्छ आकाश, शान्त वायुमण्डल अनुकूल दशाएं हैं। (2) ओसांक को संघनन तापमान भी कहा जाता है।
(3) विकिरण द्वारा ठण्डे धरातल के सम्पर्क में आने वाली वायु जल्दी संतृप्त हो जाती है तथा कुछ जलवाष्प जल बूंदों में बदल जाता है। (3) जब वायु पूरी तरह संतृप्त हो जाती है और जलवाष्प ग्रहण नहीं कर सकती तो उस तापमान को ओसांक कहते हैं।
(4) ओस के निर्माण के लिए तापमान 0°C से ऊपर होना चाहिए। (4) ओसांक पर सापेक्ष आर्द्रता 100% होती है।

प्रश्न 2.
वर्षा तथा वृष्टि में अन्तर स्पष्ट करो।
उत्तर:

वर्षा (Rainfall) वृष्टि (Precipitation)
(1) जलवाष्प का जल कणों के रूप में पृथ्वी पर गिरना वर्षा कहलाता है। (1) किसी क्षेत्र में वायुमण्डल से गिरने वाली समस्त जलराशि को वृष्टि कहा जाता है।
(2) वर्षा मुख्य रूप से तीन प्रकार की है संवाहिक, पर्वतीय तथा चक्रवातीय। (2) वृष्टि मुख्यत: तरल तथा ठोस रूप में पाई जाती है।
(3) संतृप्त वायु के ठण्डा होने से वर्षा होती है। (3) ओसांक से कम तापमान पर संघनन से वृष्टि होती है।
(4) जब जल-कणों का आकार बड़ा हो जाता है तो वे वर्षा के रूप में गिरते हैं। (4) जल वर्षा, हिम वर्षा तथा ओलावृष्टि वृष्टि के मुख्य रूप हैं।

प्रश्न 3.
कोहरा तथा धुन्ध में अन्तर स्पष्ट करो।
उत्तर:

कोहरा (Fog) धुन्ध (Mist)
(1) कोहरा एक प्रकार का बादल है जो धरातल के निकट वायु में धूल-कणों पर लटके हुए जल- बिन्दुओं से बनता है। (1) हल्के कोहरे को धुन्ध कहते हैं। यह भी संघनित जल- वाष्प होता है।
(2) कोहरे में संघनता अधिक होती है तथा 200 मी० से अधिक दूरी की वस्तु दिखाई नहीं देती। (2) धुन्ध में संघनता कम होती है तथा 2 कि० मी० तक की दूरी पर भी वस्तुएं दिखाई पड़ती हैं।
(3) कोहरे में शुष्कता अधिक होती है। (3) धुन्ध में नमी अधिक होती है।
(4) शीत ऋतु में लम्बी रातों के कारण भूमि का कोहरा बनता है! (4) शीत ऋतु में शीतोष्ण कटिबन्ध में धुन्ध एक साधारण- सी बात है।


निबन्धात्मक प्रश्न (Essay Type Questions)

प्रश्न 1.
मेघ कैसे बनते हैं? मेघों के प्रकार बताओ।
उत्तर:
मेघों का निर्माण वायु में उपस्थित महीन धूलकणों के केन्द्रकों के चारों ओर जलवाष्प के संघनित होने से होता है। अधिकांश दशाओं में, मेघ जल की अत्यधिक छोटी-छोटी बूंदों से बने होते हैं। लेकिन वे बर्फ़ कणों से भी निर्मित हो सकते हैं बशर्ते कि तापमान हिमांक से नीचे हो अधिकांश मेघ, गर्म एवं आर्द्र वायु के ऊपर उठने से बनते हैं। ऊपर उठती हवा फैलती है और जब तक ओसांक तक न पहुंच जाए, ठंडी होती जाती है और कुछ जलवाष्प मेघों के रूप में संघनित होती है। दो विभिन्न तापमान रखने वाली वायुराशियों के आपस में मिश्रण से भी मेघों की रचना होती है। मेघ-आधार की औसत ऊँचाई के आधार पर मेघों के 10 प्रकारों को तीन वर्गों में रखा गया है। ये निम्नलिखित हैं:

  1. उच्च मेघ (5 से 14 कि०मी०);
  2. मध्य मेघ (2 से 7 कि०मी०);
  3. निम्न मेघ (2 कि०मी० से नीचे)।

1. उच्च मेघ (High Clouds )
(i) पक्षाभ मेघ:
यह तंतुनुमा, कोमल तथा सिल्क जैसी आकृति के मेघ हैं। जब ये मेघ समूह से अलग होकर आकाश में अव्यवस्थित तरीके से तैरते नज़र आते हैं, तो ये साफ मौसम लाते हैं। इसके विपरीत जब ये व्यवस्थित तरीके से पट्टियों के रूप में अथवा पक्षाभ स्तरी अथवा मध्यस्तरी मेघों से जुड़े हुए होते हैं, तो आर्द्र मौसम की पूर्व सूचना देते हैं।

(ii) पक्षाभ स्तरी मेघ:
पतले श्वेत चादर जैसे मेघ, जो लगभग समस्त आकाश को घेरते हैं तथा इसे दूध जैसी शक्ल प्रदान करते हैं, पक्षाभ स्तरी मेघ कहलाते हैं। सामान्यतः ये मेघ सूर्य अथवा चन्द्रमा के चारों ओर प्रभामण्डल बनाते हैं। सामान्यतः ये आने वाले तूफान के लक्षण हैं।

(iii) पक्षाभ कपासी मेघ:
ये मेघ छोटे-छोटे श्वेत पत्रकों अथवा छोटे गोलाकार रूप में दिखाई देते हैं, इनकी कोई छाया नहीं पड़ती। सामान्यतः ये समूहों, रेखाओं अथवा उर्मिकाओं में व्यवस्थित होते हैं, जो मेघ रूपी चादर में पड़ने वाली सिलवटों से उत्पन्न होते हैं ऐसी व्यवस्था को मैकेरेल आकाश कहते हैं।

2. मध्य मेघ (Medium Clouds )
(i) मध्य स्तरी मेघ:
भूरे अथवा नीले वर्ण के मेघों की एक समान चादर, जिनकी संरचना सामान्यतः तंतुनुमा होती है, इस मेघ वर्ग से संबंधित हैं। बहुधा, ये मेघ धीरे-धीरे पक्षाभ स्तरी मेघ में बदल जाते हैं। इन मेघों से होकर सूर्य तथा चन्द्रमा धुंधले दिखाई देते हैं । कभी-कभी इनसे प्रभामण्डल की छटा दिखाई देती है। साधारणतया इनसे दूर-दूर तक और लगातार वर्षा होती है।
JAC Class 11 Geography Important Questions Chapter 11 वायुमंडल में जल  1
(ii) मध्य कपासी मेघ:
ये चपटे हुए गोलाकार मेघों के समूह हैं, जो रेखाओं अथवा तरंगों की भांति व्यवस्थित होते हैं। ये पक्षाभ कपासी मेघ से भिन्न होते हैं, क्योंकि इनकी गोलाकार आकृति काफ़ी विशाल होती है, और अक्सर इनकी छाया भूमि पर दिखाई देती है ।

3. निम्न मेघ (Low Clouds )

  1. स्तरी कपासी मेघ: कोमल एवं भूरे मेघों के बड़े गोलाकार समूह जो चमकीले अंतराल के साथ देखे जाते हैं, इस वर्ग में शामिल हैं। साधारणतः इनकी व्यवस्था नियमित प्रतिरूप में होती है।
  2. स्तरी मेघ: ये निम्न, एक समान परत वाले मेघ हैं, जो कुहरे से मिलते-जुलते होते हैं, परन्तु भूपृष्ठ पर ठहरे नहीं होते हैं। ये मेघ किरीट या प्रभामण्डल उत्पन्न करते हैं।
  3. वर्षा स्तरी मेघ: ये घने आकृतिविहीन तथा बहुधा बिखरे परतों के रूप में निम्न मेघ हैं, जो सामान्यतः लगातार वर्षा देते हैं
  4. कपासी मेघ: ये मोटे, घने ऊर्ध्वाधर विकास वाले मेघ हैं। इनका ऊपरी भाग गुम्बद की शक्ल का होता है। इसकी संरचना गोभी के फूल जैसी होती है जबकि इसका आधार लगभग क्षैतिज होता है। अधिकाँश कपासी मेघ साफ़ मौसम रखते हैं। हालांकि इनका आधार कपासी वर्षा में मेघ में बदल कर तूफानों को जन्म देता है।
  5. वर्षा कपासी मेघ: ऊर्ध्वाधर विकास वाले मेघों के भारी समूह जिनके शिखर पर्वत अथवा टॉवर की भांति होते हैं, कपासी वर्षा मेघ कहलाते हैं। इसकी विशेषता इसके ऊपरी भाग का निहाई आकृति का होना है। इसके साथ भारी वर्षा, झंझा, तड़ितझंझा और कभी-कभी ओलों का गिरना भी जुड़ा हुआ है।

JAC Class 11 Geography Important Questions Chapter 11 वायुमंडल में जल 

प्रश्न 2.
निरपेक्ष आर्द्रता तथा सापेक्ष आर्द्रता में क्या अन्तर है?
उत्तर:
निरपेक्ष आर्द्रता (Absolute Humidity):
किसी समय किसी तापमान पर वायु में जितनी नमी मौजूद हो, उसे वायु की निरपेक्ष आर्द्रता कहते हैं। यह ग्राम प्रति घन से० मी० द्वारा प्रकट की जाती है। इस प्रकार निरपेक्ष आर्द्रता को वायु के प्रति आयतन जलवाष्प के भार के रूप में परिभाषित किया जाता है। तापमान बढ़ने या घटने पर भी वायु की वास्तविक आर्द्रता वही रहेगी जब तक उसमें और अधिक जल वाष्प शामिल न हों या कुछ जल वाष्प पृथक् न हो जाएं। वाष्पीकरण द्वारा निरपेक्ष आर्द्रता बढ़ जाती है तथा वर्षा द्वारा कम हो जाती है। वायु के ऊपर उठकर फैलने या नीचे उतर कर सिकुड़ने से भी यह यात्रा बढ़ या घट जाती है।

वितरण (Distribution ):

  1. भूमध्य रेखा पर सबसे अधिक निरपेक्ष आर्द्रता होती है तथा ध्रुवों की ओर घटती जाती है।
  2. ग्रीष्म ऋतु में शीतकाल की अपेक्षा तथा दिन में रात की अपेक्षा वायु की निरपेक्ष आर्द्रता अधिक होती है।
  3. निरपेक्ष आर्द्रता महासागरों पर स्थल खण्डों की अपेक्षा अधिक होती है।
  4. उच्च वायु दबाव क्षेत्रों में नीचे उतरती हुई पवनों के कारण निरपेक्ष आर्द्रता कम हो जाती है।
  5. इससे वर्षा के सम्बन्ध में अनुमान लगाने में सहायता नहीं मिलती।

सापेक्ष आर्द्रता (Relative Humidity ):
किसी तापमान पर वायु में कुल जितनी नमी समा सकती है उसका जितना प्रतिशत अंश उस वायु में मौजूद हो, उसे सापेक्ष आर्द्रता कहते हैं।
JAC Class 11 Geography Important Questions Chapter 11 वायुमंडल में जल  2
दूसरे शब्दों में यह वायु की निरपेक्ष नमी तथा उसकी वाष्प धारण करने की क्षमता में प्रतिशत अनुपात है।
JAC Class 11 Geography Important Questions Chapter 11 वायुमंडल में जल  3
उदाहरण (Example ):
किसी वायु में 70°F पर 4 Grain प्रति घन फुट आर्द्रता मौजूद है परन्तु वायु 70°F तापमान पर 8 Grain प्रति घन फुट ग्रहण कर सकती है तो उस वायु की सापेक्ष आर्द्रता \(=\frac{4}{8} \times 100\)
= 50% होगी।

नोट: सापेक्ष आर्द्रता को भिन्न की बजाय % में प्रकट किया जाता है। इसे वायु संतृप्तता का प्रतिशत अंश भी कहा जाता है। वायु के फैलने व सिकुड़ने में सापेक्ष आर्द्रता बदल जाती है। तापमान के बढ़ने से वायु की वाष्प ग्रहण करने की क्षमता बढ़ जाती है तथा सापेक्ष आर्द्रता कम हो जाती है। तापमान घटने से वायु ठण्डी हो जाती है तथा वाष्प ग्रहण क्षमता कम हो जाती है। इस प्रकार सापेक्ष आर्द्रता बढ़ जाती है।

वितरण (Distribution):

  1. भूमध्य रेखा पर सापेक्ष आर्द्रता अधिक होती है।
  2. उष्ण मरुस्थलों में सापेक्ष आर्द्रता कम होती है।
  3. कम वायु दबाव क्षेत्रों में सापेक्ष आर्द्रता अधिक परन्तु अधिक दबाव क्षेत्रों में कम होती है।
  4. महाद्वीपों के भीतरी क्षेत्रों में सापेक्ष आर्द्रता कम होती है।
  5. दिन को सापेक्ष आर्द्रता कम होती है परन्तु रात्रि को अधिक।

JAC Class 11 Geography Important Questions Chapter 11 वायुमंडल में जल 

प्रश्न 3.
वर्षा किस प्रकार होती है? वर्षा के विभिन्न प्रकारों का उदाहरण सहित वर्णन करो।
उत्तर:
वायु की आर्द्रता ही वर्षा का आधार है। वर्षा होने का मुख्य कारण संतृप्त वायु का ठण्डा होना है। (“Rainfall is caused due to the cooling of saturated air.”) वर्षा होने की क्रिया कई पदों (Stages) में होती है-

(i) संघनन होना (Condensation): नम वायु के ऊपर उठने से उसका तापक्रम प्रति 1000 फुट (300 मीटर) पर 5.6°F घटता जाता है। तापमान के निरन्तर घटने से वायु की वाष्प – शक्ति घट जाती है। वायु संतृप्त हो जाती है तथा संघनन क्रिया (Condensation) होती है।

(ii) मेघों का बनना (Formation of Clouds ):वायु में लाखों धूल-कण तैरते-फिरते हैं। जल वाष्प इन कणों पर जमा हो जाते हैं। ये मेघों का रूप धारण कर लेते हैं।

(iii) जल-कणों का बनना (Formation of Rain Drops): छोटे-छोटे मेघ कणों के आपस में मिलने से जल की बूंदें (Rain drops) बनती हैं। जब इन जल कणों का आकार व भार बढ़ जाता है तो वायु इसे सहार नहीं सकती। ये जल-कण पृथ्वी पर वर्षा के रूप में गिरते हैं।

  • इस प्रकार जल-कणों का पृथ्वी पर गिरना ही वर्षा कहलाता है। वर्षा होने के लिए आवश्यक है कि:
    1. वायु में पर्याप्त नमी हो।
    2. वायु का किसी प्रकार से ठण्डा होना या द्रवीभवन क्रिया का होना।
    3. वायु में धूल कणों का होना।
  • वर्षा के प्रकार (Types of Rainfall): वायु तीन दशाओं में ठण्डी होती है
    1. गर्म तथा नम वायु का संवाहिक धाराओं के रूप में ऊपर उठना।
    2. किसी पर्वत से टकराकर नम वायु का ऊपर उठना।
    3. ठण्डी तथा गर्म वायु का आपस में मिलना इन दशाओं के आधार पर वर्षा तीन प्रकार की है

1. संवहनीय वर्षा (Convectional Rainfall):
स्थल पर अधिक गर्मी के कारण वायु गर्म होकर फैलती है। तथा हल्की हो जाती है। यह वायु हल्की होकर ऊपर उठती है और इस वायु का स्थान लेने के लिए पास वाले अधिक दबाव वाले खण्ड की ठण्डी वायु आती है। यह वायु भी गर्म होकर ऊपर उठ जाती है। इस प्रकार संवाहिक धाराएं उत्पन्न हो जाती हैं। इन धाराओं के कारण कुछ ही ऊंचाई पर वायु ठण्डी हो जाती है तथा वर्षा होती है। इसे संवहनीय वर्षा (Convectional Rainfall) कहते हैं। यह वर्षा घनघोर, अधिक मात्रा तथा तीव्र बौछारों के रूप में होती है। यह वर्षा स्थानिक गर्मी के कारण उपस्थित होती है। भूमध्य रेखा पर प्रतिदिन वायु दोपहर तक गर्म हो उठती है और शाम को वर्षा होती है।
JAC Class 11 Geography Important Questions Chapter 11 वायुमंडल में जल  4

2. पर्वतीय वर्षा ( Orographic or Relief Rainfall): किसी पर्वत के सहारे ऊपर उठती हुई नम पवनों द्वारा वर्षा को पर्वतीय वर्षा (Relief Rainfall) कहते हैं। इस प्रकार की वर्षा के लिए आवश्यक है कि वायु में पर्याप्त मात्रा में नमी हो तथा पर्वतों का विस्तार पवन दिशा में लम्बवत् हो। संसार की अधिकांश वर्षा इसी प्रकार होती है, परन्तु यह वर्षा पर्वतों की पवन सम्मुख ढाल (Windward Slope) पर अधिक होती है। पर्वतों की पवन विमुख या दूसरी ओर की ढाल (Leeward Slope) पर बहुत कम वर्षा होती है। ऐसे प्रदेश वृष्टि छाया (Rain Shadow) के प्रदेश कहलाते हैं। पर्वत के दूसरी ओर नीचे उतरती हुई पवनें (Descending Winds) वर्षा नहीं करतीं। उतरने की क्रिया में दबाव से वायु का तापमान बढ़ जाता है तथा द्रवीभवन क्रिया नहीं हो पाती। पर्वत के दूसरी ओर तक पहुंचते-पहुंचते वायु की आर्द्रता समाप्त हो जाती है।
JAC Class 11 Geography Important Questions Chapter 11 वायुमंडल में जल  5
भारत में पश्चिमी घाट पर समुद्र से आने वाली पवनें लगभग 400 सम वर्षा करती हैं परन्तु पूर्वी ढाल पर वृष्टि छाया (Rain Shadow) के कारण केवल 65 सम वर्षा होती है।

3. चक्रवातीय वर्षा (Cyclonic Rainfall):
चक्रवात में गर्म व नम वायु ठण्डी शुष्क वायु के मिलने से ठण्डी वायु गर्म वायु को ऊपर उठा देती है। आर्द्र वायु उष्ण अग्र (Warm Front) के सहारे वायु से ऊपर चढ़ जाती है। ऊपर उठने पर गर्म वायु का जलवाष्प ठण्डा होकर वर्षा के रूप में गिरता है। इसे चक्रवातीय (Cyclonic) वर्षा कहते हैं। वह वर्षा लगातार बहुत देर तक परन्तु थोड़ी मात्रा में होती है।
JAC Class 11 Geography Important Questions Chapter 11 वायुमंडल में जल  6
शीतोष्ण कटिबन्ध में शीत काल में पश्चिमी यूरोप में इसी प्रकार वर्षा होती है। पंजाब में शीतकाल में कुछ वर्षा रूम सागर से आने वाले चक्रवातों द्वारा होती है।

JAC Class 11 Geography Important Questions Chapter 10 वायुमंडलीय परिसंचरण तथा मौसम प्रणालियाँ 

Jharkhand Board JAC Class 11 Geography Important Questions Chapter 10 वायुमंडलीय परिसंचरण तथा मौसम प्रणालियाँ Important Questions and Answers.

JAC Board Class 11 Geography Important Questions Chapter 10 वायुमंडलीय परिसंचरण तथा मौसम प्रणालियाँ

बहु-विकल्पी प्रश्न (Multiple Choice Questions)

दिए गए चार वैकल्पिक उत्तरों में से सही उत्तर चुनिए
1. सामान्य वायुदाब कितना होता है?
(A) 34 मिलीबार
(B) 300 मिलीबार
(C) 1013 मिलीबार
(D) 900 मिलीबार।
उत्तर:
(C) 1013 मिलीबार।

2. भूमध्य रेखीय खण्ड में निम्न वायुदाब पेटी का मुख्य कारण क्या है?
(A) दैनिक गति
(B) चक्रवात
(C) समुद्री धाराएं
(D) संवाहिक धाराएं।
उत्तर:
(D) संवाहिक धाराएं।

3. उपध्रुवीय क्षेत्र में निम्न वायु दाब का मुख्य कारण क्या है?
(A) दैनिक गति
(B) उच्च तापमान
(C) संवाहिक धाराएं
(D) चक्रवात।
उत्तर:
(A) दैनिक गति।

JAC Class 11 Geography Important Questions Chapter 10 वायुमंडलीय परिसंचरण तथा मौसम प्रणालियाँ

4. डोलड्रमज़ किस क्षेत्र को कहा जाता है?
(A) भू-मध्य रेखीय क्षेत्र
(B) ध्रुवीय क्षेत्र
(C) शीत ऊष्ण कटिबन्ध क्षेत्र
(D) शीत कटिबन्ध क्षेत्र।
उत्तर:
(C) भू-मध्य रेखीय क्षेत्र।

5. उत्तरी गोलार्द्ध में व्यापारिक पवनों की दिशा क्या होती है?
(A) उत्तर-पूर्वी
(B) दक्षिण-पूर्वी
(C) पश्चिमी
(D) दक्षिणी।
उत्तर:
(A) उत्तर-पूर्वी।

6. वायुदाब निम्नतम होता है जब वायु
(A) उष्ण तथा आर्द्र होती है
(B) ठण्डी तथा शुष्क होती है
(C) उष्ण तथा शुष्क होती है
(D) ठण्डी तथा आर्द्र होती है।
उत्तर:
(A) उष्ण और आर्द्र होती है।

7. वायुमण्डलीय दाब के मापन के लिए इकाई होती है:
(A) वार
(B) मिलीबार
(C) कैलोरी
(D) मीटर।
उत्तर:
(B) मिलीबार।

8. ऊंचाई के बढ़ने के साथ वायुदाब तथा तापमान में क्या अन्तर आता है?
(A) दोनों कम होते जाते हैं
(B) दोनों बढ़ते जाते हैं
(C) दोनों स्थाई रहते हैं।
(D) वायुदाब कम और तापमान बढ़ने लगता है।
उत्तर:
(A) दोनों कम होते जाते हैं।

9. पवन का वेग मापने के लिए उपयोग किया जाता है:
(A) हाइग्रोमीटर
(B) एनिमोमीटर
(C) बैरोमीटर
(D) थर्मामीटर।
उत्तर:
(B) एनिमोमीटर।

JAC Class 11 Geography Important Questions Chapter 10 वायुमंडलीय परिसंचरण तथा मौसम प्रणालियाँ

10. घोड़ों के अक्षांश भूगोलिक नाम है-
(A) उत्तरी उपोष्णीय उच्च वायु दाब कटिबन्ध का
(B) विषुवतरेखीय निम्न वायुदाब कटिबन्ध का
(C) विषुवतरेखीय शान्त मण्डल का
(D) ध्रुवीय क्षेत्रों का ।
उत्तर:
(A) उत्तरी उपोष्णीय उच्च वायु दाब कटिबन्ध का।

11. चिनूक पवनें किन पर्वतों के पूर्वी ढलानों से उतरती हैं?
(A) हिमालय
(B) एंडीज
(C) रॉकीज
(D) आल्पस।
उत्तर:
(C) रॉकीज।

अति लघु उत्तरीय प्रश्न (Very Short Answer Type Questions)

प्रश्न 1.
वायुमण्डलीय दाब में भिन्नता का क्या कारण है?
उत्तर:
वायु गर्म होने पर फैलती है और ठण्डा होने पर सिकुड़ती है। इससे वायुमण्डलीय दाब में भिन्नता आती है। परिणामस्वरूप वायु गतिमान होकर अधिक दाब वाले क्षेत्रों से न्यून दाब वाले क्षेत्रों में प्रवाहित होती है। क्षैतिज गतिमान वायु ही पवन है।

प्रश्न 2.
वायुमण्डलीय दाब के क्या प्रभाव हैं?
उत्तर:
वायुमण्डलीय दाब यह निर्धारित करता है कि कब वायु ऊपर उठेगी व कब नीचे बैठेगी। पवनें पृथ्वी पर तापमान व आर्द्रता का पुनर्वितरण करती हैं जिससे पूरी पृथ्वी का तापमान स्थिर बना रहता है। ऊपर उठती हुई वायु का तापमान कम होता जाता है। बादल बनते हैं और वर्षा होती है जैसे हम ऊपर ऊंचाई पर चढ़ते हैं, वायु विरल होती जाती है और सांस लेने में कठिनाई होती है।

प्रश्न 3.
वायुमण्डल का सामान्य परिसंचरण किन बातों पर निर्भर करता है?
उत्तर:
वायुमण्डल का सामान्य परिसंचरण: भूमण्डलीय पवनों का प्रारूप मुख्यतः निम्न बातों पर निर्भर है

  1. वायुमण्डलीय ताप में अक्षांशीय भिन्नता,
  2. वायुदाब पट्टियों की उपस्थिति,
  3. वायुदाब पट्टियों का सौर किरणों के साथ विस्थापन,
  4. महासागरों व महाद्वीपों का वितरण तथां
  5. पृथ्वी का घूर्णन। वायुमण्डलीय पवनों के प्रवाह प्रारूप को वायुमण्डलीय सामान्य परिसंचरण भी कहा जाता है। यह वायुमण्डलीय परिसंचरण महासागरीय जल को भी गतिमान करता है, जो पृथ्वी की जलवायु को प्रभावित करता है।

लघु उत्तरीय प्रश्न (Short Answer Type Questions)

प्रश्न 1.
वायुमण्डलीय दाब से क्या अभिप्राय है?
उत्तर:
वायुमण्डलीय दाब (Atmospheric Pressure ):
वायुमण्डल पृथ्वी के धरातल पर पृथ्वी की गुरुत्वाकर्षण शक्ति के कारण टिका है। गुरुत्वाकर्षण शक्ति के कारण प्रत्येक वस्तु में भार होता है। वायु में भी एक घनफुट में 1.2 . औंस भार होता है। इस भार के कारण पृथ्वी के धरातल पर दबाव पड़ता है। वायुमण्डलीय दाब का अर्थ है किसी भी स्थान पर वहां की हवा की उच्चतम सीमा के स्तम्भ का भार।

JAC Class 11 Geography Important Questions Chapter 10 वायुमंडलीय परिसंचरण तथा मौसम प्रणालियाँ

प्रश्न 2.
‘वाताग्र’ कैसे बनते हैं?
उत्तर:
जहां दो भिन्न वायुराशियां (शीत व उष्ण) वायुराशियां परस्पर टकराती हैं, तो उसे वायुराशि का वाताग्र कहते हैं। वाताग्र न तो धरातलीय सतह के सामान्तर होता है और न ही उस पर लम्बवत् होता है बल्कि कुछ कोण पर झुका होता है।

प्रश्न 2.
सामान्य वायु दाब किसे कहते हैं?
उत्तर:
सामान्य वायु दाब (Normal Atmosphere Pressure ): समुद्र तल पर प्रति वर्ग इंच पर वायुमण्डल का दबाव 6.68 किलोग्राम या 1.03 किलोग्राम प्रति वर्ग सेंटीमीटर होता है। वायुमण्डल का औसत या सामान्य दाब 45° अक्षांश पर समुद्र तल पर 29.92 इंच या 76 सेंटीमीटर या 1013.2 मिलीबार होता है।

प्रश्न 3.
एक मिलीबार से क्या अभिप्राय है? वायुदाब की माप इकाइयों में क्या सम्बन्ध है?
उत्तर:
एक वर्ग से०मी० पर एक ग्राम भार के बल को एक मिलीबार कहते हैं। दूसरे शब्दों में 1000 डाइन (Dynes) प्रति वर्ग से०मी० के वायु भार को एक मिलीबार कहते हैं। 1000 मिली बार के वायुभार को एक बार (Bar) कहते हैं। विभिन्न माप इकाइयों में सम्बन्ध:
30 इंच वायुदाब = 76 से०मी० = 1013.2 मिलीबार
1 इंच वायुदाब = 34 मिलीबार
1 से०मी० वायु दाब = 13.3 मिलीबार

प्रश्न 4.
दाब प्रवणता (Pressure Gradient) को परिभाषित करें
उत्तर:
वायुदाब का वितरण समदाब रेखाओं द्वारा प्रदर्शित किया जाता है। समदाब रेखाओं के अन्तर को दाब प्रवणता कहते हैं। यह दो समदाब रेखाओं पर समकोण बनाती हुई होती है। यह दाब प्रवणता वायु दिशा तथा वायु वेग को प्रदर्शित करती है। यदि समदाब रेखाएं एक-दूसरे के निकट हों तो दाब प्रवणता तीव्र होती है तथा तेज़ पवनें चलती हैं। यदि समदाब रेखाएं दूर-दूर हों तो वायु की गति मन्द होती है।

JAC Class 11 Geography Important Questions Chapter 10 वायुमंडलीय परिसंचरण तथा मौसम प्रणालियाँ

प्रश्न 5.
ऊँचाई के साथ वायु दाब में कमी क्यों होती है?
उत्तर:
ऊँचाई के साथ साथ 34 मिलीबार प्रति 300 मीटर की दर से वायुदाब कम होता है। इस कमी का मुख्य कारण वायु का घनत्व तथा सम्पीड़न क्रिया है। वायुमण्डल की ऊपरी परतें हल्की होती हैं। ऊपरी परतों के बोझ तथा दबाव के कारण नीचे की परतों पर सम्पीड़न क्रिया होती है इसलिए धरातल के निकट की परतों में वायुदाब अधिक होता है।

निश्चित ऊँचाई पर मानक तापमान व वायुदाब

स्तर वायुदाब (mle) समुद्रतल
समुद्रतल 1013.25 15.2
1 कि०मी० 898.76 8.7
5 कि०मी० 540.48 – 17.3
10 कि०मी० 265.00 -49.7

प्रश्न 6.
कॉरिऑलिस प्रभाव किस प्रकार पवन की दिशा प्रभावित करता है?
उत्तर:
व्यापारिक पवनों की दिशा पर कॉरिऑलिस शक्ति का प्रभाव पड़ता है। उत्तरी गोलार्द्ध में इन पवनों की दिशा उत्तर-दक्षिण की अपेक्षा उत्तर-पूर्व हो जाती है। दैनिक गति के कारण उत्पन्न कॉरिऑलिस प्रभाव से इन पवनों की दिशा उत्तरी गोलार्द्ध में दाईं ओर तथा दक्षिणी गोलार्द्ध में बाईं ओर मुड़ जाती है।

प्रश्न 7.
वाताग्र के विभिन्न प्रकार बताओ।
उत्तर:
वाताग्र (Fronts ): जब दो भिन्न प्रकार की वायुराशियाँ मिलती हैं तो उनके मध्य सीमा क्षेत्र को वाताग्र कहते हैं। वाताग्रों के बनने की प्रक्रिया को वाताग्र जनन (Frontogenesis) कहते हैं। वाताग्र चार प्रकार के होते हैं:

  1. शीत वाताग्र
  2. उष्ण वाताग्र
  3. अचर वाताग्र
  4. अधिविष्ट वाताग्र।

जब वाताग्र स्थिर हो जाए तो इन्हें अचर वाताग्र कहा जाता है (अर्थात् ऐसे वाताग्र जब कोई भी वायु ऊपर नहीं उठती)। जब शीतल व भारी वायु आक्रामक रूप में उष्ण वायुराशियों को ऊपर धकेलती है, इस सम्पर्क क्षेत्र को शीत वाताग्र कहते हैं। यदि गर्म वायुराशियाँ आक्रामक रूप में ठण्डी वायुराशियों के ऊपर चढ़ती हैं तो इस सम्पर्क क्षेत्र को उष्ण वाताग्र कहते हैं। यदि एक वायुराशि पूर्णतः धरातल के ऊपर उठ जाए तो ऐसे वाताग्र को अधिविष्ट वाताग्र कहते हैं। वाताग्र मध्य अक्षांशों में ही निर्मित होते हैं और तीव्र वायुदाब प्रवणता व तापमान इनकी विशेषता है। ये तापमान में अचानक बदलाव लाते हैं तथा इसी कारण वायु ऊपर ऊठती है, बादल बनते हैं तथा वर्षा होती है।

प्रश्न 8.
फ़ैरल का नियम क्या है? चित्र द्वारा स्पष्ट करो।
उत्तर:
फ़ैरल का नियम (Ferral’s Law):
धरातल पर पवनें कभी सीधे उत्तर से दक्षिण को नहीं चलतीं। सभी पवनें उत्तरी गोलार्द्ध में अपनी दायीं ओर तथा दक्षिणी गोलार्द्ध में बाईं ओर मुड़ जाती हैं। इसे फ़ैरल का नियम कहते हैं। ” All moving bodies are deflected to the right in the northern hemisphere and to left in the southern hemisphere.”
JAC Class 11 Geography Important Questions Chapter 10 वायुमंडलीय परिसंचरण तथा मौसम प्रणालियाँ  1
वायु की दिशा में परिर्वतन का कारण पृथ्वी की दैनिक गति है। जब हवाएं कम चाल वाले भागों से अधिक चाल वाले भागों की ओर आती हैं तो पीछे रह जाती हैं। इस विक्षेप शक्ति (Deffective Force) को कोरोलिस बल भी कहा जाता है।

JAC Class 11 Geography Important Questions Chapter 10 वायुमंडलीय परिसंचरण तथा मौसम प्रणालियाँ

प्रश्न 9.
वायुमण्डल का सामान्य परिसंचरण महासागरीय धाराओं की गति को कैसे प्रभावित करता है तथा ENSO घटना का वर्णन करें।
उत्तर:
वायुमण्डल का सामान्य परिसंचरण और उसका महासागरों पर प्रभाव- वायुमण्डल का सामान्य परिसंचरण के सन्दर्भ में प्रशान्त महासागर का गर्म या ठण्डा होना अत्यधिक महत्त्वपूर्ण है। मध्य प्रशान्त महासागर की गर्म जलधाराएं दक्षिणी अमेरिका के तट की ओर प्रवाहित होती हैं और पीरू की ठण्डी धाराओं का स्थान ले लेती हैं। पीरू के तट पर इन गर्म धाराओं की उपस्थिति एल-निनो कहलाता है। एल-निनो घटना का मध्य प्रशान्त महासागर और ऑस्ट्रेलिया के वायुदाब परिवर्तन से गहरा सम्बन्ध है। प्रशान्त महासागर पर बायुदाब में यह परिवर्तन दक्षिणी दोलन कहलाता है।

इन दोनों (दक्षिणी दोलन बदलाव व एल निनो) की संयुक्त घटना को ईएनएसओ (ENSO) के नाम से जाना जाता है। जिन वर्षों में ईएनएसओ (ENSO) शक्तिशाली होता है, विश्व में वृहत् मौसम सम्बन्धी भिन्नताएँ देखी जाती हैं। दक्षिण अमेरिका के पश्चिमी शुष्क तट पर भारी वर्षा होती है, ऑस्ट्रेलिया और कभी-कभी भारत अकालग्रस्त होते हैं तथा चीन में बाढ़ आती है। इन घटनाओं के ध्यानपूर्वक आकलन से संसार के अन्य भागों की मौसम सम्बन्धी भविष्यवाणी के रूप में इनका प्रयोग किया जाता है।

प्रश्न 10.
वायु राशि क्या होती है? पवन से यह किस प्रकार भिन्न है?
उत्तर:
वायु राशि वायुमण्डल का एक मोटा तथा विस्तृत भाग है जिसमें तापमान और आर्द्रता के लक्षण एक समान होते हैं। इस प्रकार वायुराशियों में एकरूपता का पाया जाना इसका मुख्य लक्षण है। एक वायु राशि में एक-दूसरे के ऊपर वायु की विभिन्न परतें होती हैं। इस प्रकार वायुमण्डल में पर्याप्त ऊँचाई तक एक विशाल वायु राशि में एकरूपता पाई जाती है। वायु राशि तथा वायु में कई प्रकार से भिन्नता पाई जाती है। वायु धरातल के समानान्तर चलती है तथा इसकी विभिन्न पर्तों में ताप एवं आर्द्रता में विभिन्नता पाई जाती है। वायु राशि में वायु धाराएं ऊपर उठती हैं तथा एक विशाल क्षेत्र में तापमान एवं आर्द्रता में समानता पाई जाती है।

प्रश्न 11.
स्रोत प्रदेश किसे कहते हैं? ध्रुवीय महाद्वीपीय स्रोत प्रदेशों का वर्णन करो
उत्तर:
धरातल के ऐसे समान क्षेत्र जहां वायु राशियों की उत्पत्ति होती है, स्रोत प्रदेश (Source Region) कहलाते हैं। यह प्रदेश पृथ्वी के धरातल पर विस्तृत क्षेत्र हैं जहां एक सम लक्षण पाए जाते हैं। ऐसे प्रदेश में एक सम धरातल तथा प्रति चक्रवातीय वायु व्यवस्था पाई जाती है। इस अवस्था में अपसारी वायु संचरण (Divergent) होता है। प्रायः स्रोत प्रदेश ध्रुवीय तथा उष्ण कटिबन्धीय क्षेत्र में पाए जाते हैं। शीतकाल में उत्तरी अमेरिका तथा यूरेशिया के ध्रुवीय प्रदेश बर्फ से ढके रहते हैं। यहां प्रति चक्रवात चलते हैं। वायु स्थिर तथा शुष्क होती है। तापमान तथा आर्द्रता में एकरूपता पाई जाती है।

प्रश्न 12.
विभिन्न प्रकार की वायु राशियों की उत्पत्ति, स्रोत के आधार पर वर्गीकरण करो
उत्तर:
संसार की विभिन्न राशियों के सुविधापूर्वक अध्ययन के लिए इनका वर्गीकरण आवश्यक है। उत्पत्ति, स्रोत के आधार पर वायु राशियां दो प्रकार की हैं-

  1. ध्रुवीय वायु राशियां (Polar Air Masses): वे वायु राशियां उच्च अक्षांशों में जन्म लेती हैं। ये ठण्डी तथा कम आर्द्र होती हैं ।
  2. उष्ण कटिबन्धीय वायु राशियां (Tropical Air Masses): ये वायु राशियां निम्न अक्षांशों में जन्म लेती हैं तथा इनमें तापक्रम व आर्द्रता अधिक होती है।

वायुराशियों को उत्पत्ति के आधार पर दो वर्गों में बांटा जा सकता है:

  1. समुद्री वायु राशियां (Maritime Air Masses): जो अधिक आर्द्र होती हैं।
  2. महाद्वीपीय वायु राशियां (Continental Air Masses ): जो प्रायः शुष्क होती हैं।

इस प्रकार वायु राशियों के चार प्रकार माने जाते हैं:

  1. समुद्री ध्रुवीय वायु राशियां।
  2. महाद्वीपीय ध्रुवीय वायु राशियां।
  3. समुद्री उष्ण कटिबन्धीय वायु राशियां|
  4. महाद्वीपीय उष्ण कटिबन्धीय वायु राशियां।

JAC Class 11 Geography Important Questions Chapter 10 वायुमंडलीय परिसंचरण तथा मौसम प्रणालियाँ  2

प्रश्न 13.
उत्तरी गोलार्द्ध में व्यापारिक पवनों की दिशा उत्तरी पूर्वी क्यों है? व्याख्या करें।
उत्तर:
व्यापारिक पवनें उपोष्ण उच्च वायुदाब पेटी से भूमध्य रेखीय निम्न दाब पेटी की ओर चलती हैं। उत्तरी गोलार्द्ध में इनकी दिशा उत्तर-पूर्वी होती है। यदि पृथ्वी स्थिर होती तो ये पवनें उत्तर-दक्षिण दिशा में चलतीं, परन्तु पृथ्वी की दैनिक गति के कारण ये पवनें लम्बवत् दिशा से हट कर एक ओर चली जाती हैं। उत्तरी गोलार्द्ध में विक्षेप शक्ति (Deflective force) दाईं ओर कार्य करती है। इसलिए ये पवनें दाईं ओर मुड़ जाती हैं। इस शक्ति को कोरोलिस बल भी कहा जाता है। इसी बल पर फैरल का सिद्धान्त आधारित है। इस सिद्धान्त के अनुसार ये पवनें उत्तरी गोलार्द्ध में दाईं ओर मुड़ जाती हैं तथा इनकी दिशा उत्तरी-पूर्वी हो जाती है।

JAC Class 11 Geography Important Questions Chapter 10 वायुमंडलीय परिसंचरण तथा मौसम प्रणालियाँ

प्रश्न 14.
चक्रवात किसे कहते हैं?
उत्तर:
चक्रवात न्यून वायुदाब की ऐसी व्यवस्था है जिसमें पवनें चारों ओर से केन्द्र की ओर चलती हैं। इस प्रकार चक्रवात न्यून वायुदाब के केन्द्र होते हैं जिनमें समदाब रेखाएं वृत्ताकार होती हैं। उत्तरी गोलार्द्ध में पवनों की दिशा घड़ी की सुइयों के विपरीत (Anti-clockwise) तथा दक्षिणी गोलार्द्ध में घड़ी की सुइयों के अनुसार (Clock-wise) होती है । चक्रवात उत्पत्ति क्षेत्र की दृष्टि से उष्ण कटिबन्धीय तथा शीत उष्ण कटिबन्धीय होते हैं।

प्रश्न 15.
उष्ण कटिबन्धीय चक्रवातों के प्रमुख गुणों का वर्णन करो।
उत्तर:
उष्ण कटिबन्धीय चक्रवात (Tropical Cyclones):

  1. ये चक्रवात 5° से 30° अक्षांशों के बीच व्यापारिक पवनों के साथ – साथ पूर्व से पश्चिम में चलते हैं।
  2. इनके केन्द्र में अत्यधिक निम्न दाब होता है तथा समदाब रेखाएं वृत्ताकार होती हैं।
  3. साधारणतः इनका आकार तथा विस्तार छोटा होता है। इसका व्यास 150 से 500 किलोमीटर तक होता है।
  4. चक्रवात के केन्द्रीय भाग को ‘आंधी की आंख’ (Eye of the Storm) कहते हैं। यह प्रदेश शान्त तथा वर्षाहीन होता है। यह गर्म वायु की धाराओं के रूप में ऊपर उठने से बनता है तथा इनकी ऊर्जा
  5. के स्रोत संघनन की गुप्त ऊष्मा है।
  6. शीत ऋतु की अपेक्षा ग्रीष्म ऋतु में इनका अधिक विकास होता है।
  7. इन चक्रवातों में हरीकेन तथा टाइफून बहुत विनाशकारी होते हैं।
  8. इन चक्रवातों द्वारा भारी वर्षा होती है।

प्रश्न 16.
उष्ण कटिबन्धीय चक्रवात इतने वृत्ताकार क्यों होते हैं?
उत्तर:
उष्ण कटिबन्धीय चक्रवात में समदाब रेखाएं वृत्ताकार होती हैं। केन्द्र में अत्यधिक निम्न दाब होता है। इनकी उत्पत्ति दो विभिन्न गुणों वाली वायु राशियों के मिलने के कारण होती है। परन्तु तीव्र हवाओं के कारण इन वायु राशियों के सीमान्त प्रदेश समाप्त हो जाते हैं। एक चक्रधार वायु व्यवस्था उत्पन्न हो जाती है जिसके चारों ओर अधिक वायुदाब होता है। यह वायुदाब समान रूप से बढ़ता है इसलिए ये चक्रवात वृत्ताकार होते हैं।

प्रश्न 17.
उष्ण कटिबन्धीय चक्रवातों में भारी वर्षा कैसे होती है?
उत्तर:
चक्रवात के केन्द्रीय भाग में पवनें नीचे उतरती हैं। इसलिए यह भाग शान्त तथा वर्षाहीन होता है। अन्य भागों दाब प्रवणता काफ़ी तीव्र होती है तथा तेज़ हवाएं चलती हैं। चारों ओर से ऊपर उठती हुई पवनों में संघनन की क्रिया से घनघोर वर्षा होती है। ये चक्रवात महासागर में उत्पन्न होते हैं इसलिए नमी से भरपूर होने के कारण अधिक वर्षा करते हैं।

प्रश्न 18.
उष्ण कटिबन्धीय चक्रवातों का प्रभाव विनाशकारी क्यों होता है?
उत्तर:
उष्ण कटिबन्धीय चक्रवात निम्न दाब के केन्द्र होते हैं। अतः बाहर से तीव्र हवाएं भीतर आती हैं। इनकी गति लगभग 200 कि० मी० प्रति घण्टा होती है। ये चक्रवात महासागरों पर बिना रोक-टोक के चलते हैं। समुद्र में ऊंची- ऊंची लहरें होती हैं जिससे समुद्री जहाज़ों को हानि होती है। समुद्री तटों पर छोटे-छोटे द्वीपों पर भयंकर लहरें अपार धन-जन की हानि करती हैं हज़ारों लोग समुद्र में डूब जाते हैं। समुद्री यातायात ठप्प हो जाता है। डैल्टा क्षेत्रों में ऊंची लहरों से भारी हानि होती है। सन् 1970 में बांग्ला देश में ऐसे ही चक्रवात से धन-जन की भारी हानि हुई थी।

JAC Class 11 Geography Important Questions Chapter 10 वायुमंडलीय परिसंचरण तथा मौसम प्रणालियाँ

प्रश्न 19.
उष्ण कटिबन्धीय चक्रवातों के क्षेत्रों का वर्णन करो।
उत्तर:
ये चक्रवात भिन्न-भिन्न सागरों में विभिन्न नामों से प्रसिद्ध हैं। खाड़ी बंगाल में इन्हें चक्रवात (Depression), पश्चिमी द्वीप समूह में हरीकेन (Huricane), चीन सागर में टाइफून (Typhoon) तथा अन्ध महासागर में टारनेडोज़. (Tornadoes) कहते हैं। ऑस्ट्रेलिया में विली – विली ( Willy-Willy) कहते हैं।

प्रश्न 20.
उष्ण कटिबन्धीय चक्रवात तथा शीतोष्ण कटिबन्धीय चक्रवातों में अन्तर स्पष्ट करो
उत्तर:

उष्ण कटिबन्धीय चक्रवात (Tropical Cyclones) शीतोष्ण कटिबन्धीय चक्रवात (Temperate Cyclones)
1. स्थिति-यह चक्रवात उष्ण कटिबन्ध में 5° से 3°अक्षांश तक चलते हैं। 1. यह चक्रवात शीतोष्ण कटिबन्ध में 35° से 65° अक्षांश तक चलते हैं।
2. दिशा-यह व्यापारिक पवनों के साथ-साथ पूर्व से पश्चिम की ओर चलते हैं। 2. यह पश्चिमी पवनों के साथ-साथ पश्चिम से पूर्व की ओर चलते हैं।
3. विस्तार-इनका व्यास 150 से 500 कि० मी० तक होता है। 3. इनका व्यास 1000 कि० मी० से अधिक होता है।
4. आकार-यह वृत्ताकार होते हैं। 4. यह प्राय: V आकार के होते हैं।
5. उत्पत्ति-यह संवाहिक धाराओं के कारण जन्म लेते हैं। 5. यह उष्ण तथा शीत वायु के मिलने से जन्म लेते हैं।
6. गति-इनमें वायु गति 100-200 कि० मी० प्रति घण्टा होती है। 6. इनमें वायु गति 30-40 कि० मी० प्रति घण्टा होती है।
7. रचना-यह प्राय: ग्रीष्मकाल में उत्पन्न होते हैं। इसके केन्द्रीय भाग को आंधी की आंख कहा जाता है। 7. यह प्राय: शीतकाल में उत्पन्न होते हैं। इसमें दो भाग उष्ण वाताग्र तथा शीत वाताग्र होते हैं।
8. मौसम-इसमें थोड़े समय के लिए तेज़ हवाएं चलती हैं तथा भारी वर्षा होती है। 8. इसमें शीत लहर चलती है तथा कई दिनों तक थोड़ीथोड़ी वर्षा होती रहती है।

निबन्धात्मक प्रश्न (Essay Type Questions)

प्रश्न 1.
वायुमण्डलीय दाब से क्या अभिप्राय है? वायुदाब किन तत्त्वों पर निर्भर करता है?
उत्तर:
वायुमण्डलीय दाब (Atmospheric Pressure ):
वायुमण्डल पृथ्वी के धरातल पर पृथ्वी की गुरुत्वाकर्षण शक्ति के कारण टिका है। गुरुत्वाकर्षण शक्ति के कारण प्रत्येक वस्तु में भार होता है। वायु में भी एक घनफुट में 1.2 औंस भार होता है। इस भार के कारण पृथ्वी के धरातल पर दबाव पड़ता है। वायुमण्डलीय दाब का अर्थ है किसी भी स्थान पर वहां की हवा की उच्चतम सीमा के स्तम्भ का भार।

सामान्य वायुदाब (Normal Atmosphere Pressure ):
समुद्र तल पर प्रति वर्ग इंच पर वायुमण्डल का दबाव 6.68 किलोग्राम या 1.03 किलोग्राम प्रति वर्ग सेंटीमीटर होता है। वायुमण्डल का औसत या सामान्य दाब 45° अक्षांश पर समुद्र तल पर 29.92 इंच या 76 सेंटीमीटर या 1013.2 मिली बार होता है।

मिलीबार (Millibar):
एक वर्ग सें० मी० पर एक ग्राम भार के बल को एक मिलीबार कहते हैं। दूसरे शब्दों में 1000 डाइन (Dynes) प्रति वर्ग से० मी० के वायु भार को एक मिलीबार कहते हैं। वायुदाब मापने की इकाई मिलीबार तथा पास्कल है। व्यापक रूप से प्रयोग किए जाने वाली इकाई किलो पास्कल है जिसे hpa लिखते हैं। 1000 मिलीबार के वायुभार को एक बार (Bar) कहते हैं।

विभिन्न माप इकाइयों में सम्बन्ध:
30 इंच वायुदाब = 76 से० मी० = 1013.2 मिलीबार
1 इंच वायुदाब = 34 मिलीबार
1 सें०मी० वायुदाब = 13.3 मिलीबार

वायुदाब को प्रभावित करने वाले तत्त्व:
1. तापमान (Temperature ):
गर्म होने पर वायु फैल कर हल्की हो जाती है। ठण्डी वायु सिकुड़ कर भारी हो जाती है। इसलिए यदि तापमान अधिक होगा तो वायु दबाव कम होगा। यदि तापमान कम होगा तो वायु दबाव अधिक होगा जैसे कहा जाता है कि ” A rising thermometer shows a falling barometer.” यही कारण है कि दिन को कम तथा रात को अधिक वायु दबाव होता है।

2. ऊँचाई (Altitude):
वायु की ऊपरी सतहों का भार निचली सतह पर पड़ता है। नीचे की हवा भारी तथा घनी हो जाती है। ऊपर जाने पर प्रत्येक 300 मीटर की ऊँचाई पर वायु दबाव 1 इंच या 34 मिलीबार गिर जाता है। प्रत्येक दस मीटर की ऊंचाई पर 1 hpa (किलो पास्कल) वायुदाब घटता है। अनुमान है कि वायुमण्डल का आधा दबाव केवल 5000 मी० की ऊँचाई तक सीमित है।

3. जलवाष्प (Water Vapour ):
जलवाष्प वायु की अपेक्षा हल्का होता है, इसलिए शुष्क वायु नम वायु की अपेक्षा भारी होती है। यही कारण है कि स्थलीय पवनें (Land Winds) शुष्क होने के कारण समुद्री पवनों (Sea Winds) की अपेक्षा भारी होती हैं।

4. दैनिक गति (Rotation ):
पृथ्वी की दैनिक गति के कारण कई स्थानों पर वायु इकट्ठी होती है तथा दूसरे स्थानों पर वायु दबाव कम हो जाता है। 60° अक्षांश पर कम वायु दबाव पृथ्वी की दैनिक गति के ही कारण है।

JAC Class 11 Geography Important Questions Chapter 10 वायुमंडलीय परिसंचरण तथा मौसम प्रणालियाँ

प्रश्न 2.
समदाब रेखाएं क्या होती हैं? इनकी विशेषताएं बताओ।
अथवा
मौसम मानचित्र बनाते समय किसी स्थान के वायुदाब को समुद्र तल तक क्यों घटाया जाता है?
उत्तर:
समदाब रेखाएं (Isobars) – Iso शब्द का अर्थ है:
समान और Bar का अर्थ है – दाब । इसलिए Isobars का अर्थ है – समदाब रेखाएं (Lines of Equal Pressure) “Isobars are lines joining the places of same pressure reduced to sea level.” (धरातल पर समान वायु दबाव वाले स्थानों को मिलाने वाली रेखाओं को समदाब रेखाएं कहते हैं।) इस वायु दबाव को समुद्र – तल पर घटा कर दिखाया जाता है। ऊँचाई के प्रभाव को वायु दबाव में से घटा लेते हैं। यह कल्पना की जाती है कि सभी स्थान समुद्र – तल पर स्थित हैं। यदि कोई स्थान 300 मीटर ऊँचा है और उसका वास्तविक वायु दबाव 900 मिलीबार है तो उसका समुद्र तल पर वायु दबाव = 900 + 34 = 934 मिलीबार होगा क्योंकि प्रति 300 मीटर पर 34 मिलीबार वायु दबाव कम हो जाता है।

विशेषताएं (Characteristics):

  1. ये रेखाएं पूर्व – पश्चिम दिशा में फैली हुई होती हैं।
  2. ये दक्षिणी गोलार्द्ध में अक्षांश रेखाओं के लगभग समानान्तर हैं।
  3. ये रेखाएं अधिक दबाव से कम दबाव की ओर खींची जाती हैं।
  4. ये रेखाएं समुद्र पर स्थल की अपेक्षा अधिक नियमित (Regular) होती हैं।
  5. जलवायु मानचित्रों में वायुभार समदाब रेखाओं से दिखाया जाता है।
  6. इससे पवन की दिशा व गति का पता चलता है।

प्रश्न 3.
व्यापारिक तथा पश्चिमी पवनें क्या होती हैं? इनके विस्तार तथा दिशा का वर्णन करो। ये कैसे उत्पन्न होती हैं तथा अपनी दिशा क्यों बदलती हैं? इनके प्रभाव का वर्णन करो।
उत्तर:
स्थायी पवनें (Planetary Winds):
धरातल पर उच्च वायु दबाव तथा कम वायु दबाव की विभिन्न स्थायी पेटियां (Belts) मिलती हैं। उच्च वायु भार पेटियों की ओर से कम वायु भार की ओर निरन्तर पवनें चलती हैं। इन्हें स्थायी पवनें कहते हैं। ये सदा एक ही दिशा में चलती हैं। स्थायी पवनें तीन प्रकार की हैं

  1. व्यापारिक पवनें (Trade Winds)
  2. प्रतिकूल व्यापारिक या पश्चिमी पवनें (Westerlies)
  3. ध्रुवीय पवनें (Polar Winds)

1. व्यापारिक पवनें (Trade Winds):
विस्तार (Extent): व्यापारिक पवनें वे स्थायी पवनें हैं जो उष्ण कटिबन्ध (Tropics) के बीच भूमध्य रेखा की ओर चलती हैं। ये पवनें अश्व अक्षांशों (Horse Latitudes) या उपोष्ण कटिबन्धीय उच्च दबाव (Sub- Tropical High Pressure) के क्षेत्र से डोलड्रमस् (Doldrums) भूमध्य रेखा की कम वायु दबाव पेटी की ओर चलती हैं। इनका विस्तार प्रायः 5°—35° उत्तर तथा दक्षिण तक चला जाता है।
JAC Class 11 Geography Important Questions Chapter 10 वायुमंडलीय परिसंचरण तथा मौसम प्रणालियाँ  3

दिशा (Direction): ये पवनें दोनों गोलाद्ध में पूर्व से आती हुई प्रतीत होती हैं। इसलिए इन्हें पूर्वी पवनें (Easterlies) भी कहते हैं। उत्तरी गोलार्द्ध में इनकी दिशा उत्तरी-पूर्वी (North-East) तथा दक्षिणी गोलार्द्ध में दक्षिण- पूर्वी (South-East) होती है।

नाम का कारण (Why so called?): इन पवनों को Trade Winds अर्थात् व्यापारिक पवनें कहने के दो कारण

  1. प्राचीन काल में यूरोप तथा अमेरिका के बीच बादबानी जहाज़ों को इन पवनों से बहुत सहायता मिलती थी ये पवनें Backing winds के रूप में जहाज़ों की गति बढ़ा देती थीं, इसलिए व्यापार में सहायक होने के कारण इन्हें व्यापारिक पवनें कहा जाता है।
  2. अंग्रेज़ी के मुहावरे ‘To blow trade’ का अर्थ है निरन्तर चलना ये पवनें लगातार एक ही दिशा में चलती हैं इसलिए इन्हें Trade winds या Track winds कहते हैं।

उत्पत्ति का कारण (Why caused):
भूमध्य रेखा पर अधिक गर्मी के कारण कम वायु दबाव पेटी मिलती है। भूमध्य रेखा से ऊपर उठने वाली गर्म तथा हल्की वायु 30° उत्तर तथा दक्षिण के पास ठण्डी तथा भारी होकर नीचे उतरती रहती है। ध्रुवों से खिसक कर आने वाली वायु भी अक्षांशों में नीचे उतरती है । इन उतरती हुई पवनों के कारण कर्क रेखा तथा मकर रेखा के निकट उच्च वायु दबाव पेटी बन जाती है। इसलिए भूमध्य रेखा के न्यून वायु दबाव (Low Pressure) का स्थान ग्रहण करने के लिए 30° उत्तर तथा दक्षिण के उच्च वायुदाब से भूमध्य रेखा की ओर व्यापारिक पवनें चलती हैं।

दिशा परिवर्तन का कारण (Change in Direction):
यदि पृथ्वी स्थिर होती तो ये पवनें उत्तर दिशा में चलतीं परन्तु पृथ्वी की दैनिक गति के कारण ये पवनें इस लम्बवत् दिशा से हट कर एक ओर झुक जाती हैं अर्थात् परे (Deflect) हो जाती हैं। फैरल के नियम (Ferral’s Law) तथा कोरोलिस बल के कारण ये पवनें उत्तरी गोलार्द्ध में दाईं ओर तथा दक्षिणी गोलार्द्ध में बाईं ओर मुड़ जाती हैं।

प्रभाव (Effects):

  1. गर्म प्रदेशों की ओर चलने के कारण ये पवनें प्रायः शुष्क होती हैं।
  2. ये पवनें महाद्वीपों के पूर्वी भागों में वर्षा करती हैं तथा पश्चिमी भागों तक पहुंचते-पहुंचते शुष्क हो जाती हैं। यही कारण है कि पश्चिमी भागों में 20° – 30° में उष्ण मरुस्थल (Hot Deserts) मिलते हैं।
  3. ये पवनें उत्तरी भाग में उच्च दबाव (High Pressure) के निकट होने के कारण ठण्डी (Cool) तथा शुष्क (Dry) होती हैं। परन्तु भूमध्य रेखा के निकट दक्षिणी भागों में गर्म (Hot) तथा आर्द्र (Wet) होती हैं।
  4. ये पवनें समुद्रों पर निरन्तर तथा धीमी गति से चलती हैं। परन्तु महाद्वीपों पर इनकी दिशा व गति में अन्तर पड़ जाता है।

2. पश्चिमी पवनें (Westerlies):
विस्तार (Extent): ये पवनें वे स्थायी पवनें हैं जो शीतोष्ण (Temperate) खण्ड में 35° के उच्च वायु दबाव से 60° के उपध्रुवीय न्यून वायु दबाव (Subpolar Low Pressure) की ओर चलती हैं। इनका विस्तार प्राय: 35° से 65° तक पहुंच जाता है। इन पवनों की उत्तरी सीमा ध्रुवीय सीमान्त ( Polar Fronts) तथा चक्रवातों (Cyclones) के कारण सदा बदलती रहती है।

दिशा (Direction):
उत्तरी गोलार्द्ध में इन पवनों की दिशा दक्षिण-पश्चिमी ( South-West) होती है। दक्षिणी गोलार्द्ध में इन पवनों की दिशा उत्तर-पश्चिमी (North-West) होती है।

नाम का कारण (Why so called ):
दोनों गोलाद्धों में ये पवनें पश्चिम से आती हुई प्रतीत होती हैं। इसलिए इन्हें पश्चिमी पवनें कहते हैं। इनकी दिशा व्यापारिक पवनों के विपरीत होती है। इसलिए उन्हें प्रतिकूल व्यापारिक पवनें (Anti-Trade Winds) भी कहते हैं।

उत्पत्ति का कारण (Why caused?):
कर्क रेखा तथा मकर रेखा के निकट नीचे उतरती हुई पवनों (Descending Winds) के कारण उच्च वायु दबाव हो जाता है। भूमध्य रेखा से गर्म तथा हल्की वायु इन अक्षांशों से नीचे उतरती रहती है। इस प्रकार ध्रुवों से खिसक कर आने वाली वायु भी यहां उतरती है। परन्तु 60° अक्षांश के निकट Arctic circle तथा Antarctic circle पर पृथ्वी की दैनिक गति के कारण कम वायु दबाव हो जाता है। इसलिए 30° के उच्च वायु दबाव की ओर से 60° के कम वायु दबाव की ओर पश्चिमी पवनें चलती हैं।

दिशा परिवर्तन (Change in Direction ):
साधारणतया पवनों की दिशा उत्तर-दक्षिण होनी चाहिए, परन्तु पृथ्वी की दैनिक गति के कारण ये पवनें लम्बवत् दिशा से हटकर एक ओर झुक जाती हैं। फैरल के नियम (Ferral’s law) के अनुसार उत्तरी गोलार्द्ध में दाईं ओर तथा दक्षिणी गोलार्द्ध में बाईं ओर मुड़ जाती हैं।

प्रभाव (Effects):
1. समुद्रों की नमी से लदी होने के कारण ये पवनें अधिक वर्षा करती हैं।

2. ये पवनें पश्चिमी प्रदेशों में बहुत वर्षा करती हैं, परन्तु पूर्वी भाग शुष्क रह जाते हैं।

3. ये पवनें बहुत अस्थिर होती हैं । इनकी दिशा तथा शक्ति बदलती रहती है।चक्रवात (Cyclones) तथा प्रति- चक्रवात (Anti-Cyclones) इनके मार्ग में अनिश्चित मौसम ले आते हैं । वर्षा, बादल, कोहरा, बर्फ़ तथा तेज़ आंधियों के कारण मौसम लगातार बदलता रहता है।

4. यह दक्षिणी गोलार्द्ध में समुद्रों पर निरन्तर तथा तीव्र गति से चलती हैं। 40° – 50° दक्षिण के अक्षांशों में इन्हें गर्जता चलीसा (Roaring Forties) भी कहा जाता है। 50° – 60° दक्षिण में इन्हें प्रचण्ड पछुआ Furious Fifties तथा Shrieking Sixties कहते हैं। इन प्रदेशों में ये इतनी तेज़ी से चलती हैं कि दक्षिणी अमेरिका के सिरे (Cape Horn) पर समुद्री यातायात बन्द हो जाता है।

5. व्यापारिक पवनों की अपेक्षा इनका प्रवाह क्षेत्र बड़ा होता है।

JAC Class 11 Geography Important Questions Chapter 10 वायुमंडलीय परिसंचरण तथा मौसम प्रणालियाँ

प्रश्न 4.
निम्नलिखित स्थानीय पवनों का विस्तार – पूर्वक वर्णन करो-
1. स्थल तथा जल समीर।
2. पर्वतीय तथा घाटी पवनें।
3. चिनूक तथा फोएन पवनें।
उत्तर:
1. स्थल तथा जल समीर (Land and Sea Breezes):
धरातल पर स्थानीय पवनों का क्रम है। परन्तु जल तथा स्थल में तापमान की विभिन्नता के कारण कुछ स्थानीय पवनें जन्म लेती हैं। जल – समीर व स्थल – समीर वे अस्थायी पवनें हैं जो समुद्र तटीय प्रदेशों में अनुभव की जाती हैं। ये जल तथा स्थल की असमान गर्मी के कारण उत्पन्न होती हैं। इसलिए इन्हें छोटे पैमाने पर मानसून पवनें (Monsoons on a small scale) भी कहते हैं।

तीव्र गर्मी से स्थल भाग समुद्र की अपेक्
(क) जल समीर (Sea Breeze):
ये वे पवनें हैं जो दिन के समय समुद्र की ओर चलती हैं। उत्पत्ति के कारण (Origin) दिन के समय सूर्य की तीव्र गर्मी से स्थल भाग समुद्र की अपेक्षा अधिक तथा जल्दी गर्म हो जाता है। स्थल पर वायु गर्म होकर ऊपर उठती है तथा कम वायु दबाव हो जाता है, समुद्र पर स्थल की अंपेक्षा अधिक वायु दबाव रहता है। इस प्रकार स्थल के कम दबाव का स्थान लेने के लिए समुद्र की ओर से ठण्डी पवनें चलती हैं। स्थल की गर्म वायु ऊपर उठकर समुद्र की ओर चली जाती है। इस प्रकार वायु के चलने का संवहन चक्र बन जाता है।
JAC Class 11 Geography Important Questions Chapter 10 वायुमंडलीय परिसंचरण तथा मौसम प्रणालियाँ  4

प्रभाव (Effects):

  1. जल – समीर ठण्डी तथा सुहावनी (Cool and Fresh) होती ह।
  2. गर्मियों में तटीय भागों के तापक्रम को कम करती है, परन्तु सर्दियों में तटीय भागों के तापक्रम को ऊंचा करती है। इस प्रकार मौसम सुहावना तथा समान हो जाता है।
  3. इनका प्रभाव समुद्र तट से 33 कि० मी० की दूरी तक सीमित रहता है।

JAC Class 11 Geography Important Questions Chapter 10 वायुमंडलीय परिसंचरण तथा मौसम प्रणालियाँ  5

(ख) स्थल समीर (Land Breeze ): ये वे पवनें हैं जो रात के समय स्थल से समुद्र की ओर चलती हैं।
उत्पत्ति का कारण (Origin):
रात की स्थिति दिन विपरीत होती है। स्थल भाग समुद्र की अपेक्षा अधिक तथा जल्दी ठण्डे हो जाते हैं। समुद्र पर वायु दबाव कम हो जाता है। परन्तु स्थल पर अधिक वायु दबाव होता है। इस प्रकार स्थल की ओर से समुद्र की ओर पवनें चलती हैं। समुद्र की गर्म वायु ऊपर उठकर स्थल पर उतरती है जिससे वायु चलने का क्रम पूरा हो जाता है।

प्रभाव (Effects):

  1. इनका स्थल भागों पर कोई विशेष प्रभाव नहीं होता।
  2. इन पवनों का लाभ उठाकर मछली पकड़ने वाले प्रात: काल स्थल समीर (Land Breeze) की सहायता से समुद्र की ओर बढ़ जाते हैं तथा सायंकाल को जल- समीर (Sea Breeze) के साथ-साथ तट की ओर वापस आ जाते हैं।
  3.  इनका प्रभाव तभी अनुभव होता है जबकि आकाश साफ हो, दैनिक तापान्तर अधिक हो तथा तेज़ पवनों का अभाव हों।

2. पर्वतीय तथा घाटीय पवनें (Mountain and Valley Winds): ये पवनें साधारणतया दैनिक पवनें हैं जो दैनिक तापान्तर के फलस्वरूप वायु दबाव की विभिन्नता के कारण चलती हैं।
(क) पर्वतीय पवनें (Mountain Winds0): पर्वतीय प्रदेश में रात के समय पर्वत के शिखर से घाटी की ओर ठण्डी और भारी वायु बहती है जिसे पर्वतीय पवनें (Mountain Winds) कहते हैं।
उत्पत्ति ( Origin):
रात के समय तीव्र विकिरण (Rapid radiation) के कारण वायु ठण्डी तथा भारी हो जाती है। यह वायु गुरुत्वाकर्षण शक्ति (Gravity) के कारण ढलानों से होकर नीचे उतरती है। इसे वायु प्रवाह (Air Drainage) भी कहते हैं। इन्हें अवरोही पवनें (Katabatic Winds) भी कहते हैं।
JAC Class 11 Geography Important Questions Chapter 10 वायुमंडलीय परिसंचरण तथा मौसम प्रणालियाँ  6
प्रभाव (Effects): इन पवनों के कारण घाटियां (Valleys ) ठण्डी वायु से भर जाती हैं जिससे घाटी के निचले भाग पर पाला पड़ता है। इसलिए कैलीफोर्निया (California) में फलों के बाग तथा ब्राज़ील में कहवा के बाग ढलानों पर लगाए जाते हैं।

(ख) घाटीय पवनें (Valley Winds ): दिन के समय घाटी की गर्म वायु ढाल से होकर शिखर की ओर ऊपर चढ़ती है। इसे घाटीय पवनें (Valley Winds) कहते हैं।
उत्पत्ति (Origin):
दिन के समय पर्वत के शिखर पर तीव्र गर्मी तथा विकिरण के कारण वायु गर्म होकर ऊपर उठती है तथा वायु दबाव कम हो जाता है। उसका स्थान लेने के लिए घाटी से हवाएं ऊपर चढ़ती हैं। ज्यों-ज्यों पवनें ऊपर चढ़ती हैं, ठण्डी होती जाती हैं। इन्हें आरोही पवनें (Anabatic winds) भी कहते हैं।

प्रभाव (Effects):

  1. ऊपर चढ़ने के कारण ये पवनें ठण्डी होकर घनघोर वर्षा करती हैं।
  2. ये ठण्डी पवनें गहरी घाटियों में गर्मी की तीव्रता को कम करती हैं।

3. चिनूक तथा फोएन पवनें (Chinook and Foehn Winds):
उत्पत्ति (Origin):
ये गर्म तथा शुष्क पवनें हैं। ये पवनें पर्वतों के सम्मुख ढाल पर टकराकर ऊपर चढ़ती हैं। इस क्रिया के कारण ये ठण्डी होकर पवन के सामने वाले ढाल (Windward slope ) पर काफ़ी वर्षा करती हैं। फिर ये पवनें पर्वत के विपरीत ढाल पर नीचे उतरती हैं। ये नीचे उतरती हुई पवनें (Descending Winds) दबाव से गर्म तथा शुष्क हो जाती हैं तथा वर्षा नहीं करतीं। मैदानी भागों में उतर कर उनका तापमान बढ़ा देती हैं।

(क) चिनूक पवनें ( Chinook Winds):
अमेरिका में रॉकीज (Rockies) पर्वतों को पार करके प्रेयरी के मैदान में चलने वाली ऐसी पवनों को चिनूक (Chinook) पवनें कहते हैं। चिनूक का अर्थ है – बर्फ खाऊ (Snow Eater) क्योंकि ये पवनें अधिक तापक्रम के कारण बर्फ को पिघला देती हैं । कई बार 24 घण्टों के समय में 50°F (10° C ) तापमान बढ़ जाता है

(ख) फोएन पवनें (Foehn Winds ):
यूरोप में एल्पस (Alps) को पार करके स्विट्ज़रलैण्ड में उतरने वाली पवनों को फोएन (Foehn) पवनें कहते हैं।
JAC Class 11 Geography Important Questions Chapter 10 वायुमंडलीय परिसंचरण तथा मौसम प्रणालियाँ  7

प्रभाव (Effects):

  1. ये पवनें तापमान बढ़ा देती हैं जिससे बर्फ पिघल जाती है और फसलों के पकने में सहायता मिलती है।
  2. ये पवनें शीतकाल की कठोरता को कम करती हैं।
  3. बर्फ के पिघल जाने से पहाड़ी चरागाह सारा साल खुले रहते हैं तथा पशु-पालन की सुविधा रहती है।

JAC Class 11 Geography Important Questions Chapter 10 वायुमंडलीय परिसंचरण तथा मौसम प्रणालियाँ

प्रश्न 5.
शीतोष्ण चक्रवात कैसे निर्मित होते हैं ? इनकी मुख्य विशेषताओं का वर्णन करो।
उत्तर:
शीतोष्ण चक्रवात ( Temperate Cyclones): ये चक्रवात पश्चिमी पवनों के क्षेत्र में 350 से 650 अक्षांशों के बीच तरंगों की तरह जन्म लेते हैं तथा पश्चिम से पूर्व दिशा में आगे बढ़ते हैं।
शीतोष्ण चक्रवातों की उत्पत्ति दो प्रकार से होती है

  1. धरातलीय अग्र ( Surface Front ) की अस्थिर पवनों से।
  2. उच्च-वायु (Upper Air) द्रोणी के नीचे की ओर प्रसार से।

इन चक्रवातों की उत्पत्ति के विषय में ध्रुवीय सीमान्त सिद्धान्त (Polar Front Theory ) प्रस्तुत किया गया है। इस सिद्धान्त के अनुसार चक्रवात की उत्पत्ति दो भिन्न तापमान वाली वायु राशियों के मिलने से होती है। इस सिद्धान्त के अनुसार चक्रवात के जीवन के इतिहास में अवस्थाओं का एक क्रम देखा जा सकता है:

  1. पहली अवस्था: इस अवस्था में दो वायु राशियां एक-दूसरे के निकट आती हैं तथा अग्र (front ) की रचना होती है। ध्रुवों की तरफ से शीतल वायु राशि तथा भू-मध्य रेखा की ओर से गर्म वायु विपरीत दिशाओं में आती है।
  2. दूसरी अवस्था: इस अवस्था में उष्ण वायु राशि में एक उभार उत्पन्न हो जाता है तथा अग्र एक तरंग का रूप धारण कर लेता है। अग्र के दो भाग हो जाते हैं- उष्ण अग्र तथा शीतल अग्र । गर्म वायु राशि उष्ण उग्र (Warm
    front) की शीतल वायु से टकराती है।
  3. तीसरी अवस्था: इस अवस्था में शीतल अग्र तेज़ी से आगे बढ़ता है। तरंगों की ऊँचाई तथा वेग में वृद्धि हो जाती है। गर्म वायु राशि का भाग छोटा हो जाता है।
  4. चौथी अवस्था: इस अवस्था में तरंगों की ऊँचाई अधिकतम हो जाती है। दोनों वायु राशियों में धाराएं वृत्ताकार गति प्राप्त कर लेती हैं तथा चक्रवात का विकास होता है।
  5. पांचवीं अवस्था: इस अवस्था में शीतल अग्र उष्ण अग्र को पकड़ लेता है। शीतल वायु उष्ण वायु को धरातल पर दबा देती है।
  6. अन्तिम अवस्था: इस अवस्था में उष्ण वायु अपने स्रोत से हट कर ऊपर उठ जाती है धरातल पर एक शीतल वायु का विशाल भंवर (whirl) चक्रवात का निर्माण करता है

JAC Class 11 Geography Important Questions Chapter 10 वायुमंडलीय परिसंचरण तथा मौसम प्रणालियाँ  8

शीतोष्ण चक्रवात की मुख्य विशेषताएं:

  1. ये चक्रवात पश्चिमी पवनों के क्षेत्र में 35° से 65° के अक्षांश के बीच पश्चिम पूर्व दिशा में चलते हैं।
  2. साधारणतः ये वृत्ताकार होते हैं परन्तु कुछ चक्रवात ‘V’ आकार के होते हैं।
  3. इनकी लम्बवत् मोटाई 9 से 11 किलोमीटर तथा व्यास 1000 किलोमीटर चौड़ा होता है।
  4. चक्रवात में अभिसारी पवनें केन्द्र में वायु को ऊपर उठा देती हैं। इसके परिणामस्वरूप मेघों का निर्माण तथा वर्षा होती है।
  5. साधारणतः इनकी गति 50 किलोमीटर प्रति घण्टा होती है। ग्रीष्म ऋतु की अपेक्षा शीतकाल में इनकी गति अधिक होती है।
  6. इसकी संरचना में दो अग्र (Fronts ), दो खण्ड (Sectors) तथा चार वृत्तपाद (Quadrants) होते हैं। उष्ण अग्र दक्षिणी-पूर्वी वृत्तपाद में होता है जबकि शीतल अग्र दक्षिणी-पश्चिमी वृत्तपाद में होता है।

JAC Class 11 Geography Important Questions Chapter 9 सौर विकिरण, ऊष्मा संतुलन एवं तापमान 

Jharkhand Board JAC Class 11 Geography Important Questions Chapter 9 सौर विकिरण, ऊष्मा संतुलन एवं तापमान Important Questions and Answers.

JAC Board Class 11 Geography Important Questions Chapter 9 सौर विकिरण, ऊष्मा संतुलन एवं तापमान

बहु-विकल्पी प्रश्न (Multiple Choice Questions )

दिए गए चार वैकल्पिक उत्तरों में से सही उत्तर चुनिए-
1. प्रकाश की क्या गति है?
(A) 3 लाख कि०मी० प्रति सै०
(B) 5000 कि०मी० प्रति सै०
(C) 10 कि०मी० प्रति सै०
(D) 100 कि०मी० प्रति सै०।
उत्तर:
(A) 3 लाख कि०मी० प्रति सै०।

2. सूर्य से पृथ्वी को प्राप्त होने वाली ऊर्जा को क्या कहते हैं?
(A) तापमान
(B) ऊर्जा
(C) सूर्यातप
(D) सौर विकिरण।
उत्तर:
(C) सूर्यातप।

3. सूर्य से आने वाले ताप का कितने प्रतिशत भाग पृथ्वी पर पहुंचता है?
(A) 51%
(B) 47%
(C) 65%
(D) 44%
उत्तर:
(A) 51%

4. सूर्यातप को नापने की कौन-सी इकाई प्रयोग की जाती है?
(A) डिग्री फार्नहाइट
(B) प्रतिशत
(C) कैलोरी
(D) डिग्री सैंटीग्रेड।
उत्तर:
(C) कैलोरी।

5. कर्क रेखा पर सूर्य की किरणें लम्बवत् किस दिन चमकती हैं?
(A) 21 मार्च
(B) 23 सितम्बर
(C) 22 दिसम्बर
(D) 21 जून।
उत्तर:
(D) 21 जून।

6. वायुमण्डल के बाह्य संस्तर तक पहुंचने वाले सूर्यातप का कितना भाग पृथ्वी के धरातल तक पहुंच पाता है?
(A) 49 प्रतिशत
(B) 51 प्रतिशत
(C) 27 प्रतिशत
(D) 14 प्रतिशत।
उत्तर:
(B) 51 प्रतिशत

JAC Class 11 Geography Important Questions Chapter 9 सौर विकिरण, ऊष्मा संतुलन एवं तापमान

7. पृथ्वी के धरातल द्वारा विकिरित ऊर्जा को कहते हैं
(A) सूर्यातप
(B) सौर विकिरण
(C) तापमान
(D) पार्थिव या भौमिक विकिरण
उत्तर:
(D) पार्थिव या भौमिक विकिरण।

8. वायुमण्डल मुख्यतः गर्म होता है:
(A) सीधे ही सूर्य की किरणों द्वारा
(B) पार्थिव विकिरण द्वारा
(C) पृथ्वी के भीतर की ऊष्मा द्वारा
(D) ज्वालामुखी क्रिया द्वारा।
उत्तर:
(B) पार्थिव विकिरण द्वारा।

9. सामान्यतः तापमान कम होता जाता है
(A) विषुवत रेखा से उत्तर की ओर
(B) विषुवत रेखा से दक्षिण की ओर
(C) विषुवत रेखा के दोनों ओर
(D) ध्रुवों से विषुवत रेखा की ओर ।
उत्तर:
(C) विषुवत रेखा के दोनों ओर।

10. स्थल तथा महासागरों के बीच तापमान का अन्तर अधिक होता है :
(A) उत्तरी गोलार्द्ध में
(B) दक्षिणी गोलार्द्ध में
(C) ध्रुवों के निकट
(D) ग्रीष्म ऋतु में
उत्तर:
(A) उत्तरी गोलार्द्ध में।

अति लघु उत्तरीय प्रश्न (Very Short Answer Type Questions)

प्रश्न 1.
‘पृथ्वी न तो अधिक समय के लिए गर्म होती है और न ही अधिक ठंडी’ व्याख्या करो।
उत्तर:
पृथ्वी अपनी ऊर्जा का लगभग सम्पूर्ण भाग सूर्य से प्राप्त करती है। इसके बदले पृथ्वी सूर्य से प्राप्त ऊर्जा को अन्तरिक्ष में वापस विकरित कर देती है। परिणामस्वरूप पृथ्वी पर ताप सन्तुलन बना रहता है।

प्रश्न 2.
‘पृथ्वी के अलग-अलग भागों से प्राप्त ताप की मात्रा समान नहीं होती’ क्यों?
उत्तर:
स्थल तथा जल में गर्म होने तथा ठण्डा होने की दर में विभिन्नता है। कोई भी भाग अधिक देर तक गर्म नहीं रहता तथा कोई भी भाग अधिक देर तक ठण्डा नहीं रहता। इसलिए विभिन्न भागों में तापमान की मात्रा समान नहीं होती।

JAC Class 11 Geography Important Questions Chapter 9 सौर विकिरण, ऊष्मा संतुलन एवं तापमान

प्रश्न 3.
तापमान की अलग-अलग भागों में भिन्नता के प्रभाव बताओ।
उत्तर:

  1. इस विभिन्नता के कारण वायुमण्डल के दाब में भिन्नता होती है।
  2. इसी कारण पवनों के द्वारा ताप का स्थानान्तरण एक स्थान से दूसरे स्थान पर होता है।

प्रश्न 4.
पार्थिव विकिरण तथा सूर्यातप में अन्तर बताओ।
उत्तर:
सूर्य से पृथ्वी को प्राप्त होने वाली ऊर्जा को सूर्यातप कहते हैं या आगामी सौर विकिरण (Insolation) कहते हैं। यह अधिकतर लघु तरंगों द्वारा प्राप्त होती है। पृथ्वी के धरातल से विकिरण हुई ऊर्जा को पार्थिव विकिरण कहते हैं। यह दीर्घ तरंगों द्वारा अन्तरिक्ष में वापस चली जाती है।

प्रश्न 5.
पृथ्वी के गोलाभ आकार का क्या प्रभाव है?
उत्तर:
पृथ्वी का आकार गोलाभ (Geoid) है। सूर्य की किरणें इस कारण वायुमण्डल के ऊपरी भाग पर तिरछी पड़ती हैं। इससे पृथ्वी सौर ऊर्जा का बहुत कम अंश प्राप्त करती है।

प्रश्न 6.
वायुमण्डल की ऊपरी सतह को कितनी सौर ऊर्जा प्राप्त होती है?.
उत्तर:
गोलाभ पृथ्वी के कारण वायुमण्डल की ऊपरी सतह पर औसत रूप से 0.5 कैलोरी प्रति वर्ग सैं०मी० प्रति मिनट ऊर्जा प्राप्त होती है। इसमें थोड़ा सा परिवर्तन होता रहता है।

प्रश्न 7.
पृथ्वी द्वारा 3 जनवरी की अपेक्षा 4 जुलाई को अधिक सूर्यातप प्राप्त होता है। क्यों ?
उत्तर:
सूर्य के चारों ओर परिक्रमण के दौरान पृथ्वी 4 जुलाई को सूर्य से सबसे दूर अर्थात् 15 करोड़ 20 लाख कि०मी० दूर होती है। इसे अपसौर (Aphelion) कहते हैं। 3 जनवरी को पृथ्वी सूर्य के सबसे निकट 14 करोड़ 70 लाख कि०मी० दूर होती है। इसे उप सौर (Perihelion) कहते हैं। इसलिए पृथ्वी द्वारा प्राप्त वार्षिक सूर्यातप (insolation ) 3 जनवरी की अपेक्षा 4 जुलाई को अधिक होता है।

JAC Class 11 Geography Important Questions Chapter 9 सौर विकिरण, ऊष्मा संतुलन एवं तापमान

प्रश्न 8.
सूर्यातप में विभिन्नता लाने वाले कारक लिखो।
उत्तर:

  1. पृथ्वी का धुरी पर घूमना
  2. सूर्य की किरणों का नति कोण
  3. दिन की अवधि
  4. वायुमण्डल की पारदर्शिता
  5. स्थल विन्यास

प्रश्न 9.
आकाश में रंगों की विभिन्नता क्यों है?
उत्तर:
पृथ्वी की सतह की ओर विकीर्ण के कारण सूर्य उदय एवं अस्त होने के समय सूर्य लाल दिखाई देता है। वायुमण्डल के प्रकाश के प्रकीर्णन (Scattering) के कारण नीला रंग दिखाई देता है।

प्रश्न 10.
पृथ्वी का एलिबडो किसे कहते हैं?
उत्तर:
सूर्य से ताप विकिरण का 30% सीधे रूप में परावर्तित हो कर अन्तरिक्ष में वापस चला जाता है। इसे पृथ्वी का एलिबडो कहते हैं।

प्रश्न 11.
सौर कलंक से क्या अभिप्राय है?
उत्तर:
सूर्य की सतह पर काले रंग के गहरे व उथले बनते-बिगड़ते धब्बों को सौर कलंक कहते हैं।

लघु उत्तरीय प्रश्न (Short Answer Type Questions)

प्रश्न 1.
सूर्यातप क्या है?
उत्तर:
पृथ्वी के धरातल पर पहुंचने वाले सौर विकिरण को सूर्यातप कहते हैं। यह ऊर्जा लघु तरंगों के रूप में 3 लाख कि० मी० प्रति सेकण्ड की दर से पृथ्वी पर पहुंचती है। पृथ्वी पर सौर विकिरण का केवल 2 अरबवां भाग ही पहुंचता है।

प्रश्न 2.
ऊष्मा तथा तापमान में क्या अन्तर है?
उत्तर:
दोनों शब्द एक-दूसरे से सम्बन्धित हैं। ऊष्मा वास्तव में ऊर्जा का रूप है जो वस्तुओं को गर्म करती है। तापमान ऊष्मा की मात्रा का माप है। ऊष्मा के बढ़ने या घटने से तापमान बढ़ता या घटता है।

प्रश्न 3.
सूर्यातप तथा किसी स्थान के तापमान में क्या अन्तर है?
उत्तर:
सूर्यातप एक प्रकार की ऊर्जा है। सूर्य से पृथ्वी को प्राप्त होने वाली ऊर्जा या ऊष्मा को सूर्यातप कहते हैं। यह लघु तरंगों के रूप में पृथ्वी के तल को गर्म करती है। किसी स्थान के तापमान से अभिप्राय उस स्थान पर धरातल से एक मीटर ऊपर वायु में ऊष्मा की मात्रा है। यह वायुमण्डल का तापमान है। वायु धरातल द्वारा छोड़ी गई ऊर्जा के विकिरण से गर्म होती है।

प्रश्न 4.
तापमान के क्षैतिज वितरण से क्या अर्थ है?
उत्तर:
तापमान का क्षैतिज वितरण अक्षांशों के अनुसार तापमान घटता-बढ़ता रहता है। अक्षांशों के अनुसार तापमान के वितरण को क्षैतिज वितरण कहते हैं। यह वितरण समताप रेखाओं द्वारा प्रकट किया जाता है।

प्रश्न 5.
तापमान के ऊर्ध्वाधर वितरण का क्या अर्थ है?
उत्तर:
वायुमण्डल मुख्यतः नीचे से ऊपर की ओर गर्म होता है। इसलिए ऊंचाई के साथ तापमान कम होता जाता है। तापमान कम होने की दर 1°C प्रति 165 मीटर है। इसे सामान्य ह्रास दर (Normal Lapse Rate) कहते हैं।

JAC Class 11 Geography Important Questions Chapter 9 सौर विकिरण, ऊष्मा संतुलन एवं तापमान

प्रश्न 6.
विभिन्न अक्षांश सूर्यातप भिन्न मात्रा में क्यों प्राप्त करते हैं?
उत्तर:
सूर्यातप की मात्रा सूर्य किरणों के आपात् कोण तथा दिन की अवधि पर निर्भर करती है। पृथ्वी की वार्षिक गति तथा पृथ्वी के अक्ष के झुकाव कारण भिन्न अक्षांशों पर सूर्य किरणों का कोण भिन्न-भिन्न होता है तथा दिन की अवधि भी समान नहीं होती । भूमध्य रेखा से ध्रुवों की ओर सूर्य की किरणों का तिरछापन बढ़ता जाता है तथा दिन की अवधि भी बढ़ती जाती है। इसलिए भिन्न-भिन्न अक्षांशों पर सूर्यातप की मात्रा में भिन्नता पाई जाती है। एक ही अक्षांश पर सूर्यातप की मात्रा सब स्थानों पर बराबर होती है।

प्रश्न 7.
दैनिक तापान्तर किसे कहते हैं?
उत्तर:
किसी स्थान पर उस दिन के उच्चतम तथा न्यूनतम तापमान के अन्तर को उस स्थान का दैनिक तापान्तर कहते हैं। यह तटीय प्रदेशों में कम होता है। दैनिक तापान्तर अन्दरूनी भागों तथा मरुस्थलीय प्रदेशों में अधिक होता है।

प्रश्न 8.
वार्षिक तापान्तर से क्या अभिप्राय है?
उत्तर:
किसी वर्ष के सबसे गर्म तथा ठण्डे महीनों के औसत मासिक तापमान के अन्तर को वार्षिक तापान्तर कहते हैं। प्रायः जुलाई मास को सबसे गर्म तथा जनवरी मास को सबसे ठण्डा मास लिया जाता है। सबसे अधिक वार्षिक तापान्तर साइबेरिया में वर्खोयांस्क में 38°C होता है।

प्रश्न 9.
किसी स्थान के तापमान से क्या अभिप्राय है?
उत्तर:
तापमान किसी पदार्थ में ताप की मात्रा का सूचक है। किसी स्थान पर छाया में भू-तल में फुट की ऊंची वायु की मापी हुई गर्मी को उस स्थान का तापमान कहा जाता है। प्रत्येक स्थान पर तापमान समान नहीं पाया जाता है। धरातल पर तापमान का वितरण विभिन्न संघटकों द्वारा नियन्त्रित होता है।

प्रश्न 10.
तापमान प्रतिलोम से क्या अभिप्राय है?
उत्तर:
ऊंचाई के बढ़ने के साथ-साथ 1°C प्रति 165 मीटर की दर से तापमान कम होता जाता है, परन्तु कई बार स्थानीय या अस्थाई रूप से ऊंचाई के साथ-साथ तापमान में वृद्धि होती है। ऐसी स्थिति में जब ठण्डी वायु धरातल के निकट और गर्म वायु इसके ऊपर हो तो इसे तापमान प्रतिलोम कहते हैं।

प्रश्न 11.
सूर्यातप का महत्त्व बताओ।
उत्तर:
सूर्यातप का महत्त्व (Importance of Insolation ) :
यद्यपि पृथ्वी के धरातल पर प्राप्त होने वाला सूर्यातप बहुत थोड़ा है फिर भी यह कई प्रकार से महत्त्वपूर्ण है-

  1. कई भौतिक क्रियाएं सूर्यातप पर निर्भर करती हैं।
  2. सूर्यातप के कारण पवनें तथा समुद्री धाराएं चलती हैं ।
  3. सूर्यातप ऋतु परिवर्तन का आधार है ।
  4. इसी सूर्यातप के कारण पृथ्वी मानव निवास के योग्य ग्रह है
  5. वायुमण्डलीय गतियां तथा वायुराशियाँ सूर्यातप पर ही निर्भर करती हैं।

JAC Class 11 Geography Important Questions Chapter 9 सौर विकिरण, ऊष्मा संतुलन एवं तापमान

प्रश्न 12.
वायुमण्डल पर ग्रीन हाऊस के प्रभाव का वर्णन करो।
उत्तर:
वायुमण्डल भू-तल से विकिरण द्वारा गर्म होता है। वायुमण्डल की तुलना एक शीशे के घर या ग्रीन हाऊस से की जाती है जहां ध्रुवीय क्षेत्र में फूल तथा सब्जियां उगाई जाती हैं। शीशे के घर के अन्दर सौर ताप प्रवेश कर सकता है परन्तु बाहर नहीं जा सकता इसलिए शीशे का घर अन्दर से गर्म होता है, वायुमण्डल भी एक छाते की भान्ति कार्य करके पृथ्वी को गर्म रखता है।

दिन के समय यह सूर्य को पूरी शक्ति से बचा कर तापमान बढ़ने नहीं देता तथा रात को भू-तल से विकिरण को बाहर नहीं जाने देता। पृथ्वी पर औसत तापमान 15°C रहता है। यह वायुमण्डल में कार्बन डाइऑक्साइड गैस के कारण है जो कि विकिरण को सोख कर एक छत का कार्य करती है। कार्बन डाइऑक्साइड गैस बढ़ने से पृथ्वी का औसत तापमान बढ़ रहा है। पिछली शताब्दी में भारत में 1955 का वर्ष सब से अधिक गर्म रहा है।

प्रश्न 13.
ग्लोबल वार्मिंग से क्या अभिप्राय है? इसके क्या कारण हैं? इसके प्रभाव बताओ।
उत्तर:
ग्लोबल वार्मिंग का अर्थ है पृथ्वी के औसत तापमान में वृद्धि होना। जीवाश्म ईंधनों के अत्यधिक जलने (कोयला, गैस, तेल) के कारण, अधिक कृषि, अधिक औद्योगीकरण के कारण, तीव्रगामी परिवहन साधनों के प्रयोग तथा वनों की अत्यधिक कटाई से वायुमण्डल के संघटन में एक असन्तुलन उत्पन्न हो गया है। इन क्रियाओं से कार्बन डाइऑक्साइड गैस की मात्रा बढ़ रही है। इसके ग्रीन हाऊस प्रभाव से पृथ्वी का औसत तापमान 0.5°C बढ़ गया है।

एक अनुमान है कि सन् 2040 तक विश्व तापमान में 2°C की वृद्धि हो जाएगी विभिन्न क्रियाओं के कारण वायुमण्डल में ओज़ोन गैस की कमी हो रही है। इस से पराबैंगनी किरण पृथ्वी पर पहुंच कर तापमान में वृद्धि कर रही है। इसके प्रभाव से ध्रुवीय प्रदेशों की हिम पिघलने से समुद्र तल ऊंचा हो रहा है। कई तटीय प्रदेशों के जलमग्न होने का भय है।

प्रश्न 14.
ऊंचाई के साथ तापमान में होने वाले ह्रास की व्याख्या करो
उत्तर:
विभिन्न निरीक्षणों से पता चलता है कि वायुमण्डल में ऊंचाई के साथ तापमान कम होता जाता है। तापमान कम होने की दर 1°C प्रति 165 मीटर है। इसे सामान्य ह्रास दर (Normal Lapse Rate) कहते हैं। इस तथ्य के मुख्य कारण निम्नलिखित हैं:

  1. वायुमण्डल प्रत्यक्ष रूप से धरातल द्वारा छोड़ी गई ऊष्मा से गर्म होता है। सूर्य का प्रभाव अप्रत्यक्ष होता है। धरातल के समीप वाली परत सबसे अधिक गर्म होती है, ऊंचाई पर परतें बाद में गर्म होती हैं।
  2. धरातल के समीप वायु में धूल-कण तथा जल वाष्प अधिक मात्रा में रहते हैं जो अधिक गर्मी ग्रहण कर लेते हैं। परन्तु वायुमण्डल की ऊपरी परतों में हल्की तथा स्वच्छ वायु कम मात्रा में गर्मी ग्रहण करती है। तापमान के कम होने की दर स्थानीय अवस्थाओं के कारण बदलती रहती है। रात के समय, महाद्वीपीय क्षेत्रों में ताप घटने की दर तीव्र होती है।

प्रश्न 15.
समताप रेखाओं से क्या अभिप्राय है?
उत्तर:
समताप रेखाएं ( Isotherms):
‘Iso’ शब्द का अर्थ है समान और ‘Therm’ शब्द का अर्थ है तापमान। इसलिए Isotherm का अर्थ है Lines of equal temperature या सम – तापमान रेखाएं। [Isotherms are lines joining the places of same (equal) temperature reduced to sea-level.]

धरातल पर समान तापमान वाले स्थानों को जोड़ने वाली रेखा को समताप रेखा कहते हैं। इस तापक्रम को समुद्र-तल पर घटा कर दिखाया जाता है। इस प्रकार ऊंचाई के प्रभाव को दूर करने का प्रयत्न किया जाता है। यह कल्पना जाती है कि सभी स्थान समुद्र तल पर स्थित हैं। यदि कोई स्थान 1650 मीटर ऊंचा है और उसका वास्तविक तापमान 20°C है तो उस स्थान का समुद्र तल पर तापमान 20°C + 10°C = 30°C क्योंकि प्रति 165 मीटर पर 1°C तापमान कम हो जाता है।

प्रश्न 16.
किसी स्थान का तापमान किस प्रकार अक्षांश पर निर्भर है?
उत्तर:
भूमध्य रेखा से ध्रुवों की ओर जाते हुए तापमान लगातार कम होता जाता है। किसी भी अक्षांश पर तापमान सूर्य की किरणों के कोण पर निर्भर है। लम्ब किरणें तिरछी किरणों की अपेक्षा थोड़े स्थान को घेरती हैं। अतः प्रति इकाई क्षेत्र प्राप्त ऊष्मा अधिक होती है। तिरछी किरणें वायुमण्डल में अधिक दूरी तय करती हैं तथा इनकी बहुत-सी गर्मी जलवाष्प अथवा धूलकणों द्वारा सोख ली जाती है। भूमध्य रेखा पर सारा वर्ष सूर्य की किरणें लम्बवत् पड़ती हैं तथा इन प्रदेशों में उच्च तापमान पाए जाते हैं। ध्रुवों की ओर तिरछी किरणों के कारण कम तापमान पाए जाते हैं।

JAC Class 11 Geography Important Questions Chapter 9 सौर विकिरण, ऊष्मा संतुलन एवं तापमान

प्रश्न 17.
तापमान तथा ऊंचाई में क्या सम्बन्ध है?
उत्तर:
समुद्र तल से ऊंचाई (Altitude ):
समुद्र तल से ऊंचाई के साथ तापमान घटता है। ताप के कम होने की दर 1°F प्रति 300 फुट या 0.6°C प्रति 100 मीटर है। वायुमण्डल धरातल द्वारा छोड़ी गई गर्मी (Radiation) से गर्म होता है। इसलिए निचली परतें पहले गर्म होती हैं तथा ऊपरी परतें बाद में पर्वत मैदानों की अपेक्षा ठण्डे होते हैं (Mountains are cooler than plains.) ऊंचाई के अनुसार वायु का दबाव सघनता, जलवाष्प तथा धूल कणों की कमी होती है।

इसलिए ऊंचे पर्वतीय क्षेत्र की शुद्ध तथा विरल वायु गर्मी को लीन नहीं कर सकती पर्वतीय प्रदेशों की कठोर चट्टानें शीघ्र ही गर्मी छोड़ देती हैं जो कि बिना रोक-टोक वायुमण्डल से बाहर निकल जाती हैं। इस प्रकार जो स्थान जितना ही ऊंचा होगा, उतना ही ठण्डा होगा।

प्रश्न 18.
भौमिक विकिरण किसे कहते हैं?
उत्तर:
पृथ्वी के धरातल द्वारा सौर ऊर्जा दीर्घ तरंगों के रूप में वापस लौट जाती है, इसे भौमिक विकिरण कहते हैं। इसीलिए वायुमण्डल नीचे से ऊपर की ओर गर्म होता है। भौमिक विकिरण ही वायुमण्डल को गर्म करने का प्रमुख स्रोत है।

प्रश्न 19.
ऊष्मा बजट किसे कहते हैं?
उत्तर:
पृथ्वी पर औसत तापमान एक समान रहता है। पृथ्वी का औसत तापमान 15°C है। सूर्यातप तथा भौमिक विकिरण के कारण पृथ्वी के ताप में सन्तुलन रहता है, पृथ्वी जितनी मात्रा में सौर ऊर्जा प्राप्त करती है, उतनी ही मात्रा में ऊर्जा भौमिक विकिरण द्वारा अन्तरिक्ष में लौट जाती है। इसे ऊष्मा बजट कहते हैं

प्रश्न 20.
हिमालय पर्वत की दक्षिणी ढलानों पर मानव बस्तियाँ हैं परन्तु उत्तरी ढलानों पर वन पाए जाते हैं। क्यों?
उत्तर:
हिमालय पर्वत की दक्षिणी ढलानें सूर्य की ओर झुकी रहती हैं, तथा अधिक गर्म होती हैं। परन्तु उत्तरी ढलानें सूर्य से परे झुकी होती हैं तथा ठण्डी होती हैं। इसलिए दक्षिणी ढलानों पर कृषि होती है तथा मानव बस्तियां हैं। परन्तु तिब्बत की ओर उत्तरी ढलान ठंडी है तथा यहां वन पाए जाते हैं।

प्रश्न 21.
दैनिक तापान्तर किसे कहते हैं?
उत्तर:
किसी स्थान के एक दिन (24 घण्टे) के अधिकतम तापमान तथा न्यूनतम तापमान के अन्तर को दैनिक तापान्तर कहते हैं।

निबन्धात्मक प्रश्न (Essay Type Questions)

प्रश्न 1.
सूर्यातप किसे कहते हैं? सूर्यातप का वितरण किन तत्त्वों पर निर्भर करता है?
उत्तर:
सूर्य वायुमण्डल को गर्मी तथा प्रकाश प्रदान करने वाला मुख्य तथा मूल स्रोत है। सूर्य का व्यास पृथ्वी से सौ गुना बड़ा है। सूर्य के धरातल पर 10,000°F से भी अधिक तापक्रम है। इस विशाल तेज पिण्ड से सभी दिशाओं में ताप तरंगें फैलती हैं। सूर्य ताप प्रकाश की गति से (186,000 मील या 3000,000 कि० मी० प्रति सेकण्ड की दर से) वायुमण्डल में से गुज़रता है।

पृथ्वी पर सूर्यातप का केवल दो अरबवां भाग ही (1/2000,000,000) प्राप्त होता है। अनुमानतः पृथ्वी प्रति मिनट 1.94 Calories गर्मी प्रति वर्ग सेंटीमीटर प्राप्त करती है। इसे सौर स्थिरांक (Solar Constant) कहते हैं। इस प्रकार धरातल पर प्राप्त होने वाले सौर्य विकिरण को सूर्यातप कहते हैं। (Insolation means incoming Solar Radiation on the Surface of the Earth.) सूर्यातप तीन शब्दों के जोड़ से बना है:

सूर्यातप (Insolation)= In + sol + ation
In = In coming
Sol = solar
ation = Radiation

सूर्यातप का महत्त्व (Importance of Insolation): यद्यपि पृथ्वी के धरातल पर प्राप्त होने वाला सूर्यातप बहुत थोड़ा है फिर भी यह कई प्रकार से महत्त्वपूर्ण है

  1. कई भौतिक क्रियाएं सूर्यातप पर निर्भर करती हैं।
  2. सूर्यातप के कारण पवनें तथा समुद्री धाराएं चलती हैं।
  3. सूर्यातप ऋतु परिवर्तन का आधार है।
  4. इसी सूर्यातप के कारण पृथ्वी मानव निवास के योग्य ग्रह हैं।
  5. वायुमण्डलीय गतियां तथा वायु राशियाँ सूर्यातप पर ही निर्भर करती हैं।

सूर्यातप वितरण के प्रमुख घटक (Major Factors affecting the distribution of Insolation)
प्रमुख घटक (Major Factors): पृथ्वी के हर स्थान पर सूर्यातप की मात्रा समान नहीं है। सूर्यातप की मात्रा निम्नलिखित प्रमुख घटकों पर निर्भर करती है

1. सौर विकिरण की तीव्रता (Intensity of Insolation ):
सौर विकिरण की तीव्रता सूर्य किरणों के आपात् कोण पर निर्भर करती है। पृथ्वी की गोल सतह पर सूर्य की किरणों का कोण हर स्थान पर भिन्न-भिन्न होता है। यह कोण भूमध्य रेखा पर लम्ब और ध्रुवों की ओर कम होता जाता है। इस प्रकार सूर्य की तिरछी व लम्ब किरणों के कारण सूर्यातप की मात्रा में अन्तर पाया जाता है। लम्ब किरणें तिरछी किरणों की अपेक्षा एक छोटे क्षेत्र में प्रभाव डालती हैं तथा प्रति इकाई क्षेत्र अधिक सूर्यातप प्राप्त होता है।

लम्बवत् किरणों को वायुमण्डल का थोड़ा भाग पार करना पड़ता है। इसलिए वायुमण्डल में मिली गैसें जल वाष्प द्वारा अवशोषण, प्रकीर्णन तथा परावर्तन से सूर्यातप की मात्रा कम नष्ट होती है। इसके विपरीत तिरछी किरणें अधिक स्थान घेरती हैं तथा वायुमण्डल का अधिक भाग पार करती हैं। इसके परिणामस्वरूप प्रति इकाई क्षेत्र प्राप्त सूर्यातप की मात्रा कम होती है तथा तिरछी किरणों के ताप का अधिक भाग वायुमण्डल में नष्ट हो जाता है।

निम्नलिखित तालिका से भी यह स्पष्ट हो जाता है:

सूर्य की किरणों का आपतन कोण (Angle of Sun’s Rays) वायुमण्डल में सूर्य की किरणों की लम्बाई (Length of Sun’s Rays in Atmosphere) तीव्रता (Intensity)
90° 1.00 78
60° 1.15 55
30° 2.00 31
45.00 0

JAC Class 11 Geography Important Questions Chapter 9 सौर विकिरण, ऊष्मा संतुलन एवं तापमान  1

2. दिन की अवधि (Length of Day ):
पृथ्वी के अक्ष के झुके होने के कारण हर स्थान पर दिन-रात की लम्बाई समान नहीं होती है। भूमध्य रेखा से दूर जाने पर ग्रीष्म काल में दिन बड़े हो जाते हैं और शीत काल में रातें बड़ी हो जाती हैं। दिन की अवधि बढ़ने का अर्थ है कि लम्बे समय तक सूर्यातप का प्राप्त होना। दिन की अवधि कम होने से सूर्यातप की मात्रा कम हो जाती है। परन्तु पृथ्वी पर सूर्यातप की मात्रा के वितरण पर इन दोनों घटकों का सम्मिलित प्रभाव पड़ता है उच्च अक्षांशों में दिन की अवधि अधिक होते हुए भी सूर्यातप की मात्रा कम होती है क्योंकि सूर्य की किरणें अधिक तिरछी पड़ती हैं जैसा कि निम्नलिखित तालिका से स्पष्ट है:

विभिन्न अक्षांशों पर दिन की अधिकतम अवधि (Length of Longest day at different Latitudes)
JAC Class 11 Geography Important Questions Chapter 9 सौर विकिरण, ऊष्मा संतुलन एवं तापमान  2

3. वायुमण्डल की मोटाई (Thickness of Atmosphere ):
वायुमण्डल के मोटे आवरण में से गुज़रते समय सूर्यातप का 49% भाग मार्ग में नष्ट हो जाता है। वायुमण्डल की मोटाई निम्नलिखित रूप से सूर्यातप की मात्रा पर प्रभाव डालती है।
(क) प्रकीर्णन (Scattering): सूर्यातप का कुछ भाग अणुओं तथा धूल कणों द्वारा बिखेर दिया जाता है जिसके कारण आकाश नीला प्रतीत होता है।
(ख) परावर्तन (Reflection ): धूल कण, जलवाष्प तथा कुछ सूर्यातप अन्तरिक्ष में लौट जाता है।
(ग) अवशोषण (Absorption ): जलवाष्प तथा गैसों द्वारा काफ़ी मात्रा में सूर्यातप जज़ब कर लिया जाता है।
जो सूर्यातप पृथ्वी की सतह से अंतरिक्ष में लौट जाता है उसका कुछ अंश आकाश के नीले तथा मन्द प्रकाश के रूप में उपयोगी है। उच्च अक्षांशों में यह मन्द प्रकाश (Diffused day light) पूर्ण अन्धकार नहीं होने देता।

वायुमण्डल में नष्ट होने वाले सूर्यातप की मात्रा मान लो कि वायुमण्डल की ऊपरी सतह पर प्राप्त होने वाला ताप 100 इकाई है। इसमें से केवल 51 इकाई ताप ही पृथ्वी पर पहुंचता है। इसमें 49 इकाई ताप वायुमण्डल तथा अन्तरिक्ष में लौट जाता है। शून्य में परावर्तन हुई उस ऊष्मा को एल्बेडो (Albedo of the earth) कहते हैं।
वायुमण्डल की ऊपरी सतह पर प्राप्त ऊष्मा
JAC Class 11 Geography Important Questions Chapter 9 सौर विकिरण, ऊष्मा संतुलन एवं तापमान  3

पृथ्वी की सतह पर प्राप्त ऊष्मा 100 – 49 = 51%
इस प्रकार सूर्यातप का केवल 51% भाग ही पृथ्वी की सतह पर प्राप्त होता है। यह ताप पृथ्वी से दीर्घ किरणों द्वारा विकिरण के माध्यम से वायुमण्डल को गर्म करता है।

अन्य घटक (Minor Factors):
4.  जल एवम् स्थल का वितरण (Distribution of land and water ):
पानी की अपेक्षा स्थल भाग अधिक तथा शीघ्र गर्म हो जाते हैं तथा शीघ्र ही ठण्डे हो जाते हैं। जल का आक्षिक ताप (Specific Heat) स्थल के आक्षिक ताप से 2 गुना अधिक है। महासागरों में सूर्य की किरणों को अधिक गहराई तक जल को गर्म करना पड़ता है। जल गतिशील भी है। इसलिए जल-भागों को गर्म करने के लिए अधिक ताप की आवश्यकता होती है। इसी कारण ग्रीष्म ऋतु में महाद्वीप अधिक गर्म हो जाते हैं जबकि शीतकाल में महासागर अधिक गर्म होते हैं।

5. धरातल का स्वभाव (Nature of the Surface ): धरातल में विभिन्नता के कारण ताप के अवशोषण तथा परावर्तन की महत्ता भिन्न-भिन्न होती है। जैसे विभिन्न भूतलों पर सौर किरणों का परावर्तन अग्रलिखित प्रकार से होता है
वनस्पति – 7%
रेत – 13%
जल – 30%
बर्फ़ – 80%
इसी कारण हिम से ढका भूतल कम ताप ग्रहण करता है जबकि मरुस्थल में रेतीली भूमि अधिक ताप प्राप्त करती है।

6. पृथ्वी से सूर्य की दूरी (Distance between Earth and Sun):
पृथ्वी तथा सूर्य के मध्य दूरी सदा समान नहीं रहती। उत्तरायण (Aphelion) के समय यह दूरी 940 लाख मील होती है तथा दक्षिणायन (Perihelion) के समय यह दूरी 915 लाख मील होती है। इस प्रकार उत्तरायण के समय तथा दक्षिणायन के समय सूर्यातप में 7% का अन्तर पड़ जाता है।

7. सौर कलंकों की संख्या (Number of Sun Spots ):
सौर कलंकों की संख्या घटती-बढ़ती रहती है । अधिक संख्या के कारण सूर्यातप की मात्रा अधिक प्राप्त होती है।

JAC Class 11 Geography Important Questions Chapter 9 सौर विकिरण, ऊष्मा संतुलन एवं तापमान

प्रश्न 2.
तापमान के क्षैतिज वितरण को कौन-से संघटक नियन्त्रित करते हैं?
उत्तर:
तापमान किसी पदार्थ में ताप की मात्रा का सूचक है। किसी स्थान पर छाया में भूतल 4 फुट की ऊंची वायु की मापी हुई गर्मी को उस स्थान का तापमान कहा जाता है। प्रत्येक स्थान पर तापमान समान नहीं पाया जाता है। धरातल पर तापमान का वितरण निम्नलिखित संघटकों द्वारा नियन्त्रित होता है:

1. भूमध्य रेखा से दूरी (Distance from the Equator):
भूमध्य रेखा से ध्रुवों की ओर जाते हुए तापमान लगातार कम होता जाता है। किसी भी अक्षांश पर तापमान सूर्य की किरणों के कोण पर निर्भर है। लम्ब किरणें तिरछी किरणों की अपेक्षा थोड़े स्थान को घेरती हैं। अतः प्रति इकाई क्षेत्र प्राप्त ऊष्मा अधिक होती है। तिरछी किरणें वायुमण्डल में से अधिक दूरी तय करती हैं तथा इनकी बहुत-सी गर्मी जलवाष्प अथवा धूलकणों द्वारा सोख ली जाती है। भूमध्य रेखा पर सारा वर्ष सूर्य की किरणें लम्बवत् पड़ती हैं तथा इन प्रदेशों में उच्च तापमान पाए जाते हैं। ध्रुवों की ओर तिरछी किरणों के कारण कम तापमान पाए जाते हैं।

2. समुद्र तल से ऊंचाई (Altitude):
समुद्र तल से ऊंचाई के साथ तापमान घटता है। ताप के कम होने की दर 1°F प्रति 300 फुट या 0.6°C प्रति 100 मीटर है। वायुमण्डल धरातल द्वारा छोड़ी गई गर्मी (Radiation) से गर्म होता है। इसलिए निचली परतें पहले गर्म होती हैं तथा ऊपरी परतें बाद में पर्वत मैदानों की अपेक्षा ठण्डे होते हैं (Mountains are cooler than plains)। ऊंचाई के अनुसार वायु का दबाव सघनता, जलवाष्प तथा धूल के कणों की कमी होती है। इसलिए ऊंचे पर्वतीय क्षेत्र की शुद्ध तथा विरल वायु गर्मी को लीन नहीं कर सकती पर्वतीय प्रदेशों की कठोर चट्टानें शीघ्र ही गर्मी छोड़ देती हैं जो कि बिना रोक-टोक वायुमण्डल से बाहर निकल जाती हैं। इस प्रकार जो स्थान जितना ही ऊंचा होगा, उतना ही ठण्डा होगा।

3. समुद्र से दूरी (Distance from the Sea) समुद्र का प्रभाव जलवायु के लिए समकारी होता है। समुद्र के समीप के प्रदेशों में सम (Equable) जलवायु होती है। परन्तु भीतरी (Inland) या समुद्र से दूर प्रदेशों में कठोर (Extreme) जलवायु मिलती है। जल स्थल की अपेक्षा धीरे-धीरे गर्म तथा ठण्डा होता है इसलिए तटीय प्रदेशों में जल समीर (Sea Breeze) के कारण दिन का ताप अधिक नहीं होता है। स्थल समीर (Land Breeze) के कारण रात का ताप अधिक कम नहीं होता। इसलिए इनके प्रभाव से गर्मी तथा सर्दी दोनों ही अधिक नहीं होतीं। परन्तु समुद्र तट से अधिक दूर जाने पर, स्थल के प्रभाव के कारण ग्रीष्म ऋतु अधिक गर्म तथा शीत ऋतु अधिक ठण्डी होती है।

4. प्रचलित पवनें (Prevailing Winds):
प्रचलित पवनें किसी प्रदेश के तापमान तथा वर्षा पर महत्त्वपूर्ण प्रभाव डालती हैं। समुद्र की ओर से आने वाली पवनें जलवायु को सम तथा आर्द्र रखती हैं। (A wind from the sea lowers the summer temperature and raises the winter temperature.) परन्तु स्थल की ओर से आने वाली पवनें स्थल के प्रभाव के कारण किसी प्रदेश की जलवायु को कठोर तथा शुष्क बनाती हैं। (A Wind from the land lowers the winter temperature and raises the summer temperature.) समुद्र से आने वाली पश्चिमी पवनों (Westerlies) के कारण शीत ऋतु में इंग्लैण्ड का औसत तापमान 20°F – 30°F ऊंचा रहता है।

5. समुद्री धाराएं (Ocean Currents ):
समुद्री धाराओं का प्रभाव उन पवनों द्वारा होता है जो इन धाराओं के ऊपर से गुज़रती हैं। गर्म धाराओं के ऊपर से गुज़रने वाली पवनें तटीय प्रदेशों के तापक्रम को ऊँचा कर देती हैं तथा वर्षा में सहायक होती हैं। शीत ऋतु का तापमान बढ़ जाने से जलवायु सम हो जाती है । परन्तु ठण्डी धाराओं के ऊपर से गुज़रने वाली पवनों के प्रभाव से तटीय प्रदेश ठण्डे तथा शुष्क होते हैं। (Warm currents raise and cold currents lower the temperature of coastal areas.)
JAC Class 11 Geography Important Questions Chapter 9 सौर विकिरण, ऊष्मा संतुलन एवं तापमान  4

6. पर्वतों की दिशा (Direction of Mountains):
किसी देश में पर्वतों की स्थिति तथा दिशा तापमान तथा वर्षा पर प्रभाव डालती है। अरावली पर्वत मानसून पवनों के समानान्तर स्थित होने के कारण इन्हें रोक नहीं पाता जिससे राजस्थान शुष्क रहता है। यदि हिमालय पर्वत मानसून पवनों के आड़े स्थित न होता तो उत्तरी भारत भी मरुस्थल होता । पर्वतों के सम्मुख ढाल पर वर्षा होती है परन्तु त्रिमुख ढाल वर्षा छाया (Rain Shadow) में होने के कारण शुष्क रहते हैं ।

7. भूमि की उलान (Slope of the Land):
सूर्यमुखी ढाल सूर्य त्रिमुखी ढालों की अपेक्षा गर्म होती है। उत्तरी ढाल की अपेक्षा दक्षिणी ढाल पर सूर्य की किरणें अधिक सीधी पड़ती हैं। दक्षिणी ढाल उत्तर से आने वाली ठण्डी हवाओं से सुरक्षित रहती है। उत्तरी अधिक देर तक छाया में रहती है परन्तु दक्षिणी ढाल पर सूर्य अधिक देर तक चमकता है। हिमालय पर्वत के तिब्बत की ओर ढाल वाले प्रदेश ठण्डे हैं परन्तु भारत की ओर ढाल वाले प्रदेश गर्म हैं।
JAC Class 11 Geography Important Questions Chapter 9 सौर विकिरण, ऊष्मा संतुलन एवं तापमान  5

8. भू-तल का स्वभाव (Nature of the Soil):
मैदानों की चिकनी मिट्टी प्रायः बारीक होती है तथा शनै: शनै: गर्म तथा ठण्डी होती है परन्तु रेत जल्दी ही गर्म तथा ठण्डी हो जाती है । इसी कारण मरुस्थलों में दिन को अधिक गर्मी तथा रात को अधिक सर्दी होती है ।

9. मेघ और वर्षा (Clouds and Rainfall):
जिन प्रदेशों में बादल छाए रहते हैं, वहां जलवायु सम रहती है। और अधिक वर्षा होती है। मेघ सूर्य की किरणों को दिन के समय रोकते हैं। सूर्य के ताप को लीन कर लेते हैं। मेघरहित (Cloudless) आकाश के कारण दिन को तापक्रम ऊँचा हो जाता है। ये रात को पृथ्वी द्वारा छोड़ी हुई गर्मी को बाहर नहीं जाने देते । वर्षा के कारण भी तापमान कम हो जाता है।

JAC Class 11 Geography Important Questions Chapter 9 सौर विकिरण, ऊष्मा संतुलन एवं तापमान

प्रश्न 3.
तापमान कटिबन्ध से क्या अभिप्राय है? अक्षांशों के आधार पर ताप-कटिबन्धों का वर्णन करो अक्षांशों के आधार पर तापमान का क्षैतिज वितरण
उत्तर:
अक्षांशों के आधार पर तापमान का क्षैतिज वितरण (Horizontal Distribution of Temperature Based on Latitudes):
विषुवत् रेखा से ध्रुवों की ओर सूर्यातप की प्राप्ति कम होती जाती है क्योंकि सौर – ताप की किरणों में अधिकाधिक तिरछापन आता रहता है। इस प्रकार भूतल पर तापमान का क्षैतिज वितरण अक्षांशों के अनुसार घटता-बढ़ता है। यद्यपि विषुवत् रेखा पर वर्ष पर्यन्त सूर्य लगभग लम्बवत् रहता है, किन्तु यहां आकाश अधिकांश अवधि के लिए मेघाच्छादित रहता है तथा वर्षा प्रचुर मात्रा में होती है। अतः सूर्यातप का अधिकांश भाग परावर्तित हो जाता है तथा पर्याप्त मात्रा में ऊष्मा वाष्पीकरण में व्यय हो जाती है।

इन कारणों से उच्चतम तापमान कभी भी विषुवत् रेखीय प्रदेशों में नहीं पाया जाता संसार के उच्चतम तापमान कर्क एवं मकर रेखाओं के निकट पाए जाते हैं क्योंकि यहां आकाश सदैव मेघ – विहीन तथा स्वच्छ रहता है, जिससे सूर्य शक्ति बिना किसी अवरोध के यहां पहुंचती है तथा यहां की भूमि को प्रचण्ड रूप से उष्ण कर देती है। मकर रेखा की अपेक्षा कर्क रेखा पर स्थल का विस्तार अधिक होने के कारण वहां तापमान उच्च रहते हैं। इस प्रकार के संसार के उच्चतम तापमान कर्क रेखा पर ही पाए जाते हैं।

अतः संसार की तापीय मध्य रेखा (Thermal or Heat Equator) कर्क रेखा के साथ-साथ पाई जाती है। विषुवत्रेखा से ध्रुवों की ओर तापमान के घटने की दर को ताप प्रवणता (Temperature Gradient) कहते हैं। यह ताप प्रवणता कर्क एवं मकर रेखाओं के बीच के भू-भाग में अति न्यून होती है। इस प्रकार आयतन का क्षैतिज वितरण विषुवत् रेखा से दूरी अर्थात् अक्षांशों का अनुसरण करता पाया जाता है। प्राचीन यूनानी (Greek) विचारकों ने अक्षांशों के आधार पर पृथ्वी के तल को निम्नलिखित तीन भागों में विभक्त किया है:

ताप कटिबन्ध (Temperature Zone):
1. उष्ण कटिबन्ध (Torrid or Hot Zone):
उत्तरी गोलार्द्ध में कर्क रेखा \(\left(23 \frac{1}{2}^{\circ} \text { North }\right)\) तथा दक्षिणी गोलार्द्ध में मकर रेखा \(\left(23 \frac{1}{2}^{\circ} \text { South }\right)\) के बीच विस्तृत भू-भाग में सौर किरणें वर्ष भर में एक बार अवश्य ही ऊर्ध्वाधर (Vertical) पड़ती हैं। परन्तु इस भू-भाग से दूर सूर्य कभी भी ऊर्ध्वाधर नहीं होता। दूसरे शब्दों में, सूर्य इस भू-भाग में वर्ष पर्यन्त लगभग लम्बवत् रहता है। इस भू-भाग में दिन एवं रात्रि की अवधि में भी बहुत कम अन्तर होता है। अतः इस भू-खण्ड में सदैव प्रचण्ड रूप से उष्णता रहती है, जिससे इसे उष्ण कटिबन्ध (Hot or Torrid Zone) कहते हैं।
JAC Class 11 Geography Important Questions Chapter 9 सौर विकिरण, ऊष्मा संतुलन एवं तापमान  6

2. शीतोष्ण कटिबन्ध (Temperature Zones):
कर्क रेखा एवं उत्तरी ध्रुववृत्त \(\left(66 \frac{1}{2}\right)^{\circ}\) उत्तरी अक्षांश Article Circle,\( \left(66 \frac{1}{2}^{\circ} \text { North }\right)\) तथा मकर रेखा, \(23 \frac{1}{2} \circ\)दक्षिण (Tropic of copricorn) से अंटार्कटिक वृत्त \(66\frac{1}{2} \circ\) दक्षिणी मध्य के भू-भाग में सूर्य की किरणें सदा तिरछी पड़ती हैं। इसके परिणामस्वरूप शीतकाल में बहुत निम्न तापमान तथा ग्रीष्म काल में दरमियाने तापमान रहते हैं। इन क्षेत्रों को उत्तरी शीत ऊष्ण कटिबन्ध (North Temperature Zone) तथा दक्षिणी शीत-उष्ण कटिबन्ध (South Temperature Zone) कहते हैं।

3. शीत कटिबन्ध ( Frigid Zone ):
उत्तरी ध्रुव तथा उत्तरी ध्रुवीय वृत्त, दक्षिणी ध्रुव तथा दक्षिणी ध्रुवीय वृत्त के मध्य स्थित भागों को क्रमवार उत्तरी शीत खण्ड तथा दक्षिणी शीत खण्ड कहते हैं। इन भागों में सारा वर्ष सूर्य की किरणें बहुत तिरछी पड़ती हैं। इसलिए तापमान बहुत कम रहते हैं तथा कठोर शीत पड़ती है।

JAC Class 11 Geography Important Questions Chapter 4 महासागरों और महाद्वीपों का वितरण 

Jharkhand Board JAC Class 11 Geography Important Questions Chapter 4 महासागरों और महाद्वीपों का वितरण Important Questions and Answers.

JAC Board Class 11 Geography Important Questions Chapter 4 महासागरों और महाद्वीपों का वितरण

बह-विकल्पी प्रश्न (Multiple Choice Questions)

प्रश्न-दिए गए प्रश्नों के चार वैकल्पिक उत्तरों में से सही उत्तर चुनकर लिखो
1. महाद्वीपीय विस्थापन का सिद्धान्त किसने प्रस्तुत किया?
(A) होल्मस
(B) वैगनर
(C) टेलर
(D) काण्ट।
उत्तर:
(B) वैगनर।

2. आरम्भ में सभी स्थल खण्ड एक बड़े भू-भाग के रूप में जुड़े थे जिसे कहते हैं
(A) गोंडवाना लैंड
(B) लारेशिया
(C) पेंजिया
(D) पेन्थालासा।
उत्तर:
(C) पेंजिया।

3. प्रशान्त महासागर के दक्षिण-पूर्व में स्थित प्लेट को कहते हैं
(A) कोकोस प्लेट
(B) नाज़का प्लेट
(C) भारतीय प्लेट
(D) फिलीपाइन प्लेट।
उत्तर:
(B) नाज़का प्लेट।

4. वेगनर ने विस्थापन सिद्धान्त कब प्रस्तुत किया?
(A) 1911
(B) 1912
(C) 1913
(D) 1914
उत्तर:
(B) 1912

JAC Class 11 Geography Important Questions Chapter 4 महासागरों और महाद्वीपों का वितरण

5. गोंडवाना लैंड पेंजिया से कब अलग हुआ?
(A) 4.5 करोड़ वर्ष पूर्व
(B) 5.5 करोड़ वर्ष पूर्व।
(C) 6.5 करोड़ वर्ष पूर्व
(D) 7.5 करोड़ वर्ष पूर्व।
उत्तर:
(B) 5.5 करोड़ वर्ष पूर्व।

6. कौन-सा महाद्वीप गोंडवाना लैंड का भाग नहीं था?
(A) अफ्रीका
(B) ऑस्ट्रेलिया
(C) अंटार्कटिका
(D) एशिया।
उत्तर:
(D) एशिया।

7. जलोढ़ में स्वर्ण निक्षेप कहां मिले हैं?
(A) घाना तट
(B) ऑस्ट्रेलिया तट
(C) चिल्ली तट
(D) गियाना तट।
उत्तर:
(A) घाना तट।

8. मध्यवर्ती महासागरीय कटक किस महासागर में है?
(A) हिन्द महासागर
(B) प्रशान्त महासागर
(C) आर्कटिक महासागर
(D) अन्ध महासागर।
उत्तर:
(D) अन्ध महासागर।

9. किस स्तर पर भू-प्लेटें विपरीत दिशा में खिसकती हैं?
(A) अभिसरण क्षेत्र
(B) अपसरण क्षेत्र
(C) रूपान्तर क्षेत्र
(D) ज्वालामुखी क्षेत्र।
उत्तर:
(C) रूपान्तर क्षेत्र।

JAC Class 11 Geography Important Questions Chapter 4 महासागरों और महाद्वीपों का वितरण

10. संवहन धाराओं की संकल्पना किसने प्रस्तुत की?
(A) वेगनर
(B) होम्स
(C) टेलर
(D) ट्रिवार्था।
उत्तर:
(B) होम्स।

अति लघु उत्तरीय प्रश्न (Very Short Answer Type Questions)

प्रश्न 1.
किसने और कब महाद्वीपीय संचलन सिद्धान्त प्रस्तुत किया?
उत्तर:
अल्फ्रेड वैगनर ने 1912 ई० में।

प्रश्न 2.
मूल महाद्वीप का क्या नाम था? यह कब बना?
उत्तर:
पेंजिया-काल्पनिक कल्प में 280 मिलियन वर्ष पूर्व।

प्रश्न 3.
पेंजिया से पृथक् होने वाले उत्तरी महाद्वीप का नाम लिखो।
उत्तर:
लारेशिया।

प्रश्न 4.
पेंजिया से पृथक् होने वाले दक्षिणी महाद्वीप का नाम लिखो।
उत्तर:
गोंडवानालैंड।

JAC Class 11 Geography Important Questions Chapter 4 महासागरों और महाद्वीपों का वितरण

प्रश्न 5.
गोंडवानालैंड में शामिल भू-खण्डों के नाम लिखो।
उत्तर:
दक्षिणी अमेरिका, अफ्रीका, ऑस्ट्रेलिया तथा अंटार्कटिका।

प्रश्न 6.
अफ्रीका तथा दक्षिणी अमेरिका में स्वर्ण निक्षेप कहां पाये जाते हैं?
उत्तर:
घाना तथा ब्राज़ील में।

प्रश्न 7.
ध्रुवों के घूमने से क्या अभिप्राय है?
उत्तर:
विभिन्न युगों में ध्रुवों की स्थिति का बदलना।

प्रश्न 8.
समुद्र के अधस्तल के विस्तारण से क्या अभिप्राय है?
उत्तर:
महासागरीय द्रोणी का फैलना तथा चौड़ा होना।

प्रश्न 9.
प्लेटों के संचलन का क्या कारण है?
उत्तर:
तापीय संवहन क्रिया।

प्रश्न 10.
संवहन क्रिया सिद्धान्त किसने प्रस्तुत किया?
उत्तर:
सन् 1928 में आर्थर होम्स ने।

प्रश्न 11.
स्थलमण्डल पर कुल कितनी प्लेटें हैं?
उत्तर:
7.

प्रश्न 12.
सबसे बड़ी भू-प्लेट कौन-सी है?
उत्तर:
प्रशान्त महासागरीय प्लेट।

प्रश्न 13.
हिमालय पर्वत की उत्पत्ति का क्या कारण था?
उत्तर:
भारतीय प्लेट तथा यूरेशियन प्लेट का आपसी टकराव।

प्रश्न 14.
पेंजिया शब्द का अर्थ क्या है?
उत्तर:
सम्पूर्ण पृथ्वी।

JAC Class 11 Geography Important Questions Chapter 4 महासागरों और महाद्वीपों का वितरण

प्रश्न 15.
पैंथालासा शब्द का अर्थ क्या है?
उत्तर:
जल ही जल।

प्रश्न 16.
प्लेट शब्द का प्रयोग सर्वप्रथम किसने किया था?
उत्तर:
टूजो विल्सन।

प्रश्न 17.
प्लेटों में गति का क्या कारण है?
उत्तर:
प्लेटों में गति का कारण तापीय संवहन क्रिया है।

प्रश्न 18.
‘टेक्टोनिकोज’ किस भाषा का शब्द है? इसका क्या अर्थ है?
उत्तर:
यह यूनानी भाषा का शब्द है जिसका अर्थ निर्माण है।

लघु उत्तरीय प्रश्न (Short Answer Type Questions)

प्रश्न 1.
पेंजिया किसे कहते हैं? इसकी उत्पत्ति कब हुई? इसमें मिलने वाले भू-खण्ड बताओ। पेंजिया के टूटने की क्रिया बताओ।
उत्तर:
विश्व के सभी भू-खण्ड पेंजिया नामक एक महा-महाद्वीपीय से विलग होकर बने हैं, यह बात अल्फ्रेड वैगनर ने 1912 में कही। पेंजिया नामक यह महाद्वीप 28 करोड़ वर्ष पूर्व, कार्बनी कल्प के अन्त में अस्तित्व में आया। मध्य जुरैसिक कल्प तक यानि 15 करोड़ वर्ष पूर्व, पेंजिया उत्तरी महाद्वीप लॉरेशिया तथा दक्षिणी महाद्वीप गोंडवानालैंड में विभक्त हो गया था।

लगभग 6.5 करोड़ वर्ष पूर्व अर्थात् क्रिटेशस कल्प के अन्त में गोंडवानालैंड फिर से खंडित हुआ और इससे कई अन्य महाद्वीपों जैसे दक्षिण अमेरिका, अफ्रीका, ऑस्ट्रेलिया और अंटार्कटिका की रचना हुई। भारत इससे टूटकर स्वतंत्र रूप से एक अलग पथ पर उत्तर-पूर्व की ओर अग्रसर हुआ।

JAC Class 11 Geography Important Questions Chapter 4 महासागरों और महाद्वीपों का वितरण

प्रश्न 2.
Jig-saw-fit से क्या अभिप्राय है? अन्ध-महासागर के दोनों तटों पर मिलने वाली समानताएं बताओ। इससे क्या निष्कर्ष निकलता है?
उत्तर:
Jig-saw- fit का अर्थ है कि अन्ध महासागर का पूर्वी तथा पश्चिमी तट किसी समय एक साथ जुड़े हुए थे। इन तटों पर कई समानताएं हैं।

  1. गिन्नी की खाड़ी ब्राज़ील के तट के साथ जोड़ी जा सकती है। अफ्रीका का पश्चिमी भाग खाड़ी मैक्सिको में जोड़ा जा सकता है। पश्चिमी यूरोप तथा उत्तरी अमेरिका का पूर्वी तट तथा ग्रीनलैंड अफ्रीका जोड़े जा सकते हैं।
  2. पूर्वी तट पर घाना में तथा पश्चिमी तट पर ब्राज़ील में अमेरिका स्वर्ण निक्षेप पाये जाते हैं।
  3. गोंडवानालैंड के सभी भू-खण्डों में हिमानी निक्षेप मिलते हैं। इन समानताओं से निष्कर्ष निकलता है कि ये महाद्वीप किसी प्राचीन भू-वैज्ञानिक काल में इकट्ठे थे।

JAC Class 11 Geography Important Questions Chapter 4 महासागरों और महाद्वीपों का वितरण  1

प्रश्न 3.
ध्रुवों के घूमने से क्या अभिप्राय है?
उत्तर:
ध्रुवों का घूमना (Polar Wandering):
पहले महाद्वीप पेंजिया के रूप में परस्पर एक-दूसरे से जुड़े हुए थे, इसका सबसे शक्तिशाली प्रमाण पुरा चुम्बकत्व से प्राप्त हुआ है। मैग्मा, लावा तथा असंगठित अवसाद में उपस्थित चुम्बकीय प्रवृत्ति वाले खनिज जैसे मैग्नेटाइट, हेमेटाइट, इल्मेनाइट और पाइरोटाइट इसी प्रवृत्ति के कारण उस समय के चुंबकीय क्षेत्र के समानान्तर एकत्र हो गए। यह गुण शैलों में स्थाई चुम्बकत्व के रूप में रह जाता है।

चुम्बकीय ध्रुव की स्थिति में कालिक परिवर्तन होता रहा है, जो शैलों में स्थाई चुम्बकत्व के रूप में अभिलेखित किया जाता है। वैज्ञानिक विधियों द्वारा पुराने शैलों में हुए ऐसे परिवर्तनों को जाना जा सकता है, जिनसे भूवैज्ञानिक काल में ध्रुवों की बदलती हुई स्थिति की जानकारी होती है। इसे ही ध्रुवों का घूमना कहते हैं। धूवों का घूमना यह स्पष्ट करता है कि महाद्वीपों का समय-समय पर संचलन होता रहा है और वे अपनी गति की दिशा भी बदलते रहे हैं।

प्रश्न 4.
अपसरण क्षेत्र तथा अभिसरण क्षेत्र में अन्तर स्पष्ट करो।
उत्तर:
अपसरण क्षेत्र-ये वे सीभाएं हैं जहां प्लेटें एक-दूसरे से अलग होती हैं। भूगर्भ से मैग्मा बाहर आता है। ये महासागरीय कटकों के साथ-साथ देखा जाता है। इन सीमाओं के साथ ज्वालामुखी तथा भूकम्प मिलते हैं। इसका उदाहरण मध्य अटलांटिक कटक है जहां से अमेरिकी प्लेटें तथा यूरेशियम व अफ्रीकी प्लेटें अलग होती हैं। अभिसरण क्षेत्र-ये वे सीमाएं हैं जहां एक प्लेट का किनारा दूसरे के ऊपर चढ़ जाता है। इनसे गहरी खाइयों तथा वलित श्रेणियों की रचना होती है।

ज्वालामुखी तथा गहरे भूकम्प उत्पन्न होते हैं। रूपांतर सीमा-जहां न तो नई पर्पटी का निर्माण होता है और न ही विनाश होता है, उसे रूपांतर सीमा कहते हैं। इसका कारण है कि इस सीमा पर प्लेटें एक-दूसरे के साथ-साथ क्षैतिज दिशा में सरक जाती हैं। रूपांतर भ्रंश (Transform faults) दो प्लेट को अलग करने वाले तल हैं जो सामान्यतः मध्य-महासागरीय कटकों से लंबवत स्थिति में पाए जाते हैं।

JAC Class 11 Geography Important Questions Chapter 4 महासागरों और महाद्वीपों का वितरण

प्रश्न 5.
समुद्र अधस्तल के विस्तारण का क्या महाद्वीपीय अर्थ है ?
उत्तर:
मध्यवर्ती महासागरीय कटक महासागर के अधस्तल पर स्थित दरारें हैं। इनसे लावा बाहर निकलता है। पिघला हुआ पदार्थ एक नये धरातल की रचना करता है। यह अधस्तल कटक से दूर फैलता है। इस प्रकार महासागरीय द्रोणी चौड़ी हो जाती है। इसे समुद्र अधस्तल मैंटल विस्तारण कहते हैं।
JAC Class 11 Geography Important Questions Chapter 4 महासागरों और महाद्वीपों का वितरण  2

प्रश्न 6.
महत्त्वपूर्ण छोटी प्लेटों का वर्णन करो।
उत्तर:
कुछ महत्त्वपूर्ण छोटी प्लेटें निम्नलिखित हैं

  1. कोकोस (Cocoas) प्लेट: यह प्लेट मध्यवर्ती अमेरिका और प्रशान्त महासागरीय प्लेट के बीच स्थित है।
  2. नाजका प्लेट (Nazca plate): यह दक्षिण अमेरिका व प्रशान्त महासागरीय प्लेट के बीच स्थित है।
  3. अरेबियन प्लेट (Arabian plate): इसमें अधिकतर साऊदी अरब का भू-भाग सम्मिलित है।
  4. फिलिपाइन प्लेट (Philippine plate): यह एशिया महाद्वीप और प्रशान्त महासागरीय प्लेट के बीच स्थित है।
  5. कैरोलिन प्लेट (Caroline plate): यह न्यू गिनी के उत्तर में फिलिपियन वे इंडियन प्लेट के बीच स्थित है।
  6. फ्यूजी प्लेट (Fuji plate): यह ऑस्ट्रेलिया के उत्तर-पूर्व में स्थित है।

निबन्धात्मक प्रश्न (Essay Type Questions)

प्रश्न 1.
प्लेट विवर्तनिक सिद्धान्त और इसकी क्रियाविधि की व्याख्या कीजिए।
अथवा
प्लेट विवर्तन की अवधारणा का वर्णन करें।
उत्तर:
भूमण्डलीय प्लेट विवर्तनिक सिद्धान्त के अनुसार स्थलमण्डल मध्यम दृढ़ प्लेटों में विभक्त है। ये प्लेटें निरन्तर संचलन कर रही हैं और उनकी गति-दिशा सापेक्ष है। प्लेटों के सीमान्त (Plate Boundaries): प्लेटों की सापेक्ष संचलन के आधार पर तीन विभिन्न प्रकार की प्लेट-सीमाएं या सीमान्त क्षेत्रों की रचना होती है:

  1. अपसरण अथवा विस्तारण क्षेत्र या सीमान्त
  2. अभिसरण क्षेत्र या सीमान्त; तथा
  3. विभंग क्षेत्र अथवा रूपान्तर भ्रंश।

1. अपसरण क्षेत्र (Zones of Divergence):
वे सीमाएं हैं, जहां प्लेटें एक-दूसरे से अलग होती हैं और पृथक्करण की इस प्रक्रिया में भूगर्भ से मैग्मा बाहर आता है। सामान्यतः ऐसा रैखिक महासागरी कटकों के साथ-साथ देखा जाता है, जहां नए महासागरीय अधस्तल के रूप में नवीन स्थलमण्डल का निर्माण हो रहा है। ऐसे सीमान्तों की विशिष्टता सक्रिय ज्वालामुखी उद्भव तथा उथले उद्गम केन्द्रों वाले भूकम्प हैं।

2. अभिसरण क्षेत्र (Zones of Convergence):
वे सीमाएं हैं, जहां एक प्लेट का किनारा दूसरे के ऊपर चढ जाता है, जिससे नीचे की प्लेट मैंटल में फिसल कर इसी में विलीन हो जाती है। इस प्रक्रिया को प्रविष्ठन (Subduction) कहते हैं। इन सीमाओं पर ज्वालामुखी उद्भव तथा उथले से गहरे उद्गम केन्द्रों वाले भूकम्पों की उत्पत्ति के अतिरिक्त गहरी महासागरीय खाइयों, द्रोणियों तथा वलित पर्वत श्रेणियों की रचना होती है।

3. रूपांतर (Transform faults):
भ्रंश पर न तो भूपर्पटी का निर्माण होता है और न विनाश। यहां स्थलमण्डलीय प्लेटें एक-दूसरे के विपरीत दिशा में साथ-साथ खिसकती हैं।

प्लेट संचलन के कारण (Causes of Plate Movement)
1. तापीय संवहन:
आर्थर होम्स ने 1928 में यह बताया कि अधोपर्पटी संवहन धाराएं तापीय संवहन की क्रियाविधि आरम्भ करती हैं, जो प्लेटों के संचलन के लिए प्रेरक बल के रूप में काम करता है।

2. उष्ण धाराएं:
उष्ण धाराएं ऊपर उठती हैं। जैसे ही वे भूपृष्ठ पर पहुंचती हैं, वे ठण्डी हो जाती हैं और नीचे की ओर चलने लगती हैं। इस प्रकार यह संवहनी संचलन भूपर्पटी प्लेटों को गतिशील कर देता है।

3. प्लेटों का तैरना:
संचलन के कारण स्थल मण्डल की प्लेटें, जो नीचे के अधिक गतिशील एस्थेनोस्फीयर पर तैर रही हैं, निरन्तर गति में रहती हैं।

4. ज्वालामुखी क्रिया:
अतीत में हुई ज्वालामुखी क्रिया के छोटे केन्द्र, जो बहुधा किसी सक्रिय प्लेट सीमा से दूर स्थलमण्डल पर स्थित हैं, संवहन धाराओं के प्रभाव का संकेत देते हैं। ज्वालामुखी क्रिया के ये केन्द्र तप्त स्थल कहलाते हैं।

5. ज्वालामुखी:
डब्ल्यू० जैसन मॉर्गन ने 1971 में तप्त स्थल की परिकल्पना की। उनके अनुसार मैंटल में मैग्मा का स्रोत अपने स्थान पर स्थिर रहता है, जबकि इसके ऊपर स्थित स्थलमण्डलीय प्लेटें निरन्तर संचलित होती हैं। इस प्रकार किसी तप्त स्थल के ऊपर ज्वालामुखियों की रचना होती है, लेकिन वे उसके बाद मैग्मा-स्रोत से दूर खिसक जाते हैं और मृत हो जाते हैं। ये मृत ज्वालामुखी एक श्रृंखला की रचना करते हैं, जो प्लेट संचलन के अभिलेख हैं।

JAC Class 11 Geography Important Questions Chapter 4 महासागरों और महाद्वीपों का वितरण  3

प्लेट सीमाएं (Plate Boundaries):
प्लेट सीमाएं पृथ्वी के सर्वाधिक विशिष्ट संरचनात्मक लक्षण हैं। प्लेट सीमाओं की पहचान कठिन नहीं है। ये प्रमुख स्थलाकृतिक लक्षणों से चिन्हित हैं। स्थलमण्डल, सात मुख्य प्लेटों और अनेक छोटी उप-प्लेटों में विभक्त है। मुख्य प्लेटों की बहिर्रेखा नवीन पर्वत तंत्रों, महासागरीय कटकों तथा खाइयों से बनी है। ये प्लेटें निम्नलिखित हैं

  1. प्रशान्त प्लेट;
  2. यूरेशियन प्लेट;
  3. इंडो-ऑस्ट्रेलियन प्लेट;
  4. अफ्रीकन प्लेट;
  5. उत्तरी अमेरिकन प्लेट;
  6. दक्षिण अमेरिकन प्लेट;
  7. अंटार्कटिक प्लेट;

मुख्य विशेषताएं:

  1. इनमें से सर्वाधिक नवीन प्रशान्त प्लेट है जो लगभग पूरी तरह महासागरीय पटल से बनी है और भूपृष्ठ के 20 प्रतिशत भाग पर विस्तृत है।
  2. कोई भी प्लेट केवल महाद्वीपीय पटल से निर्मित नहीं है। (3) प्लेटों की मोटाई में अन्तर महासागरों के नीचे 70 कि०मी० से लेकर महाद्वीपों के नीचे 150 कि०मी०
  3. तक है।
  4. प्लेट स्थाई लक्षण नहीं है। इनकी आकृति तथा आकार में अन्तर होता रहता है। वे प्लेटें जो महाद्वीपीय पटल से नहीं बनी हैं, प्रविष्ठन का शिकार हो सकती हैं।
  5. कोई भी प्लेट टूट सकती है अथवा अन्य प्लेट के साथ जुड़ सकती है। (6) प्रत्येक विवर्तनिक प्लेट दृढ़ है और एक इकाई के रूप में संचलन करती है।
  6. लगभग सभी विवर्तनिक क्रियाएं प्लेट सीमाओं पर होती हैं, यही कारण है कि भू-वैज्ञानिक तथा भूगोलवेत्ता प्लेट सीमाओं पर अपना ध्यान केन्द्रित करते हैं।

JAC Class 11 Geography Important Questions Chapter 4 महासागरों और महाद्वीपों का वितरण

प्रश्न 2.
भारतीय प्लेट की मुख्य विशेषताएं बताओ।
उत्तर:
भारतीय, प्लेट का संचलन (Movement of the Indian Plate):
इंडियन प्लेट में प्रायद्वीप भारत और ऑस्ट्रेलिया महाद्वीपीय भाग सम्मिलित हैं। हिमालय पर्वत श्रेणियों के साथ-साथ पाया जाने वाला प्रविष्ठन क्षेत्र (Subduction zone), इसकी उत्तरी सीमा निर्धारित करता है जो महाद्वीपीय-महाद्वीपीय अभिसरण (Continentcontinent convergence) के रूप में हैं। (अर्थात् दो महाद्वीप प्लेटों की सीमा है) यह पूर्व दिशा में म्यांमार के राकिन्योमा पर्वत से होते हुए एक चाप के रूप में जावा खाई तक फैला हुआ है। इसकी पूर्वी सीमा एक विस्तारित तल (Spreading site) है, जो ऑस्ट्रेलिया के पूर्व में दक्षिणी पश्चिमी प्रशान्त महासागर में महासागरीय कटक के रूप में है।

इसकी पश्चिमी सीमा पाकिस्तान की किरथर श्रेणियों का अनुसरण करती है। यह आगे मकरान तट के साथ-साथ होती हुई दक्षिण-पूर्वी चागोस द्वीप समूह (Chagos archipelago) के साथ-साथ लाल सागर द्रोणी (जो विस्तारण तल है) में जा मिलती है। भारतीय तथा आर्कटिक प्लेट की सीमा भी महासागरीय कटक से निर्धारित होती है। जो एक अपसारी सीमा (Divergent boundary) है और यह लगभग पूर्व-पश्चिम दिशा में होती हुई न्यूज़ीलैंड के दक्षिण में विस्तारित तल में मिल जाती है।

हिमालय पर्वत का उत्थान: भारत एक वृहत् द्वीप था, जो ऑस्ट्रेलियाई तट से दूर एक विशाल महासागर में स्थित था।

  1. लगभग 22.5 करोड़ वर्ष पहले तक टेथीस सागर इसे एशिया महाद्वीप से अलग करता था।
  2. ऐसा माना जाता है कि लगभग 20 करोड़ वर्ष पहले, जब पैंजिया विभक्त हुआ तब भारत ने उत्तर दिशा की ओर खिसकना आरम्भ किया।
  3. लगभग 4 से 5 करोड़ वर्ष पहले भारत एशिया से टकराया व परिणामस्वरूप हिमालय पर्वत का उत्थान हुआ।
  4. 7.1 करोड़ वर्ष पहले से आज तक की भारत की स्थिति आज से लगभग 14 करोड़ वर्ष पहले यह उपमहाद्वीप सुदूर दक्षिण में 50″ दक्षिणी अक्षांश पर स्थित था। इन दो प्रमुख प्लेटों को टिथीस सागर अलग करता था और तिब्बतीय खंड, एशियाई स्थलखण्ड के करीब था।
  5. इंडियन प्लेट के एशियाई प्लेट की तरफ प्रवाह के दौरान एक प्रमुख घटना घटी-वह थी लावा प्रवाह से दक्कन ट्रैप का निर्माण होना। ऐसा लगभग 6 करोड़ वर्ष पहले आरम्भ हुआ और एक लम्बे समय तक यह जारी रहा । याद रहे कि यह उपमहाद्वीप तब भी भूमध्यरेखा के निकट था।
  6.  लगभग 4 करोड़ वर्ष पहले और इसके पश्चात् हिमालय की उत्पत्ति आरम्भ हुई। वैज्ञानिकों का मानना है कि यह प्रक्रिया अभी भी जारी है और हिमालय की ऊँचाई अब भी बढ़ रही है।

JAC Class 11 Geography Important Questions Chapter 4 महासागरों और महाद्वीपों का वितरण  4

JAC Class 11 Geography Important Questions Chapter 3 पृथ्वी की आंतरिक संरचना

Jharkhand Board JAC Class 11 Geography Important Questions Chapter 3 पृथ्वी की आंतरिक संरचना Important Questions and Answers.

JAC Board Class 11 Geography Important Questions Chapter 3 पृथ्वी की आंतरिक संरचना

बहु-विकल्पी प्रश्न (Multiple Choice Questions)

प्रश्न-दिए गए प्रश्नों के चार वैकल्पिक उत्तरों में से सही उत्तर चुनकर लिखें
1. निम्नलिखित में से पृथ्वी की संरचना का मुख्य स्त्रोत क्या है?
(A) भूकम्पीय तरंगें
(B) ज्वालामुखी
(C) पृथ्वी का तापमान
(D) पृथ्वी का घनत्व।
उत्तर:
भूकम्पीय तरंगें।

2. भू-पृष्ठ का घनत्व बताओ
(A) 17.2
(B) 5.68
(C) 2.75
(D) 5.53.
उत्तर:
(C) 2.75.

JAC Class 11 Geography Important Questions Chapter 3 पृथ्वी की आंतरिक संरचना

3. पृथ्वी की अभ्यान्तर परत को क्या कहा जाता है?
(A) सियाल
(B) सीमा
(C) नाइफ
(D) मैंटल।
उत्तर:
(C) नाइफ।

4. अभ्यान्तर में अधिकतम घनत्व का मुख्य कारण क्या है?
(A) अधिक गहराई
(B) अधिक तापमान
(C) तरल पदार्थ
(D) निक्कल तथा लौह धातुएं।
उत्तर:
निक्कल तथा लौह धातुएं।

5. पृथ्वी के भीतरी भाग में तापमान की वृद्धि की औसत दर क्या है?
(A) 1°C प्रति कि०मी०
(B) 12°C प्रति कि० मी०
(C) 1°C प्रति 30 मीटर
(D) 10°C प्रति 100 मीटर।
उत्तर:
(C) 1°C प्रति 30 मीटर।

6. निम्नलिखित में से कौन-सा तथ्य पृथ्वी की आन्तरिक बनावट पर सर्वाधिक प्रकाश डालता है?
(A) गहरी खदानों का खोदा जाना
(B) कुओं का खोदा जाना
(C) भूकम्पीय तरंगें
(D) ज्वालामुखी उद्भेदन।
उत्तर:
(C) भूकम्पीय तरंगें।

JAC Class 11 Geography Important Questions Chapter 3 पृथ्वी की आंतरिक संरचना

7. कौन-सी भूकम्पीय तरंगें पृथ्वी से 2900 किलोमीटर की गहराई के पश्चात् लुप्त हो जाती हैं?
(A) प्राथमिक
(B) गौण
(C) धरातलीय
(D) अनुदैर्ध्य।
उत्तर:
(B) गौण।

8. भूकम्पीय तरंगों को रेखांकित करने वाला यन्त्र
(A) थर्मोग्राफ
(B) सिस्मोग्राफ
(C) हाइग्रोग्राफ
(D) बैरोग्राफ।
उत्तर:
(B) सिस्मोग्राफ।

9. भूपर्पटी में विकसित थरथराहट को कहते हैं
(A) भूसंचरण
(B) पृथ्वी की गति
(C) भूकम्प
(D) पृथ्वी की कामुकता।
उत्तर:
(C) भूकम्प।

10. भूकम्पों के कारण समुद्रों में विकसित विशाल तरंगें होती हैं
(A) समुद्री तरंगें
(B) ज्वारीय तरंगें
(C) सुनामी तरंगें
(D) धरातलीय तरंगें।
उत्तर:
सुनामी तरंगें।

11. पृथ्वी की त्रिज्या कितनी है?
(A) 5370 कि०मी०
(B) 6370 कि०मी०
(C) 7370 कि०मी०
(D) 8370 कि०मी०।
उत्तर:
(B) 6370 कि०मी०।

12. दक्षिणी अफ्रीका की सोने की खानें कितनी गहरी हैं?
(A) 3-4 कि०मी०
(B) 4-5 कि०मी०
(C) 5-6 कि०मी०
(D) 6-7 कि०मी०
उत्तर:
(A) 3-4 कि०मी०

JAC Class 11 Geography Important Questions Chapter 3 पृथ्वी की आंतरिक संरचना

13. सबसे गहरा प्रवेधन कितना गहरा है?
(A) 10 कि०मी०
(B) 11 कि०मी०
(C) 12 कि०मी०
(D) 13 कि०मी०
उत्तर:
(C) 12 कि०मी०

14. प्राकृतिक भूकम्प किस भू-भाग में आते हैं?
(A) स्थल खण्ड
(B) मैंटल
(C) नाईफ्
(D) क्रोड ।
उत्तर:
(A) स्थल खण्ड

15.’P’ व ‘S’ तरंगों के छाया क्षेत्र का अधि केन्द्र से विस्तार है ।
(A) 105-145°
(B) 115-1550
(C) 125-165°
(D) 135-175°
उत्तर:
(A) 105-145°

16. भूकंप की तीव्रता का माप किस वैज्ञानिक के नाम पर है?
(A) कान्ट
(B) लाप्लेस
(C) मरकैली
(D) ऑटो शिमिड।
उत्तर:
(C) मरकैली

अति लघु उत्तरीय प्रश्न (Very Short Answer Type Questions)

प्रश्न 1.
भूकम्पीय तरंगों के प्रकार बताओ।
उत्तर:

  1. प्राथमिक तरंगें
  2. माध्यमिक तरंगें
  3. धरातलीय तरंगें ।

प्रश्न 2.
धात्विक क्रोड के दो प्रमुख पदार्थ बताओ ।
उत्तर:
निकिल, लोहा

प्रश्न 3.
पृथ्वी की तीन परतों के नाम लिखो
उतर:
सियाल, सीमा, नाइफ

JAC Class 11 Geography Important Questions Chapter 3 पृथ्वी की आंतरिक संरचना

प्रश्न 4.
पृथ्वी की संरचना की जानकारी के प्रत्यक्ष साधन बताओ ।
उत्तर:
खानें, कुएं, छिद्र

प्रश्न 5.
पृथ्वी के आन्तरिक भाग में तापमान वृद्धि की औसत दर क्या है?
उत्तर:
1°C प्रति 32 मीटर।

प्रश्न 6.
पृथ्वी की संरचना की जानकारी प्रदान करने वाले परोक्ष साधन कौन-से हैं?
उत्तर:

  1. तापमान
  2. दबाव
  3. परतों का घनत्व
  4. भूकम्पीय तरंगें
  5. उल्काएं।

प्रश्न 7.
भूकम्पीय तरंगों के अध्ययन करने वाले यन्त्र का क्या नाम है?
उत्तर:
सीस्मोग्राफ (Seismograph)।

प्रश्न 8. कितनी गहराई के पश्चात् ‘S’ तरंगें लुप्त हो जाती हैं?
उत्तर:
2900km.

प्रश्न 9.
कौन-सी तरंगें केवल ठोस माध्यम से ही गुज़र सकती हैं?
उत्तर:
अनुप्रस्थ तरंगें।

प्रश्न 10.
किन प्रमाणों से पता चलता है कि पृथ्वी के आन्तरिक भाग का तापमान अधिक है?
उत्तर:

  1. ज्वालामुखी
  2. गर्म जल के झरने
  3. खानों से।

प्रश्न 11.
सियाल (Sial) किन दो शब्दों के संयोग से बना है?
उत्तर:
सियाल शब्द सिलिका तथा एल्यूमीनियम (Si+Al) के संयोग से बना है।

JAC Class 11 Geography Important Questions Chapter 3 पृथ्वी की आंतरिक संरचना

प्रश्न 12.
सीमा (Sima) किन दो शब्दों के संयोग से बना है?
उत्तर:
सीमा शब्द सिलिका तथा मैग्नीशियम (Si+Mg) के संयोग से बना है।

प्रश्न 13.
निफे (Nife) शब्द किन दो शब्दों के संयोग से बना है?
उत्तर:
निफे शब्द निकिल तथा फैरस (Ni+Fe) के संयोग से बना है।

प्रश्न 14.
पृथ्वी की केन्द्रीय परत को क्या कहते हैं?
उत्तर:
अभ्यान्तर या क्रोड या गुरुमण्डल।

प्रश्न 15.
पृथ्वी का औसत घनत्व कितना है?
उत्तर:
5.53.

प्रश्न 16.
सबसे धीमी गति वाली तरंगें कौन-सी हैं?
उत्तर:
धरातलीय तरंगें।

प्रश्न 17.
पृथ्वी के अभ्यान्तर का घनत्व कितना है?
उत्तर:
13.

प्रश्न 18.
अभ्यान्तर का घनत्व सबसे अधिक क्यों है?
उत्तर:
अभ्यान्तर में निकिल तथा लोहे के कारण।

प्रश्न 19.
Volcano ज्वाला शब्द यूनानी भाषा के किस शब्द से बना है?
उत्तर:
यूनानी शब्द ‘Vulan’ का अर्थ-पाताल देवता है।

प्रश्न 20.
पृथ्वी के अन्दर पिघले पदार्थ को क्या कहते हैं?
उत्तर:
मैग्मा।

प्रश्न 21.
जब मैग्मा पृथ्वी के धरातल से बाहर आ जाता है तो उसे क्या कहते हैं?
उत्तर:
लावा।

प्रश्न 22.
ज्वालामुखी के तीन प्रकार कौन-कौन से हैं?
उत्तर:

  1. सक्रिय ज्वालामुखी
  2. प्रसुप्त ज्वालामुखी
  3. मृत ज्वालामुखी।

प्रश्न 23.
भारत में सक्रिय ज्वालामुखी का नाम बताएं।
उत्तर:
अण्डमान द्वीप के निकट बैरन द्वीप।

JAC Class 11 Geography Important Questions Chapter 3 पृथ्वी की आंतरिक संरचना

प्रश्न 24.
ज्वालामुखी शंकु किसे कहते हैं?
उत्तर:
ज्वालामुखी के मुख से निकले पदार्थ मुख के आस-पास जमा हो जाते हैं। धीरे-धीरे ये शंकु का रूप धारण कर लेते हैं। इन्हें ज्वालामुखी शंकु कहते हैं।

प्रश्न 25.
क्रेटर किसे कहते हैं?
उत्तर:
ज्वालामुखी के केन्द्र में कटोरे के समान या कीपाकार गर्त को क्रेटर कहते हैं।

प्रश्न 26.
काल्डेरा किसे कहते हैं?
उत्तर:
विशाल क्रेटर को काल्डेरा कहते हैं।

प्रश्न 27.
भूकम्प किसे कहते हैं?
उत्तर:
पृथ्वी का अचानक हिलना।

प्रश्न 28.
भूकम्प आने के तीन कारण लिखो।
उत्तर:

  1. ज्वालामुखी विस्फोट
  2. विवर्तनिक कारण
  3. लचक शक्ति।

प्रश्न 29.
उद्गम केन्द्र किसे कहते हैं?
उत्तर:
पृथ्वी के अन्दर जहां भूकम्प उत्पन्न होता है, उसे उद्गम केन्द्र कहते हैं।

प्रश्न 30.
अधिकेन्द्र से क्या अभिप्राय है?
उत्तर:
भूकम्प केन्द्र के ठीक ऊपर धरातल पर स्थिर बिन्दु या स्थान को अधिकेन्द्र कहते हैं।

JAC Class 11 Geography Important Questions Chapter 3 पृथ्वी की आंतरिक संरचना

प्रश्न 31.
आग का गोला (Ring of fire) किसे कहा जाता है?
उत्तर:
परिप्रशान्त महासागरीय पेटी।

प्रश्न 32.
संसार की तीन प्रमुख ज्वालामुखी पेटियों के नाम लिखें।
उत्तर:

  1. प्रशांत महासागरीय पेटी
  2. अन्ध महासागरीय पेटी
  3. मध्य महाद्वीपीय पेटी।

लघु उत्तरीय प्रश्न (Short Answer Type Questions)

प्रश्न 1.
सियाल (Sial) से क्या अभिप्राय है?
उत्तर:
यह पृथ्वी की सबसे ऊपरी परत है। इसमें सिलिका तथा एल्यूमीनियम के अंश अधिक मात्रा में हैं। इन दोनों धातुओं के संयोग के कारण इस परत को सियाल (Sial = Silica + Aluminium) कहते हैं। इस परत की औसत गहराई 60 km. है। इस परत का घनत्व 2.75 है। इस परत से महाद्वीपों का निर्माण हुआ है।

प्रश्न 2.
सीमा (Sima ) से क्या अभिप्राय है?
उत्तर:
सियाल से निचली परत को सीमा कहा जाता है। इस परत से सिलिका तथा मैग्नीशियम धातुएं अधिक मात्रा में मिलती हैं। इसलिए इस परत को सीमा (Sima = Silica + Magnesium) कहा जाता है। इस परत की मोटाई 2800 km है। इसका औसत घनत्व 4.75 है। महासागरीय तल इसी परत से बना हुआ है।

प्रश्न 3.
निफे (Nife) से क्या अभिप्राय है?
उत्तर:
यह पृथ्वी की सबसे निचली तथा केन्द्रीय परत है। यह सबसे भारी परत है जिसका घनत्व 13 है। इसमें निकिल तथा फैरस (लोहा) धातुएं अधिक हैं। इसलिए इसे (Nife = Nickle + Ferrous) कहा जाता है। इस परत की मोटाई 3500 km. 1

JAC Class 11 Geography Important Questions Chapter 3 पृथ्वी की आंतरिक संरचना

प्रश्न 4.
पृथ्वी की तीन मौलिक परतों के नाम, गहराई, विस्तार तथा घनत्व बताओ।
उत्तर:
पृथ्वी का निर्माण करने वाली तीन मूल परतें हैं जिनकी रचना भिन्न घनत्व वाले पदार्थों से हुई है

  1. भू-पृष्ठ (Crust),
  2. मैण्टल (Mantle, )
  3. क्रोड (Core)
परत क्षेत्र मोटाई कुल का \% घनत्व
(1) भू-पृष्ठ सियाल 60 कि॰ मी० 0.5 2.75
(2) मैण्टल सीमा 2840 कि॰ मी० 16.5 5.68
(3) क्रोड नाइफ 3500 कि॰ मी० 83.0 17.2

प्रश्न 5.
सिस्मोग्राफ किसे कहते हैं? इसका प्रयोग किस उद्देश्य के लिए किया जाता है?
उत्तर:
सिस्मोग्राफ (Seismograph) एक यन्त्र है जिसके द्वारा भूकम्पीय तरंगें तथा तीव्रता मापी जाती है। इस यन्त्र में लगी एक सूई द्वारा ग्राफ पेपर पर भूकम्पीय तरंगों को रेखांकित किया जाता है। इस यन्त्र द्वारा भूकम्प का उद्गम (Focus), भूकम्पीय तरंगों की गति, मार्ग तथा तीव्रता का ज्ञान होता है।

प्रश्न 6.
भूकम्पीय तरंगों के मुख्य प्रकार बताओ। कौन-सी तरंगें धीमी गति वाली हैं तथा कौन-सी तरंगें तेज़ गति वाली हैं?

  1. प्राथमिक या अनुदैर्ध्य तरंगें।
  2. गौण या अनुप्रस्थ तरंगें
  3. धरातलीय या लम्बी तरंगें।

धरातलीय तरंगें सब से धीमी गति से चलती हैं। प्राथमिक तरंगें सब से तेज़ गति से चलती हैं।

प्रश्न 7.
गुरुत्वाकर्षण तथा चुम्बकीय क्षेत्र किस प्रकार भूकम्प सम्बन्धी सूचना देते हैं?
उत्तर:
अन्य अप्रत्यक्ष स्रोतों में गुरुत्वाकर्षण, चुम्बकीय क्षेत्र व भूकम्प सम्बन्धी क्रियाएं शामिल हैं। पृथ्वी के धरातल पर भी विभिन्न अक्षांशों पर गुरुत्वाकर्षण बल एक समान नहीं होता है। यह (गुरुत्वाकर्षण बल) ध्रुवों पर अधिक एवं भूमध्यरेखा पर कम होता है। पृथ्वी के केन्द्र से दूरी के कारण गुरुत्वाकर्षण बल ध्रुवों पर कम और भूमध्य रेखा पर अधिक होता है । गुरुत्व का मान पदार्थ के द्रव्यमान के अनुसार भी बदलता है । पृथ्वी के भीतर पदार्थों का असमान वितरण भी इस भिन्नता को प्रभावित करता है।

अलग-अलग स्थानों पर गुरुत्वाकर्षण की भिन्नता अनेक अन्य कारकों से भी प्रभावित होती है। इस भिन्नता को गुरुत्व विसंगति (Gravity anomaly ) कहा जाता है गुरुत्व विसंगति हमें भूपर्पटी में पदार्थ के द्रव्यमान के वितरण की जानकारी देती है। चुम्बकीय सर्वेक्षण भी भूपर्पटी में चुम्बकीय पदार्थ के वितरण की जानकारी देते हैं । भूकम्पीय गतिविधियां भी पृथ्वी की आन्तरिक जानकारी का एक महत्त्वपूर्ण स्रोत है ।

JAC Class 11 Geography Important Questions Chapter 3 पृथ्वी की आंतरिक संरचना

प्रश्न 8.
उल्काएं पृथ्वी की आन्तरिक बनावट के विषय में जानकारी देने में किस प्रकार सहायक हैं?
उत्तर:
कभी-कभी जलते हुए पदार्थ आकाश से पृथ्वी की ओर गिरते दिखाई देते हैं। इन्हें उल्का (Meteorites) कहते हैं। यह सौर मण्डल का एक भाग है। इनके अध्ययन से पता चलता है कि इनके निर्माण में लोहा और निकिल की प्रधानता है। पृथ्वी की रचना उल्काओं से मिलती-जुलती है तथा पृथ्वी का आंतरिक क्रोड भी भारी पदार्थों (लोहा + निकिल ) से बना हुआ है।

प्रश्न 9.
गुरुमण्डल से क्या अभिप्राय है?
उत्तर:
गुरुमण्डल (Barysphere):
पृथ्वी के केन्द्रीय भाग या आन्तरिक क्रोड को गुरुमण्डल कहते हैं। इसकी औसत गहराई 4980 से 6400 मी० है। यह अत्यधिक भारयुक्त खनिज पदार्थों से बना हुआ है। इसका औसत घनत्व 13 से अधिक है। इसमें लोहा तथा निकिल पदार्थों की अधिकता है। इस भाग को अभ्यान्तर (Core) भी कहा जाता है।

प्रश्न 10.
ज्वालामुखी किसे कहते हैं? ज्वालामुखी के विभिन्न भाग बताओ।
उत्तर:
ज्वालामुखी (Volcano):
ज्वालामुखी क्रिया एक अन्तर्जात क्रिया है जो भू-गर्भ से सम्बन्धित है। ज्वालामुखी धरातल पर एक गहरा प्राकृतिक छिद्र है जिससे भू-गर्भ से गर्म गैसें, लावा, तरल व ठोस पदार्थ बाहर निकलते हैं। सबसे पहले एक छिद्र की रचना होती है जिसे ज्वालामुखी (Volcano) कहते हैं। इस छिद्र से निकलने वाले लावा पदार्थों के चारों ओर फैलने तथा ठण्डा होकर ठोस होने से एक उच्च भूमि का निर्माण होता है जिसे ज्वालामुखी पर्वत कहते हैं। एक लम्बे समय में कई बार निकासन, शीतलन तथा ठोसीकरण की क्रिया से ज्वालामुखी का निर्माण होता है।

ज्वालामुखी के भाग (Parts of a Volcano):

  1. ज्वालामुखी निकास-एक छेद जिस से लावा का निष्कासन होता है ज्वालामुखी निकास कहलाता है।
  2. विवर-ज्वालामुखी विकास के चारों ओर एक तश्तरीनुमा गर्त को विवर कहते हैं।
  3. कैल्डेरा–विस्फोट से बने खड़ी दीवारों वाले कंड को कैलडेरा कहते हैं।

प्रश्न 11.
भूकम्पों के विभिन्न प्रकार बताओ।
उत्तर:
उत्पत्ति के आधार पर भूकम्प निम्नलिखित प्रकार के हैं

  1. विवर्तनिक भूकम्प-सामान्यतः विवर्तनिक (Tectonic) भूकम्प ही अधिक आते हैं। ये भूकम्प भ्रंश तल के किनारे चट्टानों के सरक जाने के कारण उत्पन्न होते हैं।
  2. ज्वालामुखी भूकम्प-एक विशिष्ट वर्ग के विवर्तनिक भूकम्प को ही ज्वालामुखीजन्य (Volcanic) भूकम्प समझा जाता है। ये भूकम्प अधिकांशतः सक्रिय ज्वालामुखी क्षेत्रों तक ही सीमित रहते हैं।
  3. नियात भूकम्प-खनन क्षेत्रों में कभी-कभी अत्यधिक खनन कार्य से भूमिगत खानों की छत ढह जाती है, जिससे हल्के झटके महसूस किये जाते हैं। इन्हें नियात (Collapse) भूकम्प कहा जाता है।
  4. विस्फोट भूकम्प-कभी-कभी परमाणु व रासायनिक विस्फोट से भी भूमि में कम्पन होती है। इस तरह के झटकों को विस्फोट (Explosion) भूकम्प कहते हैं।
  5. बांध जनित भूकम्प-जो भूकम्प बड़े बांध वाले क्षेत्रों में आते हैं, उन्हें बांध जनित (Reservoir induced) भूकम्प कहा जाता है।

JAC Class 11 Geography Important Questions Chapter 3 पृथ्वी की आंतरिक संरचना

प्रश्न 12.
सुनामी किसे कहते हैं? इनके विनाशकारी प्रभाव की एक उदाहरण दो।
उत्तर:
सुनामी (Tsunami):
कई बार भूकम्प के कारण सागरीय लहरें बहुत ऊंची उठ जाती हैं। जापान में इन्हें सुनामी कहते हैं। ये तूफानी लहरें तटीय प्रदेशों में जान-माल की हानि करती हैं। सन् 1883 में क्राकटोआ विस्फोट से जो भूकम्प आया जिससे 15 मीटर ऊंची लहरें उठीं तथा पश्चिमी जावा में 36,000 व्यक्तियों की मृत्यु हो गई। 26 दिसम्बर, 2004 को इण्डोनेशिया के निकट केन्द्रित भूकम्प से हिन्दमहासागर में 30 मीटर ऊंची सुनामी लहरों से लगभग 3 लाख व्यक्तियों की जानें गईं। इण्डोनेशिया, थाइलैंड, अण्डमान द्वीप, तमिलनाडु तट तथा श्रीलंका के तटीय भागों में प्रलय समान तबाही हुई।

प्रश्न 13.
दुर्बलता मण्डल क्या है?
उत्तर:
दुर्बलता मण्डल (Asthenosphere) ऊपरी मैंटल परत का एक भाग है। यह 650 कि० मी० गहरा है। यह परत ठोस तथा लचीले गुट रखती है। इस परत का अनुमान प्रसिद्ध भूकम्प वैज्ञानिक गुट्नबर्ग ने लगाया था। यहां भूकम्पीय लहरों की गति कम होती है। Asthenosphere का अर्थ है दुर्बलता।

प्रश्न 14.
पृथ्वी के धरातल का विन्यास किन प्रक्रियाओं का परिणाम है?
उत्तर:
पृथ्वी पर भू-आकृतियों का विकास बहिर्जात एवं अन्तर्जात क्रियाओं का परिणाम है। दोनों प्रक्रियाएं निरन्तर कार्य करती हैं तथा भू-आकृतियों को जन्म देती हैं।

प्रश्न 15.
पृथ्वी की संरचना की जानकारी के प्रत्यक्ष साधनों के नाम लिखो।
उत्तर:
पृथ्वी की संरचना की जानकारी के प्रत्यक्ष साधन निम्नलिखित है

  1. धरातलीय चट्टानें : धरातलीय चट्टानें सुगमता से प्राप्त होती हैं तथा इन से भू-गर्भ की जानकारी मिलती है।
  2. खनन क्षेत्र : दक्षिणी अफ्रीका की सोने की खानों की गहराई 3-4 किलोमीटर तक है जहां तक जाना सम्भव है।
  3. विभिन्न परियोजनाएं : गहरे समुद्र में गहराई से पदार्थ प्राप्त करने की योजनाएं भी प्रत्यक्ष साधन हैं।
  4. ज्वालामुखी उद्गार : ज्वालामुखी उद्गार से लावा पृथ्वी के धरातल पर आता है तो पृथ्वी की संरचना की जानकारी मिलती है।

प्रश्न 16.
उन दो वैज्ञानिक परियोजनाओं का वर्णन करो जिनसे पृथ्वी के भूगर्भ की जानकारी प्राप्त होगी।
उत्तर:
पृथ्वी के वैज्ञानिक ऐसी दो परियोजनाओं पर काम कर रहे हैं।

  1. गहरे समुद्र में प्रवेश परियोजना (Deep ocean drilling project).
  2. समन्नित महासागरीय परिवेधन परियोजना (Integrated ocean drilling project) आज तक सबसे गहरा प्रवेधक (Drill) आर्कटिक महासागर के कोला (Kola) क्षेत्र में 12km की गहराई तक किया गया है।

प्रश्न 17.
भूकम्प की माप किस पैमाने पर की जाती है?
उत्तर:
भूकम्पों की माप-भूकम्पीय घटनाओं का मापन भूकम्पीय तीव्रता के आधार पर अथवा आघात की तीव्रता के आधार पर किया जाता है। भूकम्पीय तीव्रता की मापनी ‘रिक्टर स्केल’ (Richter scale) के नाम से जानी जाती है। भूकम्पीय तीव्रता भूकम्प के दौरान ऊर्जा विमोचन से सम्बन्धित है। इस मापनी के अनुसार भूकम्प की तीव्रता 0 से 10 तक होती है। तीव्रता गहनता (Intensity scale) ‘हानि की तीव्रता’ मापनी इटली के भूकम्प वैज्ञानिक मरकैली (Mercalli) के नाम पर है। यह मापनी झटकों से हुई प्रत्यक्ष हानि द्वारा निर्धारित की गई है। इसका गहनता का विस्तार 1 से 12 तक है।

तुलनात्मक प्रश्न
(Comparison Type Questions)

प्रश्न 1.
अनुदैर्ध्य तथा अनुप्रस्थ तरंगों में अन्तर स्पष्ट करो।
उत्तर:

अनुदैर्ध्य तरंगें (Longitudinal waves) अनुप्रस्थ तरंगें (Transverse waves)
(1) इन तरंगों में कण आगे बढ़ने की दिशा में चलते हैं। (1) ये तरंगें दोलन की दिशा पर समकोण चलती हैं।
(2) इन्हें प्राथामिक तरंगें, ध्वनि तरंगें या P-waves कहा जाता है। (2) इन्हें द्वितीयक या S-waves भी कहा जाता है।
(3) इनकी गति कुछ तेज़ होती है। (3) इसकी गति धीमी होती है।
(4) ये तरल, गैस तथा ठोस तीनों माध्यमों से गुज़र सकती हैं। (4) ये केवल ठोस माध्यम से ही गुज़र सकती हैं।
अनुदैर्ध्य तरंगें (Longitudinal waves) अनुप्रस्थ तरंगें (Transverse waves)

JAC Class 11 Geography Important Questions Chapter 3 पृथ्वी की आंतरिक संरचना

प्रश्न 2.
भू-पृष्ठ तथा अभ्यान्तर में अन्तर स्पष्ट करो।
उत्तर:

भू-पृष्ठ (Crust) अभ्यान्तर (Core)
(1) यह पृथ्वी की बाहरी परत है। (1) यह पृथ्वी की भीतरी परत है।
(2) यह सबसे हल्की परत है। (2) यह सब से भारी परत है।
(3) इस परत का औसत घनत्व $2.75$ है। (3) इस परत का औसत घनत्व $17.2$ है।
(4) यह पृथ्वी के $0.5 \%$ भाग को घेरे हुए है। (4) यह पृथ्वी के $83 \%$ भाग को घेरे हुए है।
(5) इसमें सिलिका तथा एल्यूमीनियम की अधिकता है। (5) इसमें निकल तथा लोहे की अधिकता है।

प्रश्न 3.
मैग्मा तथा लावा में अन्तर स्पष्ट करो।
उत्तर:

मैग्मा (Magma) लावा (Lava)
(1) पृथ्वी के भीतरी भाग में पिघले हुए गर्म घोल को मैग्मा कहते हैं। (1) जब उद्भेदन के कारण मैग्मा धरती के बाहर आकर ठण्डा तथा ठोस रूप धारण कर लेता है तो उसे लावा कहते हैं।
(2) इसमें जल व अन्य गैसें भी मिली होती हैं। (2) इसमें जल व गैसों के अंश नहीं होते।
(3) यह पृथ्वी के भीतरी भागों में ऊपरी मैंटल में उत्पन्न होता है। (Magma is hot Sticky molten material.) (3) यह पृथ्वी के धरातल पर वायुमण्डल के सम्पर्क से ठण्डा व ठोस होता है। (The Solidifed magma is called Lava.)

निबन्धात्मक प्रश्न (Essay Type Questions)

प्रश्न 1.
पृथ्वी की आन्तरिक संरचना का वर्णन करो। इसकी प्रत्येक पर्त का विवरण दो। अपने कथन के पक्ष में विवेकपूर्ण तर्क दो।
अथवा
पृथ्वी की भूपर्पटी, मैंटल व क्रोड का वर्णन कीजिए।
उत्तर:
पृथ्वी की आन्तरिक संरचना (Structure of the Earth):
पृथ्वी के भू-गर्भ के बारे में विद्वानों ने अलग-अलग विचार प्रस्तुत किए हैं। कुछ विद्वानों ने पृथ्वी के भू-गर्भ को ठोस अवस्था माना है तो कुछ ने इसे गैसीय अथवा तरल अवस्था में माना है। भू-गर्भ की जानकारी प्राप्त करने के लिए मनुष्य के पास कोई प्रत्यक्ष साधन नहीं है। भीतरी भाग में अत्यधिक तापमान एक बड़ी रुकावट है। पृथ्वी की सबसे गहरी खान केवल 4 km गहरी है। इसीलिए भू-गर्भ की जानकारी के सभी साधन अप्रत्यक्ष (Indirect ) हैं तथा केवल अनुमान ही हैं।

पृथ्वी की विभिन्न परतें (Different Layers of the Earth): वान्डर ग्राट (Vander Gracht) के अनुसार, पृथ्वी की आन्तरिक संरचना निम्नलिखित परतों में हुई है:
1. बाहरी सियाल परत ( Sial):
यह पृथ्वी की ऊपरी परत है। इस परत में सिलिका तथा एल्यूमीनियम का अंश अधिक है। यह सबसे हल्की परत है जिसका घनत्व 2.7 के बीच है। इस परत की औसत गहराई 45 कि० मी० तक है। इससे अधिकतर महाद्वीपों का निर्माण हुआ है। (Sial = Silica + Aluminium)

पृथ्वी की परतें (Layers of the Earth)

JAC Class 11 Geography Important Questions Chapter 3 पृथ्वी की आंतरिक संरचना 1

2. बाहरी सिलिकेट परत:
भूपर्पटी की इस परत का विस्तार महासागरों के नीचे है। इसमें अधिकतर तलछटी चट्टानें तथा ग्रेनाइट का विस्तार है। इसकी गहराई 45 से 100 कि०मी० तक है। इसका घनत्व 2.75-2.90 तक है

3. आन्तरिक सिलिकेट परत:
सियाल से निचली परत को आन्तरिक सिलिकेट परत कहा जाता है। बाहरी तथा आन्तरिक परत के बीच के अन्तराल को कॉनरैड अन्तराल कहा जाता है। इस परत का घनत्व 3.10 से 4.75 तक है। इसकी गहराई 1700 कि० मी० तक है। इसमें सिलिकेट तथा मैग्नीशियम की अधिकता होती है।

4. सिलिकेट तथा मिश्रित धातु परत (Sima ):
सियाल के नीचे दूसरी मुख्य परत को सीमा कहा जाता है। इसमें सिलिका तथा मैग्नीशियम अधिक मात्रा में पाए जाते हैं। (Sima = Silica + Magnesium) इस परत की मोटाई 1700 कि० मी० से 2900 कि०मी० है तथा इसका घनत्व 4.75 से 5 तक है। सियाल तथा सीमा को पृथक् करने वाले अन्तराल को मोहरोविसिक अन्तराल कहते हैं। इसकी रचना बैसाल्ट चट्टानों से हुई है।

5. धात्विक नाभि (Nife ):
यह सबसे निचली केन्द्रीय तथा भारी परत है। इसमें निकिल तथा फैरस अधिक मात्रा में पाया जाता है (Nife = Nickle + Ferrous) । इस परत का घनत्व 4.75 से 11 तक है। इस परत की गहराई 2900-4980 कि०मी० है। इस परत को बाह्य धात्विक क्रोड भी कहा जाता है।
JAC Class 11 Geography Important Questions Chapter 3 पृथ्वी की आंतरिक संरचना 2

6. धात्विक क्रोड (Core):
इस परत में भारी धातुओं की अधिकता है। इसलिए इसका घनत्व 13 से 17 तक है। इसकी गहराई 4980-6400 कि० मी० तक है। इस परत को गुरुमण्डल (Bary sphere) भी कहते हैं।

JAC Class 11 Geography Important Questions Chapter 3 पृथ्वी की आंतरिक संरचना

प्रश्न 2.
भूकम्प किसे कहते हैं? यह कैसे उत्पन्न होता है?
उत्तर:
भूकम्प (Earthquake):
भूपृष्ठ के किसी भी भाग के अचानक हिल जाने को भूकम्प कहते हैं। (An earthquake is a sudden movement on the crust of the earth.) इस प्रकार भूकम्प धरातल का कम्पन तथा दोलन है जिसके द्वारा चट्टानें ऊपर नीचे सरकती हैं। यह एक आकस्मिक एवं अस्थायी गति है। भूकम्प अपने केन्द्र से चारों ओर तरंगों के माध्यम से आगे बढ़ता है। भूकम्प के कारण – प्राचीन काल में लोग भूकम्प को भगवान् का कोप मानते थे, परन्तु वैज्ञानिकों के अनुसार भूकम्प के निम्नलिखित कारण हैं

  1. ज्वालामुखी उद्गार (Volcanic Eruption ): ज्वालामुखी विस्फोट में शक्ति होती है जिससे विस्फोट स्थान के समीपवर्ती क्षेत्र कांप उठते हैं। सन् 1883 में क्राकटोआ विस्फोट से दूर-दूर तक भूकम्प अनुभव किए गए।
  2. विवर्तनिक कारण (Tectonic Causes ): पृथ्वी की भीतरी हलचलों के कारण धरातल पर चट्टानों में मोड़ तथा दरारें पड़ जाती हैं । दरारों के सहारे हलचल होती है और भूकम्प आते हैं।
  3. पृथ्वी का सिकुड़ना (Contraction of Earth): तापमान कम होने से पृथ्वी सिकुड़ती है तथा चट्टानों में हलचल के कारण भूकम्प आते हैं ।
  4. लचक शक्ति (Elasticity of Rocks): जब किसी चट्टान पर दबाव पड़ता है तो वह चट्टान उस दबाव को वापस धकेलती है।
  5. सामान्य कारण (General Causes ): पर्वतीय भागों में भूस्खलन कार्स्ट प्रदेशों में गुफ़ाओं की छतों के धंसने से, तूफानी लहरों के कारण तथा अणु बमों के विस्फोट से साधारण भूकम्प उत्पन्न होते हैं ।

प्रश्न 3.
ज्वालामुखी स्थलाकृति के मुख्य लक्षणों का वर्णन करो।
उत्तर:
ज्वालामुखी क्रिया (Vulcanicity ) वह क्रिया है जिससे गर्म पदार्थ धरातल के नीचे या बाहर प्रकट होते हैं। ज्वालामुखी पृथ्वी की भीतरी शक्तियों (Internal Forces) में से एक है। ज्वालामुखी के मुख से निकले पदार्थ मुख के आस-पास जमा हो जाते हैं। धीरे-धीरे ये शंकु (Cone) का रूप धारण कर लेते हैं। इन्हें ज्वालामुखी शंकु (Volcanic Cones) कहते हैं। ज्वालामुखी स्थल के रूप
1. क्रेटर (Crater):
ज्वालामुखी के छिद्र के ऊपर गर्त बन जाता है। यह कटोरे के समान या कीपाकार (Funnel Shaped) होता है। क्रेटर में जल भर जाने से झील की रचना होती है। जैसे U.S.A. की क्रेटर लेक (Crater Lake) तथा महाराष्ट्र की लोनार झील।

2. काल्डेरा (Caldera ): विशाल क्रेटर को काल्डेरा कहते हैं। तीव्र विस्फोट से शंकु का ऊपरी भाग उड़ जाता है या क्रेटर के धंस जाने से इसका विस्तार बढ़ जाता है। जापान का काल्डेरा इतना बड़ा है कि इसे “Volcano of a Hundred Villages” कहते हैं।
JAC Class 11 Geography Important Questions Chapter 3 पृथ्वी की आंतरिक संरचना 3
3. राख शंकु (Ash Cone): यह कम ऊंचे शंकु होते हैं जिनकी रचना धूल तथा राख से होती है। इनके किनारे अवतल (Concave) ढाल वाले होते हैं। इसे सिंडर शंकु (Cinder Cone) भी कहते हैं।
JAC Class 11 Geography Important Questions Chapter 3 पृथ्वी की आंतरिक संरचना 4
4. शील्ड शंकु (Shield Cones): इनका निर्माण पैठिक लावा (Basic Lava) से होता है। पैठिक लावा हल्का तथा पतला होता है। इसमें सिलिका की मात्रा कम होती है। यह लावा दूर तक फैल जाता है। इस प्रकार लम्बे तथा कम ऊंचे शंकु का निर्माण होता है। जैसे हवाई द्वीप समूह के ज्वालामुखी शंकु।

5. गुम्बद शंकु (Lava Dome ): इनका निर्माण एसिड लावा से होता है। यह लावा काफ़ी गाढ़ा तथा चिपचिपा होता है। इसमें सिलिका की मात्रा अधिक होती है। यह मुख के निकट ही जल्दी जम कर गुम्बद बन जाता है। इस प्रकार तीव्र ढाल वाले ऊंचे शंकु का निर्माण होता है। फ्रांस में पाई डी डोम (Puy de dome) 1500 मीटर ऊंचा है।
JAC Class 11 Geography Important Questions Chapter 3 पृथ्वी की आंतरिक संरचना 5
6. लावा डॉट (Volcanic Plugs ):
जब शंकु पूरी तरह नष्ट हो जाता है तो नली व छिद्र ठोस लावा से भर जाते हैं। यह नली एक डॉट या प्लग की तरह दिखाई देती है। जैसे संयुक्त राज्य अमेरिका (USA) में लैसेन चोटी (Lassen Peak)।
JAC Class 11 Geography Important Questions Chapter 3 पृथ्वी की आंतरिक संरचना 6
7. मिश्रित शंकु (Composite Cones): ये सबसे बड़े व ऊंचे शंकुओं में गिने जाते हैं। इसका निर्माण लावा, राख तथा दूसरे पदार्थों के बारी-बारी जमा होने से होता है। यह जमाव समानान्तर
परतों में होता है। इटली का स्ट्रॉम्बोली (Stromboli) इसका मुख्य उदाहरण है जिसमें प्रति घण्टा के बाद उद्गार होता है। इसे रूम क्रेटर सागर का प्रकाश स्तम्भ (Light house of the Mediterranean) कहते हैं। जापान का फ्यूजीयामा पर्वत इसका सुन्दर उदाहरण है। ढलानों पर बनने वाले छोटे-छोटे शंकुओं को परजीवी शंकु (Parastic Cone) कहते हैं।
JAC Class 11 Geography Important Questions Chapter 3 पृथ्वी की आंतरिक संरचना 7
8. बेसाल्ट प्रवाह क्षेत्र (Flood basalt provinces): ये ज्वालामुखी अत्यधिक तरल लावा उगलते हैं जो बहुत दूर तक बह निकलता है। संसार के कुछ भाग हजारों वर्ग कि० मी० घने लावा प्रवाह से ढके हैं। इनमें लावा मैग्मा प्रवाह क्रमानुसार होता है और कुछ प्रवाह 50 मीटर से भी।

अधिक मोटे हो जाते हैं। कई बार अकेला प्रवाह सैंकड़ों कि० मी० दूर तक फैल जाता है। भारत का दक्कन ट्रैप, जिस पर वर्तमान महाराष्ट्र पठार पर ज्यादातर भाग पाया जाता है, वृहत् बेसाल्ट लावा प्रवाह क्षेत्र है। ऐसा विश्वास किया जाता है कि आज की अपेक्षा, आरंभ में एक अधिक वृहत् क्षेत्र इस प्रवाह से ढका था।

9. मध्य-महासागरीय कटक ज्वालामुखी-इन ज्वालामुखियों का उद्गार महासागरों में होता है। मध्य महासागरीय कटक एक श्रृंखला है जो 70,000 कि० मी० से अधिक लंबी है और जो सभी महासागरीय बेसिनों में फैली है। इस कटक के मध्यवर्ती भाग में लगातार उद्गार होता रहता है। अगले अध्याय में हम इसे विस्तारपूर्वक पढ़ेंगे।